Oswal Maths QB

You might also like

Download as pdf or txt
Download as pdf or txt
You are on page 1of 345

PRO CBSE

EDUCATE-EMPOWER-EVOLVE

Donate US
Your 1rs will help
thousands of students
Upi-Arvishm@YBL

Follow us -
+++ ProCbse.in

@PROCBSE doesn't own these books/materials,neither


created nor scanned. it is uploaded to help financialy
weak students. read our dmca
UNIT I
NUMBER SYSTEMS
CHAPTER

1 REAL
NUMBERS
F Syllabus Fundamental
done earlier Theorem of
and after Arithmetic-Statements after reviewtng two
proofs irrationality of illustrating
of and motivating through examples,
N2, N3 and y5.
nthis chapter you will
Pundamental theorem of. study
Arithmetic. To find
List of Topics
fnctorization method. Rational and Irrational HCF and LCM bå prime Topic-1: Fundamental
numbers. Theorem of Arithmetic
Page No. 1
Topic-2: Irrational
Numbers
Page No. 10

Fundamental'Theoremn of Arithmetic
Concets Covered Fundamental Theorem of
Topic-1 For any two posttive integers a b
and
Artthnetie:
We have HCF (a, b) x LCM (a, bj a
xb
HCF (a, b)
LCM (à, b) LCM (a, b) BCE (a,b)

ERevision Notes
The Fundamental Theorem of Arithmetic
(iii) 15 can be written as 3 X 5 or 5 X 3, where 3 and 5
Every composite number can be expressed as a are prime numbers.
product of primes and this factorisation is unique, The prime factorization of a natural number is
apart from the order in which the prime factors occur. unique, except to the order of its factors.
Fundamental theorem of arithmetic is also called a e.g, 12 detained by multiplying
Unique Factorization Theorem. the prime numbers 2, 2 and 3 Scan to know
more about
Lomposite number = Product of prime numbers together, this topic
Or 12 = 2x 2x3
We would probably write it as
integer greater than 1 can either be a prime
12 = 2 x 3
humbers Or can be written as aunique product of Fundamental
prime numbers. e.g. By using Fundamental Theorem theorem of
of Arithmetic, we shall find the arithmetic
i) 2x
11= 22 is the same as 11 x 2= 22. HCF and LCM of given numbers
6can be written as 2 X 3 or 3 x 2, where 2and 3
(twoor more).
are prime numbers.
Topicwise, MATHEMATICS (8TANDARD), Class->
Oswaal CBSE Ouestion Bank Chapterwise &

prouct the Number


Composite
Pa numbers
prime
are
this from
composite
Every the0CCuT
factorS
prime
.
numbe factorization
be
can is Level
and which P2,
a apart p1, Third
as primnes,
expressed in where Map
unique,
order
Level
of Mind
Second
the
Trace
Arithmetic
of
Theorem Level
First
Fundamental
º

Factorization
b
x
=a Method
qeaNumbe.
integers,
)
L.C.M. (a,
L.C.M.
Prime
& positive f()=3y
62y2
H.C.F. 6xy2H.C.F.=3xy
x =
two b) Example
For = L.C.M,
()
obtain (4,
anyband
a H.C.F. g
and and
To For

Theorems

which
non-negative
be expansion of recurring
can are
qis
of such the non-negativesuchnot expansion
whose &q non
rational
number of
a, number x
Then of
number
is number,
is
number. integer
divides p are g g
wherefactorisation
of decimal of are and
m negative
integers terminates.
which factorisation
irrational
terminates. n, factorisation
rational are rational repeating
m decimal
p positive
primne where n,
then m a where
form, , has
a non-terminating
a a², a are 25, a
prime be where
r
a
=be5has
Letpbe is
divides expansion
the prime Then, prime ,
a a x
whereJ2,J3be in thefom r= M5m,
2
Then,
expressed z of
1. p Let coprime, Let the integers. Let theform
If 2. the form that integers.
3.
decimal 4. that 5.
the is
REAL NUMBERS 3

methodis also called Prime Factorization


This
Method. Fundamental Facts
M Factorization Method to find HCF and LCM:
T Sndallthe prime factors of given numbers. (1) The Euclidean algorithm is useful for
(, HCF of two or more numbers = Product of the reducing acommon fraction to lowest terms.
smallest power of each common prime factor, 714 51x14 14
involved in the numbers.
For example: 65 51x15 15
ot ICM of two or more numbers = Product of the
greatest power of each prime factor, involved in
the numbers. (2) The concept of LCM is important to solve
problem related to racetracks, traffic light
o Key Words etc.

Highest Common Factor (HCF): The HCE (3) In Mathematics problem, where we pair two
f WO or more numbers is the highest number objects against each other, the LCM value is
among all the common factors of the given
numbers.
useful in optimizing the quantities of the
given objects.
Prime Numbers: A number that can be
divided exactly only by itself and 1.

Example
Fnd the LCM of 40, 36 and 126 by applying the prime factorization method.
Step 1. Factorise each of the given positive integers such as:
40 =2 x 2 x 2 x5
36 =2 x 2 x3 x3
and 126 = 2x 3 x3×7
Step 2. Express them as aproduct of powers of primes in ascending order of magnitudes of primes:
40 = 2 x 5,36 = 22x 3 and 126= 2 x 3x7
Step 3. To find LCM, list all prime factors of 40, 36 and 126 with their greatest exponents as:
LCM =2x3x5x7
=8 x9 x5x7
= 2520.

Mnemonics
Concept: Euclid's Division Lemma (a = b¡ + r)
Mnemonics: Alibaba's best product quotation is assent reward.
Interpretation:
Alibaba's A = a
best B = b
quotation Q=9
assets A = addition of bg and r
! reward R=r
Then a=bXq+r.

OBJECTIVE TYPE QUESTIONS (1 mark each)

(A) 3 (B) 4
Multiple Choice Questions (C) 5 (D) 6
[CBSE Board Term-l, 2021|
Q.1.3750
The isexponent of 5in the prime factorization of Ans. Option (B) is correct.
Explanation: According to the prime factorisation,
3750 can be written as
Chapterwise &, Topicwise, MATHEMATICS (STANDARD), Clas$-X
Oswaal CBSE Question Bank
1251 -1=1250
3750 = 5x 5 x 5 x 5 x 3 x 2=5 x3 x 2 9377-2 =9375
It is clear from above, that exponent of 5 in the 15628-3 = 15625 Q.
prime factorisation of 3750 is 4. According to the prime
factorisation,
5x5
1250 =2x5x5 X
Commonly Made Error 9375 =3 X5X5 5x5x5
15625 =5 x5X5 x5×5x5
%5=62:
Sometimes students write answer as 5* = 625, HCF(1250, 9375, 15625) = 5x5x5
which is wrong decision. (CBSE Marking Scheme, 212
then a an.
d5.]f a and b are two coprime numbers,
[CBSE Board Term-I, 2,
are
Answering Tip (A) Coprime (B) Not coprime
(C) Even
(D) Odd
The answer is given 5 = 625 by the student,
it is not clear, so students should read question Ans. Option (A) is correct.
carefully. Explanation: As a and b are co-prime then aar,
b are also co-prime.
Q.2. What is the greatest possible speed at which
a girl Wecan understand above situation with the he,
exact number of
can walk 95 m and 171 m in an of an example.
minutes? AL CBSE Board Term-I, 2021] Let a= 3 and b = 4
(A) 17 nm/min. (B) 19 m/min. a'=3= 27 andb= 4'= 64
=1
Clearly, HCF(a, b) = HCF(3, 4)
(C) 23 m/min. (D) 13 m/min.
Ans. Option (B) is correct. Then, HCF(, b) = HCF(27, 64) = 1
Explanation: As the girlneeds to walk 95 m and 171 [CBSEMarking Scheme, 2021
m at the exact number of minutes.
o6)If nis a natural number, then 2(5" + 6") alwat
So, we have to find HCF of 95 and 171. ends with
According to prime factorization of 95 and 171 (A) 1 (B) 4
95 =5 x 19
171 = 3 x 3 X 19 (C) 3 (D) 2
HCF(95, 171) = 19 AO CBSE Board Term-I, 202:
Hence, greatest possible speed is 19 m/min. Ans. Option (D) is correct.
0.3. Three alarm clocks ring their alarms at regular Explanation: Let us take an example of different
intervals of 20 min., 25 min. and 30 min. powers of 5.
respectively. If they first beep together at 12 noon, 5' = 5
As,
at what time will they beep again for the first time? 5 = 25
[CBSE Board Term-I, 2021]
5 = 125
(A) 4:00 pm (B) 4:30 pm
(D) 5:30 pm 5 =625
(C) 5:00 pm
Ans. Option (C) is correct. It is clear from above example that 5" will always
end with 5.
Explanation: Time when they ring together
= LCM (20, 25, 30) Similarly, 6" will always end with 6.
According toprime factorisation, So,5" + 6" will always end with 5 + 6 = 11
20 = 2 X 2 x 5
Also, 2(5" + 6) always ends with 2 X 11 = 22
i.e.,it will always end with 2.
25 = 5X5
30 = 2 X 3 x 5 [CBSE Marking Scheme, 2021])
LCM (20, 25, 30)= 2 x 2 × 3 × 5 ×5= 300 zThe LCM of two numbers is 2400. Which of the
Thus, 3 bells ring together after 300 minutes or 5 following can not be their HCF?
hours. (A) 300 (B) 400
Since, they rang together first at 12 noon, then they (C) 500 (D) 600
ring together again at 5pm [CBSE Board Term-1, 2021
Q(4)The greatest number which when divides 1251, Ans. Option (C) is correct.
9377 and 15628 leaves remainder 1, 2 and 3
respectively is [CBSE Board Term-1, 2021] Explanation: According to the property, HCF of
(A) 575 (B)) 450 two numbers is also a factor of LCM of same two
(C) 750 numbers.
(D) 625
Ans. Option (D) is correct. Out of all the options, only (C)500 is not a factor
First subtract the remainders from their of 2400.
numbers, respective Therefore, 500 cannot be the HCF.
[CBSE Marking Scheme, 2021|
REAL NUMBERS
8,Which of these numbers can be
Q.8. expressed as a Q. 12. The HCF and the LCM of 12, 21, 15 respectively are
oroduct of two or more prime number ?
()
15 AO[CBSE Delhi&O.D., 2020]
(ü)34568
(A) 3, 140 (B) 12,420
(C) 3, 420 (D) 420, 3
(iü)(15x 13) ADICBSE Q.B, 2021-22|
(4) only(i) Ans. Option (C) is correct.
(B) only (i)
Explanation:
() only () and (i) (D) all - (i), (i) and (iiüi) 12 = 2 X 2x3
Ans. Option (D) is correct. 21 =3x7
Explanation: Prime factors of 15=3x5 and 15 = 3 x5
Prime factors of 34568 = 2 x 29 x 149 HCF of 12, 21 and 15 =3
Prime factors of 15 × 13 =3x5x 13 and LCM of 12, 21 and 15
It is clear that the given numbers in part =2 x 2 X 3×5 x7= 420.
(i), (i)
and (ii) have product of two or more
prime [CBSE Marking Scheme, 2020]
numbers. [CBSE Marking Scheme, 2021] Q. 13. The largest number which divides 70 and 125,
1245 is afactor of the numbers pand g. leaving remainders 5and 8, respectively, is:
(A) 13. (B) 65.
Which of the following will always has 1245 as a
() 875. (D) 1,750.
factor?
()p+9 [NCERT Exemplar]
Ans. Option (A) is correct.
(ü)p X4
Explanation: Required largest number = HCF of
(i) p+9 [CBSE Q.B, 2021-22] (70- 5) and (125 -8) = HCF of 65 and 117= 13
(A) only (iü) (B) only (i) and (iü) Q. 14. If two positive integers a and bare
written as
() only (ii) (D) all - (1),(ü) and (iüi) a= and b = x: x, y are prime numbers, then
HCF (4, b) is:
Ans. Option (A) is correct.
(A) xy. (B) x.
Explanation: Factors of 1245 =1,3,5,15, 83, 249, () . (D) .
415, 1245
Let p = 83 and q = 15
GNCERT Exemplar
Q. 15. If twopositive integers p and g can be expressed as
.pxq= 83 x 15 =1245 p= ab and q = a'b; a, bbeing prime numbers, then
Hence, p Xq has 1245 as a factor.
LCM (p,q) is
(A) ab. (B) a
[CBSE Marking Scheme, 2021] (C) at. (D) a
Q. 10.P and Qare two positive integers such that P=Pg
and Q = (pg, where p and q are prime numbers. Ans. Option (C) is correct. ANCERT Exemplar]
what is LCM (P Q) ? [CBSE Q.B, 2021-22] Explanation: Since p= ab' =axbxb
(A) pg (B) p'g and q=a'b=axaxaxb
(D) p' Thus, LCM of pandg = axaxaxbxb=a'b'
Ans. Option (C) is correct. Q(18, The least number that is divisible by all the
numbers from 1to 10(both inclusive) is
Explanation: P=p'q=pxpxpX9 (A) 10. (B) 100.
Q=(p9 =pxp Xqx4 (C) 504. (D) 2,520.
:. LCM (P and )=p xp Xp XqX
=pxg [NCERT Exemplar]
Ans. Option (D) is correct.
[CBSE Marking Scheme, 2021] Explanation: Required number
Q.11. The total number of factors of prime number is = LCM (1, 2,3, 4, 5,6,8,9, 10)
[CBSE Delhi &OD, 2020] =1x2X 2x 2 x 3 x3×5x7=2,520
(A) 1 (B) 0 Q. 17. The exponent of 2 in the prime factorisation of 484
is:
(C) 2 (D) 3
(A) 1 (B) 2
Ans. Option (C) is correct.
(C) 3 (D) 4
Explanation: We have only two factors (1 and Ans. Option (B) is correct.
number itself) of any prime number.
[CBSE Marking Scheme, 2020] Explanation: Prime factors of 484 = 2 x 11?
Hence, theexponent of 2in 2 x 11 = 2.
These questions are for practice and their solutions are available at the end ofthe chapter
MATHEMATICS (9 Class-A
6 Oswaal CBSE Question Bank Chapterwise &Topicwise,
Q.19)Ifx = 2 x 7, y= 2 x 3 x 5 andz=3 y
Q. 18.If nis an even prime number, then 2(7" + 8") ends LCM (X, M, z) = 2 x 3 x 5 x7, then n
with D
(A) 2 (B) 3
(A) 1 (B) 4
(C) 4 (D) 5
(C) 2 (D) 6
Ans. Option (D) is correct. Ans. Option (C) is correct.
Explanation: As we know that, the geat
Explanation: Since 2is an even prime number, then
n =2
exponents and raise each prime factor fo.
Putting the value of nis 2(7 + 8), we get greatest exponent and multiply them to get LO,
So, n = 4
2(7 +8) =2(49 +64)
=2 x 113 0. 20.If the product of two numbers is 1080 and h.
LCM is60, then their HCF is:
= 226
(A) 18 (B) 30
Hence, 2(7" + 8) ends with 6,where n is an even (C) 45 (D) 90
prime number.

SUBJECTIVE TYPE QUESTIONS


Q.5. Express 429 as a product of itsprime factors.
Very Short Answer Type AP [CBSE Delhi Set-I, 20
Questions (1 mark each)
Topper Answer, 2019
Q.1. If xy=180 and HCF (, y) =3, then find the LCM
(<, y) ADRCBSESQP 2020-21] Ans.
Q.2. The LCM of two numbers is 182 and their HCF is
13. If one of the numbers is 26, find the other. 1| t43
429 3X1||3. T913
U [CBSE Delhi & OD, 20201

S
Topper Answer, 2020 Q. 6.Two positive integers a and b can be written a
Ans.
a = yand b = xy', where x and y are prim
numbers. Find LCM (a, b).
et oth yumles be e. A [CBSE Delhi Set-III, 201
Sol. LCM (y, ) =y. 1
[CBSE Marking Scheme, 2019]
Detailed Solution:
Q.3. Find the sum of exponents of prime factors in the Given, positive integers are
prime factorisation of 196. a=
and
Sol.
AOU|CBSE OD Set-1, 2020]
Prime factors of 196 = 2 x 72
b =n
The sum of exponents of prime factors Here, x and y are prime numbers
:. LCM (, b) = product of
Q.4. Find the LCM of
=2+2 =4. greatest power of x and y
smallest two digit
number and smallest composite number.composite
Q.7.If HCF (336, 54) = 6, find LCM (336, 54)
Sol. Since, the smallest AOUCBSE SOP 2020]
composite number =4
A
and smallest 2digit O CBSE OD Set-I, II, IIL, 2019
.:. LCM of 4 and 10. composite number = 10 Q. 8.What is the HCF of
smallest prime number and the
4 =2X 2 smallest composite number ?
10 = 5 X2
L.C.M. =2x5x 2 ARICBSE Delhi, OD, 2018
= 20 Sol. The required numbers are 2 and 4 and the
: LCM of 4 and 10 = 20 2 and 4 is 2. HCF of
[CBSE Marking Scheme, 2018] 1
These questions are for practice and their
solutions are available at the end of the chapter
REAL NUMBERS 7
DetailedSolution:
Q.3. The HCF and LCM of two numbers are 9 and 360

Topper Answer, 2018 respectively. If one number is 45, find the other
number. RCBSE SQP 2019]
Q.4f two positive integers p and qare written as p =
a and g = b, where a and b are prime numbers
then verify.
1 LCM (P, q) x HCF (P, q) =p.q.
Q.9. Explain why 13233343563715 is a composite DO AP [CBSE SQP 2017]
number ?
R(Board Term-1, 2016) Sol. Since, LCM (. g) = d
Cat Since, the given number ends in 5. Hence, it is a and HCF (P g) =tb
muliple of 5. Therefore, it is acomposite number. LHS, LCM (p. g) x HCF (p) =t xdb
[CBSE Marking Scheme, 2016|1
=pg= RHS.
Short Answer Type 6Explain whether 3x 12 x101 +4is aprime number
Questions-I (2 &3 markseach) or a composite number.
0.1.3bells ring at an interval of 4, 7 and 14 minutes. AI U [Board Term-I 2017, 2015]
three bells rang at 6am, when the three bells will be Sol. 3 x 12 x 101 + 4 = 4(3x3x101 + 1) 1
ring together next ?
= 4(909 + 1)
AOA +E[CBSE SQ, 2020-21] = 4910)
Sol. We know that, =2x2 x 2x5x7x13 1
=a compositenumber
The three bells again ring together on that time
which is the LCM of individual time of each bell [:Product of more than two prime factors]
[CBSE Marking Scheme, 2015]
4 =2x 2
Q.6. Find the HCF and LCM of 90and 144 by the method
7=7x1
of prime factorization. AOUBoard Term-1, 2016]
14 = 2 x7
Sol. Since, 90 =2x 3 x 5
LCM = 2 x 2x7= 28 144 =24 x3
and
The three bells will ring together again at 6: 28 am Hence, HCF =2x3=18 1
[CBSE Marking Scheme, 2020-21] 2 and LCM =2 x3 x5=720 1

0.2. Write the smallest number which is divisible by Q.7. The length, breadth and height of a room are 8 m
both 306 and 657. UCBSE OD Set-I, II, III, 2019] 50 cm, 6 m 25 cm and 4 m 75 cm respectively. Find
Sol. Smallest number divisible by 306 and 657 the length of the longest rod that can measure the
=LCM (306,657) 1 dimensions of the room exactly.
1
LCM (306, 657) = 22338 A Board Term-I, 2016]
[CBSE Marking Scheme, 2019]
Sol. Given, Length=8m 50 cm= 850 cm
Detailed Solution: breadth =6m 25 cm = 625 cm
By using prime factorisation method, height = 4m 75 cm = 475 cm
306 = 2 x 3 x 17 Since, the length of the longest rod is equal to
and 657 = 3 x 73 HCF of 850,625 and 475
HCF(657, 306) =9 850 = 2 x 5 x 17
Product of two numbers
LCM = 625 = 5
HCF(657, 306)
and 475 = 5 x 19
657 x 306
= 657 X 34 Hence, HCE (625, 850, 475) = 5= 25
9
LCM(657, 306) = 22338
Thus, the longest rod that can measure the
divisible by dimensions of the room exactly = 25 cm.
Hence, the smallest number which is
[CBSE Marking Scheme, 2016]
306 and 657 is 22338.

solutions are available at the end of the chapter


y These questions are for practice and their
Osaaal CRSS QUesthon RankChapterwzse &Topvcwise. MATHEMATICs ($TANDARD), Class-X
o12Complete the following factor tree and fns
Commonly Made Brrors composite number :

sh candidates are unable to determine


about what thev have to find. Actually, most
f the candidates don't get to know that the
quesion is about HCF or LCM.
Sometimes students cakulate the kongest
length of ri that ies in the room by finding its |3
iagonal
371

Answeriìng Tips
Adequate practice is necessarr for sucth tvpe of
questions and basic concept of HCF and LCM
shouki be dear.
Sol.
ADuBoard Term-I.
Students should read the question properly. 11130 =565x 2

a$ ind the smallest natural number by which 1,200


55 =185×3
should be multiplied so that the square root of the 2
product is a rational number.
QOU Board Term-I, 2015] 3 185

Sol. Since, 1200 =4x3 x (2 x 5) 1

= x3x 5 1 371
Hence, the required smallest natural number is 3.
[CBSE Marking Scheme, 2015] 7
0.9. Can two numbers have 15 as their HCF and 175 as
their LCM? Give reasons. RBoard Term-I, 2015)
X=11,130
Sol Since, 15 does not divide 175 and
LCM of two numbers should be exactly divisible by Short Answer Type
their HCE 1 Questions-II (4 marks each
.: Two numbers cannot have their HCF as 15 and 0.1. If HCF of 144 and 180 is expressed in the fo.
LCM as 175. 1 131n - 16. Find the value of m.
Q. 10. Check whether 4 can end with the digit0 for any AP (CBSE SQP 02
natural number n. A [Board Term-1, 2015] Sol. Prime factors of 180 = 2 x 3 x5
Sol. If the number 4", for any natural number n,were to Prime factors of 144 = 2x3
end with the digit zero, then it would be divisible HCF (180, 144) = 2x3=36
by 5 and 2. 36 = 13m - 16
1
That is, the prime factorization of4" would contain Solving, we get m= 4.
the prime 5 and 2. This is not possible because [CBSE SQP Marking Scheme, 02
4"= (2)"; so, the only prime in the factorization of Detailed Solution:
4" is 2. So, the uniqueness of the Fundamental Given, HCF of 144 and 180 is in form of 13n: - lo
Theorem of Arithmetic guarantees that there
Now, HCF of 144 and 180.
are noother primes in the factorization of 4". So,
there is no natural number n for which 4" ends 144 = 2 x 2 x 2x 2x3X3
2 =2x3²
with the digit zero.
[CBSE Marking Scheme, 2015] 180 = 2 x2x3x3 X5
= 2 x3?x5
Q1]Find HCF of the numbers given below:
k, 2k, 3k, 4k and 5k, where k is a positive integer. HCF(144, 180) = 2² x 3
UBoard Term-1, 2015] = 36

These questions arefor practice and their solutions are available at the end of the chapter
REAL NUMBERS 9
: HCF of 144 and 180 is given in the form of
13m- 16.
Q.2. Three sets of Science, History and Drawing books
have to be stacked in such a way that all the bookS
131m - 16 =36
are stored topic wise and the height of each stack is
13m =36 + 16 the same. The number of Science books is 192, the
13m =52
number of History books is 480 and the number of
52
Drawing books is 672. Assuming that the books are of
1M = the same thickness, determine the number of stacks
13 of Science, History and Drawing books. A+
m = 4 Sol. In order to arrange these books, we have to find
HCF that divides 192, 480 and 672 exactly.
So, 192 =2 x3
Commonly Made Error 480 = 2° x3x5
and 672 =25 x 3 x7
Chudents often commit errors in finding HCF
dLCM. Some students find HCF but used .:. HCF (192, 480 and 672) = 2 x3 = 96 2
rule LCM. There must be 96 books in each stack.
192
.:. Number of stacks of Science books = =2
Answering Tip 96 1

480
Iinderstand the rule of the difference between No. of stacks of History books= =5 1
LCM and HCE 96
672
o2 Fnd HCF and LCM of 404 and 96 and verify that No. of stacks of Drawing books = =7. 1
HCF XLCM = Product of the two given numbers. 96

UCBSE Delhi,OD, 2018] Q.3. The HCF of two numbers is 145 and their LCM is
2175. If one number is 725. Find the other number.
0.3. Find the greatest number of six digits exactly
divisible by 18, 24 and 36. R
Sol. Given, HCF of two numbers = 145
AdA Board Term-I, 2016]
LCM of two numbers = 2175
Sol. LCM of 18,24 and 36
18 = 2 x 3 One number = 725 1

24 =2 3 As we know that,
36 =2 x32 1 One number x other number = HCF x LCM 1
ach 725 X other number = 145 x 2175 1
LCM (18, 24, 36) = 2° x 3
= 72 1 145x2175
e for other number = 1
The largest 6 digit number is 999999 1 725
13888 Quotient
other number = 435. 1
P,202 72 ) 999999
-72
1
279
Long Answer Type
-216 Questions -1I (6 marks each)
1
639 Q.1.Afruit vendor has 990 apples and 945 oranges. He
-576 packs them into baskets. Each basket contains only
1 639 one of the two fruits but in equal number. Find the
-576 number of fruits to be put in each basket in order to
,2020) have minimum number of baskets.
639
-576 A Board Term-1, 20161
1-16, 63 ’ Remainder
Sol. For finding minimum number of basket, we get
i. The required number = 999999 -63 = 999936.1 HCF of 990 and 945.
x3 [CBSE Marking Scheme, 2016] Prime factors of 990 = 2 x 3 x 5x 11 1
Prime factors of 945 =3 x5x7 1
Long Answer Type Since, HCF of 990 and 945 is 3 x 5or 45. 2
Questions -I (5 markS each) Thus, the fruit vendor should put 45 fruits in each
ate Fundamental theorem of Arithmetic. Find LCM basket to have minimum number of baskets. 2
of numbers 2520 and 10530 by prime factorization
method. OR|Board Term-1, 2016] [CBSE Marking Scheme, 2016]
Hse questions are for practice and their solutions are available at the end of the chapter
10 Oswaal CBSE Question Bank Chapterwise &Topicwisc, MATHEMATICS (8TANDARD), Class-X
Q.2. Can the number 6", n being a natural number, end
with the digit 5 ? Givereasons. .. 6" = (2 x 3)" = 2" x 3"
From the fundamental theorem of arithre.
A |Board Term-I, 2015] prime factorization of every composite nu
Sol. If 6"ends withO or 5, then it must have 5as a factor. unique.
or 5.
.. 6" can never end with 0
But only prime factors of 6" are 2and 3. [CBSE Marking Schene:

Irrational Numbers
Topic-2 COncepts CoveredRattonat& Irrutionat Numbers.

Revision Notes

Rational Numbers: A number in the form , where p O- Key Words


and qare co-prime numbers and g#0, is known as Co-Prime Numbers: Co-prime numbers
only one
rational number. are those numbers that have
3 2
etc. are rational numbers. common factor. For example: 3 and 5. 11
For example:2,-3, 5 and 13 etc.
irrational if it
Irrational Numbers: A number is called
where p and q are
cannot be writtern in the form E, Fundamental Facts
example, V2, V3, V5, r are
integers and q * 0. For
irational numbers.
(1) The discovery of irrational numbers i
Scan to know
usually attributed to Pythagoras, mor
number. If
Let p be a prime divides a, more about specifically to the Pythagorean Hippasus o
p divides a, then p this topic metapontum who produced a proof of the
where a is a positive integer.
of a irrationalityof V2,
The sum or difference
rational and an irrational number cannot
(2) Irrational number are numbers that
of two whole
is irrational. be expressed as the ratio
and quotient of
The product rational Rational and
numbers.
and an
a non-zero irrational number
irrational number is irrational.

Example
irrational numbe.
Show that 2/3 +7 is an
can be expressed as
wh

2/3+7 be a rational number. Since, a rational number


Step 1. Let
b+0 anda&bare integers.
2v3+7 =
Step 2. Then

25 =-7

Here, L.HS. = V3 is an irrational.


But, R.H.S. = is a rational.

So, it is not possible.


rational is incorrect.
Step 3. Hence, our assumption that 2/3+7 is a
Hence, 2/3 +7 is an irrational number.
REAL NUMBERS 11

OBJECTIVE TYPE QUESTIONS (1mark each)

Q4Which of these is a rational number ?


Multiple Choice Questions (B) 545
(C) 0.3466666.. (D) 0.345210651372849.
Theproducttof anon-zero 1rational and an irrational [CBSE Additional Ouestion, 2021-22]
Q.1.numberis:
(A) always irational. Ans. Option (C) is correct.
(B) always rational. Explanation: As we know that 3z, 5v5 and
I) rational or irrational. 0.345210651372849..are irrational numbers.
(D)one. INCERT Exemplarl But 0.3466666..is arational number.
.:. Let X= 0.3466666...
is a/an:
100x =34.66666..
or 1000x = 346.66666..
(A) fraction (B) rational number
irational number (D) none of these Then (1000x - 100x) = (346.66666. - 34.66666..)
correct. 900x = 312
Ans. Option (C) is 312 78
1 X=
Esplanation: 900 225

1 Q.5.The number 1.732 is:


We have
2
is a rational but 2 is irrational. (A) an irational number
(B) a rational number
So. the product of a rational and an irrational is (C) an integer
irational. (D) a whole number
1
Hence, N2 is irrational. Ans.Option (B) is correct.
Explanation: We have,
1732 433
oQ.3. (3+V5) is: 1.732 =
250
1000
(A) an integer which is a rational numbe.
(B) a rational number 0.6. 2.1311 3111 311113 .....is
(C) an irrational number (A) an integer
(D) none of these (B) a rational number
Ans. Option (C) is correct. (C) an irrational number
Explanation: Here 3 is a rational and V5 is an (D) none of these.
iational. Ans. Option (C) is correct.
. The sum (3+\5) i.e., rational and irrational is Explanation: Given number is non-terminating,
non-repeating decimal.
irrational.
Hence (3 +V5) is an irrational number. So, it is an irrational number.

SUBJECTIVE TYPEOUESTIONS
Very Short Answer Type 6V5 +65
Sol. For writing
Questions (1 mark each) 2/5
= 6which is rational. %
Q.4. Write one rational and one irrational number lying [CBSE Marking Scheme, 2018]
between 0.25 and 0.32. mCBSE SQP, 2020] Detailed Solution:
2/45 +3/20 2/9x5+3/4x5
Q2 Findarational number between and y3.
2/5 2V5
UCBSE Delhi Set- I, II, II, 2019]
2x3/5+3x 2V5
2/45 +3/20
whether on simplification 2/5
2V5
gives an irrational or a rational number. 6V5+6V5
AP [CBSEComptt. Set I, II, III 2018] 2/5
ne questions are for practice and their solutions are available at the end of the chapter
(STANDARD), Class-X
MATHEMATIC8
&Topicwise,
12 Oswaal CBSE Question Bank Chapterwise
and q 0
(6+6)V5
2V5
= 55 is rational. whi,
12V5 =6 74-P is rational
2/5
contradictory. irrational.
which is a rational number. Hence, 7-V5 is
(CBSE Marking Scheme, 2
Short Answer Type
(2 marks each) number 0.3178 in the form of rax
a.3)Express the
Questions-I
that (5+3/2) number4
Q.1. Given that is irrational,prove
Delhi OD, 2018] number.
numberAP/CBSE irrational
7y5 is an
is an irrational [CBSE Comptt. Set I,
I, III, 2018] 0.4. Prove that AP INQ
2/7 is an
irrational number, w
(5+3/2) is a rational number. 0.5. Show that 5+
Sol. Let us assune
be an irrational number.
V7is given to
AP[CBSE Delhi & OD, ;
5+3/2 =
integers) Sol.
(where, q+0 and p and gare TopperAnawer, 2020
p-5q
J2 = 34
thatto Yatienal
censa:
is irrational. ht.us assum.tetheuf.infomwhu Land q ae lunu
This contradicts the given fact that v2 Fhan $taVT.ui.
amd qyto.
number.
Hence, (5 +3/2) is an irrational
[CBSE MarkingScheme, 2018]1
Detailed Solution:

Topper Answer, 2018

Te prove: $43 is
iohbnal.A -SY is atienal as.famd q ae
2
unkgera
Thus corlsadiet the quscn yact thatV7us ltatika

Boved.

nalbna! Short Answer Type


Questions-II (4 marks eac
But we kroy het V2 B. na howa
Q.1. Prove that 2-V3 is irrational, given that v
irrational. AOAP (CBSE SQP 2020
Q.2. Show that 7-45 is irrational, iven that v5 is Sol. Let 2-3 be a rational number
irrational. U [SQP 2018-19] We can find co-prime numbers a and b(b 0) su
that
Sol. Let us assume, to the contrary that 7-5 is
rational.
2-V3 =
7-5 2 ,where pand qare co-prime 2- = 3
These questions are for practice and their solutions are available at the end of the
chapter
REAL NUMBERS
So, we get 2b--a 13
Detailed Solution:
Let us assume 2 +5N3 be a
Sine «andib are 2b-a rational number.
integers, we get Let 2+5/3 a
is irrational and so N3 is
irrationalnumber. rational. But J3 is an [b0, aand bare integers]
1
Bat rational number cannot
irrational number. be equal to an
Which contradicts our statement is a rational
number.
Therefore 2-N3 is irrational. But this contradicts the fact
1 that 3 is an
number. irrational
[CBSE Marking Scheme, So, our
Q.2. Giventhat N5 is 2020-21] assumption is
wrong
an irrational irrational, prove that 2N5-3 is Therefore, 2+5V3 is an irrational number.
number. AU AP [CBSE SQP- 20201
Sol Let us assume, to the contrary, that 2/5-3 is a
Long Answer Type
rational number Questions-I (5 marks each)
.2/5-3 P Q.1. Prove that 2+V3 is an
,where pand q are integers and q0 5 irrational number, given
that J3 is an irrational
V5 P+34 number.
2q ..(1) 1 AP [CBSEDelhi
Set-III, 2019]
Since p and q are integers a.2. Prove that 2 is an
irrational number.
p+34 is a rational A UCBSE Delhi Set-I, I,
20 number 1 III, 2019]
[Borad Term-1, 2015]
.J5 is a rational number Sol. Let us assume J2 be a
which is contradiction rational number and its
as 5 is an irrational
number 1 Ssimplest form be ;,4
4 and b
are coprime positive
Hence, our assumption is wrong and hence
is an irrational number.
2V5-3 integers and b+ 0.
So
[CBSE SQP Marking Scheme, 2020j1 b
o.B Prove that 3 is an irrational a= 2
number.
Thus a' is amultiple of 2
AP [CBSE Board, 2019) ’ais amultiple of 2.
Q.4. Prove that 2+ 5/3 is an
irrational number, given Let a = 2m for some 1
that v3 is an irrational integer m
number. =2m
1
AUAP |CBSE OD, Set-I, 2019| Thus b' is amultiple of 2
Sol. Let 2+5/3 = a, >bis amultiple of 2
2 where'a is a rational number. 1 Hence, 2 is a common factor of a and b. 1

Then, This contradicts the fact that a b


1 and are co-primes
5 Hence, 2 is an irrational number.
)S 1
Which is a contradiction as LHS is
RHS is rational irrational and [CBSE Marking Scheme, 2019]
Detailed Solution:
: 2+5/3 can not be
rational. Let 2 be a rational number,
Hence, 2+ 5V3 is irrational. P.qare integers, q0.
Then, v2 = E, where
2
If HCF (., ) 1, then by
[CBSE Marking Scheme, 2019] dividing p and q by HCF
(p, ), V2 can be reduced as
These questions are for practice and their solutions are available at the end of the chapter
Class-X
MATHEMATIC8 (8TANDARD),
Topicwise,
Bank Chapterwise & Here pand gare divisible by 5, which cort,
14 Oswaal CBSE Question
co-primes.
...) p and g are
the fact thatassumption
where HCF (4, b) = 1 is false
Hence,our Se
number.
an irrational
Hence Pr
/5 is
W2b =a [CBSEMarking 5cheme,;
20 =a
ais divisible by 2 LongAnswerType
.i)
a is divisible by 2 Questions-II (6 marks eo
integer
a = 2c, where c is an
J2b = 2c Provethat pt a is an irrational, where p,
26 = 4c?
.).
are primes. contrad ).1.
using the method of
=2 Sol. We prove this by rational number
is divisitble by 2
..1)
Assume that Vpt9 is a
bis divisible by 2
(a, b) = 1
From (ü) and (ii), 2 is a
common factor of a and b, Then, yp +Va = # where HCF
which contradicts (i). get
Hence Proved. Squaring both sides, we Ans.

So, 2 is an irrational number.


3)Prove that v5 is an irrational number.
AOUCBSEOD Set-I, 2020]
p+q+2/pg
Sol. Let 5 be a rational number. ).2.

1 2/Pg =

where p and qare co-prime integers and g 0 Ans

On squaring both the sides, we get


Since Vpg is an irrational number whe

is a rational number.
1
or p'= 5
pis divisible by 5
So, contradicts
:.pis divisible by 5
Let p = 5r for some positive integer r, assumption.
Ar
p=25,2 1 Hence, Vp+yg is an irrational number.
5 = 25-2 Q.2. Prove that(v2 +N3) is an irrational number
or f=5 Q.3. Provethat 6+/2 is an irrational number.
*gis divisible by 5
uNCE
:.q is divisible by 5.

COMPETENCY BASED QUESTIONS (4 marks each

In other words, The word derives from the Li


O Case based MCOs liber, "book", whereas a Latinized Greek wo
I. Read the following and answer any four
questions bibliotheca, the origin of the word for librar
from Q.1. to Q.5. German, Russian and the Ronman language.
A place in which literary, musical, Artistic or Toenhance the reading skills of grade Xstude
reference materials (such as books, manuscripts, the school nominates youand two of your friend
recordings or films) are kept for use but not for sale. set up a class library. There are two sections-sect
These questions are for practice and their solutions are available at
the end of the chapter
REAL NUMBERS 15
section Bof grade X. There are 32
Aand students in Ans. Option (B) 0s correct.
section Aand 36 students in section B.
Explanation: Given, p= a =a %bxb
(CBSE QB, 2021| q =db=aXaxb
LCM of (p, q) =
II. Read the following and answer any four questions
from Q.1. to Q.5.
Educational organization means any organization
within this state that is not organised for profit,
the primary purpose of which is to educate and
develop the capabilities of individuals through
instruction by means of operating or contributing
to the support of a school, acadeny, colege or
2.1. What isthe minimum number of books you will
university.
Aseminar is being conducted by an Educational
acquire for the class library, so that they can be Orgarisation, where the participants will be
distributed equally among students of Section A educators of different subjects. The number of
orSection B? participants in Hindi, English and Mathematics are
(A) 144 (B) 128 60, 84 and 108 respectively. [CBSE QB, 2021]
(C) 288 (D) 272
Ans. Option (C) is correct.
Ezolanation: We have to find the LCM of 32 and 36.
LCM(32, 36) = 2 x9= 288
Hence, the minimum number of books required to
distribute equally among students of section Aand
section Bare 288.
). 2. If the product of two positive integers is equal to
the product of their HCF and LCM is true then, the
HCF (32, 36) is
Q.1. In each room the same number of participants
(A) 2 (B) 4 are to be seated and all of them being in the same
(C) 6 (D) 8 subject, hence maimum number participants that
Ans. Option (B) is correct. can accommodated in each room are
Explanation: LCM (32, 36) = 288 (Proved above) (A) 14 (B) 12
By using Formula, (C) 16 (D) 18
Product of both given numbers Ans. Option (B) is correct.
HCF (32, 36) = Explanation: No. of participants seated in each
LCM (32, 36)
room would be HCF of all the three values above.
32×36 60 =2x 2 x3× 5
= 4.
288 84 =2x 2x3x7
).3.36 can be expressed as a product of its primes as 108 =2X2 x3x 3 x 3
(A)22x 32 (B) 2x 33 Hence, HCF = 12.
(C) 2°x 3! (D) 2° x 30 0.2. What is the minimum number of rooms required
Ans. Option (A) is correct. during the event?
Explanation: Prime factorisation of 36 (A) 11 (B) 31
= 2 x 2x3x3 (C) 41 (D) 21
=2 x 3 Ans. Option (D) is correct.
Q. 4.7 x 11 x 13 × 15 + 15 is a Explanation: Minimum no. of rooms required are
(A) Prime number total number of students divided by number of
CE
(B) Composite number students in each room.
(C) Neither prime nor composite 60+84+108
No. of rooms =
(D) None of the above 12
Ans. Option (B) is correct.
= 21
Explanation: We have,
7x 11 x 13 × 15 + 15 =15(7 x 11 x 13+ 1) 0.3. The LCM of 60,84 and 108 is
= 15(1001 + 1) (A) 3780 (B) 3680
W = 15 x 1002 (C) 4780 (D) 4680
ar
=3 x 5x 1002 Ans. Option (A) is correct.
Hence, it is a composite number because it has more Explanation: Prime factorization of
than 2 factors. 60 =2 x 3x 5
de0.lfp and gare positive integers such that p = ab 84 =2 x3×7
and q = ab, where a, bare prime numbers, then and 108 = 2² x 3
et the LCM (p, q) is .. LCM (60, 84 and 108) =2x3 x5x7
(A) ab (B) a2 =4x 27 x 35
() (D) ab =3780.
Clas3-X
Topicwisc, MATHEMATICS (STANDARD),
Bank Chaptcrwisc & plants
16 Oswaal CBSE Question totalnumber of
is 0. 2. Find the (B) 225
and LCM of 60,84 and 108 (A) 135
Q. 4. The product of HCF (D) 45
(A) 55360 (B) 35360 (C) 360
(C) is correct.
(C) 45500
(D) 45360 Ans. Option
Explanation: Total number of plants 135 +22
= 360 plants
Ans. Option (D) is correct. = 12 the prime fact
Explanation: HCF (60, 84 and 108) |Prove in Q.1.] of exponents of
0.3. Find the sum number of columns in
the maximum which
[Prove in Q.3.] can be planted.
3780
and LCM (60, 84 and 108) = and 108 (B) 3
The product of HCF and LCM of 60, 84 (A) 5
(D) 6
..
3780
= 12 x (C) 4
is correct.
= 45360. Ans. Option (B) that the max
primes as Explanation: We have proved
as a product of its = 45
Q.5. 108 can be expressed (B) 2°x 33 number of columns
of 45= 3 x3x5
(A) 2°x 3² So, prime factors =3 x 5!
(D) 2²x33
() 2²x 3² 2+ 1 =3.
correct. Sum of exponents = thev.
Ans. Option (D) is numbers of rowin which
Explanation: Prime factorisation
of 108 0.4. What is total
= 2X 2x 3 ×3X 3 planted
(B) 5
= 22 x 33 (A) 3 (D) 15
answer any four questions (C) 8
correct.
III. Read the following and Ans. Option (C)is Rose plants
from Q.1. to Q.5. Explanation: Number of rows of
usually near a house, where 135
Aplot of ground, fruits or herbs are
=3
flowers, shrubs, vegetables, ground or other
45
cultivated. In the words, a piece of plants, trees = 45
225
space, commonly with
ornamental Number of rows of marigold plants
public recreation area,
etc., used as a park or other +5=8
and delightful spot or Total number of rows = 3
a public garden, a fertile rose plants planted of the prime facto
region. Agarden consists of 135 Q.5. Find the sum of exponents
There are another set
columns. total number of plants.
in certain number of
is to be planted in the (B) 3
of 225 marigold plants, which (A) 2
(D) 6
same number of columns. (C) 5
Ans. Option (D) is correct.
Explanation:
Total number of plants = 135 + 225
= 360
x 3x3)
The primefactors of 360 = 2x 2 x 2
= 2 x 3² x 51
Sum of exponents =3+2+1=6.
Case based Subjective
Questions
columns in
Q.1. What is the maximum number of LRead the following text and answer any
which they can be planted ? questions from Q1 to Q5.
(A) 45 (B) 40 Mathematics exhibitions are organized at
(C) 15 (D) 35 schools and colleges to help the students
Ans. Option (A) is correct. understand mathematics more interestin;
Explanation: No. of rose plants = 135 Mathematics exhibitions are very important pat
No. of marigold plants = 225 the mathematics curriculum. The main objectiv
mathematics exhibitions are:
The maximum number of columns in which they
can be planted To inculcate interest in mathematics among
be
students. A Mathematics Exhibition is
= HCFof 135 and 225
conducted in your School and one of your fre
.:.Prime factors of 135 = 3 x3×3 X 5 SO
is making a model of a factor tree. He has
and 225 =3x3 ×5 x 5 difficulty and asks for your help in completit
..Max, number of column =3x3 x 5 = 45. quiz for the audience.
REAL NUMBERS 17

II. Read the following text and answer any four


questions from Q1 to Q5.
A morning walk may help improve your mental
clarity and ability to focus throughout the day.
A recent study found that amongst older adults,
5 2783
those who started their days with a morning walk
improved their cognitive function, compared to
253 those who remained sedentary. Walking may also
help you thínk more creatively. In a morning walk
three students step off together, their steps measure
80cm, 85 cm and 90 cm respectively.

2783

253 Q.1. What is the minimum distance each should walk


so that he can cover the distance in complete steps?
Sol. We have to find the LCM of 80, 85 and 90 by using
prime factorisation method.
80 =2 x5
85 =5 x 17
and 90 = 2x 3x5
.:. LCM (80, 85 and 90) = 2 x 3² x 5x 17
= 12240. 1

0.1. What will be the value of x? Hence, the minimum distance each should walk so
Sol
X =2783 x 5 that he can cover the distance 122240 cm or 122 m
X= 13915 1 40cm.
0. 2. What is the HCF of 80 and 90?
0. 2. What will be the value of y? Sol. From question 1, we have to proved
Sol. 2783 = y x 253 80 = 2 >x 5
2783 and 90 = 2 x3 x 5
y 253 HCF (80, 90) = 2 X5= 10. 1
So
1 Q.3. Find the sum of exponents of the prime factors of
y=11
total distance.
0.3. What will be the value of z? Sol. Total distance = (80 + 85 + 90) cm
Sol. 253 = 11 x z = 255 cm
253 .: The prime factors of 255 = 3 x5× 17
Hence, the sum of exponents =1+1+1=3.
Z = 1
11

z = 23 1 Q.4. What is the product of HCF and LCM of 80, 85 and


90?
Q.4. Find the prime factorisation of 13915. Sol. LCM (80, 85 and 90) = 12240 [From Q. 1.]
Sol. Prime factorisation of 13915 and HCF (80, 85 and 90) = 5
=5X 11 x 11 x 23. and product of HCFand LCM= 12240 X5
= 61200 1
=5x 11x 23. 1

5. Find LCM (x, y and z). Q.5.Find the product of exponents of the prime factor
of LCM (80, 85 and 90).
Sol. x= 13915 = 5 X 11'x 23
[Proved in Q. 4.] Sol. From question 1,
[Proved in Q.2.] LCM (80, 85and 90) = 2'x3 x5 x 17
y=11 ..Product of exponernts of its prime
z = 23 [Proved in Q.3.] = 4X 2 X1×1
. LCM (13915, 11 and 23) = 5 x 11 x 23 = 8. 1
= 13915. 1
18 Oswaai CBSE Question Bank Chapterwise &Topicwise, MATHEMATICs (8TANDARD), Class-X
II. Read the following text and answer the following Hence, the longest rod which can
questions on the basis of the same: given dimensions of the roorr exactly 75 om
Afamily room is an informal, all purpose room is 0.2 What is LCM of thegiven three measuremens.
a house. The family room is designed to be a place Sol For LCM, taking the geatest exPonent ard.
where familyand guests gather for group recreation each prime factor to the reatest exponert
like talking reading. watching TV and other family multiply them.
activities. The length, breadth and height of aroom From question 1,
are &m 25 cm, 6 m 75 cm and 4 m 50 cm. LCM (825, 675 and 450) = 2x 3² x sx11
= 14830.
0.3. If the HCF (825 and 675) = 75, then find LCV
and 675).
Sol. By using fundamental theorem of arithmes.
get
825x675
LCM (825 and 675)= 75
= 7425.
0.4. Find the sum of exponents of the prime factor.
LCM (825 and 675).
Sol. From question 3,
LCM (825 and 675) = 7425
Q. 1. Determine the longest rod which can measure the Prime factors of 7425 = 3 x 5x 11
three dimensions of the room exactly. Now, the sum of exponents =3+2+1=6.
Sol. Given, Length = 8m 25 cm = 825 m 0.5. Prove that: Product of LCM and HCF (825 and E
Breadth=6cm 75 cm = 675 cm = Product of 825 and 675.
and height 4m 50 m= 450Cm Sol. LHS = Product ofLCM and HCF (825,6
we have to find the HCF of 825, 675 and 450 by =7425 x 75 [From question.;
factorization method. = 556875.
825 =3 x 5x 11 RHS = Product of 825 and 675
675 =33x5² = 825 X 675
and 450 = 2x 3x52 1 =556875.
HCF (825, 675 and 450) =3 x5 Hence, LHS = RHS Prove
=75

Solutions for Practice Questions (Topic-1)


Multiple Choice Questions
Detailed Solution:
i Sol. 14: (B) Since a=xy=xx*x*xyxy and
b=xy' =xxyxyxy, Product of two numbers,
Thus, HCF ofa and b= xxyxy=y Xy = 180
Ans. 20: (A) Since, HCF x LCM = Product of HCF (*, y) =3
Numbers By using fundamental theorem,
HCF × 60 = 1080 HCF (x, y) x LCM (x, y) =x Xy
1080 3x LCM (x, y) =180
HCF =
60
LCM (x, ) = 60.
HCF = 18.
Sol. 7:Since, HCF XLCM = Product of numbers
i Very Short Answer Type Questions 6x LCM=336 x 54
Sol. 1:We know that,
"LCM XHCF = Product of two numbers". LCM = 336x 54
6
(LCM) (3)=180
LCM = 60, LCM =56 x 54
[CBSE Marking Scheme, 2020-21] LCM =3024
[CBSE Marking Scheme, 2019] 1
REAL N¯MBERS 19

Short Answer lype Questions-l Detailed Solution:


Ans. 3: Since, HCF xLCM = Product of two numbers
Then, 9 x 360 = 45 x 2nd number 1 Topper Answer.2018
2nd number = (9x 360)
45
PubeSi 404, 6.
Thus, 2nd number 72 1
02, 40
[CBSE SQr Marking Scheme, 2019) o, 24

Ans, 11: HCF of and


k.2 HCF" greaksl cDMnon atohe 2 4.
k.3 is k.
k.2?
k. 5
2
[CBSE SQP Marking Scheme, 2015] :384
Short Answer Type Questions-l|
Sol. 2: Since, 404 = 2 X2x 101= 2 x 101 3t84.
and Henee, HCFY LCH" prode of_to ronbes.
96 =2 x 2 x 2x2 x2 x3
4
=2° x3 1
.HCF of 404 and 96 = 2 = 4 1
Long Answer Type Questions
LCM of 404 and 96 = 101 x 2° x3 = 9696 Sol. 1: Fundamental theorem of arithmetic: Every
HCF × LCM =4x 9696 = 38784 composite number can be expressed as the product
1 of powers of primes and this factorization is
Also, 404 x 96 =38784 unique.
Since, 2520 = 2 x 3 x5x7 12
Hence, HCF × LCM = Product of 404 and 96. and 10530 =2 x 3+ x 5x 13 1
Hence Verified. 1 LCM =2 x3x5 x7× 13
= 294840 236
[CBSE Marking Scheme, 2018]
[CBSE Marking Scheme, 2016] :

Solutions for Practice Questions (Topic-2)


Multiple Choice Questions Short Answer Type Questions-l
Ans. 1: (A) The product of a non-zero rational with
and an irrational number is always irrational. Sol. 3: Let, x= 03178
Very Short Answer Type Questions or x =0.3178178178
Ans. 1: rational number = 0.3
irrational number = 0.30300300030000 Now, 10,000 x = 3178.178178 ..)
and 10 x = 3.178178
Sol. 2: Any one rational number between 2 (1.41 Subtracting, equation (ü) from eq. (1), we get
approx.) and 43 (1.73 approx.) 9990 x = 3175
eg, 1.5, 1.6, 1.63 etc. Or r=
3175
[CBSE Marking Scheme, 2019] 1 9990
iDetailed Solution:
635
Since, V2 = 1414... Hence,
1998
and V3 = 1.732.
Now, we can write n' rational numbers between Sol. 4: We prove this by using the method of
them e.g., just greater than 1.414 and less than 1.732 contradiction. Assume that 7V5 is a rational
and it should be terminating or not terminating number:
recurring.
eg., 1.415659, 1.416893, 1.715644, Then, 7V5 =
Therefore,one rational number betweern V2 and
3 is 1.416893.
(8TANDARD), Class-X
20 OswaalCBSE Question Bank Chapterwise &Topicwise, MATHEMATICS

where pand qare coprime numbers (q +0) i.e., J3 is arational number


1
which contradicts the fact that 3 iss
7q
2+3 is an irrational number.
irrational ;
Since V5 is an irrational number and is a Therefore
5 1
7g
rational number. [CBSE Marking Scheme, 201% Funo
:. which contradicts our assumption. Detailed Solution:
(a)
Therefore, 7V5 is irrational. 1
2+ V3 is a rational number (b)
Short Answer Type Questions-l Let
5
Sol. 3: Therefore, we can write it in the form of
Topper Answer, 2019 2+V3
5

2+/3 = 5p

V3 = 5p.-2
5p-24
J3 =
Since, p and q are co-prime integers, then 5p -2
is a rational number.

Amdegu 'a!. But this contradicts the fact that v3 is an irrationa


number.
So, our assumption is wrong.
2+W3
Therefore, 5 is an irrational number,
Hence Proved.
Long Answer Type Questions-ll
Sol. 3: We prove this by using the method of
contradiction.
fout, pandsca-peims. Assume that 6+2 is a rational number. 1

Then, 6+v2 = P
Long Answer Type Questions-l 1
where HCF (P, 4) = 1
Sol. Let us assume 2+v3 be a rational number. 6+V2 = P
Now
5

Let 2+V3 1
5
(b +0, a and bare integers) J2 = -6
5a-2b
V3 = 1
P-69 ..)2
’4,bare integers
5a-2b Since, LHS of equation (i) is an irrational number
is a rational number 1
whereas RHS of equations (i) is a rational number1
the equation (1) contradicts our assumption.
1
Thus, 6+2 is an irrational number
REAL NUMBERS 21

REFLECTIONS
Fundamental
(a)
theorem of Arithmetic has very deep and significant application in the field of mathematics.
Willyou be able to write any one of its application?
1

() How can you prove that 2, V3 & V5 are irrational numbers?


SELF ASSESSMENT PAPER -1

Maximum Time: 1 hour MM:)


[1 x6,
I. Choose the correct alternative from the given options.
Q.1.Ifa Xb= 1600 and LCM of aand b is 320., then the HCF of aand b is:
(A) 10 (B) 5
(C) 20 (D) 16
Q.2. The difference of the irrational numbers (3+ 2/3) and (3-2v3) is:
(A) 4v3 (B) 6+2V3
(9) 7+4W3 (D) 6
Q.3. The LCM of 25 x 3 x 5 and 2 x 7x5 is:
(A) 140 (B) 3260
(C) 1360 (D) 3360
Q.4. Thequotient of (5+3/2) and (5-3/2) is:
43+30/2 4330/2
(A) (B)
7

(C) 43+302 (D) - (43+30/2)


Q.5. Given that LCM of twonumbers is 1449 and their HCF is 23. If one of the numbers is 207, then other number:
(A) 261 (B) 161
(C) 361 (D) 167
Q.6. r is a/an:
(A) rational (B) composite
(C) irrational (D) prime
II. Case-Based Question [1 x4=
Q.7.Asweetseller has 420 Bundi Laddoo and 130 badam barfis. He wants to stack them is such away that each sa
has the sarme number, and they take up the least area of the tray.

Read carefully the above paragraph and answer any four


(i) The prime factorsof 420 are: questions:
(A) 2² x 3 x5 X7 (B) 2 X 3 x5 x7
(C) 2x 3 x5 x7
(D) 2 x3x 5x7
(i) Using Euclid's algorithm, the HCF of 420 and 130 is:
(A) 10
(C) 20 (B) 30
(ii)The LCM of 420 and 130 is: (D) 15
(A) 4560
(C) 6540 (B) 5460
(D) 5640
REAL NUMBERS 23
(iv) The sum of exponents of prime factors in the prime factorization of 420 is:
(A) 2
(B) 3
(C) 5 (D) 4
(v) The sum of exponentsof prime factors in the prime factorization of 130 is:
(A) 1
(B) 3
(C) 2 (D) 4
IL. Very Short Answer Type Questions [1x3 =3]
Q. 8.aandlb aretwo positive integers such that the least prime factor of ais 3and the least prime factor of bis 5. Then
calculate the least prime factor of (a + b).
Q.9.If HCF of two numbers is 18 and their product is 12960, then find the LCM.
Q. 10. If pis a prime number,then find LCM of p. p and p.
TV Short Answer Type Questions-I [2 x 3 = 6]
2/125 +/25
Q. 11. Write whether on simplification gives an irrational or a rational number.
3V5
o. 12. Ona morning walkthree persons steps off together and their steps measure 40 cm, 42 cm and 45 cm respectively.
What is the minimum distance each should walk so that each can cover the same distance in complete steps ?
o. 13. Prove that 6+2 is an irrational number.
V. Short Answer Type Questions-II [3 x 2= 6]

Q. 14. Ifp is a prime number, then prove that yp is an irrational.


0.15. Three bells tollat intervals of 9, 12, 15 minutes respectively. If they start tolling together, after what time will they
next toll together ?
[1x 5 = 5]
VI.Long Answer Type Questions
yn-1+n+1 is rational.
Q. 16. Showthat there is no positive integer n, for which

OSWAAL COGNITIVE
Finished Solving the Paper?
Time to evaluate yourself ! LEARNING TOOLS

OR
SCAN THE CODE

For elaborated
Solutions
UNIT II ALGEBRA

CHAPTER

POLYNOMIALS
Relationship between zeroes and coefficients
Zeroes of apolynomial.
BME Syllabus quadraticpolymomials.

study
In thischapter you will
polynomial, zeroes of polynomial, degree
ofpolynomial and relationshin
Graphical representation ofa
coefficients of quadraticor cubic polynomials.
between zeroes and

Revision Notes
form of Quadratic Polynomial: (i) Graph of a quadrat
Polynomial: An algebraic expression in the polynomial p(x) = ax + bx t cis a
parabol
atta,-t......t a,r+a,x + a, (where nis a and interset
a, agy ...... a, are real which opens upwards, if a > 0
whole number and a two distinct points.
variablex of X-axis at a maximum of
mumbers) is calleda polynomial in one
degree n. (ii) Graph of a quadratic Scan to know

Value of aPolynomial at a given Scan to know


polynomial p() = a + more about
this topic
polynomial in x bx + c is a parabola which
point: Ifpr) is a more about
number, then opens downwards, if a < 0
and ´a is any real thís topic
and intersects X-axis at a
the value obtained by putting
value maximum of two distinct
x=a in p(), is called the Relationship
of p(z) atx = a. points. between zeroes

Zero of a Polynomial: A real Zeroes of Cubic polynomial: Graph and coefficients


number kis said to be a zero of a polynomials of cubic polynomial p(r) = of quadratic
ax + bx t cx + d intersects polynomials
polynomial p(x), if p()= 0.
Geometrically, the zeroes of a polynomial p(r) are X-axis at a maximum of three
where the
precisely the X-coordinates of the points, distinct points.
and the
graph of y = plr) intersects the X-axis. Relationship between the Zeroes
(i) Alinear polynomial has one and only one zero. Coefficients of a Polynomial:
quadratic polynomial has at most two zeroes.
(ii) A () Zero of alinear polynomial
cubic polynomial has at most three zeroes.
(iii) A (-1)'Constant term
(iv) Ingeneral, a polynomial of degree n has at most Coefficient of x
n zeroes.
polynomial, then zero 0
Graphs of Different types of Polynomials: If ax + b is a given linear
-b
Linear Polynomial: The graph of a inear linear polynomial is
polynomial p() = ax + b is a straight line that
intersects X-axis at one point only. (ii) Ina quadratic polynomial,
POLYNOMIALS 26

intercepts
z p()
Highest
power
of
5,6)
Polynomial, 0 fx)=
r+rtc
GGeneral
Form
#
Y
Axis
of
symmetry 3,-9
Vertex
$t, a
f)=rtb
Parabola y=-3r-4
in
4)
x Types
Polynomial
Degree
(-2,64 (-1,0) 1 3

Quadratic
Linear Cubic

Polynomial
ofDegree
Representation
Quadratic
of Polynomial
Graphical

X-axis
Graph 0
=
Zero
cut
: of
3 not Number
Case
does
polynomic
:
Polynomial
Graphically
point
cuts
one =1 of
Graph Zero Zeroes
exactly
of of Level
: Number
2 at Quadratic
Polynomials
Coefficients
Relationship
Between
Case Third
X-axis
Map
Level
Mind
Second
and the
Trace
Zeroes Level

Quadratic First
cuts
points 2
=
Graph Zeroes

2 -
at time
: X-axis
of zeroes zeroes, the
Polynomial Product
zeroes
of
I Number zeroes
of zeroes
Quadratic +d y=-2
a+
B+
Case Prodnct
of
zeroes of ata
Quadratic
are bx+cB=-b
ast+ a+ are products
two
Polynomial Cubic
B of aß
= y + of
and Sum andCubic Sum taken
of bx
a + of
Then, B Then,
If , ax Sumzeroes
If
(8TANDARD), Class-X
Chapterwise &Topicwise, MATHEMATICs
26 Oswaal CBSE Question Bank
Sum of zeroes of aquadratic polynomial
(-1) Coefficient of x 3
Coefficient of x
Product of zeroes of aquadratic polynomial X X+

(-1 Constant term


Coefficient ofx
quadratic
:If a and B are the zeroes of a
polynomial ax + bx +t c, then
one zero' and we can
a + B= and aß = .:.f(x) will have only are
roots/zeroes of the given polynomial
polynomial
(ii) Ifa,Band yare the zeroes of acubic equal. = ax+ br
a + bx+ r+ d, then Case II: IfD < 0,graph ofthef()
X-axis.
touch nor intersect
a+B+y=(-1)! = aß + By +ya = (-1) neither
Y (1)
Y
C
and aßy ==(-1)°
Polynomial: For
Discriminant of a Quadratic 4ac is called its
where a 0,
f(x)= ax+ bx + c, discriminant X+
D determines the X (2)
discriminant D. The
polynomial.
nature of roots/zeroes of a quadratic ax+ bx + c
Case I: If D > 0, graph of f () =
points,
distinct
will intersect the X-axis at two
I-C0ordinates of points of intersection with X-axis is
known as 'zeroes off ().
zero.
Y
. f(x) will not have any real

Key Words
used to
Distinct: The term distinct number is
X equal to
X ! refer to anumber in a set that is not
i another number.
Graph: A diagram that represents the
variation of a variable in comparison with that
of one or more other variables.
.f() wil have twO zeroes and we can say that
roots/
and
Polynomial: An expression of more than two
are real
zeroes of the two given polynomials algebraic terms, especially the sum of several
unequal. terms that contain different powers of the
will
Case II: fD = 0, graph of f() = a + bx + c same variable(s).
touch the X-axis at one point only.

O- Key Formulae
Polynomial:
Relationship between the zeroes and the coefficients of a
S Type of General form
Maximum Relationship between zeroes and coefficie
Number of zeroes
No. polynomial b -Constant term
1 Linear ax + b, where a 0 k =-,ie,k= Coefficient of x

2. Quadratic ar + bx + c Where 2 Sum of zeroes,


-Coefficient of x =-b
a#0 (a + B) = Coefficient ofx?
Product of zeroes,
Constant term
(aB) Coefficient of x
OL/NOMIALAS 27
Cubic
where a 0 Sumof zeoKn,
Coelfidentof'
(u t )
Cocfident of r
|Product of nun ofzeroes taken two at atime,
Cocffcient of x
Coeflient of r'
Product of zeroes,
Constant term
(aby)
Coefficient of x

Fundamental Pacts Mnemonics


(1) Polynomial are also an essential tool in Concept: a.ß
C

describing and predicting traffic patterns so


appropriate traffic control measures,such as Mnemonics: Amitabh Bachchan went
traffic lights, can be implemented. Canada by aeroplane.
(2) Economists use polynomials to model Interpretation:
economic growth patterns, and medical Amitabhs A >Alpha (a)
researchers used them to describe the Bachchan's B > Beta (b)
behaviour of bacterialcolonies. Canada'sC »Constant (c)
By for Divide by and aeroplane's a
Variable.

Example1
If(x + a) is a factor of 2r + 2ax + 5x + 10, then find a. Also find its zeroes.
Solution: Let p(x) = 2x + 2ax+ 5x + 10
Step 1. If (x + a) is a factor of px), then -a is a zero of p(x).
p-a) =0
5(-) + 10 = 0
Step 2. Putting x=-a in p(x), we get 2(-a)+2a(-a) +
2a-- 2a-5a + 10 0
5a 10

Step 3. Putting the value of a = 2 in p(),


p(x) = 2x+ 4x + 5x + 10
= 2x(x + 2) + 5(r + 2)
= (x + 2)(2x + 5)
Step 4. For finding zero, we get, p(x) =0
(x + 2)(2x + 5) = 0
x+ 2= )>x=-2
Either, 5
2x +5 0>x=
2

5
Hence,- 2 and are zeroes of p(x).
2
28
Oswaal CBSE Question Bank Chapterwise &Topicwise, MATHEMATIC8 (8TANDARD), Class-X

OBJECTIVE TYPE QUESTIONS (1 mark eac or

Q.6.
C
5= -
Multiple Choice Questions
5 Q.7
Q.1The graph of a polynomial P(r) cuts the X-axis at 3
1
pomts and touches it at 2other points. The number
of zeroes of P(r) is or c= 5k, a = 1k
(A) 1 (B) 2 Polynomial whose sum of zeroes or prod.
(C) 3 (D) 5 zeroes are givern,
R(CBSE, Board Term-1, 2021] Required Polynomial == ar + bx + c An
k'-8kx + 5k
Ans. Option (D) is correct.
Explanation: According to the property of the -k(?-8x + 5)
polynomials, Q.4. Ifx -1is a factor 4,of the polynomial plr) =,
then
Number of zeroes = Number of points at which + 2b anda + b=
(B) a = 9, b = -5
graph intersects the X-axis. (A) a = 5, b = -1
(C) a= 7, b= -3 (D) a = 3, b = 1
It is mentioned in the question that, the graph ACBSE, Board Term-I, ;
intersects X-axis at 3 points and it touches it at Ans. Option (B) is correct.
2 further points. Explanation: Given,
This means that the graph intersects the X-axis at 5 plr) = +a+ 26
different points. a+b= 4
Therefore, number of zeroes=5.
x-lis a factor of the polynomial P(),
Q.2. In figure, the graph of a polynomial P(r)is shown. which means x = 1 is a zero of the polynomial
The number of zeroes of P(*) is p1)= 0
Y (1 + a(1)+ 26 = 0
1+a+ 25 = 0
or a+ 2b =-1
P(x) Subtracting (i) from (ü), we get
b=-5
ox Substituting the value of bin (i), we get a =9
a =9&b=-5
Q.5. If a, Bare the zeroes of the quadraticpolynor
p() =-(k+ 6)x + 2(2k-1), then the valueofi
1
a+ß=aß,
2 is
(B) 2 (A) -7 (B) 7
(D) 4 (C) -3 (D) 3
ORICBSE, Board Term-1, 2021] A[CBSE, Board Term-L, 2
Q.3. Aquadratic polynomial, the product and sum of Ans. Option (B) is correct.
whose zeroes are 5 and 8 respectively is Explanation: p(x) = *- (k+ 6)x + 2(2% 1) is:
(A) k[-&x + 5j (B) k[ + 8x + 5] given polynomial
(C) k<-5x + 8] (D) k[ + 5x + 8] Here, a= 1, b=-(k+ 6) &c = 2(2k- 1)
A[CBSE, Board Term-I, 2021] Sum of zeroes = a+ B
Ans. Option (A) is correct. -b
Explanation: For any quadratic polynomial,
ax +bx +c =k+ 6
-b Product of zeroes = aß
Sum of zeroes
a

-b
8=
2(2k-1) =2(2k-1)
8 -b 1
It is given that,
1 1
or b=8k, a= 1k a+ß= aß
2
C
Also, product of zeroes = k+6 = 2(2k-1)
These questions are for practice and their solutions are available at the end of the chapter
POLYNOMIALS 29

k+6=2k-1 Ans. Option (D) is correct.


-k=-7 Explanation: Given polynomial is 4y-4y + 1
k=7
)6 How many zero(es) does (x- 2)(x +3) have ? :. Sum of polynomial =
UCBSE, Q.B, 2021-22]
(B) One 1
(A) Zero and product of polynomial =
( Two (D) Three 4

Which of these is the polynomial whose zeroes are 11


Now, -+-+ pg = +pq
1 and R[CBSE, Q.B., 2021-22]
3 1
(A) 12x+ 5x3 (B) 12x-5x-3 We Have proved above, p+ q=1 and pq = 4
12r+ 13r +3 (D) 12- 13x-3
(C)
(A) is correct. 1 1
Ans. Option So, 1/4 4
Explanation: Sum of zeroes =
1_17
= 4+,A
4-9 -5 4

12 12
Q. 10. If onezero of the quadratic polynomial + 3x +k
is 2, then the value of k is
and product of zeroes = C+ U [CBSE, Delhi Set-I, 2020]
(A) 10 (B) -10
1 (D) - 2
(C) -7
Ans. Option (B) is correct.
Explanation: Let plx) =+3x+k
: Required polynomial = -(sum of zeroes)r 2is a zero of p(), then
+ product of zeroes
P2) = 0
(2) + 3(2) +k= 0
4 +6+ k=0
12x² +5x-3 10+ k= 0
k=-10
12 sum of whose
Q. 11. The quadratic polynomial, the6, is
zeroes is -5 and their product is
-12r+5z-3J AJCBSE Delhi Set-I, 2020]
(B) -5x +6
(A) +5x + 6
1
(C) -5r-6 (D) - + 5x + 6
Hence, 12r + 5x-3 is a polynomial and
is a
12 Ans.Option (A)is corect.
Explanation: Let a and B be the zeroes of the
constant. quadratic polynomial, then
the polynomial
Q.8. How many zerofes) does[CBSE, Q.B., 2021-22] a+B =-5
293:- 293x have ? A and aß =6
(A) 0 (B) 1 So, required polynomial is
(D)3 t-(a+ B)x + aß = -5)x + 6
() 2 =t+5x +6.
Ans. Option (C) is correct. -3r - m(m+ 3)
Explanation: Given polynomial is 293:-
293x Q14. The zeroes of the polynomial
are
’ 293x(x - 1) (A) m, m+3 (B) -m, m+3
For the property of zeroes, (C) m,-(m +3) (D) - m, - (m +3)
293x(x-1) = 0 U [CBSE Outside Delhi Set-I, 2020]
293x = 0’r=0
Either, Ans. Option (B) is correct.
X-1=0’r=1 Explanation: Given, -3xm (m + 3)
Hence, it has two zeroes. puttingx=-m, we get
+1. =(-m-3(- m)- m(m + 3)
and gare the zeroes of the polynonmial 4y' -4y = m +3m - m'-3n = 0,
What is the value of 4-+pg? puttingx=m + 3, weget
= (m + 3)- 3(m +3)- m(m + 3)
A+E [CBSE, Q.B., 2021-22] = (m + 3) [m + 3-3-m]
15 3 = (m + 3) [0] = 0.
(A) - 4 (B) 4
Hence, m and m + 3 are the zeroes
of given

5 17 polynomial.
() (D)
4 4
end of the chapter
These questions arefefor practice and their solutions are available at the
Chapterwise & Topicwise, MATHEMATICS (STANDARD), Class-X
30 Oswaal CBSE Question Bank
One root is zero (given) so, a = 0.
Commonly Made Error
Students often make mistakes in analyzing the .6 one of the zeroes of
the cubic
zeroes as they get confused with the different + ax' + bx + c is -1, then poly,
the product
other two zeroes is:
terms. (B) b-a-1
(A) b-a+1
(C) a-b+1 (D) a-b-1
Answering Tip ANCERT Exe
Ans.Option (A)is correct.
Understand the different cases for zeroes. Explanation: Letfr) =r+ ax+ bx +c
One of the zeroes offx) is -1 so
Q. 13. The degree of polynomial having zeroes-3 and 4 f-1) =0
A[CBSE Board, 2020]
only is (-1)° +a(-1)' +b(-1) +c=0
(A) 2 (B) 1 -1+a-b+c=0
(C) more than 3 (D) 3 a-b+c=1
c=1+b-a
Topper Answer, 2020 -d
Now, aßy = [:a=l;
Ans. (A)2A
-1ßy = 1
Detailed Solution:
By the definition of the polynomial, By =c
By = 1+b-a
Apolynomialof degree n has at most n zeroes.
:. Number of zeroes i.e., n = 2. Q.17. The zeroes of the quadratic polynomial +
Hence, thedegree of polynomial zeroes - 3 and 4 127 are:
only is 2. (A) both positive
Q. 14. The number of polynomials having zeroes as -2 (B) both negative
and 5 is: (C) one positive and one negative
(A) 1 (B) 2 (D) both equal U NCERT Exem
(C) 3 (D) more than 3 Ans. Option (B) is correct.
U [NCERT Exemplar] Explanation: Let given quadratic polynomial b
Ans. Option (D) is correct. P(r) = +99r +127
Explanation: We know that if we divide or multiply On comparing p(r) with ax + bx + c. we get
a polynomial by any constant (real number), then a= 1,b =99 andc= 127
the zeroes of polynomial remains same. We know that,
Here, a = -2and B= +5
.. a+B= -2+5= 3and aßB=-2x 5 = -10 X=
-btb-4ac [By quadratic formulk
2a
So, required polynomial is
-(a+ B)x + aß =-3x-10 -99 ±J(99) -4x1x127
If we multiply this polynomial by any real number, 2x1
let 5 and2, we get 5x-15x - 50 and 2x'-6x -20 -99 t V9801 - 508
which are different polynomials but having same 2
zeroes -2 and 5. So, we can obtain so many (infinite
polynomials) from two given zeroes. -99tV9293 -99 +96.4
15. Given that one of the zeroes of the cubicpolynomial 2 2
as + br + cx + d is zero, the product of the other -99 +96.4 -9996.4
two zeroes is: 2
C -2.6 -195.4
(A) (B) 2
=-1.3, -97.7
(C) 0 (D) Hence, both zeroes of the given quad
A [NCERT Exemplar] polynomial p(x) are negative.
Ans. Option (B) is correct. Q.18. The zeroes of the quadratic polynomial + kt
Explanation: Letf) = a + bx+ cx + d k*0,
If a, B, yare the zeroes of f), then (A) cannot both be positive
(B) cannot both be negative
aß +By+ = (C) are always unequal
(D) are always equal UNCERT Exempl
POLYNOMIALS 31
correct.
6. Option(A)is (A) has no linear term and the constant term is
Explanation: Let fx))x+ kr +k,k 0. negative.
O comparing the given polynomial with (B) has no inear term and the Constant term is
+bx+ c, we get a =1,b= k, c = k positive.
If aand Bbe tne zeroes of the polynomial (x). (C) can have a linear term but the constant term is
We knowthat,
negative.
= (D) can have a linear term but the constant term is
Sum of zeroes, a + B U (NCERT Exemplar]
positive.
Ans. Option (A) is correct.
..) Explanation: Let fr) t+ ax+ band a,Bare the
roots of it.
And product of zeroes, aß = C Then, B=-a(Given)
C
a +B= and aß =
aß= sk
..(i)
A A

b
Case I: If kis negative, ap [from equation (i)) is and a(-a) =
1
negative.Itmeans a and ßare of opposite sign.
-a =0 and -a= b
Case I: Ifk is positive, then aß [from equation a =0’b<0 or bis negative.
1 is positive but a + ßis negative. If, the product So, fx) =t+b shows that it has no linear term.
of two numbers is positive, then either both are
negative or both are positive. But the sum of these . 21. Which of the following is not the graph of a
numbers is negative, S0 numbers must be negative. quadratic polynomial?
Hence, in any case zeroes of the given quadratic (B)
polynomial cannot both be positive. (A)
.19.If the zeroes of the quadratic polynomial ar + bx +c,
a 0areequal, then:
(A) cand a have opposite signs
(B) cand bhave opposite signs
(C) (D)
(C) canda have the same sign
(D) cand b have the same sign
U[NCERT Exemplar]
Ans. Option (C) is correct. UINCERT Exemplar]
Explanation: For equal roots b - 4ac = 0 or
Ans. Option (D) is correct.
B is always positive so 4ac must be positive, ie., Explanation: Graph (D) intersect at three points
product of aand cmust be positive, i.e.,a and cmust on X-axis so the roots of polynomial of graph is
have same sign either positive or negative. three, so it is cubic polynomial. Other graphs are
of the
20. If one of the zeroes of a quadratic polynomial it of quadratic polynomial. Graph (A) have no real
other, then zeroes and Graph (B) has coincident zeroes.
fom + ax+ bis the negative of the

SUBJECTIVE TYPE QUESTIONS


Y

Very Short Answer Type


Questions (1 mark each)
Q11fhe sum ofthe zeroes of the quadraticpolynomial
3-kx + 6is 3, then find the value of k.
OC+U [CBSE SQP, 2020-21]
Find the number of zeroes of p(x).
2 The graph of y = p), where p(r) is a polynomial OAO R[CBSE SQP 2020)
in variable x, is as follows:

end of the chapter


These questions are for practice and their solutions are available at the
32 Oswaal CBSE Question Bank Chapterwise 'lople wise, MATIIEMATICS (9TAMDARD), lns 7
0.3. Froma quadraticpolynomial, the sumand product
of whose zeroes are (- 3) and 2respectively.
A|CBSE Delhi Board, 20201

Topper Answer, a020


Answer fhe following, questit:
Sol. Sum ef XKSLS E3 ()How nany of the abtrve tet, ate nt ptolynn
ato
(i) How many of the ahrye er,
polynomials ?
ThL quinad folynomiad:
o3,Tind the value of , for which one tut
Detailed Solution: quadratic polynomial pr14 h-0is 4.
Given, sum of zeroes = 3 the other.
and Product of zeroes =2
Topper Anewer, 2017
Here, Required Polynomial
P(x) = k[x- (sum of zeroes)x
+ (product of zeroes)] tquatior.

= k[-(-3)x + 2] Her, a
= k[ + 3r + 2]
Short Answer Type
Questions-I (2 marks each)
Q.1. Find a quadratic polynomial where zeroes are
5-3/2 and 5+32 AOA[CBSE SQ, 2020-21]

Sol. Sum of zeroes = 5-3W2 +5+3N2 = 10 %

Product of zeroes = (5-3V2)(5 +3/2)


from ).
= 25-18 =7
1

Polynomial is given by
-(sum of zeroes)x + (product of zeroes) =0
P) = -10x +7
[CBSE Marking Scheme, 2020-21]

Commonly Made Error


Q4If2 zeroes ofthepolynomial + 4x + 2a are a
Students often commit errors in finding then find the value of a.
quadratic polynomial. Some students find sum
of zeroes and product of zeroes but not find a C+0 [Board Term, 2
complete polynomial. Q.5. Find the quadratic polynomial whose sum
product of the zeroes are 21 5
and respective
16
O Answering Tip AOA(Board Term-1, 2
Sol. Given,
Students should read the question properly Sum of zeroes = 21
and solved step by step.
.A teacher asked 10 of his students to write a and Product of zeroes = 5
16
polynomial in one variable on a paper and then
to handover the paper. The following were the So, quadratic polynomial
answers given by the students: =-(Sum of zeroes)x
+ Product of zerce
These questions are for practice and their solutions are available at the
end of the chapter
POLYNOMIALS 33

1
16 42P+4
3 3 =0
16
(16-42x + 5) 1 On solving we get
p=-8
Q.6.If a and B are the zeroes of a polynomial 3r- 8x + 4 = 0
243x+ 3, then find the value of a+ B- aß. 3r-6x- 2x + 4 = 0
G+A|Board Term-1, 2015] 3x(r - 2)-2(x -2) =0
2
Sol. Let, t-4V3x + 3 =0 X= or x = 2

If aand Bare the zeroes of -43x+3 Hence, X=2


b Sothe other root is 2.
Then, a+ß = [CBSE Marking Scheme, 2020-21]
a+B (-4V3) Q.2. The roots a and B of the quadratic equation
x-5x + 3(k -1) = 0are such that a - B= 1. Find
1 the value of k. AOUCBSE SQ, 2020-21]
a+ß= 4 V3
Sol. We have a+B=5
C
and aß = a-ß=1
Solving (i) and (ü), we get
a =3 and B= 2 1
3
aß = also aß =6
1
aß =3 3(k-1) = 6
k-1 =2
a+ß-aß = 4/3 -3. 1 k=3 1

a and B are the zeroes of the polynomial [CBSE Marking Scheme, 2020-21]
fz) = -6x + k, find the value of k, such that Q.3,/Pind a quadratic polynomial whose zeroes are
a'+B°=40. C+A BoardTerm-1, 2015] reciprocals of the zeroes of the polynomial
f(x) = ax+ bx + G, a z0, c+0.
ACBSE DelhiSet-I, 2020]
SoL.
a+B=-.b .4.f4isa zero of thecubic polynomialx-3r-10r +24,
ind its other two zeroes.
A+E[CBSE Delhi Set-I, 2020]
9)-6 1
Sol. x-3x-10x + 24
Let a, ß and ybe the zeroes of given polynomial
and =k
1
1 a+ß+y=3 ...)
aß + By+ ya = -10
Given, a'+B = (a + B-2xß =40 aßy = -24 ...i) 1
(6)-2k = 40 Given: a = 4
36- 2k = 40 from eqn. (i) B+y =-1
from eqn (ii) By = -6
-2k = 4 (B- = (B +Y-4By
k=-2 1
=(-1'-4-6)
[CBSE Marking Scheme, 2015] = 25 1
B-y =t5
B-y =5 ..iv) 1
Short Answer Type B+y=-1
Questions-II (4 marks each) 2ß = 4 B=2
and y= -3
Q.1.Ifoneroot of the quadratic equation 3* +px +4=0 Hence zeroes are -3, 2 and 4. 1
is 2 then find the value ofpand the other root of [CBSE Marking Scheme, 2020]
3' 0.5. Find the value of k such that the polynomial
the equation. AOC+A[CBSE SQr, 2020-21] r-(k + 6)x + 2(2k - 1) has sum of its zeroes equal
to half of their product. RCBSE Delhi Set-1, 2019)
Sol. 3a + px +4 = 0
Sum of zeroes = k+6 1
2 Sol.
is a root so it must satisfy the given 1
Product of zeroes = 2(2k - 1)
1
equation Hence k+6=x2 (k-1)
1 k=7 1
[CBSE Marking Scheme, 2019)

qMestions are for practice and their solutions are available at the end of the chapter
34 Oswaal CBSE Question Bank Chapterwise &Topicwise, MATHEMATICS (STANDARD), Class-X
Detailed Solution: 2
Let a and Bbe the roots of given quadratic equation y=;andy=
3
t-(k+ 6)x + 2(2k-1) =0 On comparing eq (i) witha+ bx t c=0,
Now, sum of roots, a +ß = (Hk+6)} - k+6 a= 21, b=-1l and c=-2
1

product of roots,
1
Now, sum of zeroes =

2
9)
According to given condition, 3 7

Sum of roots (zeroes) = 1 X product of roots(zeroes) 14-3


2
21
11
k+6=22k-1)) 2 21
k+6=2k-1
k=7 Hence veri
Hence, the value of kis 7.
C6. ind the zeroes of the quadratic polynomial
11 2
and product of roots
)-2
7y- and verify the relationship between 21
the zeroes and the coefficients.
A [CBSE Outside Delhi-IlI, 2019]
Hence verit
11
Sol.
P) =7-y-2y-1ly-2) Q.7. Find the zeroes of the following polynomial:
5/5r+30x +8/5 UCBSE SQP:
-I7y +1)3y-2)] 1
Sol. 5/5x +30x +8/5
2 -1
.:Zeroes are 1 = 5/5x +20x +10x +8V5
3' 7
2 1 11 = 5x(V5x +4) +2/5(V5x +4)
Sum of zeroes = --
3 7 21 = (/5x+4)(5x +2V5)
-b 11
21
|sum of zeroes = 1 Thus, zeroes are
4 4N5 and -2V5
5 5 5
Product of zeroes =
6I)- [CBSE Marking Scheme, 20

21
Commonly Made Error
1 Candidates commit error in
equation 5/5r+30x+8/5. simplifying
th
[CBSE Marking Scheme, 2019]
Detailed Solution:

Given, polynomial is?yf-y-%2 ..) Answering Tip


Qy-1y-9) Adequate Practice is necessary for factorization
problems.
(21y-14y +3y-2) Q.8. Quadratic polynomial 2x -3x + 1has
zeroesi
a and ß. Now form a quadratic
polynomial wha
3 I7y3y -2) +13y -2)) zeroes are 3a and 3B. A Board Term-2, 20
Sol. If a and Bare the zeroes of 2-3x + 1,
then
a By-2)07y +1)
a +ß =
2 or y =
3
Hence, zeros of given polynomial are a+ß=
2
POLYNOMIALS 35

> x(r+ 4) +3(x + 4) = 0


and (r + 4)(x + 3) =0
X=-4 or-3 ..)1
Since, p'(r) =+ 7 + 7 t px tqthen
aß =
2
1
If p(r) is exactly divisible byt 7x + 12,
-4
X=-4 andx=-3are its zeroes.So, putting x=
New quadratic polynomial whose zeroes are 3a and x=-3.
and3ßis : p-4) =(-4) +7-4) + 7(-4 +p-4) + q
2-Sum of the roots) x + Product of the roots 1 but p(-4) =0
=-(30a + 38)x + 3a. x 3ß = 256- 448 + 112- Ap tq
=t-3a+ B)x +9aß 0 =-4p + q-80
’ 4p-g =-80 ...i) 1
and p(-3) =(-3) + 7(-3) + 7(-3) + p(-3) + q
but p(-3) = 0
0 = 81- 189 + 63-3p + q
1
0=-3p + q-45
’ 3p -q =-45 ..(i)
1 Subtracting equation (iü) from equation (i),
p-q=-80
Hence, required quadratic polynomial is 3p-q =-45
1
1 +
p=-35
1
0.9. Show that and
-3
are the zeroes of the On putting the value of p in eq. (i),
2 2 4(-35) -q=-80
polynomial 4x + 4x -3and verify the relationship -140-q=-80
between zeroes and coefficients of the -9= 140-80
polynomial. U Board Term-1, 2015] -q = 60
Sol, Let fr) = 4+ 4x-3 q=-60
p=-35 and q= - 60 1
Hence,
Thus,
1)-4)-4)- =1+2-3
.2. If a and B are the zeroes of the polynomial
plx) = 2x + 5x + k satisfying the relation,
21
a +B² +ß = then find the value of k.
=0 4
C+A
and
(-43-4 =9-6-3
Sol.Given, plx) =2:+ 5x + k
coefficient of x
Then, sum of zeroes =
=0 coefficient of x
1 3
and-are
2
zeroes of polynomial 4x + 4x- 3.1 a+ß =
-5
1
2
1 3 -4 Constant term
Sum of zeroes
2 2
=-1» 4 and product of zeroes =
coefficient of x
Coefficient of x 1
aß 1
Coefficient of x 2

According to equation,
Product of zeroes
-9)
Constant term Hence verified. 1
a+p+ caß =
21
4

21
Coefficient of x or(a + B)-2aß + aß = 1

Long Answer Type


Questions (5 markseach) ( 21

Q:1.Eolynomial x + 7+ 7 + px + q is exactly k 25 21 4
=4 1
divisible by + 7x + 12, then find the value of p 4 4
and q. A Board Term-1, 2015]
Hence, k= 2. 1
Sol. Factorsof x+7x + 12:
+7x + 12 = 0
Q.3. If a and Bare zeroes of the quadratic polynomial

t-6r +; find the value of a' if 3a + 2ß = 20.
+ 4x + 3x + 12 =0
These questions are for practice and their solutions are available at the end of the chapter
MATIHEMATIC (8TANDARD), Clnss X
Bank Chaperwise & T'opicwise,
36 OswaalCBSE Queution
COMPETENCY BASED QUESTIONS (4 markS eGc
polyn,
formn of quadratic
0.1. In the standard c are
+ bx,c, a, b and
numbers,
(A) AIlare real
O Case based MCOs rational numbers, and.
questions (B) Allare zero real number and bh
non
Read the following and answer any four is a
(C) Any real numbers,
I.
from Q.1. toQ.5. examples of
are few naBural quadratic (D) All are integers.
correct.
picture
The below
which is represented by a Ans.Option (C) is the quadratic polynomial are e
parabolic shapeparabolic the shape
arch is an arch inrepresents
of
0.2. If theroots discriminant D 4uc, then
polynomial. A their curve where the (B) D <0
of a parabola. In structures, found in (A) D> 0 (D) D )
efficient method of load, and so can be (C) D0
an ina variety of forms.
bridges and in architecture correct.
JCBSE QB, 2021| Ans.Option (D) isIftheroots of the quadraticpolyng
Explanation:
discriminant is cqual to zero
arecqual, then
D= -4ac = )
quadr
and
1
are the zeroes of the
3.f a
is
polynomial 2r-x + 8k, thenk
1
(A) 4 (B) 4

(D) 2
(C) 4
Ans. Option (B) is correct. 8k
Explanation: Given equation, 2x-x +
1
Sum of zeroes = u+

Product of zeroes = 0. =1

C 8k
Product of zeroes = =

2
8k
= 1
So, 2
2
k =

1
k=
4
Q.4. Thegraph of +1=0
(A) Intersects X-axis at two distinct points.
(B) Touches X-axis at a point.
(C) Neither touches nor intersects X-axis.
(D) Either touches or intersects X-axis.
Ans. Option (C) is correct.
Explanation:
4.
3
2

-4 -3 -2 -1 12 3 4
+2
+3
POLYNOMIALS 37
0.5. If the sum of the roots is p and product of the
Q.1. The shape of the poses shown is
rootsis , then the quadratic polynomial is (A) Spiral (B) Ellipse
(C) Linear (D) Parabola
Ans. Option (D) is correct.
(A) (B Thegraph of parabola opens downwards, if
(A) a >0 (B) a = 0
(C) a< 0 (D) a>0
(C) (D)
Ans. Option (C) is correct.
Q.3.In the graph, how many zeroes are there for the
Ans.Option (C) is correct. polynomial?
Erplanation: Required polynomial,
P(x) = k[-(sum of zeroes)x
+ (product of zeroes)]

In Read the following text and answer any four


questions from Q1 to 05.
An asana is a body posture, originally and still
a general term for a sitting meditation pose,
and later extended in hatha yoga and mnodern (A) 0 (B) 1
yoga as exercise, to any type of pose or position, (C) 2
adding reclining, standing, inverted, twisting, and (D) 3
balancing poses. In the figure, one can observe that Ans. Option (C) is corect.
poses can be related to representation of quadratic Explanation: Since, graph, cuts X-axis at two points.
polynomial. [CBSE QB, 2021] Hence, zeroes of polynomial are 2.
Q.4. The two zeroes in the above shown graph are
(A) 2,4 (B) -2, 4
(C) -8,4 (D) 2, -8
Ans. Option (B) is correct.
Q.5. The zeroes of the quadratic polynomial
4W3x +5x-2/3 are
2 V3 2 v3
(A) 5' 4 (B) 4
2 W3 2 V3
(C) B 4 (D) 4
Ans. Option (B) is correct.
Explanation: 4/3x+5x-2/3 (given)
=4V3* +(8-3)x 2V3
= 4V3r² +8x-3x-23
=4x(V3x +2)-N3(V3r+ 2)
= (Vr+2)(4r-V3)
-2 V3
Hence, zeroes of polynomial =
J3'4
III, Read the following text and answer the following
questions on the basis of the same:
Basketball and soccer are played with a spherical
ball. Even though an athlete dribbles the ball in
both sports, a basketball player uses his hands
and a soccer player uses his feet. Usually, soccer is
played outdoors on a large field and basketball is
(STANDARD), Class-X
Question Bank Chapterwise & Topicwise,MATHEMATICS
38 Oswaal CBSE shown grank
of wood. The 0.4. The three zeroes in the above
plaved indoor on a wurt made out (B) -2, 3, 1
and basketball (A) 2,3,-1
projectile (path trad) of soer ball (D) -2, -3, 1 0. 1.
nepresenting quadratic (C) 3,-1,2
are in the form of paratola
[CBSE QB, 20211 Ans. Option (C) is correct.
polynomial. Sol.
o.3What will be the expression of the polynomia
A) +2-5r-6 (B) +2r-5r-6
(C) + 2r+ 5x-6 (D) x'+2r +5x 44
Ans. Option (A)is correct.
Explanation: Since, the three zeroes =-3, -1, 0. 2
Hence, the expression is (r + 3)(r + 1)(r -2)
=[a+x+ 3r + 3](x-2) So
=r+ 4r+ 3r-2r-8r -6
= +2r-5:-6

Case Based Subjective


Questions
I. Read the following text and answer any s
questions from Q1 to Q5.
Applications of Parabolas: Highway Overpas Q.
Underpasses
Ahighway underpass is parabolic in shape.

Q.1. The shape of the path traced shown is


(A) Spiral (B) Ellipse
() Linear (D) Parabola
Ans. Option (D) is correct.
Q.2The graph of parabola opens upwards, if
YA) a=0 (B) a < 0
(C) a>0 (D) a > 0 Shope of cross slope:
Ans. Option (C) is correct. X

Q.3. Observe the following graph and answer Y


linn

B.
a. Paraboic camber
-2 y=2r/nw
4-2 1 2 3

Parabola
t-2
Aparabola is the graph that results from
plr) = a+ br + c.
Parabolas are symmetric about a vertical line know:
as the Axis of Symmetry.
The Axis of Symmetry runs through the maximu
Inthe above graph, how many zeroes are there for or minimum point of the parabola which is called th
vertex.
the polynomial?
(A) 0 axis
(B) 1 axis

() 2 (D) 3 vertex
Ans. Option (D) is correct.
Explanation: The number of zeroes of polynomial is
the number of times the curve intersects the X-axis,
ie. attains the value 0. vertex

Here, the polynomial meets the X-axis at 3 points.


So, number of zeroes = 3.
AO C+ AE ÊCBSE SQP 2020-21)
POLYNOMIALS 39
0.1.IIfthe highway overpass is represented by -2x-8.
Findits zeroes.
Q. 1. Ifa linear polynomial is 2x + 3, then find the zero of
2x + 3.
-2x-8 = 0 Sol. Given, polynomial =2r +3
Sol.
P-4r+2r-8 = 0 Let plx) = 2x +3
or. x(x-4) + 2(*-4) = 0 For a zeroof p(x),
(r-4) (x + 2) = 0 2r +3 =0
X =4, x=-2 2x =-3
Q.2. Find the product of zeroes of given polynomial in 3
X=
question1. 2

Sol. Given polynomial is x-2x-8 = 0 Q.2. The graph of y= plr) is given in figure below for
Comparing with ax +bx +c=0, we get
a=1,b=-2andc=-8 Some polynomial p(). Find the number of zero/
zeroes of pr).
constant term
product of zeroes =
coefficient of x?
-8
1

0.3. Write the name of graph, which represent above X+


case.

Sol. Here, the given graph of a quadratic polynomial is


aparabola.
(o4 Whichof the following representation of Highway
Underpass whose one zero is 6 and sum of the Sol. Since the graph dpes not intersect the X-axis,
zeroes is 0 ? therefore it has no zero.
t-36,x-6 Q3Ifaand Bare the zeroes ofthe quadratic polynomial
t-36 = 0 ’=36
Sol. -5x + k such that a - B= 1, then find the value
x = tv36 ’X= 6,-6. of k.
Hence, x-36 has one zero6 and sum of zeroes is 0.
Sol. p(r) =t-5x +k
(5) =5
0.5. Find the number of zeroes that polynomial f(r)= Then a-B =
1
(x-2) + 4 can have.
Sol. We have, and aß = =k
fx) =(-2) + 4 1
=t+4-4x +4 Also givern, a-B = 1
=t-4x +8.
ie, It has D=-4ac = 16-32 < 0
Hence no real value of X is possible, i.e.,no zero.
four =1
II. Read the following text and answer any
questions from Q1 to Q5. 254k =1 (Squaring both sides)
For alinear polynomial kx + c, k0, thethe graph of -4k = 1- 25 = - 24
intersects X-axis
y=kr+ cis a straightline which k=6.

at exactly one point, namely, 70Therefore, Q.4. Ifaand Bare the zeroes ofthe quadraticpolynomial
plz) = 4r+ 5x +1,then find the product of zeroes.
the linear polynomial kx + c, k 0, has exactly one Sol. We have, p(x) = 41 + 5x +1
the point where
Zero, namely, the X-coordinate of aß =
X-axis.
the graph of y= kx + cintersects the 4
C+ AE quadratic
Q.5.If the product of the zeroes of the
polynomial p(x) = ax - 6r - 6 is 4, then find the
value of a.
Sol. We have,
plx) = ar-6x-6
X+ Let a and Bbe the zeroes of the given polynomial,
then
aß =
Toncwise MATHEMATICS (STANDARD) Clas
40 7eWA) CiSEt iterik 'hhter
O.2 lf a and are zerves of rit) - 1 ).
1
ir, the value of a + 8

Sol. iNe have provd zees in question 1.


.a= and ß= 1
4 2
1
III. Read the following text and answer any four
questions from Q1to 05.
Aparabolic shape object used for receiving signals - 4-1-=5
fron a batellite, especially television signais. A
satellite dish works in the 5ane way as the reflector
for a torch orcar headlights.A signal is produced or
reflected frorn a focal point. Aparabola is the graph
that results frorn plr) = ax+ bx + C. Q.3. If one zero of the quadraticpolynomial
Ifthe values of a, band care 1, -3 and 2respectively, is 1, then what is the value of A?
then give answer of following questions: Sol. Let,

P) = ()+ i() + 2
1+k+2=0
[:: lis a zero of given polyno
k=-3.
0.4. Find the quadratic polynomial whose zeroes)
and- 3.
Sol.: 5and-3 are zeroes of p(), then (t-5) and (.
are the factors of p(r).
Q.1. Find the zeroes of polynomial p(x). P() = (r-5) (r + 3)
Sol. Given, =t-5r + 3r- 15
p(x) =-3z +2 =t-2r- 15.
=t-2x-x+2 Q.5.Find the product of zeroes of the polynon
= x(x-2) - 1(x-2) -2r.
= (x-2)(x-1) Sol. Given polynomial = -2r
For zero, plr) = 0 Constant term
.. Product of zeroes =
(x-2)(x-1) = 0 coefficient of r?
X-2= 0’x=2
Either,
Or, x-1= 0 ’x=1 -0.
Hence, the zeroes of p(x) are 1 and 2.

Solutions for Practice Questions


Explanation: For zeroes
Multiple Choice Questions (r-2)(x +t 3) = 0
Ans. 2: Option (C) is correct.
Explanation: According to the property of the Either, X-2 =0’r=2
x+3 = 0>r=-3
polynomials, Or

Number of zeroes = Number of points at which So, we get two values of x i.e.,
graph intersects the x-axis. X =2 or-3.
Y Hence, it is quadratic polynomial intersects x-a
at two points.
P()
Very Short Answer Type Questions
-+X Sol. 1: Let the roots of the given quadratic equation be
a and ß
So we have,
a+B
3

From the figure it is clear that the graph intersects 3=


3
X-axis at three different pojnts. Therefore, the
polynomial has 3 zeroes. k =9
[CBSE Marking Scheme, 2020-21)
(Ans. 6: Option (C) is correct.
POLYNOMIALS 41

Sol. 2: Since the graph touches the X-axis 5 times. 1 1 1


Hence, the number of zeroes of p(r) is 5. 1
1 p= a. B aß C
[CBSE SQP Marking Scheme, 2020]
Short Answer Type Questions-| Quadratic polynomial

5
Sol. 2:(i) +3x+7, 2x +3 and x+ are not

polynomials. 1
P(r) =
C
(cx +bxta) 1

(ü) 3+7x + 2is only one quadratic polynomial. 1 [CBSE Marking Scheme, 2020]
[CBSE Marking Scheme, 2020]
Long Answer Type Questions
C Sol. 3: We have quadratic polynomial = -6x + a
Sol. 4: Product of (zeroes) roots = -(-6) =6
a + B=
1
2a 2
1
1 1
and aß =
1
Or, 2a = 2
a=1 1 It is given that; 3a + 2ß = 20 ...i)
and a + B =6 ...ü)
[CBSE Marking Scheme, 2016]
Questions-II Multiplying by 2 in eq. (i), we get
ero Short Answer Type 2a + 2ß = 12
Sol. 3:Let a and Bbe zero of the given polynomial Subtracting eq. (ii) from eq. (), we get
nd ax + bx + c a =8
b
a + B= and aß = 1 Substituting a = 8in eq. (ii), we get
8+ B= 6
B = 6-8=-2 1
1,1 B+a -b
1 aß = a
oß C C
Then, a = 8(- 2) =-16. 1

REFLECTIONS

polynomial?
relationship between the zeroes and the co-efficients of a
Did you understand the polynomial /3x-10x +
3
sum and product of zero in given 5
Will yoube able to evaluate the
CHAPTER

E Syllabus
5 ARITHMETIC
PROGRESSIONS
Motivationfor studying Arithmetic Progression. Derivation of the n"
term and sum of the first n terms of A.P and their application in solving
daily life problems.

in this chapter you will study List of Topics


Topic-1: To Find nth
IÜnderstand the condition when the pattern will be in A.P. Term of the Arithmetic
Know various terms associated with A.P. ie., flrst term, last term and Progression Page No. 95
common difference.
Topic-2 : Sum of n
Understand fnite and infinite A.P. terms of an Arithmetic
Understand the formula to find n term and sum of n terms.of A.P. Progression Page No. 104

To Find nh Term of the Arithmetic Progression


Topic-1 Concepts Covered Defne fArst term, common dference.
Define finite and infinite A.P,
"Formula for fnding nth term of an A.P.

Revision Notes

An arithmnetic progression is a sequence of numbers A list of numbers a1, a,, da ..... is an AP, if the
in whích each term is obtained by adding or differences a, - A, ag - ayy ay - Q3 give the same
subtracting a fixed number d to the preceding term, value i.e., aya1- a, is same for all Scan to know

except the first term. different values of k. more about


this topic
he difference between the two successive terms of The general form of an A.P is a, a
an A.P is called the common difference. + d, a t 2d, a + 3d, .....
Each number in the sequence of arithmetic If the A.Pa, a+ d, a+ 2d,......,
Progression is called a term of an A.P. l is reversed to ll- d, l- 2d,
the arithmetic progression having finite number of ,a, the common difference nh termm of AP
terms is called a finite arithmnetic changes to negative of original
The arithmetic progression having progression.
infinite number of sequence common difference.
terms is called an infinite arithmetic
progression.
MATHEMATICS (STANDARD), Clas5-X
Chapterwise & Topicwisc,
96 Oswaal CBSE Question Bank dierene
difference
’common
d

3by
numbers term
99 term am’nn
term common
divisible
99
= 29 -1)dlast
37 a, -1)3 first
by ... d=3, 87= n=30 l-(n
2-digit 18, (n 3 ’
adding/subtracting divisible
numbers
15, 12+ =
n-1
i.e., Ihere,
a-’
d
the 12, = here,
arithmetic negative. many 12,a= = a,
number.
provides are are 99
How 2-digithere,
or
in whichpresent
the
from
end eRrOm beginning
positive
number
...a+{n-1)d by
terms
progression t
the

"Fixed be Om

fro "Can
Difference
Common th
termn
3d, and
a+
a+2d,
to duexq

A.!
in
+d,
a
4,

2
b
arithmetic
mean

General

Arithnetic
mean
form

prithmetic
progress
S$0on:

are
, Then
b
b, here,
a,
If
Definition

adding
whichpreceding
common difference
in by ternm.
:
-1)d)
numbers given
obtained
the
to
first and (n termn
terms
d
common

the
number are +
of is except
term =(2a
difference
first total
Sequence
ternm first
fixed 2 a ’ -’
each term When
S, 1
here,
a Sum
(S)
Third
Level

(a+)
given:
integers Map
*
* l=1+n) are MindLevel
Second
Ltterm,
S,1+2+3+
= teIMS
positive S, a
teTms ’ teIm
first
term I
last
term

last
or total
nth
the
last=a+an) Trace
n 1, ’ ’ Level
First
first a= &
fst n
of here, here,
Sum arduexi S,
When
ARITHMETIC PROGRESSIONS 97

O Key Formulae (3j If aconstant is added or subtracted:


from each term of an A.P, the resulting
sequence is also an A.!
The general (rh) term of an A.P is expressed
(4) If each term of an A.P.is multiplíed or
divided by a non-zero constant, the
as:

a+ (n-l)d. ...from the starting. resulting sequence is also an A..


where, ais the first term and dis the common (5) If the n term is in linear form i.e., an
difterence. + b=a the sequence is in A.P.
The general (n) term of an A.P ,|-d,1 (6) If the terms are selected at a regular
2d,....., a 0S givern by: interval, the given sequence is in A.P?
A=l+ (n-11- a) = l-(n-1)d....t.on (7) Ifthree consecutive numbers a, b and
numbers is
the end. care in A.P, the sum two
where, is the last term, dis the common twice the middle number i.e., 2h = a +
difference and n is the number of terms. C
ordered
(8) A sequence is defined as an
O Key Words list of numbers.
The first, second and third terms of a
Term: A term is a word or expression sequence are denoted by t, t, and t3
used with a particular meaning. respectively.
Seguence: A sequence is an enumer (9) If the terms of sequence are connected
ated collection of objects in which rep with plus (+) or minus (-), the pattern
etitions are allowed and order matters. is called a series.
Example: 2 + 4+ 6 + 8+ ... is a
series.
Fundamental of numbers 0, 1,1,2,3, 5,
(10) The sequence discovered
8, 13,.. was by a famous
Facts Italian Mathematician Leonasalo
1) An A.P or arithmetic sequence is a Fibonacci, when he was dealing with
sequence of numbers such that the the problem of rabbit population.
difference between the consecutive (11) If the terms of asequence or a series
terms is constant. are written under specific conditions,
(2) The7hCentury Indian Mathematician then the sequence or series is called a
and astronomer Brahmagupta is the progression.
father of Arithmetic.

Mnemonics
i Concept: n" Ternm of Arithmetic Progression n =a+ (n-1)d.
Mnemonics: Nokia Offers Additional Programmers in English To Attract Positive
New One Buyer Daily
Interpretation:
Nokia's 'Nis nth term.
Offer's 'O' is of
Additional's 'A' is Arithmetic Programmer's 'P is Progression
In's I is is.
English's E' is Equal
To's T' is To
Attract's 'A' is a
Positive's 'P is +
New's 'N is n
One buyer is -1
Daily's D' is d

Example
Which term of the A.P6, 13, 20, 27,.... is 98 more than its 24 term ?
Step 1. The given A.P is 6, 13, 20,27,.
Here, first term, a=6
common difference, d= 13-6= 20-13 =7
4, =4,4 + 98
Step 2. According to question: a+ (n+ 1)d =a+ (24-1) d+ 98
7(n--1) = 23 X7+ 98
n-1=23 +14
1=38
Hence, 38th term is the required term.
MATHEMATICS (STANDARD), Class-X
98 Oswaal CBSE Question Bank Chapterwise &Topicwise,

OBJECTIVETYPE QUESTIONS (1 mark eae


0.5.30h term of the A.P.,: 10, 7, 4,-......., is:
Multiple Choice Ouestions (A) 97 (B) 77
(C) -77 (D) -87
0.1. The value of xfor which 2x, (x + 10) and (3x + 2)
are the three consecutive terms of an A.P, is RINCERT E
(A) 6 (B) -6 Q.6.11th term of theA.P.,; -3,-2,
2
is:
(C) 18 (D) -18
(A) 28 (B) 22
AORCBSE DelhiSet-I, 2020]
0.2. The first term of A.P. is p and the common (C) -38 (D) -482
difference is q, then its 10th term is
(A) q+ 9p
(C) p+9q
(B) p-9q
(D) 2p + 9q Ans.Option (B) is correct.
RINCERT Exe
RICBSE Delhi Set-I, 2020] Explanation: In the given A.P,
Ans. Option (C) is correct.
Explanation: a=pand d=g(given)
a=-3 and d=
*3
10th term =a+ (10-1)d
-p+ 9q Thus, the 11 term, k=3+(11-1) -2
[CBSE Marking Scheme, 2020]
0.7. In an A.P., if d=4,n =7, a, = 4, then ais:
1-2p
Q.3. The commondifference of the A.P. 1-p (A) 6 (B) 7
(C) 20 (D) 28

Ans. Option (D) is correct.


RINCERT Exer
1
(A) 1 (B) Explanation: Given,
d=4,n =7, a, = 4
(C) -1 (D)1 nth term,
4, =a+(1-1)d ’ 4=a+ (7-1)(-4)’a= 8
RICBSE OD Set-I, 2020] Q. 8. In an A.P., ifa 3.5, d = 0, n = 101, then 4, ily
Ans. Option (C) is correct. (A) 0 (B), 3.5
Explanation: Given, (C) 103.5 (D) 104.5
A.P = 1 1-p 1-22
p p p RINCERT Exen:
Q.9. The list of numbers - 10, -6, -2, 2,.....s:
-P 1 (A) an A.P., withd=-16
.. Common difference =
p p (B) an A.P., with d=4
1-p-1 (C) an A.P., with d=-4
(D) not an A.P., U[NCERT Exe
Ans. Option (B) is correct.
=-1. Explanation: In the given numbers
Q4. Thenh term of the A.P. a, 3a, 5a, ... is - 10, -6, -2, 2,..
(A) na (B) (2n-1)a
(C) (2rn + 1)a (D) 2na (-6)-(-10) =4
RICBSE OD Set-I, 2020] (-2) -(-6)=4
Ans. Option (B) is correct. 2-(-2)=4
Explanation: Given Since, (-6)-(-10) =(-2)-(-6) =2-(-2) =4,
AP =a,3a, 5a,.
thus, the given numbers are in A.P. with d =
Here first term, a =a and d= 3aa= 2a 5
5
nh term =a+(n-1)d .10. The 11h term of the A.P., -5, - 0,
a+(n-1))21
at 2na 20 (A) -20 (B) 20
(D) 30RINCERT Exe
=2n0 -a=(2n 1)a. (C) -30
CBSE Marking Scheme, 2020} Ans. Option (B) is correct.
These auestions are for practice and their solutions are available at the end of the chanter
ARITHMETIC PROGRESSIONS 99

Explanation: In the given A.P. Q. 13. Which of the followingis not an A.P. ?
5 (A) -1.2, 0.8, 2.8,...
a=-5,d=7 -5)= ,n=11 (B) 3,3+ J2,3+ 2V2,3+ 3/2,..
4 79 12
() 3'3'3' 3
4, =Q+(1-1)d=
-1 -2 -3
(D) 5 5 5 ' UCBSE Delhi & OD, 2020)
of an A.P., whose first term is
o.41) The first four terms
2 andthe common difference is -2, are:
(A) -2, 0, 2, 4 (B) -2,4, -8, 16 TopperAnewer, 2020
(C) -2,-4,- 6, -8 (D) 2,-4, -8, -16
Ans.
0. 12. The21st term of the
OUNCERT Exemp.)
A.P, whose first two terms are
-3 and 4 is: Detailed Solution:
4 79 12
(A) 17 (B) 137 Taking option (C) 3'3'3'3
(C) 143 (D) -143
Here, for the condition of common difference:
RÊNCERT Exemp.] 7 4
Ans. Option (B) is correct. 33 2=1
3

Explanation: Given, 9 7 2
t,=-3 andt, = 4 3 3 3

d=t,-t=4-(-3)=7 12 9 3
and ==1
t, =a+(n-1)d 3 3

ty =(-3) +(21-1)(7)=137 So, their differences are not equal. Hence, it is not
theform an A.P.

SUBJECTIVE TYPE QUESTIONS


Sol. We know that
Very Short Answer Type 4 =a+6 x(-4)
Questions (1 mark each) a = 28
O.1 Which term of the following A.P. 27, 24, 21, .....is [CBSE Marking Scheme, 2020-21
zero ? ALACBSE SQP,2020-21] Detailed Solution:
We have, d = -4,n=7, and a, =4
Sol. We know that a, =a+ (n1)d
l=0 4 =a+(7-1)-4)
4 =a+6(-4) =a-24
0= 27 +(n-1)(-3) a =4 + 24
30 =3n a = 28.

n= 10 Q.3. How many twodigits numbers are divisible by3?


U[CBSE Delhi Set-1, 2019]
10th term of the given A.P. is zero.
(CBSE Marking Scheme, 2020-21] Sol. Numbers are 12, 15, 18,.., 99
99 = 12 + (n-1) x 3
Detailed Solution: n=30
Given A.P.= 27,24, 21,.... [CBSE Marking Scheme, 2019}
Here, a= 27 and d= 24-27=-3
Detailed Solution:
and, I= 0= 4, Numbers divisible by 3are 3,6,9, 12, 15, ---, 6, 99
Lowest two digit number divisible by 3. is 12. and
0 =27 +(n-1)(-3) highest two digit number divisible by 3 is 99.
-3n +3 =-27 Hence, the sequence start with 12 ends with 99 and
-3n = - 27-3=-30 Common dìfference is 3.
n=10,
So,the A.P will be 12, 15, 18, ---, 96, 99
2. In an Arithmetic Progression, if d = -4, n= 7, Here, a= 12, d=3, l=99
a, =4, then finda. AOACBSE SQ, 2020-21] 1 +(n -1)d
O These guestionsare for yractice and their solutions are availabie at the end of the chayter
(8TANDARD), Class X
100 s w CRNEst Rank Chartewse &Tbptse, MATHEMATICS
y
0.9, For what value ofÅ will A+9, 2A -lànd
of an A.!?
the consecutive terms
G+OD se
Topper Answer, a016

Sol.
Theref the a N. wodigit numtes divisible

Q4 in an AP. if thecommon dittenene ()=-4 and (9X2+7) 2(2k-)


term.
the seventh term ía)is 4 then find the first 2t+7 4A-2
Delh: OD 2018)
be 72
Q.3 Which term of the A.RS 14. 0, 26, ... will 16t2 43
Ore than its 41 term.
ACBSE Outside Delhi Set-Il 2017}
CBSE Boand Comptt. Set-llL 20171 0. 10. Find the 10th term of the sequence:
V2, V8.h
0.6 Wite the term of the A.P
1 1-z 1+2 USQr, 2016] [Foreign Set-l, II, IL
Q. 11. Is series V3, V6,V9,V12, . an A.P ? Give rreas
ADACBSE Delhi Compt. Set-I. IL. IT 20171 UICBSE, Term-2,s
1
Sol We have Sol. Common difference,
d, = 6-3
1+m=l =
/-1)
a=+(n-1)1 Again,
= 3-/6
Hence, 4, = -+n-1
D
1+(n-1)m 1 = 2/3-3
As common differences are not equal.
0.7,Ifthe n term of the A.P-1,4,9, 14, ...is 129. Hence, the given series is not an A.P.
ind the value of n.
[CBSE Marking Scheme, 20131
ACBSE Delhi Comptt. Set L, II, IIL, 2017]
SoL Given, a=-1 and d= 4-(-1) = 5 Short Answer ype
a, =-1+ (n-1) x5= 129 Questions-I (2 marks each
O, (n-1)5 = 130 0.1. Find the number of natural numbers between
(n-1) = 26 and 998 which are divisible by 2 and 5 both.
n=27 AOA CBSE SQP, 2
Hence, 27 term= 129.
Sol. 110, 120,130,..,.990
(CBSE Marking Schene, 2027| a, = 990’ 110 +(n-1)×10 =990
which n= 89
0.8; What is the common difference of an A.P in
Mn-ay 84? {CBSESQPMarking Scheme 2
ACBSEOutside DelhiSet-I, IL, III, 2017| Detailed Solution:
The number which ends with 0is divisible by 2a
Topper Answer, 2017 both.
..Such numbers between 102 and 998 are:
Sol. 110,120, 130,....., 990%
Last ternm, A,=990

at (zt-snd+-(a +(7-1)4J- 84 a+ (1 +1)d =990


110 +(n-1) × 10 =990
24 110 + 1On-10 = 990
10n + 100 =990

tire and thrir olutions are available at the end of the chapter
10n =90 - 100 ARITHMETIC PROGRESSIONS 101
101 =890 Detailed Solution:
890
1
10
89,
1
Given, 7a, 11a,

2.Show that(4 -b' (a'+ b) andI(a + b are in A.P. Then, 7[u + (7-1)d| = 11a + (11 -1)d]
ADACBSE Delhi Set-1, 20201
Sol..Given:(a-b),(a' + and (a + b)²
7u + 6d) = 11(a + 10d)

Common difference. 7a + 42d= 1la + 110 d


11a-7a = 42d-110d
d, = (a +b-(a -b)² 4a = -68d
=+-(a+b-2ab)
=a+--b+2ab
= 2ab a+ 17d=0
a=-17d

and d, = (a + b)- ( +b i.e., a+ (18-1)d = 0


=a+b+2ab-a Hence, a1g =0.
= 2ab 2.6.Find how many integers between 200 and 500 are
Since, divisible by 8.
Hence, (a-b),(a + b) and (a + by are is an A.P 1 ABoard Delhi comptt. Sel-I, II, II, 2017]
Hence Proved. Q.7. The fifth term of an A.P. is 26 and its 10h term is 51.
Q.3. Which term of the A.P. 3, 15, 27, 39,.. will be 120 Find the A.P AOD Comptt. Set-1, 2017
more than its 21 term ?
Sol. Here, as =a+ 4d =26 ..) ½
A CBSE DelhiSet-l, 2019) and Ayo = + 9d =51
Sol. 4n=a + 120 Solving Eqns. (i) and (ii), we get
= (3 + 20 x 12) + 120 or, 5d = 25
=363 d =5
363 =3 + (n-1) × 12 and Q=6
n=31 1
or 31t term is 120 more than az1.
Hence, the A.P is 6, 11, 16. . .
¡CBSE Marking Scheme, 2017|
TCBSE Marking Scheme, 2019)
Deiailed Solution: Q.8. How many two digit numbers are divisible by 7?
Given A.Pis: 3, 15, 27,39 ACKSE SQP,2016
Here, first term, a = 3 and common difference. 9In a certain A.P. 32h term is twice the 12h term.
d= 12 Prove that 70th term is twice the 31 term.
Now, 21 term of A.P is AOABoard Term-2, 2015]
t = a+ (21-1)d ,=a+(n-1)d] Sol. Let the first term be a and common difference be d.
ty = 3 +20x 12 = 243
Therefore, 21 termn is 243 According to the question, ag2 = 2412
We need to calculate term which is 120 more than
a +31d =2a + 11d)
a+31d =24 + 22d
21$ term 1
ie.,it should be 243 +120 =363 1
a= 9d
Again, azo at 69d
Therefore, t,= 363 =9d + 69d =78d
t, = a + (n-1)d
a3 =@+ 30d
363 = 3+ (n-1)12
360 = 12(n -1) = 9d + 30d =39d
n-1 = 30 Hence, Hence Proved. 1
n = 31 [CBSE Marking Scheme, 2013}
So,31s term is 120 more than 21 term. 1 The 8h term of an A.P is zero. Prove that its 3sth
Q.4.Find the 20th term from the last term of the A.P: term is triple of its 18th term.
j3, 8, 13, .....253. ACBSE SOR 2018) AOC8SE Board Term 2, 2013)
Q45.If7 times the 7th term of an A.P is equal to 11 times Sol. Given, a,= 0 or, a + 7d =0 or, a=-7d
its 11h term, then find its 18th term.
ACBSE SQP-2018| [Foreign Set-2017]
ICBSE Board Term-IL, 2016} Ag 7d +37d = 30d
Sol. 7a, = 11aj1 And,
7{a + 6d)= 11(a+ 10d) 1 =-7d+ 17d =10d
a+ 17d =0 Or, Bag =30d=3x 10d 3 Xays
1 Hence Proved. ½
[BSE Marking Seheme, 2018| CHSE Narking Seheme 20131
hese questions are fur practice and their solutions are available at the end of thechapter
Topícwise, MATHEMATICS (STANDARD), Class.y
To2 Oswaal CBSE QUestion Bank Chapterwise & the A.P.7,
Q2. ind the middle term of 13,:
20 and the sum of its
Q. 11. The fifth term of an A.P. is is 64. Find the common AOUCBSE. OD,1Set9, 1l1,0
terms 7 =6
Sol. In this A.P, a =7, d= 13 -
seventh and eleventh
difference. A|Foreign Set II, 2015| I, = 247
(CBSE Board Tern-II, 2015| and
t, =a + (n-1)d
d.
Sol. Let the first term be a and common difference be 247 = 7+ (n-1)6
Then, a +4d 20 6(n -1) = 40
and a + 6d + a + 10d = 64 n-1= 40
a+ 8d 32 ..(i)1
n = 41
Solving equations (i) and (i), we get Hence,
d=3
n+1
Hence, common difference, d=3 the middle term
[CBSE Marking Schene, 2015] 2
197, ...
Q12. ind the middle term of the A.P 213, 205,
41+1
2015] = 21th term
37. ACBSE DelhiBoard Term, 2

Sol. Here, a= 213, d 205- 213 = -8 and =37 a1 4+ (n-1)d


Let the number of terms be n. =7+ 20 × 6
1=a+ (n-1)d = 127
37 = 213 + (n-1)-8) 0.3. For what value of , are the nth terms of two
OL, 37- 213 =-8(n-1) 3, 10, 17,.. equal ?
63, 65, 67,.C+andA[CBSE
-176 Outside Delhi Set-II,
n-1= = 22
Or, -8
n= 22 +1= 23 Topper Answer, 2017
23+1 = 12th
The middle term will be =
2 Sol.
Qnd A,D be the ]S em and comn
a2 =a + (n-1)d letdifferenee
a,d of 4he 2 APs
= 213 + (12-1)(-8) rorestey
= 213- 88 2

= 125 Ae3 D 7

Thus, the middle term willbe 125.


[CBSEEMarking
M Scheme, 2015) an

’ a4 Cn-1)d : A + (n-1)D
634 Cn-102
Short Answer Type 63 +2n-2 7n-7
Questions-II (4 markseach) G1.2n
3 65

1 Which term of the A.P 20,19;,18;,17 13


When n is 13he n* tms aequal
isthe first negative term ?
AOA{CBSE OD Set-III, 2020]
Sol. Here, First term,a = 20
77 3 .4.N the 10th term of an A.P is 52 and the 17 term is
-20 = - 1 20 more than the 13th term, find A.P
and Common difference, d = 4 4
A|CBSE, Outside Delhi Set-i 2017
Let t, <0 p.5.The ninth term of an A.P. is equal to seven tims
iR
ty =a t (n-1) d the second termn and twelfth term exceeds
the first
times the third term by 2. FindACBSE ternm aN
the common difference.
SQP 0
ditteret
80--3n +3 <0
Sol. Let thefirst term of A.Pbe a and common
83-3n <0
be d.
83 Given,
n>
3
a+ 8d =7a+ d)

n> 27.6 and a12 =5ag +2


n= 28 Again, a+ 11d 5la + 2d) + 2
28n term. 1
Hence, the first negative term is
oiloble at the end of the chapter
From (i), a+ &d 7a+ 7d
-6a t d )
ARITHMETICPROGRESSIONS 103
Now, according to
From (ii), at 11d 5a + 10d + 2 ..ti) question,5
(14 -3d)(14 +3d)
- 4a + d =2 (14 -dy(14 +d) 6
Subtracting (iv)rom (üi), we get ..iv) 196 - 9d2
Or, 5
- 2a =-2 196-42
or,
or, 6(196- 9d =5(196- d'
From (ii), 6x 196-54d - 5 x
Or, 6 X 196 -5 x 196
196-5d
-6+ d =0 0r,
=54d-5d
(6-5) x 196 = 49d2
d 6
Or, 196
Hence, first term =4
=1 and
common difference =6 49

¡CBSE Or,
d= +2
ohe digitsare of a positive Marking
1
Scheme, 2016] :. The four parts are
number in A.P. and theirnumber
r
of three digit {14-3(2)}, {14- 2)}, {14 + 2)}, {14 + 3(2)}
sum is 15. The
abtained by reversing the digits is number Hence, first possible division will be 8, 12, 16 and
original number. Find the number.594 less than the 20.
and second possible division will be 20, 16, 12 and
ACBSE Delhi Set II, 2016| 8.
(Q.3 The pth, gth and th terms of an A.P are a, b and c
Topper Answer, 2016 respectively. Show that alq-r) +b(r-p) +c(p-9)
=0.
Ans. U[Foreign Set II, 2016]
Sol. Let the first term be a' and the common
difference
be d.
la-dtat Qtd_1s a=a+ (p -1)d, b= d +(g-1)d and
c=d+ (r-1)d 1/%
Riquuud 3diil no : 10fa-d) 41la+ atd..
1000.-100dt 10ata td alg -r) = (a + (p-1)d]lg-r]
1114-92d b(r-p) = [d + (q-1)d|[r-pl
and c(p -) = ld + (r-1)4|lp-q1
A0Da1 100d+ 10a4a-d
. a(q-r) + b(r-p) +cp-) =alg-rtr-p+p
111a+994
410 ql + a(p- 1)(q-) + (q-1)(r-p) + (r-1)(p-q)l
111a+29d = 1120-22d-s94
111a-9d-11%- 92 =dx0+ d[pq-pr t qr-p + pr-qr +(-q+r-r
594 -198d +p-p+q)] =0 Hence Proved. ½
-S94 [CBSE Marking Scheme, 2016]
193
9.Prove that the h term of an A.P. can not be + 1.
1-3da Justify your answer.
CBSE Board Tern-2 2015|
1110 ld
114x5- 39Y-3
sss+.7 8S 2
Long Answer Type
Questions (5 marks each)
(Q.1\The sum of four consecutive numbers in A.R is
Q.7Divide 56 in four parts in A.P such that the ratio 32 and the ratio of the product of the first and
of the product of their extremes (1 and 4) to the last term to the product of two middle terms is
product of middle (2nd and 3rd) is 5:6. 7:15. Find the numbers.
U|Foreign Set 1, 2016] ADUCBSE Delhi Set-l, 2020)
Sol. Let the four parts be CBSE Delhi & OD, 2018]
a-3d, a-d, a + dand a +3d. So. Let the four consecutive terms of A.P. be
i. a-3d +a-d+ a+ d+a+3d =56 (a-3d), (a -), (a + d) and (a + 3d). 1
4a = 56
1 By given conditions
a= 14
-3d +a-dtd +n+3d 32
14 - d, 14 + d and
ence, tour parts are 14 - 3d, 4a 32
14+ 3d.
the chapter
Th solutions are available at the end of
4csttons are forpractice and their
104 Oswaal CBSE Quention Bank Chapterwise &Topicwise, MATHEMATICS (STANDARD), Class X
1
a - d -bntn) +l] -o
and (a-3dXa +3d) 7
(a-dXa td) 15

7
15

' 4
d +2 1 .3. An A.P.consists of 50terms of which 3rd ter
Hence, the numbers are 2, 6, 10 and 14 or 14, 10,6 AndJast term is 106. Find the 29th term.
and 2.
ICBSE Marking Scheme, 2018]
1 UCBSE SQP
X4.the sum of three numbers in A.P is 12 and si
2. f mtimes the mh term of an Arithmetic Progression their cubes is 288. Find the numbers.
isequaltontimes its nh term andm#n,show thatthe AIDelhi Set Iil, t
(m + nh term of the A.P. is zero. Sol. Let the three numbers in A.P. be a - d, a
Then, their sum Le., 3a = 12 and a+
(CBSE Term 1, II, III, 2019) or a=4
Also, (4- d) + 4 + (4 + d' = 288
Topper Answer, 2019 or, 64 - 48d + 12d-+64 + 64 + 48d + 12
= 288
Ans. or, 24d + 192 = 288
d= 4
d= +2
Hence, the numbers are 2, 4and 6, or 6, 4and)
[CBSE Marking Scheme, 20
[m tn]_ o5.ind the value of 4, band c such that the number
To stso; a, 7, b, 23 and c are in A.P.
AOUCBSE Board Term-2,
Sol. Since, a, 7, b, 23 and care in A.P.
Let the common difference be d
a +d=7
and a+ 3d = 23
From (i) and (ii), we get
a=-1 and d= 8
Again, b=q+ 2d
b=-1+ 2x 8
or, b=-1+ 16
Or, b= 15
C=a + 4d
=-1+ 4 x 8
=-1+32
c=31 1
..a=-1,b= 15 and c=31
{CBSE Marking Scheme, 21

Sum of nTerms of an Arithmetic Progression


Topic-2 Concept Covered Understand the formula t And the sum ofn terms of A.P.
Students wtll be abte to recall some patterna which occur in thetr datly ife.

JRevision Notes Scan to know


iore abot
this topic
Sum of n terms of an A.P is given by: Sum of n terms of an A.P when
S,= " |2a + (n - 1)d) first and last term is given.
2
S, =
where, a is the first term, d is the 2
and nis the total number of terms.comnon difference
Sum a the hrst
where, a is the first term and l is
the last term.
ARITHMETIC PROGRESSIONS 105
Then"ternofan AP is the difference of the sum of first nferms and | the surn to first (n - 1) terss of it. ie.,
A, S,- S,

Fundamental Facts
(1) To find the sum of n terms of an AP. wp 1se a
formula first founded by
Iohann carl friedrich Gauss in the 19h century.
(2) A.! can be appied in real life by analysing a certain
A.P is used in straight line pattern, for exampBe,
depreciation.

OBJECTIVE TYPE QUESTIONS (1 mark eccb)

(7-1)d =19
Multiple Choice Questions
Q.1.The famous mathematician associated with
$,-[2a+(1-1ija]
inding the sum of the first 100 natural numbers is:
(A) Pythagoras (B) Newton 399=2+19]
(C) Gauss (D) Euclid n=38
RNCERT Exemp.] fo. 5.Thesum of first five multiples of 3 is:
Ans. Option (C) is correct. (A) 45 (B) 55
(D) 75
Erplanation: The famous mathematician associated (C) 65
UNCERT Exemp.
with finding the sum of the first 100 natural
numbers is Gauss. Q. 6. The sum of first five positive integers divisible by
6is:
o2f the first term of an A.P. is -5 and the common (A) 180 (B) 90
difference is 2, then the sum of the first 6terms is: (C) 45 (D) 30R [NCERT Exemp.
(A) 0 (B) 5 Ans. Option (B) is correct.
(C) 6 (D) 15 Explanation: Positive integers divisible by 6 are 6, 12,
RNCERT Exemp.] 18, 24,30
6.3 The sum of first 16 terms of the A.P..: 10, 6, 2..is: Since difference is same, its an A.P.
(A) -320 (B) 320 We need to find sum of first 5 integers
We can use formula
(C) -352 (D) -400
RNCERT Exemp.] S, = 2a + (1-1) d]
2
0.4. In an A.P., if a=1, a, = 20 and S,, = 399, then n is:
Here, n =5,d=6, a= 6
(A) 19 (B) 21
(C) 38 (D) 42 S, =[2 x6 +(5-1) x 6]
BNCERT Exemp.] 2
Ans. Option (C) is correct. 5
Explanation: In the given A.P., a = 1, 4, = 20 and S; =[12 +24]
S, =399
S, = x36
a, =a+(n-1)d 2
20 =1 +{7--1)d = 90,

SUBJECTIVE TYPE QUESTIONS

Very Short Answer Type


Questions (1markeach)
Q.1. Find the sum of the first i0 multiples of 6.
A|CBSE Board Term, 2019|

Topper Answer, 2019 330

Ans.
d.2Ifnth term of an A.P is (2 + 1), what is the sum of
its first three terms ? CBSE SQP 20I8)
These questions are for practice and their solutions are aailable at the end of the chayter
MATHEMATICS (8TANDARD), Class.y
Question Bank Chapterwisc &Topicwise,
106 Oswaal CBSE
Alternate method:
-5 and the common
0.3.If the first term of an A.P is first
S, =3n-4n
of the
difference is 2, then find the sumn S,-1 3(n-1)- 4(n -])
terms. 3r-10n + 7

Short Answer Type Hence


=(3n--4n)-(3n'-
Questions-I (2marks each) = 6n-7

Q.1.Find the sum of first 20 terms of the


following A.P: [CBSE Marking Scheme
1,4,7,10, Detailed Solutivn:
AACBSE Delhi Set-I, 2020] Given, S, = 3n'-4n
1 =-1
Sol. Given A.P.: 1, 4, ,7,10, .. Put n = 1, S, =3 x 1'-4 x -1.
of A.Pis
So, sum of first term
Here, a= 1,d=4-1=3andn= 20 But sum of first term will be the first term
.. The sum of first 20 terms, .. First team, a =-1
S =[2a +(n-1)d] Put n = 2, S, =3 x 2-4× 2 = 4
4.
.. Sum of first two terms is
=4
20 .. First term + Second term
(2x1+ (20- 1)3] -1 + a, =4
2
ay =5
= 10(2 + 57)
= 10 x 59
Hence,Common difference, d=a-a =5
nth term, a, = 4, +(n -1)d
=590. 1 a, =-1 + (n-1)6
i.e.,
[CBSE Marking 5cheme, 2020] a, = 6n -7
Therefore, nth term is 6n -7.
Q 2. The sum of the first 7 terms of an A.P is 63 and that
of its next 7 terms is 161. Find the A.P.
AAICBSE Delhi Set-III, 2020]
Commonly Made Error
Sol. Since, S, =2a +(n-1)4) Some students do not know the basic conceps
Given, S, =63 of arithmetic progression. Many students try to
solve with wrong method.
So, S, =2a +6d)
2a + 6d=18
= 63
...) ½
Answering Tip
Now,sum of 14 terms is:
Learn the concept of Arithmetic progression
S14 =Srst 7terms t Snext7 terms with different examples.
= 63 + 161 = 224
14 Q.4. Find the sum of first 8 multiples of 3.
2
2a+ 13d] = 224 ADelhi/OD 2018| |Delhi Comptt. Set-l, 21
2a+ 13d 32 Sol. Here, S =346+9 +12 +. +24
On subtracting (i) from (i), we get =3(1 +2+3+.. + 8)
(24 +13d) - (24 +6d) = 32-18 =3x 8x9
7d =14
d=2 108
Putting the value of din (), we get [CBSE Marking Scheme, 0
a 3 Detailed Solution:
Hence, the A.P willbe: 3, 5,7,9,...
[CBSE Marking Scheme, 2020| Topper Answer, 8o18
IfS,, the sum of first nterms of an A.P is given by Sol.
S, = 3n4n. Find the nth term.
AACBSE Delhi Set-1, 2019]
Sol. a4 S 3-4 -1

=(3/2)-4(2)-(-1) = 5 ½
Hence A1+(n-1) x6 6n-7 A
These questions are for practice and their solutions are available at the end of the chapter
How many terms of the A.P. -6, ARITHMETIC PROGRESSIONS 107
2-5, -
are needed to give their sum zero. 2 Sol. Given, first term, A e a
and
R[CBSE Second term h
Delhi comptt. Set-IlI, coon differenCe, d t
[CBSE Delhi 2017|
A.! of terms, the
50 Set-IIL, 2016] Last term, I C
aoInan Sum of the
last 15 terms first 10 terms A + (n --1)d = c
210and the sum
is of its
is 2565.
Find [By using, I a + (n-1 1
wanted to save atAForeign Set-III, 2017]
the A.P.
a + (n-1)d = C
5. Reshma least
6,500 for sending a + (n- 1)(b- a)
bher daughter to school next year (after
c
Shesaved 450 in the first month 12 months). (b- a)(n - 1) = c-4
savings by 20 every next month.
and raised her
How
she be able to save in next 12 months ? much wili
n-1 =
b-a
able to send her daughter to the Will she be
school
C [Foreign Set-I, II,next year?
[CBSE Delhi Term-II Set-I, II, III, 2016) C1
b-a

IIL, 2015] C-a+b-a


Sol. Here a =* 450, d =? 20, n= 12 b-a
b+c-2a
S, =[2a + (n -1)d] 1
b-a
12
S12 =2x
2 450 + 11 x 20] Now Sum
[A + ]
=6[1120] (b+c-2a) [a + c]
= 6720 > 6500
.
2 2(b- a)
Reshma wili be able to send her daughter to
school. [CBSE Marking Scheme, 2016] (a +c)(b+c-2a) 1
QE)ln an A.E, if S;t S, = 167 and S10 = 235, then find 2(b-a)
the A.P, where S,, denotes the sum of first n terms, Hence Proved.
A [CBSE Board, Term-2 2015] [CBSE Marking Scheme, 2020|
Q. 2. Solve the equation: 1 + 4 +7+ 10 + ... + x= 287.
Sol. S, = 2a +(n-1)d] AOA[CBSE DelhiOD Set-I, 20201
2
Sol. Given, a= 1 and d=4-13
Given, Ss + S, = 167
7 Let number of terms is the series be n, then
Hence,(2a
2 +4d)+(2a
2
+6d) = 167
S,, = [2a + (n -1)d] ½
Or, 244 + 62d = 334
or 12a +31d =167 ..)% n

S10 =235 2 2x1 +(n-1)3] = 287


Given,
Or, 5(2a + 9d) = 235
2a + 9d = 47 2
[2 + 3n-3| = 287
Solving (i) and (i), wet get 3n-n 574 =0
a =1and d 5 3n-42n + 41n- 574 =0
Hence A.P. = 1, 6, 11,..
[CBSE Marking Scheme, 2015] ’3n(n-14) + 41(n -14) = 0
(n-14)(3n + 41) =0
41
Short Answer Type Either n = 14 or n = t is not possible.
Ouestions-IT (3 marks each) Thus 14h termn is x
. show that the sum of all terms of an A.P whose
a + (n l)d * t
Tirst term is a, the second term is b and the last
X=It 13 x3
term is c is equal to (a+c)(b+c-2a) 40.
2(b - a)
.CBSE Marking Scheme, 2020}
AOA(CBSE OD Set-l, 20201
These equestions are for practice and their solutions are available at the end of the chapter
O8 Deailed fiolsbon

NOCSOD Set |, 20201

(pven)
m21

(given)

Here, (m- 1)is not equal to ero,


So,
Z:-) m--)+ r -(m-n)) Hence,
0.6.f the sum of first four terms of A.!. is %.,
-(m - n)-(n-)) that of first 14 terms is 280. Find the sum
n term8. AICBSE Delhi Seti
- 2 a t d(m t n- 1) Q.7. How many terms of an A.P 9, 17,. 25, -. i
Now. m+ 12 + (n + l -1)d] taken to give asum of 636?
AICBSE OD Se-tI1.
(-2)
Topper Answer, 2017
-(nt n) Hence Proved. 1
4)Find the sum of all 11 terms of an A.P whose Sol.
middle term is 30. A|CBSE OD Set-II, 2020] 4=1.) d 8 3n: 636.
Sol. ln an A.P. with 11 terms,
Sn
11+1 2
middle term = term
2
636 nfis +Cn-1)87
= 6th term 1 2

Now, sixth term i.e., a, = a + (6-1)d


a+ 5d = 30 ..) 636e
[Qmiddle term i.e., 4, = 30 (given)] 1 636
6.36 n(5+ 4n)
Now, the sum of 11 terms, 5m4 4n
4n?+5n - 63,6 0
Su =2a + (11 -1)4] 4n? + 53n- 48n -636 =D
11 2a + 10d< sn?- u8n t 53n 636o
2
unCn- 12) + 53( n-12) a
11
x2la + 5d] Gnt s3) Cn-2o
2
= 11 x30 [from (1)] as n is
=330. natural numbe, n
. 12 teims req uied to gie
Q.6. If the sum of first mterms of an A.P is the same as
the sum of its first n terms, show that the sum of its
first (m + n) termns is zero. A(CBSE SQ, 2020] a. 8. Findthe sum of n terms of the series
Sol.

(2a + (m- 1)d] = (2a + (n-1d] 1 AUACBSE Delbi Set-L. IL RU


2 2
Sol. Let sum of n term be S,
2a(m -n) +d - m-ntn) =0
(m-1)[24 +(m + n-1)d| = 0
Smtn 0
[CBSE SQP Marking Scheme, 2020)
G These gustions are for practice and their solutions are auilabde at the end of the chapter
ARITUMETIC PROGRESSiONS 109

t up to ternns)
01,
t up to n terms
and
-4t up to n tems)
-(1+2 +3+... up to nterms) Now
2
(n-1) )
+
4t llp to sterms) mmn 2
1
2m
(1+2+ 3 +..up to nterms)
1 H(27 +1) Q1zFind the sum of all two digit natural numbers
1y which are divisible by 4.
2
A |Delhi Comptt. 5et-II, 2017
4n-+1 7n-1
2 2
Sol. First two digit multiple of 4 is 12 and last is 96
So, a = 12, d = 4and{=9
Hence, Sumof terms = 7n-1 Let nth term be last term 96 1
2 a, =a + (7- 1)d=|
[CBSEMarking Scheme, 2017] 12 + (n - 1)4 = 96
Q.9.1Ifthe sum of the first 14 terms of an A.P. is 1050 and n-1= 21
n= 21 + I=22 1
is first term is10, find its 20th term.
Now,
ACBSE OD Comptt. Set-IlI, 2017
o.oFindthe sum of all odd numbers between 0 and 50. = 11 x 108
= 1183 1
ADelhi Comptt. Set-III, 2017]
[CBSE Marking Scheme, 2017]
SoL. Given, 1+ 3 +5+7+ .... + 49
Q Find the sum of the following series:
Let, total odd numbers of terms be n. 5 + (-41) + 9 + (- 39) + 13 + (-37) + 17 +
a, =1 + (n-1) x 2 = 49 +(-5) + 81 + (-3) A |Foreign Set-I, 2017]
(n-1) x 2 = 49 1l= 48 Sol. The series can be written as
n-1 = 24 (5 +9 + 13 + ... + 81) + [(-41) + (-39) + (-37) +
= 24 +1= 25 1
(-35)... (-5) + (-3)]
For the series (5+9+ 13 + .... + 81)
Sy=l+49) a =5
d = 4
= 25 x 25 and a, = 81
= 625 Then, a, = 5+ (n-1)4
= 81
Hence, sum of odd numbers between 0 and 50
or, (1-1)4= 76
1
n = 20
[CBSE Marking Scheme, 2017] 20
d ttmth term of A.P. is and nth term is 1
, find the
S,=5+ 81)
= 860
sum of first mn terms. For series (-41) + (-39) + (-37) + ...+ (-5) +(-3)
ADACBSE Set-l, I, 2017]
a=-41
Sol. Let first term of given A.P be a and common
difference be d. d =2
Then, a,, =-41 + (n 1{2)
1
am4 + (m -1)d = 1= 20O

and a,=a+ (n -1)d = s,-A1+(-3)]


On subtracting (i)from (i) we get -440
1 1
Hence, the sum of the series = 860 -- 4440
(m -n)d= 1 = 420 1
[CBSE Marking Schen ,2017]
Ihese questions are for ractice and their solutions are
available at the end of the chapter
(STANDARD), Class.Y
l10 OswaalCBSE Question Bank Chapterwise & Topicwise, MATHEMATICS
arithmetic
Q. 14, The sum of firstn terms of three a, = S = 5
( progressions are S,, S, and S, respectively.
The first term of each A.P is 1 and common 13-5
d= a,-a1 =8-5 =
differences are 1, 2 and 3 respectively. Prove that Now, A.P. is 5, 8, 11, ..
S, +S =28, AOD Set III, 2016]
nth term, a,, = a+ (n - 1)d
Sol. Since, S, = 1+ 2+3+.. + n. =5+ (n - 1)3
S, = 1+3 +5+...upto nterms =3n + 2
and S =1+ 4+7+...upto nterms
Hence, az 3 X20 + 2
n(n+1) M20 62
Or, S, = 2
[CBSE Marking Scheme,
Also, S, =2
2
x1 +(1-1)2] Q. 16. Aditi required 2500 after 12 weeks
her daughter to school. She saved { to
first week and increased her weekly 100
=[2n] =n ?20 every week. Find whether
send her daughter after 12 weeks.
saving
she will be :
and S, =;2x1
2
+(1-1)3] C[CBSE Board Term-2, Set-1, II, I
n(3n -1) Sol. Here, required money is 2500
a= saving in 1 week =100
2
d= difference in weekly saving =20
Now, S; + S, = n(n2+1).n(3n2 -1) A.P. formed by saving,
According to the question,
n[n+1+3n -1] Sequence is 100, 120, 140, ....upto 12 terms
2
S, = [2a
2
+ (n-1)4]
n[4n]
12
2
[2 x 100 ++ (12-1) x
= 2n
= 2S, Hence Proved. 1 Or, = 6[200 + 11 x 20)
[CBSE Marking Scheme, 2016] Or = 6(200 + 220]
1 Or. =6 X 420
Q. 15. If the sum of the first n terms of an A.. is
2 = 2520
[3n + 7n], then find its nth term. Hence write its Hence, she will be able to send her daughter
20th term. school after 12 weeks.
A[CBSE Board Term-2, Set-II2015] [CBSE Marking Scheme, 2015
[CBSE SQP-2016] odes, denotes, the sum of the first n terms of an A
prove that S;2 =3(Sg- S4).
Sol.
2 AJCBSE Delhi Board, Set-I, 201:
Sol. Let a be the first term and d be the comma
difference.
Since,
-+71 S1p = 6[2a + 11d]
1 = 12a + 66d
x10= 5
Sg = 4[2a + 7d]
= 8 +28d
$, =39+7x2) and
S, = 2[2a+ 3d]
= 4a + 6d
-12+ 14] Then, 3(S_- S) =3[(8a + 280)-(4a + 6
1
=3[4a + 22d]
-;x
2
26 = 120 + 66d
From equation (i) and (ii), S = 3(S_- S)
= 13
|CBSE Marking Scheme, 20
ARITHMETICPROGRESSIONS 111
14hterm of an A.R is twice its 8h
18
The
the6thtermis -8. then find the sum of itsterm,
first 201f
Q Long Answer Type
terms. Questions (LAO) (5 marks each)
|CBSEOD Set-1,
,2015||Foreign Set-1, I1,2015) 2.1. The first term of an A.P is 3, the Jast term is 83 and
numbet of
firsttermbe a and common the sum of allits terms is 903. Find the
SalLet t difference be d. terms andthe common difference of the AP.
Here,
a + 13d = 2a + 7d) AOICBSE Delhi Set-Ii, 2019
at 13d =2a + 14d 1
Sol. Here a 3,4, =83andS, =03
.)% Therefore 83 =3+(n-14
Again, ..j1
a + 5d =-8 (7-1jd = 0
or, ..i)
(ü), we get
Solving(i) anda=2,d =-2 Also
20,
S= 2x2+ (20-1)-2)] 4 -(6+8)
= 10|4 + 19 x (-2)] (using(i) 1+%
= 10(438) =43r
= 10x (-34) =-340 1 n21
(CBSE Marking Scheme, 2015] and d=4 1+4
an A.P given that
the first term (a) = 54, [CBSE Marking Scheme, 2019
Q19.In
the common difference (d) =-3 and the nth term
Detailed Solution:
=0, findn and the sum of first nterms (S.) of
(4)
theA.P. AICBSE, Delhi & OD,20201 Given:
First term, a =3
Topper Answer, 2020 Last term, 4, =83
1
Sum of n terms, S,=903
1
Sol.4 S4 Since,

903 =(3+ 83)


1806 = 86n
1806
86
n=21
n 1
Now, S,=2a+(1-1)4]
903 2 x3 +(21-14]
1806 = 21(6+ 20d)
6+ 20d=86
20d=80
d=4
1
Hence, the common difference is 4.

Commonly Made Error


they
Some students fail to find the value ofn as
between the nth term and last
get confused
term.

Answering Tip
Understand the formulae related to given
condition and use them to solve the problems.
MATHEMATICS (8TANDARD), CIase. v
112 Oswaal CBSE Qucstion Bank Chapterwise & Topicwise,
a+ 21na -2a 2ma - a
O.2.If the ratio of the sum of the first n terms of two or, a+2na - 2a
SA.Ps is (7n + 1): (4n + 27), then find the ratio of 2na - a
their 9th terms. ACBSE OD Set III 2017
[CBSE DSet-I, 2016] a(2m -1)
a(2n -1)
Topper Answer, 2017 = 21n -1:2n-1

Sol.
A,D be ihe L ts m and eommor 6.A. Ifthe ph term of an A.P. is
1 Hence Prs
and q" term is
4itlerene

Then, Prove that the Sum of tirst pg term of


nT he
TCBSE Delhi Set-l,2
244 (n-124 In+ Sol. Try yourself similar to Q.No. 11of SATO.
4nt27 Q5 Ifth,ratio of the111th term of an A.P. to its 1
Leplaing nby 7inbon LHS_and RHS.,. is 2 : 3, find the ratio of the sum of the
terms to the sum of its first 10 terms.
[Delhi Comptt. Set-1,
a+10d 2
24+l6d Sol. Since,
2A 4 16D 8427 a+17d
4Ca48d) 129. or 2(a + 17d) = 3(a + 10d)
a= 4d

19
24
Now, (2a+44)
2
10
1. rato oP q* erms ís 24:19 S10
2a +94]
5
Putting the value of a = 4d, we get
0.3/ The ratio of the sums of first m and first n terms of
an A.P is m²:n. Show that the ratio of its mth and or, S (8d +4d)
Hth terms is (2m -1) :(2n -1). 5(8d +9d)
12d 6
[CBSE Delhi Set-I, 2017]
34d 17
Sol. Let first term of given A.P. be a and common
difference be d also sum of first m and first n terms Hence, S_: Sp =6:17.
be S, and S, respectively. O(6JAn A.P. consists of 37 terms. The sum of the thre:
middle most terms is 225 and the sum of the last
m three terms is 429. Find the A.P.
1
S,
Q.ÎThe minimum age of children CBSE SQP 201"
to be eligible t
participate in apainting competition is 8years. ti
or, 2"[2a +(m- 1)d] observed that the age of youngest boy was 8yeas
1
and the ages of rest of participants are havingi
common difference of 4 months. If the sum of ages
of all the participants is 168 years, find the age
Or,
2a +(m-1)d eldest participant in the painting competition.
1
2a +(n -1)d n CICBSE SQE 201:
(Or,
m(2a + (n -1)d) = n[2a + (m-1)d] 1 Sol. Here, a = 8, d = 4 months = years and
d = 24
S, = 168
Now 4+ (m -1)d Since
a+(n-1)d S, = 2a+(1-1)a]
a+ (m-1) x2a Hence, 168 =
a+(1-1) x2a
n+ 47n -1008 =)
These questions are for practle and their solutions are available at the end of the chapter
ARITHMETIC PROGRESSIONS 113
or.
+63n- - 16n-1008 = 0
(n-16)(n + 63) = 0 Sol. Let total titne to catch the thief be nrminutes.
of.
n= 16 orn-63 Then, total distance covered by thief (100n)
or
n= 16 metres
(u cannot be negative So -63 Total distances to be covered by policeman 100
the eldest participant rejected)
Thus, the age
of
=a+ 15d
1
+ 110+ 120 +.. + (n-1)terms 1
=13years CBSE Marking Scheme, 2016] 1
Q8 thiefruns with a uniform speed of 100 m/min- .. 100n= 1-(200
2
+ (1-2)10) 1

ute. After one minute a policeman runs after, the


thiefto catch him. He goes with aspeed of 100 m/ n-31 -18 = 0
minuteinthe irst minute and increases his (n -6)(n + 3) = 0
speed
10 m/ minute every succeeding minute. After n=6
how many minutes the policeman will Policeman takes 6 minutes to catch the thief. 1

thief. CICBSE DelhiSet I, catch the


II, 2016]) [CBSE Marking Schene, 2016]

COMPETENCY BASED QUESTIONS 4 marks each)

- 2n = -22
(0 Case based MCOs n =11
Q. 3. Which of the following term is not in the A.P. of
TPoad the following passage and answer any four
questions from Q.1. to Q.5. the above given situation
Var friend Veer wants to participate in 200 (A) 41 (B) 30
(C) 37 (D) 39
m race. He can currently run that distance in 51
seConds and with each day of practice it takes him 2 Ans. Option (B) is correct.
seConds less. He wants to do in 31 seconds. OIfh term of an A.P is given by
a, = 2n +3 then common difference of an A.P. is
[CBSE QB, 2021]
(A) 2 (B) 3
(C) 5 (D) 1
Ans. Option (A) is correct.
Explanation: 4, =2n +3 (given)
. a = 2>x1+3=5
ay = 2X2+3=7
ag =2 x 3+3 =9
So,common difference = a,-4,
=7-5
= 2.
0.5.The value ofx, for which 2x, x + 10,3x + 2are three
consecutive terms of an A.P.
0,1. Which of the following terms are in A.P. for the (A) 6 (B) -6
given situation
(A) 51,53, 55... (B) 51, 49, 47... (C) 18 (D) - 18
(C) -51, -53, - 55... (D) 51,55,59... Ans. Option (A) is correct.
Ans. Option (B) is correct.
Explanation: a= 51
Explanation: Since, 2x, x + 10,3x + 2are in A.P, this
d=-2 common difference will remain same.
AP = 51, 49, 47 ... x+ 102x = (3x +2)- (x+ 10)
2 What is the minimum number of days he needs to 10 - x= 2x-8
practice till his goal is achieved ?
(A) 10 (B) 12 3x = 18
(C) 11 (D) 9 X=6
Ans. Option (C) is correct.
II. Read the following text and answer any four
Explanation: Goal = 31 second questions from Q.1. to Q.5.
n =number of days
a, = 31
Your elder brother wants to buy a car and plans to
a + (n-1)d = 31 take loan from a bank for his car. He repays his total
51 + (n-1)(-2) = 31 loan of 1,18,000 by paying every month starting
51 -2n+2 = 31 with the first instalment of 1000. If he increases
the instalment by 100 every month, answer the
- 2n = 31-53
following: [CBSE QB, 2021]
114 Oswaal CBSE Question Bank Chapterwise &Topicwise, MATHEMATIC8 (STANDARD), Class-X
Hence, Ratio of Ist
instalment
1000
to the
4900
= 10:49.
III, Read the following text
questions from Q.1. to 0.5. and
RANK
answer
Jaspal Singh takes aloan fron a bank
Jaspal Singh repays his total loan
by paying every month
instalment of 1000. If he starting with
increases the
by ? 100 every month.
Q.1. The amount paid by him in 30th instalment is
(A) 3900 (B) 3500
(C) 3700 (D) 3600 LOAN
Ans.Option (A)is correct.
Explanation: a= 1000
d= 100
agn a + (n -1)d
= 1000 + (30 1)100
= 1000 ++ 2900
Q. 2. The amount paid by him in 30 instalments is
(A) 37000 (B) 73500
(C) 75300 (D) 75000 Q.1. Ifthe given problem
Ans. Option (B) is correct.
based on A.P,
the first term and common difference then
?
Explanation: Sum of 30 instalments (A) 1000, 100 (B) 100, 1000
¡2a +(n-1)d] (C) 100, 100 (D) 1000, 1000
Ans. Option (A) is correct.
30
2x 1000+ (30 -1)100] Explanation:The number involved in this case
2 an A.P in which first termn (a) = 1000 and com
difference (d) = 100.
= 15(2000 + 2900] Q. 2. The amount paid by him in 25h instalmenti
= 15 X 4900
(A) 73300 (B) 3200
= 73500
(C) 3400 (D) 3500
Total Amount paid in 30 instalments =73500
Q.3 What amount does he still have to pay after 30th Ans. Option (C) is correct.
instalment ? Explanation: The amount paid by him in
(A) 45500 instalment is:
(B) 49000
(C) 44500 (D) 54000 T; = 4 + 24d
= 1000 -+ 24 X 100
Ans. Option (C) is correct. = 1000 + 2400
Q.4. If total instalments are 40 then amount paid in the
last instalment ? =3400.
(A) 4900 (B) 3900 Q.3. The amount paid by him in 30th instalmentis
(A) 3900 (B) 3500
(C) 5900 (D) 9400 (C) 3000 (D) 3600
Ans. Option (A) is correct. Ans. Option (A) is correct.
Explanaton: Amount paid in 40th instalment, 440 Explanation: The amount paid by him in
=a+ (n-1)d instalment,
= 1000 + (40-1)100 T30 = 4 + 29d
= 1000 +3900
= 1000 -+ 29 x 100
= 4900
= 1000 + 2900
0.5. The ratio of the 1 instalment to the last instalment
is =3900.
(A) 1:49 (B) 10:49 Q.4.The total amount paid by him in 25 and
(C) 10:39 instalment is:
(D) 39: 10
(A) 7500 (B) 7300
Ans. Option (B) is correct.
(C) 7800 (D) R7600
Explanation: According to question 4, Ans. Option (B) is correct.
Amount paid in last instalment 25
=4900 Explanaton: Total amount paid by himin
30h instalment = (3400 + 3900)
Principal for Ist instalment = 1000
7300.
ARITHMETICPROGRESSIONS 11S
a difference.annount paid by him in 26h and 2glh
The 4+ 5l 16,000
instalmentis:
Q5.
a4 8d 22,6X)
(A) 400 (B) ? 100
( ) 50 (D) 200
Option(D)isi correct. -3d - 6,60)
Ans. vlanation: The amount paid by him in 26th d = ,200
instalment, Now, putting d = 2,200 in equation ()
T% =at 25d
= 1000 + 25 × 100
a+5d = 16,000
= 1000 + 2500 n+5X2,200 = 16,000
=3500 a+ 11,000 = 16.000
Theamountt paid by him in 28th instalment, a = 5,000
T8 a+ 27d Q. 2. Find the production during 8 year.
= 1000 + 27 x 100 Sol. Production during 8th year is (a + 7)
= 1000 + 2700 =5000 + 7(2200)
=3700 = 20400
The difference amount paid by him in 26th and Q.3. Find the production during first 3years.
28th instalmentis:
=* (3700-3500) Sol. Production during first 3year
= 5000+ 7200 + 9400
=200. = 21600

Case based Subjective Q. 4. In which year, the production is 29,200.


Sol. a, = 29,200
Questions a+ (n1)d =29,200
I Read the following passage and answer any four (n-1)2,200 = 29,200 5,000
questions from Q.1. to Q.5. 2,200n -2,200 = 24,200
India is competitive manufacturing location due 2200n = 26,400
to the low cost of manpower and strong technical 26,400
n=

and engineering capabilities contributing to higher 2,200


quality production runs. The production of TV sets n = 12
in a factory increases uniformly by a fixed number In 12 year, the production is 29,200.
every year. It produced 16000 sets in 6th year and Q.Find the difference of the production during 7h
22600 in 9th year. [CBSEQB, 2021] year and 4 year.
Sol. Difference = (a + 6d) - (a-3d) = 9d
=9x 2200 =19800
(INRefer the figure below and answer any four
questions from Q1 to Q5.
Aladder has rungs 25 cm apart. (see the below).

25 m

Based on the above information, answer the


following questions:
Q.1. Find the production during first year.
Sol. a; = 16,000
m
a+ (n + 1)d = 16,000
a+ (6-1)d = 16,000
a + 5d = 16,000 ..)
ag = 22,600
25 m
a + (n-1)d = 22,600
a+ (9-1)d =22,600
a + 8d = 22,600 45 m
Solving equation (i) and (i)
116 Oswaal CBSE Question Bank Chapterwisc & Topicwise, MATHEMATICs (8TANDARD), Class y

The rungs decrease uniformly in length from


45 cm at the bottom to 25 cnn at the top. The top and
the bottom rungs are 2- nm apart.
2

Q. 1. How much distance from the top to bottom rungs


are apart? m
Sol. Since the top and the bottom rungs are apart by
1 5
2; m= m
0.1. What is the volume of the required
5
X 100 cm
step 1?
Sol. For the 1 step, the volume of requiredd CoNcret
2
= 250 cm
1
¡x;x50 =
25
4
-m. concre
Q. 2. Find the number of the rungs.
Q. 2. What the volume of the required
Sol. The distance between the two rungs is 25 cm.
Hence, the total number of rungs=
250
25
+1
step 2?
Sol. For the 2nd step, the volume of required cocontncrere,y
= 11. xx50 =m.
Q.3. The given problem is based on A.P ind its first
term. 0.3. What is the volume of the required
Sol. The length of the rungs increases from 25 to 45 and step 3? concrete
total number of rungsis 11. Sol. For the 3rd step, the volume of required COn
Thus, this is in the form of an A.P, whose first term = m.
is 25.
Q.4. What is the last term of A.P.? 2x=x50
4 4

Q. 4. The given problem is based on which mathemak.


Sol. Total number of terms, n= 11 and the last term, T1
= 45. concept ?
Q.5. What is the length of the wood required for the Sol. The numbers involved in this case in A.P. in whi
25 25
rungs ? a=d= n=15
Sol. The required length of the wood, 4 4
11 0.5. What is the total volume of concrete to buildA
S1 =25 +45] terrace?
11
2
X 70 Sol. S, = [2a +(n-1)d), we hase
=385 cm.
(IILRead the following Passage and answer the
questions from Q1 to 05.
Smallterrance at a football ground comprises of 15 15 50 350
steps each of which is 50 mn long and built of solid
Concrete.
15 400
1
Each step has a rise -mand a tread of m 2 4
= 750 m.
(see figure below).

Solutions for Practice Questions (Topic-1)


Multiple Choice Questions Ans.5: Option (C) is correct.
iAns. 1: Option (A) is correct. Explanation: In the given A.P,
Explanation: 2x, (x + 10) and (3x + 2) are in A.P. a= 10 and d=7-10=3
(x+ 10) - 2x = (3x + 2) -(* + 10) Thus, the 30 term, ta) = 10 +(30 1) (-3) =-1
-x+ 10 = 2x-8 Ans. 8:Option (B) is correct.
-X-2x =-8-10
Explanation:
a= 3.5, d= 0, n = 101
-3x = -18
a, =a+(n-1)d ’ a, =3.5+(101--1)0 ’ 4, 32
X =6
ARITHMETIC PROGRESSIONS 117

( )iss correct.
Option Detailed Solution:
1
Ans11:
Esplanation: Given, a=-2, d=-, Given A.P: 3, 8, 13, ...53
4,=a+(n-1)a common difference (d) =
Here, first term (a) =3,
8-3=5and last term (l) = 253
h=(2)+(1-1)(-2)=-2 Then, 20th term from the end of the A.?
,=(2)+(2-1)(-2)=4 =|-(n-1)d
t,=(2) +(3-17(-2)=-6 = 253 + (20- 1)5
= 253 - 95
t,=(2)+(4-1)(-2)=-g = 158.
Short,Answerr Type Questions 208, 216, 224, .. 496.
Very Sol. 6: Integers divisible by 8 are 1

Sol.4:Since, a+ 6(-4) = 4
a= 28 Which is an A.P.
1 496
Given: a=208, d = 8 and
[CBSE Marking Scheme, 2018] Let the numbers of terms in A.P. be
n.
iDetailedSolution: l
4, =a+(n-1)d =
208 + (n -1)d = 496
Topper Answer, 2O18 (n-1)8 = 496-208
288
n-1= 8
The fit term is 23 = 36
n= 36+ 1= 37
divisible by 8 = 37. |
Hence, no. of required integers
divisible by 7 are:
Sol. 8:Twodigit numbers which are
14, 21, 28, .....98.
It forms an A.P.
Here, a= 14,d =7 and a, = 98 ½
a, =a+(n-1)d
and d =6. Since,
Sol. 5: Given a= 8 41" term.
98 = 14 + (n-1)7
Let nth term be 72 more than its 98 14 = 7n-7
t,-ty =72 84 +7 = 7n
6) = 72
8+ (n-1)6-(8 + 40 x Or,
7n = 91
8+ (n-1)6 = 320 n = 13
(n-1)6 = 312 Or,
[CBSE Marking Scheme, 2016]
n-1=52
Short Answer Type Questions-ll
n=53 1

A.P.
Sol. 10: Given sequence is an Sol. 4:
Azo 52
V2, V8,V18,..=V2, 2/2,3/2.. or,
a+ 9d = 52
a7-A1g = 20
..) 1

Hence, a= 2,d= y2 and n= 10 Also


a+ 16d-(a + 12d) = 20
a, =a+(n-1)d 4d = 20
Ay0 = V2+(10-1)N2 d=5
Substituting, the value of d in (i), we get
= y2+9/2 a =7 1

= 10/2 Hence, A.P =7, 12, 17, 22...


1 [CBSE Marking Scheme, 2017]
Hence, A10 = V200.

Short Answer Type Questions-l Sol. 9: Let nh term of A.P.,


a, = +1
i Sol. 4: 20th termfrom the end =|- (1-1)d Putting the values of n= 1,2, 3,.., we get
= 253- 19 × 51 a, =1+1 =2
= 158 ½ a, = 2 + 1-5
[CBSE Marking Scheme,
2018| ag =3 +1= 10
118 OswaalCBSE Question Bank Chapterwise &Topicwise, MATHEMATIC8 (9TANDARD), Clase v

Long Answer Type Questions


The obtained sequence
2, 5, 10, 17, . . . . Sol. 3: Given, n= 50, a, = 12 and asg 106
Their common difference Then a + 2d 12
and a + 49d 106 Sol
Or, 5-2+10 --5 17- 10 On solving, we get d= 2and a=
357 1 azg a+ 28d

Sincethe common difference are not equal. =8+ 28 x 2


=64
Hence, n +lis not a form of nth term of an A.P 1
[CBSE Marking Scheme, 2013] (CBSE Marking Scheme,

Solutions for Practice Questions (Topic-2)


Multiple Choice Questions
Ans. 2: Option (A) is correct.
Explanation: In the given A.P.,
S, =2x(-5)+ (6-1)z
a=5and d= 2 =3(-10 + 10)
= 0.
s,-2a+(n-14] Short Answer Type Questions-l
Sol. 5: Given a=-6 and d =
S-2x(-5) +(6-1)×2]
=0
Since,
Ans. 3: Option (A) is correct.
Explanation: In the given A.P., Let sum of n terms be zero.
a = 10, d=6-10 =4 S, =0
Thus,
S,=2+(1-1)4]
or
2x-6(n-D =0

’ So2x10 +(16-1)x(-4)] Or

=-320
Or, -0
Ans. 5: Option (A) is correct.
Explanation: In the given A.P., Or, n-25n =0
a=3,d=3 and n=5 1+4
n(n- 25) =0
S,=H2a+(n-1)d] n = 25
Hence, terms are needed = 25.
as n 0

Sol. 6: Given,
s-[x3+(5-1):3]=48 Since,
Syo = 210
Very Short Answer Type Questions S,= 2a+(1-)d]
Sol. 2: Since, a, =3, a, = 5 and a, =7 10

S, =(3+7) =15
Or,
2a+94) - 210

or, 2, + 9d = 42
{CBSE Marking Scheme, 2018] Since, a36a + 35d
Detailed Solution: and
a, = (2n + 1) Uso 2+ 49d
Hence,
a =2 x1+1=3
I= a, = 2x3+1 =7 Sum of last 15 terms = 15
Since,
a+35d +a+ 49)
2 15
Of, (2a + 84d) = 2565
Hence, S, = 3+7) OI, A+ 42d 171 ..)
S, = 15. On solving (i) and (ii), we get
Sol. 3: In the given A.P.,a = -5 and d = 2 a=3and d =4
Thus, Hence, given A.P. is 3,7,11 ....
S, =;[24 +(1-1)d] {CBSE Marking Scheme, 2011
AnswerType
Short Questions-l|
= 40 >2(2a + 3d) 40
ARITHMETICPROGRESSIONS 119
Sol.6:5,
+ 13d) >2a 3d 20
+
=280:7(2a
Sy 280 2a +13d Sol.9:LetGivtheen, a 10, and 5s 1054)
Solving to getd 2 40 Y% common difference of fhe A.P. be d.
anda = 7
Since, (2a +(n-1yd
S, = " 2
2 14 +(n- 1)2)
=n(n+ 6) or (n +6n) h S210-(14 -1)4
eatledSolution:
|CBSE Marking Scheme, 2019 1050

Since,
20+ 13d= 1050-=150
7
Sum of n terms of an A.P. 13d = 130
130
S,=2a +(n-1)d] d=
13
=10 1

[a be the first a, =a+ (n-1)d


termdifference]
the common and dbe ag =10 +19 x 10 = 200 1
According to question, S, = 40 Hence, an =200.
4
2a + (4-1)d] = 40 (CBSE Marking Scherne, 2017]
½ Long Answer Type Questions
2[2a+3d] = 40 Sol. 6:Let the middle most terms of the
A.P. be (a - d),
2a + 3d = 20 a and (a + d).
and ...)% Given, a- d + a +a+ d= 225
S14 = 280
14 Or, 3a = 225 1
72 2a +(14 - 1)d] = 280 Or, a=75

7(2a + 13d) = 280 and the middle term = 37 +1 = 19th term


2a + 13d = 40 2
.:. A.P. is
Solving eq. () and (i), we get
a=7 and d= 2 (a-18d), ..a-2d), (a - d), a, (a + d), (a+ 2d), ....

(a+ 18d)
S, = 2 ;2x7+(n-1)2]) Sum of last three terms
(a + 18d)+ (a + 17d) + (a + 16d) = 429
[14+ 2n-2] or, 3a + 51d= 429
or, 225 + 51d =429 or, d = 4
=(12
2 +2n) First term, 4, = a-18d =75-18 x 4 = 3.
= 6n+ n a, =3+ 4 =7
Hence, Sum of n terms = 6n + n. Hence, A.P. =3,7, 11,... 147. 1

REFLECTIONS

An arithmetic progression enables us to understandthings with the use of patterns.


a) Can you 1find the sum of nnumber of things used in your daily life such as stacking cups, chairs, bowls etc., using
arithmetic progression?
b) Are you able to find the last term of arithmetic progression using formula?
SELFASSESSMENT PAPER-a

LONNmNtaltenatìr fom the gienoptions,


Q . T - + l - 3 is a:
(B) Quadratic Equation
(D) Biquadratic Equation
Q i a n ofthe quadrati auaton (:-1 -5(-)-6 = 0is:
(B) 0and 1
(D) None of these
Qibehn ofthe quetion *+t-(a+a-6) = 0is:
(B) a +2, a +3
(D) a + 2, - (a + 3)
QA mn dittereN ot an AP. is 5, then ag -R is:
(B) 25
(D) None of these
QiE7she 7h tem of an A.P. is equal to 11 times its 11th term, then its 18th term will be:
(B) 1
(D) None of these
Q¬ The sun of thë zeroes of the quadratic polvnomial 3r-9x + 6= 0is:
LA 3 (B) -3
1
(D)
6
L C2se-B2sed Ouestion
Q.7. Rezd the following text andanswer the given below questions it:
napotato race, 2bucket is placed at the starting point, which is 5mfrom the first potato, and the other noh
aepiacei 3 m apart in a straight line. There are ten potatoes in the line (see figure below).

Aconpetitoz starts fron the bucket picks up nearest potato, runs to the bucket to drop it in, and the continue
the sarme way untíl allthe potatoes are in the bucket.
Read carefuly the above paragraph and answer any four questions: [1x4={
i) What is the distance travelled by competitor to pick 1" potato ?
(A) 10 m (B) 16 mn
(C; 2m (D) 48 m
(i) What is the distence travelled by competitor to pick2d potato?
(Aj 10 m (B) 16 mn
(C) 22 n (D) 48 m
(üi) What is the distance travelled by competitor to pick 3rd potato?
(A) 10 Tn (B) 16 m
(C) 22 Trn (D) 48 m
(iv) The comnon difference of A.P involve in this problem is
(Aj 2 (B) 4
(C) 6 (D) None of these
(v The sun f 1Gterrms of an A.P.whose first term is 10 and common difference is 6 is;
(B) 370
(D) None of these
Short Answer Type Questions ARITHMETIC PROGRESSIONS 121 9
Very
Q8
|I.Findthe nature of rOots of the quadratic equation 13/3x +10x +N3 =0. [1 x3 = 31
the zeroes of sof the quadratic
A.P., -1, 4, 9,polynomial. 6-x-2 =0.
Q9
Find
nthterm of an 14,..is
If
the
Answer Type Questions-I
129,then find the value of n.
Q10.Short A [2 x 3 = 6]
IV
2 andx=3 are
o 11.Ifx=7 roots of the quadratic equation what is the value of a and b?
3 ax+7x +b= 0, then
.Finda quadraticpolynomial, the sum and the
Q 12.. product of whose zeroes are V2 and -32 respectively.
second term of. an A.P., is 13 and the fifth termis 25,
Ifthe then find its 7 term.
13.
Q Short Answer Type Questions-II [3 x2= 6]
V
squareof smaller number is 4times the larger number and the difference between the squares of two
0.14.Ifthe
numbersis.45, then
find the numbers.
first two
thesum
ofthe first n terms of an AP is 4n - n', what is the first term (that is S,)? What is the sum of
0.15.If
is the second term
What ?Similarly, find the 3rd,the 10h and the n terms. [1 x5= 5]
terms ?

Long Answer Type Questions 5 marbles each, and the product of the number of
themlost
VI.
andJivanti together have 45 marbles. Both of
John marbles they had to start with.
they now have 124. Find out how many
Q.16. is
marbles
OSWAAL COGNITIVE
Finished Solving the Paper ? LEARNING TOOLS
Time to evaluate yourself!

OR...
SCAN THE CODE

For elaborated
Solutions
CO-ORDINATE GEOMETRY
UNIT II

CHAPTER

LINES (in Two.

AE Syllabus
6 Dimensions)
Concepts of coordinate geometry graphs of linear equations, Distar
formula. Sectionformula
(internaldivision)

In this chapter you will study


Coordinate axes (X-axis and Y-axis)
Distance between two points
Distance ofa pointfrom the origin
Section formula

Revision Notes

Two perpendicular number linesintersectingat origin OP Key Words


line is the
horizontal
are called co-ordinate axes. The Axis: A line, used as a reference to
X-axis (denoted by XOX) and the vertical line is the determine position, symmetry and
Y-axis (denoted by YOY). rotation.
Point: To indicate the presence or
Y-axis position of.
Plane: A level or flat surface.
6
Quadrant I
Quadrant I
(-+)
5
4
(t, +) The point of intersection of X-axis and Y-axis ise
3
origin and denoted by O. Scantoknos
Cartesian plane is a plane noreab
thistopk
Xaxis
obtained by putting the co
|o1 2 3 4 5 ordinate axes perpendicular to
14
-2 each other in the plane. It is also
-3 called co-ordinate plane or XY
Quadrant II 4 Quadrant IV Theoremof
5 (t,-) plane. ordinateGone

-6
The X-co-ordinate of a point is
-7 its perpendicular distance from
Y-axis Y-axis. peerpendi
its
The y-co-ordinate of a point is
distance from X-axis.
LINES (IN TWO-DIMENSIONS) 123

(Ordinate)
Y-axis
(Abecissa)
X-axis
Unit
=1+4 Unit Unit Unit BD.
Unit Unit +(-1-4=4 AC=
=(4,4)
=34
=4
1=34 68 (2-42= J8 equal
(XA
y)
ordinate
>X
abscissa
a 4)?
of
(4,D
= DAare square
and
vertices
-1),C=(-1,-1);
(7-421) diagonals
Vertical
(2+ (7+ Horizontal
2),(-1, +(7-2 W4+4+
Example a
CD
= is
points + + + ABCD
and Level
1
(4,(4,2);=1-4 4 +1)
following
(1,7), +
=W4 +
W1 W1 =
BC sidesHence, Third
=
= = Map
:square
B AB four
Since,
the (1,7);AB BC CD DA AC =
BD
Al Coordinate MindLevel
Are Second
A= Cxes the
Pace
Level
Centroid irst

Distance
formula
T

R
P.

Y Lines
Meaning

Quadrant
(In
X Quadrant
(t)
\Quadrant
(++)
I
IV Section
formula
(-+)
Y
Segment
Line
aofmid-point

2(-2) 1(-2)\
m
4) + 1+2 2/4) 2+1
1(2)
+
B(-7, 1(4)
of
and B (-7,4)
trisection B(2,
2) X=
C =
2/2) Y Q2,
)
Example
-2)
+ 1+2 + 2(-7)2+1
A(2, X
of 1(-7) =1,0) (4,2) (+)Externally
(-)
Internally 2
AB,
point
segment
(2-2) Q= =
(X.)
P
R
(a)
Find
A
Coordinate
of
P Q
of
Coordinate A 2
P
line
Chapterwise 1. IfAB
Bank intersect asquare.
Question kis 2 If AB= BC = CD DA; AC
Oswaai CBSE the
124
the Aais atnd the arhombus.
point uhere ooniizate of
Theco-ordinatepoint (0. 0). CD, BC = DA; AC =
has the 3. If AB =
The
point.
abscissI of a
oint is

pOint Is
the
ordinate of
the rectangle.
4. If
AB= CD, BC = DA; AC * BD,hen
1he ordinate
point.
If the abscissa
of

ofa
a

ointis
calledthe
the
ordinateof
and 0-ordinatesof
the
the
parallelogram.
Diagonals of a square,
parallelogramalways bisect each
BD,hen
hombuS, Tet,
square otb1Seher.ct ez
then (r. v)is parts
pointis v, intofour Diagonals of rhombus and
point.
the
Cartesian plaanenumberedI,I, rightangle.
divide part),
The axesquadrants(onetourth Centroidissthe point of intersection
caliedtheanticlockwisefrom OX. the figure, Gis
of atriangle. In

thecentroi
Scantoknow
IMand IV point on
co-ondinates of a moreabout
ABC.
The (x,0)
of the torm thistopic
the X-axisare point on Y-axis A
the
and that of
are (0, y). the
co-ordinates depicts
Sign of it lies. The Basicconcepts
quadrant in which are of the of
co-ordinates of a point
Co-ordinate
quadrant, Geometry
the first
form (+, +) in second quadrant, E
(- +) in the and G
third quadrant
6 ’ in the quadrant. distances
(t,-) in thefourth Care collinearif the
B and distances
Three points A, such that thesum of two
are
AB, BCand CA
the thrd. vertices of an
isequal to and C are the B D
B
Three points A,
= BC = CA.
equilateral triangle if AB the vertices of an isosceles
Centroid divides each median of a
and Care
The pointsA, B BC = CAor CA = AB. of 2:1from vertex to base of
the side triangle
triangle if AB= BCor the vertices of a
right Ifx y,,then (x, y) y, x) andi if (x, y) =(y
and C are
Three points A, B CA2
triangle, if AB + BC= To plot a point P(3, 4) in the cartesian plan:
() A distance of 3 units along X-axis.
distance of 4 units along Y-axis.
(ii) A

f6
A 5
P(3, 4)

3
Cand D:
For the given four points A, B,

^-6 -5 -4 -3 -2 -1 i2 3 4 5 67
D
-2

-3
4
5

-7

B
Key Formulae 123
distance between two points
The is ie., P(z,
V,)and
QryY)

)
Fundanental
Facts
The distance of a point P(x, y) from oriin is (ordinate
of gometry is the system
2ntry where the
points (n the plant n psiticn of
Co-ordinates of point art oxdered paiz of deartei using
divides the line segment by
y) which 12) Cartesian pianeÇ rrrrters
points (r}, y) and (Iy y) in the thejoining wàs disOVEred
by
M:ninternallyare ratio
Rene Drscartes.
(3} The rther
Ihane of
X =
mx, + nx,
zOnetry is Analytical co-ordnate
(4) Ceonetry.
Co-ordinate Ceometry acts as 2bridze
and m +n
between the
5) Mediars of aAlgebra and Geornetry.
i Co-ordinates of mid-point of the line The point of
triargie are concurtent.
segment by joining the points ( y)
and centroid. concurrency is caled the
(yY)are
{6) Trisection of a line segment measts
x=
(s) dividing it into 3 equai
points are requirei. parts, s0 2
and
y-() (7) Centroid of a riangle divides its
median in the ratio of 2:1.

oBJECTIVE TYPE QUESTIONS (1 mark each)


Multiple Choice Questions According to the distance formula,
Q.1.IfA(3, y3), B(0, 0) and C(3,,k) are the d= Nz,-x+(y
of an equilateral triangle ABC, then the three vertices
is
value ofk
AB =
(A) 2 (B) -3
(C) -V3 (D) -2 =|V9+3=2 unts
Ans. Option (C) is correct.
ACBSE, Board Term-I, 2021] BC =|N3-0j -(k-0)j
Explanation: A(3, V3 ), B(0, 0)
vertices of the triangle ABC. and C(3, are the
Now,
= V9+* units
AB = BC

A(3, V3) 12 9+

Q.2. Three vertices of a k=*3


B(-2, 3) and C(5, 8).parallelogram ABCD are
The ordinate of the A(1, 4),
vertex Dis fourth
(A) 8 (B) 9
(C) 7 {D) 6
C3, k) Ans. Option (B) is correct. BCBSE, Board Term-, 20211
B(0,0)
Explanation: Let A(1, 4)
are the vertices of a B(-2, 3) C(5, 8) and Dia, b)
As in the
equilateral parallelogram.
Midpoint of diagonal AC
Then, apply distancetriangle ABC all sides are
formula for sides AB andequal.
BC.
MATHEMATICA

126 O8wAal CBSE


Oucstion Benk
Chantenvise &
Topicwise,
(8TANDARD)
7(y-6y -49) )
v-7y t ly -- 49 0
,
yy- 7) + 1(y - 7) )
(y-7)(y t 1) =0
0.4. The ratio in which y7,1
the
Midpoint of diagonal BD line 3x
theline segment joining
-(,) is
(A) 3:2
the points
-2+a 3+b (C) 3:4 (B) 2:3
2 (D) 4:3
The diagonals of the
parallelogram bisect each
other. The diagonals share same
mid-point.
Ans. Option (C) is correct.
Explanation: Let the
A|CESE,
the point of in
(3, 6) =
-2+a
) M(x, y).

On comparing both sides, we get 3+b


-2+a and 6 =
3= 2
2

ordinate is asked,
In the question value of M
3+b 1
6= A(1, 3)
2
12 = 3+ b
b=9
circle with centre
Q.3. Points A(-1, v) and B(5,7) lie on a
O(2, -3y). The values of y are
(B) -1,7 Let the line 2. divides the line
(A) 1,-7
(C) 2,7 (D) -2, -7 According to the section formula. the ratABin
AOACBSE, Board Term-I, 2021]
M(, y) =
Ans. Option (B) is correct.
Explanation: As points A and Blie on the circle and
O is the centre. (k(2)k+1
+1(1) k(7)+1(3|)
AO and BO will be the radii of the circle.
k+1
2k +1 7k +3
(1,y) k+1 k+1
A
This point M lies on the line à.
Therefore, 3
(2k+1+ 7k+3
k+1 k+1 -9 =0
(2, -3y) or 6k +3 + 7k +3-9(k+1) =0
4k-3 = 0
B
(5,7) Or
4
The ratio is 3:4.
So, AO = BO Q. 5. If A(4, -2), B(7, -2) and C(7, 9) are the vertce
AABC, then AABC is
(A) equilateral triangle
(B) isosceles triangle
(9 right angled triangle
-a-s*(-3y-7| (D) isosceles right angled triangle
AUUCBSE, Delhi Board Ter,:4:"
(Applying distance formula on both AO and BO)
Ans. Option (C) is correct.
Explanation: A(4, -2), B(7, -2) and C(7,9
vertices ofa triangle.
(3)? + (4y)=(3)²+ (-3y -7)° Using distance formula,
9+ 16y =9+9 +49 + 42y
»16-9y-42y -49 =0
7y- 42y-49 =0
DMEAIONS
LANBS gN TWO
).9. The istance between the ptnts tm,

- 0N3+0| 3 units
() 2Vm? +n units ( m 2n snits
DC =

No+11²| -1lunits Ans.Option (C)is correct.

AC - 7-4f-[9-(-2) Toppor Answes 020

- |W3² +12
- |V9+121(=V130 units
Sot.Aftm,n)
equilaterallor isosceles.
Clearly, they are not AC = AB + BC'
Also, it is following Pythagoras theorem.
Which meanright angled triangle.
is a
.. AABC
line
segment joining the points P(-3, 2) and AR urn
. The divided by the Y-axis in the ratio
5,7)is (B) 3:4
(A) 3:1 (D) 3:5
(C) 3:2 ACBSE, Board Term-i, 2021}
AB: 2tn
of an equilateral AABC lies on the
base BC C
are (0, -3). If the
7. The The co-ordinates of
Y-axis. mid-point of the base e BC, what are the
originisthe
B?
co-ordinates of Aand
B(0,3)
(A) A(V3,0), Q. 10. The centre of a circle whose end
paints of 2
A(+3 V3,0), B(3, 0) diameter are (-6, 3) and (6, 4) is
(B)
B(0, 3)
(C) A(+3 V3,0), (A) (8, -1) (B) (4, 7
A(-3,0), B(3, 0)
(D) UCBSE, Delhi Board Term-L, 2021) () (D)
divides the join of (2, -3) and (5, 6) in the
.X-axis
2.8. AC3SE. Delhi &c O0. 2020)
ratio.
(A) 1:2 (B) 2:1
(D) 5:2 Ans. Option (C) is correct.
(C) 2:5
ACBSE, Q.B., 2021-22}
correct. TopperAnse2020
Ans. Option (A) is
Explanation: Let P(r, 0) be a point on x-axis which
divides the join of A(2, -3) and B(5, 6) in the ratio Sol. Af
m:n, thenusing Section formula
A
(2,-3)
11

(3,3) (5,6)
3t9)
22+12

mx6+ nx(-3)
7n+n
6-3It =0
2nn-n Q. 11. The distance between the points (a cos + b sin 8,
21n = 11 0) and (0, a sin 9-b cos 8), is
1 (A) a + units (B) -units
2
(C) Va+ b² units (D) va?-b units
i.e.,
DONICESEL DElhi Set-i 20201
These questions are for practice and their solutions are atilable at the dof the chapter
MATHEMATICS
Topiiwie,
Qucstion (lark Cha:teise &
14. The
Ans, Option (C) s ornt
Co-ordinales
point (-3, 5) in
of the
(A) (3,5)
a-axis
Eylanaton Here.
and (C) (3,-5)
Distance -
Atts. Option (C) is correct.
V{O-s bsin8)+ (asin0 -tOst-0)| Explanation: By
plane, we have
raxis is (-3,-5). (he
using the
N-)°(os8+ hsin @)° +(a sint-beos0)
acose+b'sin0+ 2abcos@sin
reilectim of

ta'sin'@+b ose-2nhsin Bcose " (-3,5)

Na (sin°O+ CUs°0)+ b°(sin°0+ cos 0)


X
-3
= |Na x1+b°x1|= a+h° units.
[CBSE Markitg Scheme, 2020]
joining
Q. 12. If the point PIk, 0) divides the line segment
ratio 1:2,
the points A(2, -2) and B(-7, 4) in the e(3,-5)
then the value of k is
(A) 1 (B) 2 Y
(C) -2 (D) -1
B|CBSE, Delhi Set-l, 2020]
Ans. Option (D) is correct.
CBSE Marking Sch
Q. 15. If the point P(6, 2).divides
the
ratio ine sege
Explanation: A(6, 5) and B(4, y) in the

A(2, -2)
1 (k, 0) 2
B(-7,4)
y is
(A) 4
(C) 2
(B) 3
3:1,hent
(D) 1
1(-7) +2(2)
k=
1+2 Ans. Option (D) is corect. AICBSE, ODS
Explanation:
Here, X =6, y =5
-7+4
k=
3 A(6, 5) 3 P(6,2)
3k =-3
and
k=1
[CBSE Marking Scheme, 2020] Then
Q. 13. The point P on x-axis equidistant from the points m+1
A(-1,0) and B(5, 0) is
and y= W2ty,
(A) (2,0) (B) (0, 2) M+1
(C) (3,0) (D) (2,2)
AICBSE, OD Set-I, 2020] 2= 3xy+1x5
3+1
Ans. Option (A) is correct. 3y +5 =8
Explanation: Let the position of the point P on
3y =8-5=3
X-axis be (x, 0), then
y=1.
PA =PB?
0. 16. The distance of the point P(2, 3) fron thexa
(r+ 1)² +(oj? = (5- + (0
+ 2x +1 = 25+-10x (A) 2 (B) 3
(C) 1 (D) 5 INCERTE
2x + 10x = 25--1
Ans. Option (B) is correct.
12x =24 ’r=2
Explanation: Since y-coordinate of a givea
Hence, the point P(, 0) is (2, 0). the distance of point from x-axis.
{CBSE Marking Scheine, 2020]
LINES (IN TWO-DIMENSIONS)129

P(2,3) A(0,3)
3

O(0,0) BI5, 0)

Distance between the points A (0, 3) and


distance B(5, 0) is
between the points A(0, 6) and
0.17.The
B(0,-2)is:
(A) 6 units (B) 8 units
AB =(5-oj' +(o-)
-|V25+9|
(C) 4 units (D) 2 units
=34 units
correct.
RÊNCERT Exemp.] Hence, the required length of diagonal is v34 units.
Ans. Option (B)is
Explanation: The distance between two points Q.19.The perimeter of a triangle with vertices (0, 4),
(0,0) and (3, 0) is:
(3, y) and (y y) is given as,
(A) 5 units (B) 12 units
(C) 11 units (D) 7+V5 units
Where, x =0,y = 6and x, =0, y, =-2 RNCERT Exemp.]
So. distance between A (0,6) and B(0, -2): Ans.Option (B) is correct.
Explanation: Since, Perimeter of triangle = OA +
AB-(0-0' +(2-6| AB + OB

A(0, 4)

=8 units
0.)AOBC is a rectangle whose three vertices are O{0, 0) B(3,0)
vertices A(0, 3), O(0, 0) and B(5, 0). The length of
its diagonal is:
(A) 5units (B) 3 units
(C) V34 units (D) 4 units
Here OA = 4units, AB = 5units (using pythagoras
U[NCERT Exemp.] theorem in triangle AOB) and OB =3 units.
Ans. Option (C) is correct. Therefore, Perimeter of triangle = 4+5+3
Explanation: According to the question, a triangle = 12 units.
can be represented as:

SUBJECTIVE TYPE QUESTIONS


m=1,n =2
Very Short Answer Type Given, (*y) = (8, -9)
Questions (1 mark each)
.In which quadrant lies the point which divides (y y) = (2, 3)
the line segment joining the points (8, - 9) and
(2,3) in ratio 1: 2internally ?
AOMCBSE SQ 2020|
Sol. 1V qquadrant. (CBSE SQP Marking Scheme, 2020j ( y) = 1x2+2x8 1x3+2x(-9) |
Detailed Solution: 1+2 1+2

1 |2+16 3-18|
2
A(8 -9) P(*y) B(2,3)
130 Oswaal CHSE Queatiun ak Chapterwiae Atiptewiae,
MATHKMATIU#(WAMDAH)
Deatlei hulutioi:
(1, v)

(, y) (6, 5)
Hence, the point (6, 5) lies in IV quadrant,
a 2. Find the coordinates of apoint A, where AB la
diameter of acircle whose centre in (2, -3) and n
the point (1, 4).
ODOACBSE DelhiSet-1, 1, n, 20191
C3The istance between point A(5, -3) and B(13, m)
is 10 units. Caleulate the value of n,
ICBSE Delht Board term, 2019)
lere Dis the oid point of f
Then, the coordinaten of )
Topper Anewer, 2019
Sol.

-|/%|-|49
Hence, the length of AD is 37 unit,
0.6. 1f the distance between the points (4, .
(1, 0) Is 5, then what can be the possible
v.
k?
OUDelhi Set S,I, i
Cmeldsing omey paitie yabuj Short Answer Type
Questions-I
Q.1. Find the polnt on X-axis which is equidisa i
the polnts (2, - 2) and (- 4, 2).
0.4. Find the value of a, for which point P2| is the AOICESE SQP
Sol. Let P(x, 0) be a point on Xraxis
midpoint of the line segment joining the points PA PB
Q(-5,4) and R(-1, 0). ACBSESO, 20181 PA PB?
Q.5) A(5, 1), B(1, 5) and C(-3, -1) are the vertices of
AABC. Find the length of median AD, (r- 2)² + (0 +2) (r +4)' +(0-2?
t 4- 4x +4 + 16 + 8x + 4
CBSE Compt. Set-I,11,11, 20018|
4x + 4 8x+ 16
Sol. Coordinates of Dare (-1,2)
Hence co-ordinate of required point are (-,
A(5, 1) Q. 2. P(-2, 5) and Q(3, 2) are two points. Find tb
ordlnates of the point R on PO such that *
20R. DOCBSE SQE .
Sol. PR:QR 2:1
R(, y)

B(U, 5) (
C(3,-) 2+1 2+1

AD 5+1Ý +(-2)| 37 unit


(CBSE Markingiehome, 20)18} |CBSP Masking Scthens, 242
These quntjons are fw prutte nnd Iheir solutinse are amlable ut th end of the lupter
LINES (!N TWO-DIMENSIONS; 131

esiled
Solution:
twopoints:P{--2, 5) and Q3,2)
1 Soi. Lgt
Hen
-10+ 15

Schene,2018j
r-25) [CBSEMarking
Rx. y) Detailed Solution:
Q3.2)
2
2:3 (3,-5)
3 (-2.5}

Since,

and
Y
2x3+3x(-2-0
PR = 2OR 2+3
andalsogiven, PR 2 2x(-5)+3x5
1 and
QR 2+3
PR:QR =2:1 So, the point is (0, 1).
Q.5. Find the ratio in which P4. ) divides the lin
Then R(r. y) = segment joining the points A(2, 3) andB(6,
Hence find m. + ACBSE Delhi 0D Set-2018]
2x3+1N(-2) 2x2+1x5)
2+1 2+1
Sol. A¢
(6,-3 )
(23) P(4 m)
-
Let 4P: PB k:1
=4

and k=l, ratio is I:1


0.3.In parallelogram ABCD, A(3, 1), B(5, 1) C(4, b) -3-3
D 3) are the vertices. Find vertex Cla, b). Hence
ACBSE Board Term, 2019]
[CBSE NIarking Schene, 2018]
TopperAnswer, 2019 Detailed Sclution:

Topper Answer, 2018

QFithendjointhe ofcoordinates of ithe point P which divides


A(-2, 5) and B{3, - 5) in the ratio2:3.
ACBSE SQE 2018)
Question Bank Chapterwise &Topicwise, MATHENATICs (STANDARD), Clase. v
132 OswaalCBSE

Q.6If (1) is the mid point of the line segment Topper Answer, 20t
Sol.
joining the points (2, 0) and ("3). then show

dislante formula,
that the line 5x + 3y + 2 = 0 passes through the
point (-1, Sp). AACBSE SQT 20171
Sol. Since
(3) isthe mid point of the line segment
joining the points (2. 0) and
Hence, petd.

3 ine intersects the Y-axis and X-axis at thes


0.f0 A
Pand Qrespectively. If (2, -5) is the mid-po
PO. then find the coordinates of P and 0.
UCBSE OD, Set-L
Hence, the line 5r +3uy +2= 0,passes through the
point (- 1, 1) as 5(-1) + 3(1) + 2 = 0. 1 Topper Answer, ROI7
[CBSE Marking Scheme, 2017]
Q.z in what ratio does the point P(- 4, 6) divide the Sol.
line segment joining the points A(- 6, 10) and
B(3, -8) ? A Delhi Comptt Set-I, II, III 2017]
Sol. Let AP:PB =k:1
3k-6 =-4
k+1 let coonna k efp be o y) ang of & be
OI,
3k-6 =-4-4 (a, o).
AC 2,-s) is*md poinl of Pa
or, 7k =2 sechon fomla,

and - 5
AP:PB =2:7
Hence,
{CBSE Marking Sctem, 2017] and y -J8.
points A(2, 1) and Pis (o, -10) ond & is (410)
Q.8. If the line segment joining the andQ find the
P
B(5, - 8) is trisected at the points 0. 11./The r-coordinate of a point P is twice is
coordinates P. y-coordinate. If Pis equidistant from Q(2, -5) and
2017]
AOutside Delhi Compt. Set-i,B IIL, R(-3, 6), find the co-ordinates of P
Sol. Q (5,-5) CBSE, Delhi Set I L, IL 21
(2, 1)
ratio 1:2. 1 Sol. Let the point Pbe (2v, )
Let P(x, v)divides ABin the Given, PO = PR
.. Using section formula
1x5+2X 2
1+2 Solving to get y = 8
1x-8+2x-2 1
Hence, coordinates of point P are (16, 8).
and 1+2 [CBSE Marking Schente, 2018
Detailed Solution:
are (3, - 2).
Hence cOordinates of P Let the point P be (2y, v)
P(r, y) from A(5, 1)
and B{-1, 5) Given,
Q.9. If the distances of that 3r = 2y. PQ = PR
prove
are equal, then 2015)
UCBSE OD, Set-IL, 2017,
LiNES UN TWo-DIMENSIONS)
Squaringbothsides
+ y + 25 + 1Oy
Sol.Let A(3, 0), B(6,4) and C-4,3)9+ 16= 25 unts
4-8y + 4 =4y +12y +9 +y + 36- 12y .. AR (3 - 6 + (o- 4
49 + 1 5) urats
2y = 16 or y = 8 BC = (6 4 1 (4-3 16 +9= 25 uríts
coordinates of point P are (16, 8). and CA (-]-3 + (3-0
Hence, which Y-axis
Findthe
ratio in divides the AB² = CA or, AB = CA
12 segment joining the points A(5, -6) and
Q line Also find thee co-ordinates of the point of .. Triangle is isosceles.
B-1,-4). ACBSE, Deihi Set I, I1, T(i, 20161 25 + 25 = 50
division. Also, AB? + CA = BC
Y-axis be (0, y) ) and AP: PB = k:1.
thepointon
Let
Sol.
5-k
Ç-1,3)
Therefore
= 0 gives k = 5
k+1
ratio is 5:1
Hence,requred
-4(5) -6 -13
6 3 A(3,0) B6,4)

Hence, pointon Y.axis is (0} Since, Pythagoras theorem is


verified, therefore
Hence Froved
[CBSE Marking Scheme, 2016] triangle is a right angled triangle.
[CBSE Marking Scheme, 2016]
point P(x, y) is equidistant from the points
0.13. Ifthe b, b- a) and R(a b, a + b), then prove that Q. 15. If the mid-point of the line segment joining
Q(a + [OD,Set I, II, III, 2016] find
bx= ay. and B(r + 1, y - 3) is C(5, - 2),
AD ACBSE SQE 2016]
UICBSE Board Term-2, 2016]
|PQ| = |PR|
Sol.
Nx-(a+b)) +ly-(6-a)' Sol. At mid-point of AB = =5

- Nr-(a-b)° +[y-(b+a) Or, x = 6 1

Squaring, we get
[-(a + b)j² + [y-(b-a)j?
= (x-a + by + (y-b-a)?
(y++y-3 2
=-2

or, (x-a-b<- (x-a + b] y+ 1+ 2y-6 =-8


= (y-a-b-(y- b+ a) .CBSE Marking'Scheme, 2016]
P (< y)
Short AnswerType
Questions-II 3marks ecch)
Q.1.Ifthe point C(-1, 2) divides internally the line seg
ment joining A(2, 5) and B(¢, y) in the ratio 3:4,
find the coordinates of B.
R ADRCBSE Delhi Set-I, 20201
(a + b, b-a) (a -b, a + b)
Sol. By using section formula,
OT, (x -a -b+x-a + b)(*-2-b-+a-b) A(2, 5) C(-1,2)
=y-a-b+y-b + a)(y-a -b-y+ b-a)
(2x-2a)(- 26) =(2y-2b)(- 2a)
(x-a)b = (y-b)a 3x+§
bx = 4Y. Hence proved. 1
(CBSE ariking Scheme, 2016}
Q.14.Pvertices
Prove thatof aright angledisosceles triangle.
the point (3, 0), (6, 4) and (-1, 3) are the
CBSE, OD Set L, ), II, 3016]
134 OnwanlCHSE Oucstioy Bank Chpterwisc R Topicwise, MATHEMATICS (8TANDARD),Class-X

and
ket the
3xy+4 x5 3y +20
3+4 7
A(ay4)and B(--)
3y +20 14
3y m14 - 20
he-f(o,
s 2
Hence, the coordinates of B(x, y) is(-5,-2). -2k+6
[CBSEMarking Scheme, 2020] k+)
-7k
Q. 2. If the mid-point of the line segment joining the
points A(3, 4) and B(k, 6) is P(, y) andx +y-10=0,
find the value of k.
AA + U|CBSE, OD Set-1, 2020]
Snl. Since, the mid point of A and B is P
A(3, 4) P(,y) B(k, 6)
3+k Ratio n. which yais dides. AB 3:)
2

4+6 10
Y-73)-4
and 5 1 (3)+1
2

Also, given, x +y- 10 0


3+k
Substituting the value ofx a and y 5, we

get
fourt.o intersccdien
34k
45-10 ) r(
3+k cqeent.

34 k 0 Q.4. The line segment joining the points A(2, 1) :


B(5, -8) is trisected at the points P and Qsuch th
k e 10--3 =7. P is nearer to A. If P also lies on the line given
Hence, t)he valuc of kis 7. 1 2r-y+ k=0,find the value of k.
|CBSE Marking Scheme,2020) AOACBSE Delhi Set-, , 1I, 01
(.3. Find the ratio in which the y-axis divides the line Sol. AP: PB |:2
segment joining the points (6, - 4) and (-2, - 7).
Also find the point of intersection.
A+UjCBSE, Dellhi & OD, 2020]
A(2,1)
Topper Answer, 2017

4+5 2-8
3 and y
B(-2, ) 3

"Thuspont P is (3, -2).


I'oint (3, -2) lies on 2xy k)
6+2+ ()

/CBSE Marking Sheme 2


LINES (IN TWO-DIMENSIONS) 135

the segment
ailed
Solution:
SOL. Let the linex-3y = )intersect
0= k:1, B
A
(-2, -5) (6, 3)
x-3y = 0
ratio k:1
joining A(-2, -5) and B(6, 3) in the
6k- 2
P(r, y) B(5,-8) .. Coordinates of P are
A2,1) k+1 k+1

6k-2 =3
Plies on x 3y =0
k+1
points are A(2, 1) and B(5, -8).
Thegivern Qtrisects the line segment AB k=
13
Given,P.and 3
AP PQ= B
PO+ QB = AP+ AP = 2AP .". Ratio is 13:3
PB
AP: PB AP: 2AP =1:2
:Pdividesthe line segment. AB in the ratio 1:2.
’ Coordinates of P are
G) 2019]
knowthat, coordinates of the point dividing the
[CBSE Marking Scheme,
We points (X1 y)and (Iy y) in
segmentjoiining the Detailed Solution:
line the lite
2 1 my2 t ny, Let the ratio in which line x-3s = 0 divides
given by
the ratio m:nis M+ segment is k:1
Coordinates of P

(1x5+2x2, 1x(-8)+2x1)
1+2
1+2

5+4 -8+2 1

-( A(-2, -5) P(r, y) B(6, 3)

-()-0) /x-3y=0
Point P(3, -2) lies on the given line 2x-y + k=0
2 x3-(-2) + k = 0
6+2 +k = 0 Using section formula, we get
8 +k = 0
kx6+1x(-2)
k+1
k = -8
Thus, the value of k is 8. 6k-2
k+1

Comnonly Made Error kx3+1x(-5)


and 1/ =
k+1

Some Candidates use mid-point formula instead 3k -5


of section formula to find the coordinates of P ...(ü)
k+1
Some candidates also make calculation errors.
The point P(r, y) lies on the line, hence satisfies the
equation of the given line.
Answering Tip
Understand the concept if a point divides a line
Segment in the ratio n:n (m n) then m:n 1s
6k-2
k+1 -) = 0

not equal to n : m. 6k-2-3(3k 5) = 0


6k 2-9k + 15 =
Q. Find the ratio in which the line x-3y =0 divides
the line segment joining the points (2, - 5) and -3k + 13 = 0
S). Find the coordinates of the point of 13
k=
intersection. A[CBSE OD Set-i, II, III, 2019] 3
136 Oswaal CBSE Question Bank Chapterwisc &Topicwisc, MATHEMATICS (STANDARD), Clas..
Hence, the required ratio is 13 :3
Now, substituting valueof kin xand y, we get . Mid-point of diagonal AC

6x
13
13

+1
-2
()-()
Mid-point of diagonal BD
3 (a+1 0+2
78-6
16 Mid point of diagonal AC= mid point
72 1 241
ot diagn
16 2
and 1+b
9 2= a+ 1 and 1
a =1 and
+tba
Again, AB=(-,)+y
and
y= 13
3
+1 =N+2Y +(0-1|
8x3 AB =9+1=10 unit
16

24
16
3
2
and
BC= (*,-x+b
Hence, the coordinates of point of intersection - N(4-1f+(1-of
P(x, y)
BC= |N9+1
Q.6. If A(- 2, 1), B(a, 0), C(4, b) and D(1, 2) are the = V10 unit
vertices of a parallelogram ABCD, find the values
of a and b. Hence find the lengths of its sides. ABCD is aparallelogram (Given)
AB = CD = y10 unit
ACBSE Delhi/OD, 2018]
Sol. Given, ABCD is a parallelogram and and BC = AD= V10 unit
diagonals AC and BD bisect each other Q.7. The points A(1, - 2), B(2, 3), C(k, 2) and D(-4
are the vertices of a parallelogram. Find the val
D of k.
C
/(4,b)
C+U [CBSE SQP 2M
(1,2)/ Q. 8. Find the co-ordinates of the points which divid
the line segment joining the points (5, 7) and (8,1
in 3 equal parts. [Board SQP 201=
ACBSEOD Comptt. Set-i, 201
0.9. Show that AABC with
and C(2, 0) is similar vertices
B
(-2, 1) (a, 0)
A(- 2, 0), B
to ADEF with vertice
D(-4, 0), E(0, 4) and F(4, 0).
Therefore, mid point P of BD is same as mid point ABoard Foreign Set-i, II2!
of AC CBSE Deihi Set-L, II II, 01
Sol. Using distance formula
(2+4,
22 1+b)
a+1 b+1
AB - No+2 +(2-0 -|Va*
=1and
2 2 = 2N2 units
are i0
’a=1, b= 1, Therefore, length of sides BC =
½+1
units each.
= 2V2 units
CBSE Marking Scheme, 20181
Detailed Solvtion:
parallelogramn bisect each
CA - N-2-2) +(0-of -/T
We know that diagonals of 4 uIils
other.
of the chaptr
practiceand their solutions are available at the end
These questions are for
LINES (IN TWO-DIMENSIONS) 137
and
DE -N0+4' +(4-o - Wa2 Or,
-+y 9 or, y' 9-.
4V2 units
Or,
27
or y=t 3/3
EF =N4-0 +(0-« - |Vsa 4 2
(+ ve sign to be taken)
= 4/2 units
Hence, C:
FD -N4-4 +(0-o - |Vea
=8 units
Long Answer Type
of the
Cuestions (5 maks each)
Since, ratio corresponding sides of twO Q.1. Findthe ratio in which the Y-axis divides the line
similar As is equal.
AB BC AC segment joiningthe points (-1, 4) and (5,-6). Also
iLe., DE EF DF
find the coordinates of the point of intersection.
AO ACBSE OD Set-II, 20191
2/2 2V2 =
4 Q. 2. If P(9a - 2, - b) divides the line segment joining
of, A 4/2 8 2
A(3a + 1, 3) and B(84, 5) in the ratio3:1. Find the
values of a and b.
AABC ~ ADEF Hence Proved. AO ACBSE SQP 2016]
Sol. By section formula
[CBSE Marking Scheme, 2017]
0.10.IIn the given figure AABC is an equilateral triangle 9a-2 3(8a) + 1(3a + 1)
3+1
of side 3 units. Find the co-ordinates of the other
two vertices. A [Foreign Set-I, II, 2017] and -b= 35) +1(-3) ...(iü) 1
[Foreign Set-III, 2015) 3+1

Y
From (ii),
C(*,y) 15-3
-b= =3
4

b=-3

From (i), 9a -2 = 24a +3a+1 1


X +X 4
(2, O) (5, 0) 4(9a - 2) = 27a + 1
36a 8 = 27a + 1
9a - 9

[CBSE Marking Scheme, 20i6]


Sol. The co-ordinates of B are (5, 0) (AB = 3) Q3)The base BC of an equilateral triangle ABC
Let co-ordinates of C be (x, y) lies on Y-axis. The co-ordinates of point C are
(0, - 3). The origin is the mid-point of the base.
Since AC? = BC?
Find the co-ordinates of the point A and B. Also
(sides of equilateral triangle) findthe co-ordinates of another point D such that
(I-2) + (y -0 = (x - 5) + (y -0) BACD is a rhombus, A (Foreign Set I, I, 2015}
OT, x+ 4-- 4x + ? =+
25--10x+ y Sol. Co-ordinates of point Bare (0, 3)
6x = 21
.. BC = 6 unit
7 Let the co-ordinates of point A be (t, 0).
1
2
And (x ~2 + (y -0 AB =
=9
+y = 9
e
questigns are for Practtce and their solutiois are auailabie at the gnd of the chapter
Oswaal CBSE Question Bank Chapterwise &Topicwise, MATHEMATICS (8TANDARD), Clase.y
138

Y AB

X 1 /26 unit

C(0,-3)
BC =N5-0' +(3-4

Or, = 27 o, x=3/3
Co-ordinates of point A (343,0) 1

Since ABCD is a rhombus. = 2/13 unit


AB = AC=CD = DB AB = BC+ AC
Or,
.Co-ordinates of point D= (-3W3,0). 1 or, AABCis isosceles.
Now, using mid-point formula, the co-ordinate
[CBSE Marking Schene, 2015] mid-point of BC are
Q.4.(1, -1), (0, 4) and (- 5, 3) are vertices of a triangle. -5+0
Check whether it is a scalene triangle, isosceles 22
triangle or an equilateral triangle. Also, find the
length of its median joining the vertex (1,-1) the 3+4
mid-point of the opposite side. y=

A[CBSE, Term-2, 2015]


D(x, y)
A(1,-1)
Sol. .:. Length of median, AD

S,3) D B(0,4)
|130 V130 unit
4 2
Let the vertices of AABC be A(1, -1), B(0, 4) and
C(5,3). 1 :. Length of median AD is V130
units.
. Using distance formula, 2

{CBSE Marking Scheme, 2013)

COMPETENCY BASED QUESTIONS (4 marks each)

Case based MCOs A


D

LRead the following and answer any four questions


from Q.1. to Q.5.
The diagram show the plans for a sun room. It will B Tup view
be built ontothe wall of a house. The four walls of A

the sun room are square clear glass panels. The roof RS
is made using, Not to ale

"Four clear glass panels, trapezium in shape, all of


the same size ront vie!
Scale: 1(m 1ir
"One tinted glass panel, half a regular octagon in
shape
AdcBSE SOP 2020-21
LINES (IN TWO-DIMENSIONS) 139
find the
Refer
to
Top View, mid-point of
points J(6, 17) and I(9, 16).the seg- ground ABCD, 100 flowerpots have along been placed
AD, as
joiningthe
nLmentt 3 1 at a distance of 1 m from each other
Niharika runs 1/4"
33 15
(B) 32 Shown in given figure below. posts a green
(4) 2 2 the distance AD on the 2nd line and
13 on the eighth
(G) flag. Preet runs /5th distance AD[CESE QB }021|
1533
(D) 2'2 line and posts a red (R) flag.
22
correct. D
() is
Option
insEiplanation: Mid-point of)J(6, 17) and I(9, 16) is
6+9andy =
17+16
2
2
33
15 and y =
R
front View, the distance of the point P
Reterto
02 Y-axisis:
fronmthe (B) 15
(A) 4
( )19 (D) 25
correct.
(A)is
Ans.Option
Exylanation: The distance of the point Pfrom the
Y-axis= 4. A 1 2 3 4 5 6 7 8 9 10 B

to front
view, the distance between the
0.3Refer andS is
pointsA Q.1. Find the position of green flag.
(B) 8
(A) 4 (B) (2,0.25)
(D) 20 (A) (2,25)
() 14 (C) (25, 2) (D) (0, -25)
is correct.
Ans.Option(C) A's coordinates = (1,8) Ans. Option (A) is correct.
Evvlanation:
Sscoordinates = (15,8) Explanation: From the given figure, abscissa = 2
ordinate = of 100
Then,
AS = |N(15-1 +(8-8) and

= 14. = 25
co-ordinates of the
0.4. Refer to front view, find thesegment joining the So, required point = (2, 25)
point which divides the line internally.
points Aand Bin the ratio 1:3 Q. 2. Find the position of red flag
(A) (8.5, 2.0) (B) (2.0,9.5) (A) (8,0) (B) (20, 8)
(9) (3.0,7.5) (D) (1.75, 8.5) (C) (8, 20) (D) (8, 0.2)
Ans. Option (D) is correct.
Ans. Option (C) is correct.
Explanation: The coordinates of A = (1, 8) Explanation: From the given figure,
The coordinates of B = (4, 10)
m=1 and n=3 absissa =8
Also,
1x10+3x8)
and ordinate =-th
5
of 100
then, 1+3 1+3
1
=-x100 =20.
-6) 5
So, required point = (8, 20) both the flags ?
- (1.75,8.5) between
equidistant 0.3. What is the distance
Q.5Tom
.Refertheto Q(9,
front view, if a point (, y) is
8) and S(17, 8), then (A) V41 (B) V11
(A) x+y = 13 (B) x-13 = 0 (C) V61 (D) V51
(9 y-13=0 (D) x-y 13 correct.
As. Option (B) is Ans. Option (C) is 25).:
correct. Explanation: Position of Green flag (2,
Erplanation: Let point be P(, y) Position of Red flag = (8, 20)
PQ = PS2 Distance between both the flags
0r,
(-9 +(y-8² =(-17) + (y-8)
X-13 = 0
IM. Read the following text and answer any four
= 36+25
uestions
In order tofrom O.1. toSports
conduct Q.5. DDay activities in your
J61 units
School, ines
at a distance have meach,drawn
of 1 been in a rectangular
powder
with chalk shaped
O 4, / Rashmi has to perst a blue flag, eIsctly halfway
befween the line egnent jining th twa fi,
where should ehe pnt her flag ?
(C) (2, 4.5) (D} (2.5, 2Ai)
Ans, Option (A) is ttteet.
xplanatlon: Psitiofs of biue flay
13-5f fis-2f
Mil oint of line sgroent froning fhe gyee #nd
relflags
-(2",
(5, 22.5)
Q. 5, Joy has to post a flag at onefnurth distance fron
preen flag, in the line segmnent joining the green
and redflags, then where should he post his flag ? 0. 2. (i) What is the trtal distarce travelled
(A) (3.5,2A) (B) (0.5,12.5) to resch the sffice?
(C) (225A5) (D) (25,201) (ií) What is the etra distance travelled b
Ans, Option (A) is correct. Sol. (i) Distarce betmeen daughter's scsi
Lxplanathon: osition of Joy's flag
Mid-pointof line spnent joiring green and =(13-13j +(26-14}
blue flags
-
|2+5 25 +22.5
= 12krn
(3.5, 23.75| -- (3.5, 24] Total distance (House + Bank + Schoi
travelled =5+ 10+ 12 = 27 km
Case Based Subjective (ii)Extra distance travelled by Aayush in r .
office = 27 - 24.6 = 24 km.
Questione 0.3. What is the distance between house and o
L.Read the following, text and answer any four Sol. Distance between house to office,
questions from Q1 to 05.
Aayush Starts walking frorm his house to office. V13-2' +(26-4
Instead of going to the office directly, he goes to a
bank first, from there to his daughters school and
then reaches the office.
(Assume that all distances covered are in straight - W121 +484
lines). If the house is situated at (2, 4), bank at
(5, 8), school at (13, 14) and office at (13, 26) and =|Va05
COordinatesare in km. = 24.59
= 24.6 km
(13, 14) II Read the following text and answer the bes
(S, B) Daughter's questions:
Bank Khool
In a room, 4 friends are seated at the points A
House i
and D as shown in figure. Reeta and Meetz va
into the room and after observing for a few ms
Office Reeta asks Meeta.
(2,4) (13, 26)
10

0.1. Find is the distance between (1) house and bank,


(i) Daughter's school ? B
7
Sol. Since,
Distance between two points (x, yi) and (y ya)
5

(i) Now, distance betwcen house and bank,


2

?8 10
4 5
Colunns
141
LINES(IM TWO DMENIONSj
position of Aand D?
tisthe x = 3 and y=4. (9): Equation of ineC) equation of line thray
liesin (9,4) and D(6, 1)
the
.:A(34)is correct position.
t = 6 and y = 1. i.e, )
liesat
P i n tD
positionoof Dis (6, 1).
the
Correct
So i sthe distance beetweenAand B? y-4-9
What
a0 equationof(line CD ? I-y-5 ).
Whaltisthe
ofA.=(3,4)
Possition Q.3. What is the middie positlon of BandC
sol bB=(6,7) Sol. Position of B (6, 7)
Positionof
Distance of AB= and position of C (9,4)
(6+97+4)
. Mid-point of BandC| 2

= V18 =32 unit. 1

Solutions for Practice Questions


MtoleChoice Questions |36|
correct. BC = AB
(D)is Also,
Ans.6:Option
Explanation: Let the point on Y-axis which
divides the line PQ is
M(0, y) and the ratio be NP+9] =|V36|
k:1. formula, =27
Accordingto the section or

M(r, y) = m+ m Coordinates of Aand Bare (4343, 0) and (0, 3)


respectively.
5k +(-3) k(7)+1(2) Very Short Answer Type Questions
M(0, y) = k+1 k+1
Sol. 2: Let the point Abe (x, y)
On comparing, we get x+1 =2and * 3
5k-3 2 2
0= x=3 andy -10
k+1
5k-3 = 0 . Point Ais (3, -10)
3 (CBSE Mrkin, Schene, i )
Or k=
5 Detailed Solution: the mid
Ans.7:Option (A) is correct. Since,AB is the diameter, center Cmust be
Erplanation: point of the diameter AB.
Y

B(0,3)
A(, 0) B
X O0, 0) 1,4)
(2, -3}
C(0, -3)

).
Ois the midpoint of the base BC Let the co-ordinates of point Abe (z,
Le, Ois the midpoint of Band C(0,-3) x-C00rdinate of
Iherefore, coordinates of point Bis (0, 3)
So, BC =6units.
2
Lettheecoordinates of fpoint Abe (z, 0).
Using distance formula, 4 +1

AB =
and y-coordinate of
MATHEMATICS(STANDARD), Clas5-X
Topicwise,
Chapterwisc &
142 Oswaal CBSE Question Bank

=y+4
Commonly. Made Error
Mostly candidates do not 15e the
is (3, -10).
Hence, coordinates of point A formula. Generally, they use distarce
Sol.4:/St() 4+0)
2 9)-(6) 1
Answering Tip
3 2 Candidates should Use mid-point
Scheme, 2018] which they can get correct
(CBSE Marking
time.
solution
Detailed Solution: P
R

(-1,0) A+
+

(-5, 4) (82) Sol. 8:


(5,)
trisect AB.
Pis the mid-point of QR Let P(x, y) and Q(* V)ratio 1:2
-5+(-1) .. P divides AB in the
:|
Or, 2 1(8)+2(5)
3

3 2
and
1(10)3+2\71-8
OI,
.. P(6,8) of PB.
Sol. 6: Using distance
formula, And Qis the mid point 6+8
=7
2
8+10
3² + 1= 25 /1 2
Or,
k= t4
Scheme, 2017] .. Q(7,9)
(CBSE Marking [CBSE Marking Scheme
Questions-Il
Short Answer Type Long Answer Type Questions
Y-axis is P(0y)
Sol. 7:
Ca)
Sol. 1: Any point on ink:!
Let P divides AB
D(43)

k:1

P0, y) B(5, -6
A(-1,-4)
B2,3)
A1.-2) 5k-1
0 =
k+1
bisect each other %
Diagonals ofparallelogram 1
k=ie, l:5
ofBD
midpoint of AC = midpoint 5

1
()-() 1
-6k--4
k+1
5
+1
5
1+k -2
2 2 -)[CBSE Marking Schene,
’ Pis(0, 3
k=-3 3
2018}
CBSE Marking Schete,
LINES (IN TWO-DIMENSIONS) 143

REFLECTIONS
works on coordinate
Coordinate Geometry describes the link between the location and time also
geometry and algebra through graphs involving geometry.
find the distance between two
Curves and lines. Will you be able to handle such (c) Will you be able to
points easily ? in
problems?
concent (d) Will you be able to find the ratio of dividing lines
regulated by this ?
b) Air traffic is managed and information about m:nratio in complex questions
mid-point of a line ?
andGPS which provides precise
(e) How will you find the
SELF ASSESSMENT PAPER-3

Maximum Tine: 1 hour

I. Chose the correct alternative from the given aptions.


0.1. The distance btween the poánts (s 5, sin 9) and (sin b, - s9) is
(A) 3 uráts (B) 2 urits
(C) 2 urits (D) iunit
0.2. !fthe distance between the points (4. p) and f1, 0) is 3, then p=
(A) t4 (B) 4
(C) -4 (D) 0
Q.3. Thernid-point of P(-5,0)and Q5,0) is:
(A) (0,5) (B) 5,5)
(C) (1,0) (D) (0,)
Q.4. The point on the Y-axis which is equidistart frorn the point A(6, 5) erd -4,3); is
(A) (0,9) (B) (0,5)
(C) (9,0; (D) (5,0)
Q.5. The coordinates of a point which divides the line segnent joining the points (6,3) 2nd ( , 5), in te
internaly is:
(2 21
(A)3 5 (B)

( 21
(C) (D)

Q.6. If the coordinates of the triangie whose vertices are (3, 2), (5, 4) and (1, 3), then the centroid of the trierdez
(A) (3,5) (B) (3,3)
(C) (5,5) (D) (2, 2)
H. Case-Based Ouestion
Q.7. Read the following text and answer the given belovw questions it:
In aroorm, 4friends are seated at the points A, B, Cand Das shown in figure. Reeta and Meetawaik ine
and after observing for a few minutes Reeta asks Meeta.

1 2 3 4 5 6 78 9 10
Columns
LINES (IN TWO-DIMENSIONS) 145
carefiully the above paragraph and:answer any four
Ked positionofA 2 questions:
whatisthe
(4)
(43) (B) (3,3)
(934) (D) None of these
is
the middle position of Band C?
What
iD
(B)

(D) None of these

of D?
Whatisthe position
i) (B) (0,6)
(4)(6.0)
( )(6,1) (D) (1,6)
distancebbetween Aand B?
Whats the
i) (B) 2/3 unit
(A)3/2 unit
(C) 2V2 unit (D) 3/3 unit
between Cand D?
Whatisthe distance
(4)V2unit (B) 2/2 unit
()3/2unit (D) 4N2 unit [1x3 =3]
Answer Type Questions which ABis
1 VeryShort 5
x AB. What is the ratio of line segment in
=
such that BÌ 7
locate a point Qon line segment AB
To
0.6.
divided? the value of p.
points (4, p) and (1, 0) is 5, then diagonal.
04lfthedistance between the
are A(0, 3), O(0, 0) and B(5, 0). Find the length of its [2 x3= 6]
three vertices
o10.AOBC is arectangle whose X-axis.
INShort Answer Type
Questions-I
joining the points (6, 4) and (1, -7) is divided by the but not asquare.
line segment is arhombus
01.Find the ratio in which the S(-3,-2) be four pointsin aplane, show that PQRS a-2b = 18, find the value
of k
Q(3, 4), R(-2, 3) and 6) and B(k, 4) and
Q12.I P2,-1), the points A(10, -
the mid-point of the segment joining [3 x 2 = 6]
Q13.If(a,b) is
the distance AB. AABC
and
Questions-II - 5). Draw the figure and prove that
VShort Answer Type -1), B(- 4, 6) and C(-3,
co-ordinates of vertices of AABC are A(1, vertex.
14 The
which contains the origin, find the third
is ascalene triangle. triangle [1 x5 =5]
(-3, 2) are two vertices of an equilateral
0.it(3, 2)and AP IfP lies on the line
VLong Answer Type Questions 8) such that
and B(9, BP 1
segment joining the points A(-1, 3)
Q16. Point Pdivides the line
I-y +2= 0, find the value of k.
OSWAALCOGNITVE
LEARNINGTOOLS
Finished Solving the Paper?
!
Time to evaluate yourself

OR
SCAN THE CODE

For elaborated
GEOMETRY
UNIT IV

CHAPTER

TRIANGLES

Definitions, examples, counter examples of similar triangles.


E Syllabus 1. (Prove) If aline is drawn parallel
to one side of a
triangle
e
the other two sides in distinct points, the other two sides areto intr
the same ratio. divid
2. (Motivate) Ifa line divides tuo sides ofa triangle in the same ratio, the line is parallel to the t
3. (Motivate) If in two triangles, the corresponding angles are equal,
their
proportional and the triangles are similar: corresponding
4 (Motivate) If the corresponding sides of two triangles are proportional, their corresponding anga
equal and the two triangles are similar.
5. (Motivate) If one angle of a triangle is equal to one angle of another triangle and the sides incha,
these angles are proportional, the two triangles are similar.

In this chapter you willstudy


Similarity of Triangles
Criteria for similarity of Triangles by AAA, SSS, AA, SAS Criterian.
Basic proportionality Theorem or Thales Theorem.
Converse of basic proportionality Theorem.

Revision Notes

Atriangle is one of the basic shapes of geometry. It is (i) their corresponding angles are equal
apolygon with 3sides and 3vertices/corners.
If AABC and ADEF are similar,then this s
Two figures are said to be congruent if they have the can be written as AABC ~ ADEF.
same shape and the same size.
Those figures which have the same shape but not Criteria for Similarity of Triangles:
necessarily the same size are called sirmilar figures.
Hence, we can say that all congruent figures are
sirnilar but all similar figures are not congruent.
Similarity of Triangles: Two triangles are similar, if:
(i) their corresponding sides are M N Q
proportional.
TRIANGLES 147
=<D
ZA
DE&
LE,
ratio
ADEF
= same
ZB equal
CA FDZD, ~
AABC~ADEF AC, AABC arethe
DF
AC CE AABC
~
APQR
in
angles
BCEF = ABDE are
EFAABC-~ADEF
henrnE A then,
BC sides
If Corresponding
DE then, including
these
AB triangleare
,AB one angles
are
proportional, Corresponding
-If a similar.
(SAS
Criterion)
of triangles
to
angle
other
equal
D
triangles,. Similarity
the, oneis the two (i)
are
proportional
then sides B
sidesof) angles criterion)If triangle (ii)
of the
triangle, 5. and
the
in angle
i
and ratio two
triangles,
then
are
equal,are corresponding
proportion)
similaI.(AAA
criterion) the
simniliar.(SSS
are
sides
triangles aresame other
trìangleS,
are triangle hence
corresponding two the
angles
the
two in inof and
(or
two If one(i.e.,
sides
their
corresponding
ratio
the 4. equal
in of to
IE
3.
hence
same
their Triangles
the
then

Theorems

AEIfD
ECBC Level
Third
BC AEEC ||
|| DBDE
then
DEADDB then, Map
theIf, Level
Mínd
C points, the
Then, intersect
sides
are
divided Second
E is
to inline
side. the
parallel
distinct any
triangle
BC. Trace
the
DEI B
AE to divides third Level
triangle then
drawn a the
let
AEECACEC AC in
ratio. of First
sides ratio,
AABC, other
the
two
sidesto
line *
parallel
AD (ii)
AB
DBABDEAD
is aof same
a same
line two If two
side
In () other the
2.
a
If One
in
1.
MATHEMATICs
148 0swaal CBSE Question Bank Chapterwise &Topicwise,
ln ALMN and APQR, if Scan to know
(STANDARD). Class.X
A

(a) 1. P, ZM Q more about


and NR this tople
LM MN LN
(b)
PQ QR PR' IF AB AC
DE DE
and ZA = ZD,
(i)
then ALMN~ APQR.
AAA-Criterion: In Criteria of similarity then AABC.
two of triangles
triangles, if corresponding O-w Key Words
angles are equal, then the triangles are similar Similar:A thing similar to another
and hence their corresponding sides are
the same ratio. Corresponding: Having in the same
A D Parallel:It means thatttwo lines that
never intere relationi,
Some theorems based on
similarity of
B (i) Ifa line is drawn triangle:
parallelto one side ofa
to intersect the other two sidesSat
the other twosides are divided in thedistindt
If AABC and ADEF are similar
LA= 2D, ZB = ZE and ZC = ZF.
It is known as 'Basic Proportionalitysame
Then, AB
DE
BC
E DE
AC
or "Thales Theorem. Thenm
Remark: If two angles of a triangle are
respectively equal to the two angles of another
triangle, then by the angle sum property of
atriangle their third angles will also be equal. D
Therefore, AAA similarity criterion can also be
stated as follows: B

AA-Criterion: If two angles of one triangle are


respectively equal to two angles of another In AABC, let DE|| BC, then
triangle, then the two triangles are similar. AD_ AE AB AC
(a) (b) (c) AD AE
As we know that the sum of all angles in a DB EC DE EC AB AC
triangle is 180° so if two angles in AABC and A
APQR are same i.e., ZA= P, ZB = ZQ.
P Scan to kiOW
more about
D this topik

B
B
(ii) SSS-Criterion: In two triangles if the sides of one (ii) If a line divides any two
triangle are proportional to the sides of another sides of a triangle in the Thales theoret
triangle, then the two triangles are similar and same ratio, then the line is
hence corresponding angles are equal. parallel to the third side.
A D It is the 'Converse of Basic Proportionali
Theorem!.
If AD AE
B C E F DB EC'
BO CA then DE||BC
AB
Tf DE EF FD

AABC ADEF Fundanental Facts


ZA = ZD, ZB = E
.

then
2C= LF (1) The use of similartriangles has made possioe
and themeasurements of heights and distance
one angle of a triangle is egual
ii) SAS-Criterion: Ifanother (2) Thales ofMiletus was the great mathematicia
of triangle and the sides
to one angle angles are in the same ratio, who found the similar triangles.
includingthese
trianglesare simila.
then the two
TRIANGLES 149

Mnemonics

Concept: Area of triangle = x Base x


height
Mnemonics: Audi is the product of half of BMW and Honda
Interpretation_: > Area
B’ Base
H » Height

Q
(H) height P
D
Base
(B)

OBJECTIVE TYPE QUESTIONS (1mark each)

Multiple Choice Ouestions


0.1.In AABC and ADEF, F= ZC, ZB = LE and AB =
DE. Then the two triangles are
(A) Congruent, but not similar T S
R

B) Similar, but not congruent


(C) Neither congruent nor similar Which of these criteria can be used to prove that
ARSO is similar to ARTP ?
(D) Congruent as wellas similar
UCBSE, Board Term-I, 2021] (A) AAA similarity criterion
Ans. Option (B) is correct. (B) SAS similarity criterion
Erplanation: According to the definition of (C) SSS similarity criterion
similarity of two triangles, "Two triangles are similar (D) RHSSimilarity criterion
when their corresponding angles are equal and the CBSE, Q.B., 2021-22]
sides are in proportion" 0.3. Shown below are three triangles. The measures of
two adjacent sides and included angle are given
for each triangle.

F
B
9cm
According to the question, A60°
LF = 2Cand ZB = ZE 60
P 4 cm
4 cm
Since, AB = DE [Given]
2
(Note: The figure is not to scale.)
AB
Also Which of these triangles are similar ?
DE (A) ARPQ and AXZY
Which mneans the triangles are similar but not (B) ARPQ and AMNL
Congruent. (C) AXZY and AMNL
In the following figure, Qis a point on PR and S1s (D) ARPO, AXZY and AMNL are similar to one
Point on TR. OSis drawn and RPT = RQS. another
A|CBSE, Additional Questions, 2021-22}
questions are jor practice and their solutiorns are available at the end of the chapter
150 Oswaal CBSE Question Bank Chapterwise &Topicwise, MATHEMATICS (STANDARD), Clas5.X
Ans. Option (A) is correct. (C) 3.6 cm
(D) (cannot be calculated from
Explanation: In ARPQ and AXZY, the given data
RP 6 cm 2 ACBSE,
Q.6. In the figure given below, ZBAC
XZ 9 cm 3
Then:
=90° and ADy
PQ 4 cm
ZY 6 cm 3

RP PQ
XZ ZY
and ZRPQ = ZXZY (Each 60)
ARPQ ~ AXZY Proved. B D
Q. 4. In the figure below, DE || AC and DF || AE. Which
(A) BDx CD= BC2 (B) AB × AC= BC.
of these is equal to B (C) BD x CD = AD² (D) AB XAC =
FE =AD
NCERT
A Ans. Option (C) is correct.
D Explanation: In AABD and ACAD, ZAR:
=ZADC -90° and ZABD = ZCAD =0.
By AA Similarity, we get, AABD ~ ACAD
AD CD
’ BDxCD= AD?
BD AD
0.7. If AABC ~ AEDF and AABC is not similarto Ahr:
then which of the following is not true?
DF BE (A) BC x EF = ACx FD
(A) (B)
AE EC (B) AB X EF = AC× DE
BA (C) BC X DE = AB × EF
(C) AC
(D) EC (D) BC x DE = AB x FD UNCERT Exempl
ACBSE, Q.B., 2021-22] Ans.Option (C) is correct.
Ans. Option (B) is correct. Explanation: Since AABC ~ AEDE, then we get
Explanation: In AABC, AB AC BC
DE || AC (Given) ED EF DF
BD BE From first two, AB × EF = AC x DE. Option (B) 's
DA EC
(From BPT).i) correct.
From last two, BC x EF AC × FD. Option (A)
In AABE, correct.

DF || AE (Given) From first and last, BC × DE = AB x FD. Option ()


is correct.
BD FB
(From BP) ..ü) Thus, option (C) is incorrect.
DA FE
AB BC CA
From eg. () and (i), we get Q.8. If in two triangles ABC and PQR,
QR PR PQ
BF BE
EC
then:
FE
point S divides (A) APQR ~ ACAB (B) APQR ~ AABC
Q.5. In the following figure, ST ||= QR,
1.6 cm, what is the
POin the ratio 4:5. If ST (C) ACBA ~ APQR (D) ABCA ~ APQR
length of QR? U NCERT Exern
Ans. Option (A)is correct.
AB BC CA
Explanation: Given that, QR PR PO
by $
similarity , we get APQR ~ ACAB.
Q.9. In the figure given below, two line segments A
R and BD intersect each other at the point Psuchth
PA = 6cm, PB =3 Cm, PC= 2.5 cm, PD =5 t
is not to scale.) ZAPB = 50° and ZCDP= 300 Then. /PBA is e4*
(Note: The figure (B) 2 cm to:
(A) 0.71cm
solutions5 are available at the end of the chapter
practice and their
These questions arefor
TRIANGLES 151

D
0. 10. 1f in two triangles DEF and POR,, D 0and
5 Cm
6 cm
ZR=E, then which of the following is not true
EF DE DE FE
500 (A (B) PO RP
PR PO
3 Cm
2.5 cm EF DE
DE DF
(C .

(D) RP OR
(B) 30° OR PO
A)
50
(D) 100° OUNCERT Exemp.]
A+ UINCERT Exemp.) Q.11. Intriangles ABC andDEE. B= ZE, LF= 2Cand
Option ) is correct.
(D), AB = 3DE. Then, the two triangles are:
PA 6 (A) congruent but not similar
Eiplannation:In the given figure, ==2 and
PB 3
(B) similar but not congruent
PA PD (C) neither congruent nor similar
PD 5 =2. Thus- and ZAPB= ZDPC.
PB PC
PC 2.5 (D) congruent as well as similar
SASsimilarity, we get AAPB ~ ADPC. Hence, OUNCERT Exemp.]
By
ZPBA= 100

SUBJECTIVE TYPE QUESTIONS

Very Short Answer Type


Questions (1 mark each)
AABC, Dand E are points on side AB and
01.Inthe
AC such that DE ||BC. If AE = 2cm.
respectively
findCE.
AD =3 cm and BD=4.5 cm, then A 3 cm D
AO ACBSE SQ, 2020-21]
02ftwo triangles are similar, then find the relation AC AD
oftheir corresponding sides.A CBSE OD Set-, 2020] AB AC
3
Sol. lf two triangles are similar, then their corresponding
6
1
sides are in the same ratio. AB 6
and AD
0.3.In the figure, if ZACB = ZCDA, AC= 6cm AB = 12 cm.
=3 cm, then find the length of AB.
Q.4. In Figure, DE || BC. Find the length of side AD,
given that AE = 1.8 cm, BD = 7.2 cm and CE = 5.4
Cm.

A
1.8cm
B D E
[CBSE SQL, 2020] 7.2 cm
5.4 cm
Sol. AACB ~AADC (AA criterion) ½
AC AB B
AD AC
6 AB
A|CBSE OD Set-1, 2019)
3 6
(CBSE Term-2, 2016]
AB = 12 cm.
[CBSE SOP Marking Scheme, 2020] AD AE AD 1.8
Detailed Solution: Sol.
BD 7.2 5.4
Since ZCDA = ZACB (given)
(common)
ZCAD = LCAB 7.2x 1.8
AADC ~ AACB (AASimilarity) AD
54
= 2.4 cm.

[CBSE Marking Scheme, 2019}


These questions are for practice and their solutions are available atthe end of the chapter
Oswaal CBSE Question Bank Chaptcrwise &Topiwise, MATHEMATICS (STANDARD), Clas y
152
Detailed Soluion.
/ABC LADE
ZABC =48

(Cortespnit
lt is given that DE |BC Hence,
0.6. In AABC,. DE|| BC, find
AD AL
EC
the value né x,
DB
Hence,
I'utting the values,we get
AD 1.8 \xt3
72 5.4 D
1,8 x 7.2
AD =
5.4

AD
12.96 B
5.4

EC= 12 cm andADE = 48°, Find ZABC.


AD = 2.4 cm.
Q.5. In figure, ifAD = 6cm. DB = 9 cm, AE = 8cm and Sol. As
AD
ADIBSE, Term
DE|| BC
AE
DB EC

or,
X+3
X+1 X+5
+5x
D
or,
or,
=+4r+3
X=3

Q.7. In AABC, if Xand Y are


ICBSE Marking Schen
points on ARB
respectively such that AX
XE
U[CBSE SQP, 2018-19] YC = 9, then state whether XY and BC
Sol.
AD AE 2
Sol.
not.
UCBSE Term-1, xit
DB EC 3

DE || BC
LADE = LABC = 48°
|CBSEMarking Scheme, 2018]
Detailed Solution:
It is given that,
A B

AX 3
= ,AY= 5 and YC=9
XB 4
48
E
AX 3 AY 5
Then, and
XB 4 YC 9

12
cm AX AY
XB YC
B
Hence XY is not parallel to BC.
Here, in AABC and AADE,
Q. 8. A
AB = (9 +6) cm = 15 cm
Similarly,
AC = AE + EC
= (8 + 12) cm
= 20 cm
AD 6
Now,
AB 15 5
AE 8 2 B F
and
AC
In the figure of AABC, the points Dand
20 5
AD |.
sides CA, CB respectively suchthat DE
Then, AE
AB AC 21, DC =t+3, BE = 2r-1 and CE =.
i.e., DE || BC A|CESE,kr
OR TRIANGLES 153
thefigureof AABC, DE
In | AB. If AD Sol. In AABP,
' 2r-1 and1(CE 2x, DC | AP (Given)
x, then find the
BD BC (From BPI)..)%
value
of,
[CBSE, Term-1, 2015| DA CP

21

B 2x-1 E C
In SABC,
CD CE X+3 (Given)
Or DE || AC
AD BE 2x 2x-1 BD BE
3
(From BPT)...(iü) ½
5x =3 or, x= DA EC
5
From equations (i) and (iü), we have
thegiven figure, if ZA=90,
in /B=90, OB =4.5 em, BE BC
0904 =6Cmand AP = 4cm, then find QB. Hence Proved. ½
EC CP
UCBSE Term-1, 2015]
Q. 2. In the given figure, DEFG is a square and BAC
=90. Show that FG = BG × FC.
6
4.5
D

G C
Sol. In APA0 and AQBO,
LA = ZB= 90 (Given) AO[CBSE SQP 2020]
ZPOA = LQOB
(Vertically Opposite Angles) Sol. AADE ~AGBD and AADE ~ AFEC
Since, APAO ~AQBO, (by AA) AGBD ~ AFEC (AA Criterion)
OA PA GD G
Then, QB FC FE
6 4 GD × FE = GB x FC
4.5 QB
Or FG =BGx FG
4x4.5
[CBSE SQP Marking Scheme, 2020|
Or OB = Detailed Solution:
6
OB =3cm 1
Given, DEFG is a square and
Q10. Are two triangles having equal corresponding
sides, similar? RCBSE Term-1, 2015]
Sol. Yes, Two triangles having equal corresponding
Sides are congruent and all congruent As have equal
angles, hence they are similar too. 1

Short Answer Type


ZBÃC = 90
Questions-I (2marks each)
To prove, FC' = BG x FC
0.1.In the adjoining figure, DE || AC and DC || AP. Since, DEFG is a square, therefore we can write,
DE = EF FG = GD
Prove that BE BC In AAGE and AGDB,
EC CP ZA = 2DGB =90°
LADE = ZGBD
(Corresponding angles)
:. By AA similarity, AEAD ~AGDB ..)
Now, In AAGF and FCE,
LA = LEFC = 90°
P LAED = ZFCE

AUACBSE DelhiSet-I, 2020]


(Corresponding angles)
::By AA similarity, AAED ~AFCE ..)
154 Oswaal CBSE Question Bank Chapterwisc &Topicwise, MATHEMATCA (8TANDARD), Claan.

From (i) and(i), we get A


AGDB -AFCE
Sine, correspxonding sides of two sinilar triangles
are proortional.
GD BG
FC EF
GD x EF = BG x FC
IG² = BG × FC
Hence Proved. 2
Sol. Given, AD = BD × CD
AD BD
Q.3.X is apoint on the side BC of AABC.XM and XN are CD
AD
drawn parallel to AB and AC respectively meeting AADC ~l ABDA
AB in N and AC in M. MN produced meets CB
produced at T. Prove that TX² =TBXTC. Or LBAD = ZACD;by SAS, : .
[CBSE Comptt. Set-I, II, III, 2018] ZDAC = ZDBA
(Corresponding angles off
ZDAC +simLDBA:
ilar
Sol. ATXN ~ ATCM ZBAD+ LACD +
A

M or 2/BBAD
AD ++22DACDAC =k
/A=90 Hence pro
Short Answer Type
Questions-II (3 markss
TX XN TN Q.1. The perimeter of twossimilar
TC CM TM
triangles
15 cm respectively. If one side s e25
of the arfirst h
TX × TM = TCx TN is 9cm. Find the length of the
Again, ATBN ~ ATXM
TB BN TN
of the second triangle. corresponding
TX XM TM Sol. AABC ~ ADEF
ADC+A[CBSE SQ:
TNXTX
TM= Perimeter (AABC) AB+ BC+CA A
TB
Perimeter (ADEF) DE + EF +FD DE
Using (i) in (i), we get
25 9
TX==TCXTNTB 15
TX² =TCx TB Hence Proved. ya [Using proportionality thec
[CBSE Marking Scheme, 2018] x= 5.4 cm
Q.4. In the figure, PQRS is a trapezium in which PQ || RS. DE = 5.4 cm
On PQ and RS, there are points Eand Frespectively [CBSE Marking Scheme,22
such that EF intersects SQ at G. Prove that
Detailed Solution:
EQ X GS =GQx FS.
Given, perimeters of both triangles are 25 em
S. Let both triangles be ABC and DEF.
AB + BC + CA = 25 cm
and DE + EF + FD = 15 cm
A

UCBSE Term-1, 2016)


Sol. In 6GEQ and AGES
ZEGQ= ZFGS (vert. opp. angles)
ZEQG = LFSG (alt. angles)
AGEQ -~ AGES (AAsimilarity) 1
Or, EQ GQ
FS GS
EQ xGS = GQX FS. Hence Proved. 1 Also given, CA =9 cm
|CBSE Marking Scheme,2016] Let FD =r cm
AB +BC+CA CA
O.5. In AABC, AD BC, such that ALD = BD X CD, Then
FD
DE + EF + FD Given,A4BC".
Prove that AABC is right angled at A.
A|CBSE, Term-1, 2015]
TRIANGLES 155

Sol. Given, AAHC ~


15 ADEF
25x = 135 Then according to question,
135 AB DE
= 5.4 [From BPT| ½
25 BC EF
length of the
the correspondig side of 2r-1 18
triangleis 5.4 Cm,
Heoe
1 2x+2 3r + 9

adjoining figure,
the. (2r- 1)(3r + 9) = 18(2x + 2)
Qn
D= ZE and AD AE
DB (2x - 1)(x + 3) 6(2r + 2)
2r-x + 6x-3 = 12r + 12
that ABACis
Irove
an isosceles triangle. 21 + 5x- 12x - 15 = 0
212-7r- 15 = 0
2r- 10x + 3x- 15 = 0

E 2x(r -5) + 3(r 5) = 0


DA
(r-5)(2r + 3) = 0
-3
Either x=5 or x = which is not possible
B
So, X=5

ADA[CBSE Delhi Set-I, 2020] Then in AABC, we have


AD AE AB = 2x- 1 = 2 x5-1=9
ZD=Eand
Sol. Given: DB EC BC = 2x + 2 = 2 x5 + 2 = 12
AC = 3x = 3 x 5 = 15
and in ADEF, we have
DE = 18
DA EF =3x +9 =3 x 5 +9= 24
DE = 6x = 6 x 5 = 30.
B Q.4. Two right triangles ABC and DBC are drawn on
the same hypotenuse BC and on the same side
To prove: ABAC is an isosceles triangle. of BC. If AC and BD intersect at P, prove that
AD AE
Proof:
DB EC
AP × PC= BP x DP. A CBSE OD Set-1, 2019}
Sol. AAPB - ADPC [AA sinilarity] 1
By converse of BPT,
DE ||BC 1 AP BP
1
..

LADE = LABC (Corresponding angles) DP PC


And
LAED = LACB (Corresponding agies} ½
LADE = ZAED (Given) AP x PC= BPx DP
LABC = LACB
So, BAC is an isosceles triangle. Hence Proved. 1
V.in the given figure, if AABC ~ DEF and their sides
0t the given figure lengths (in cm) are
long them, then find the lengths of sides marked
of each
triangle.
2r-1 CBSE Marking Scheme, 2019]
31 18 6x Detailed Solution:
Given: BAC and BDC are 90° each AC and BD
2x+2 E 3r+9 F intersect at P

lAuG+ A{CBSE ODSet-I, 2020]


Oswaal CBSE Ouestion Bank Chapterwise &Topicwise, MATHEMATICA (8TANDARD), Cas y
156

To provc: AP x PC = BPx DP.


Proof: In ABAP and ACD!, Sol.
21= (2
LA = 2D (Given) PT = PS
ANSQ AMTR
BPA = ZDPC NÌS = ZMRT
(vertically opposite angles) ZPOR =ZPRQ
ABAP ~ ACDP (By AA similarity) 1 PR = PO
BP PT PS
Then AP From (i) and (i),
DP PC PR PO
Or, AP x PC =BP x DP Hence proved. 1 Also, ZTPS = ZRPQ
Q. 5. Insimilar triangles, AABC and APQR, AD and PM APTS ~ APRQ
AD AB
are the medians respectively. Prove that PM
ACBSE Board Term, 2019)
PQ

0.7. AABC is a right angled at


Hencer
(CBSE Marking Schene
C. If p
Topper Anawer, 2o19 the perpendicular from Cto AB is the
lengths of the sides opposite and
Sol.
to ZA,4,LBb,ca
1
respectively, then prove that 1

AA Board Term:
Sol.

C +

In AACB and ACDB,


aR: 28M ZACB = ZCDB = 90
ZB=2B Q.
AACB ~ ACDB (by AA Sim
b C
Or,

Or, 1 c
ab

Squaring on both sides,


1

Ap
PM Or,

1 1
Q.6. In figure Z1 = 2and ANSQ
that APTS~APRO. AMTR, then prove
Heg.
Long Answer Type
Questions (5 marks each)
TRIANGLES
157

srightangled at Q. OX LPR, XY L
BX ||BXCO
CE. is a paralelograrm then BE| XC and
ZLPQaredrawn. Prove that RO and In ABX, by
B.PL.,
XZ'=PZ xZQ. AF AO
R FB ...) 1
OX
In AAXC.
AE AO
EC .() 1
OX
Eqn. () and (i) gives,
A AE
1
FB EC
U So by
(Board
R Term-1, 2015] converse of B.PI,
EF || BC
Sol.
(ii) Given OX FB
X 1
OA AF
Adding 1on both sides
OX -+1 =
FB
OA -+1
1 AF
OX +OA
Here, RQ 1 PQ and XZ L PO FB + AE
OA
XZ|| YQ AF

.Similarly, XY ||ZQ AX AB
(from fig.)
OA AF
YOZ is arectangle. (* ZPQR = 90° 1 or OA: AX =
In AXZQ, Z1+ 22 =90° AF:AB Hence Proved. 1
and in APZX, Z3 + Z4 = 90° ...) Q3. In a trapezium ABCD, AB || DC and
..ü) EF | | AB, where E and F lie DC = 2AB.
X0L PR or, 2+ 23 =90° on BC and AD
BE
By eqs. (i) and (ii), we get respectively such that 4
Diagonal DB
EC 3
Z1 = 23
intersects EF at G. Prove that. 7EF =1LAB.
By eqs. (i) and (ii), we get
L2 = 24 Sol. A|ABoard Term-1, 2012 Set 65|
APZX ~AXZQ (AAsimilarity) 1
PZ _ XZ B

XZ

Thus, XZ = PZx ZQ 1 G
Hence Proved.
Q2 In AABC,AD is a median and Ois any point on AD.
BO and CO on producing meet AC and AB at Eand
Frespectively. Now AD is produced to Xsuch that In trapezium ABCD, AB || DC and DC = 2AB.
OD =DX as shown in figure. B 4
Also,
A
EC
In trapezium ABCD, EF || AB|| CD
AF BE 4
FD EC 3
In ABGE and ABDC,
2B = ZB (Common)
ZBEG = ZBCD
(corresponding angles)
ABGE ~ ABDC (AA similarity)1
Prove that: Or,
EG BE
..)
EF \| BC CD B
ü) A0: BE 4

Sol 0) AX= AF: AB


As,
EC 3
A Board Tern-1, 2015]
Since, BC and OX bisect each other.
Oswaal CBSE Ouestion Bank Clhapterwise &Topicwise, MATHEMATIC8 (8TANDARD), Claas.

158
01, FG 3
EC 3
Or,
BE
1
EC +1= 3 +1 A
Or,
BE 4
EC+ BE 7
or
BE 4

BC 7 BE 4 Adding eqns. () and (),


or 7
BE 4 BC EG + FG= 3
EG 4
From (1), CD 7
Or EF = 7
Or EG =
7
...i) x(2AB) +
ADGE ADBA 8
Similarly, 3
DF FG 1
Or
DA AB
7EF = 11AB.
FG 3
Or
AB

COMPETENCY BASED QUESTIONS

Case based MCOs


I. Read the following text and answer for questions
from Q1 to Q5.
SCALE FACTOR AND SIMILARITY
SCALE FACTOR
Ascale drawing of an object is of the same shape as
the object but of a different size.
The scale of adrawing is acomparison of the length Reflection or Flip Translation or Shde
used on a drawing to the length it represents. Hence, two shapes are Similar when
The value of scale is written as a ratio. become the other after a re-size, flip, shde c
SIMILAR FIGURES
The ratio of two corresponding sides in similar fig ADC +AjCBSE SQ:
ures is called the scale factor. Q.1.A modelof a boat is made on the scale otl:
length in image model is 120 cm long. The full size of the bu
Scale factor a width of 60 cm. What is the width of de
corresponding length in object model ?

(A) 20 cm (B) 25 cm
(C) 15 cm (D)240 cm
If one shape can become another using Re-sizing
then the shapes are Similar. Ans. Option (C) is correct.
Explanation: Width of the scale model 2
= 15n.

Q. 2. What will eftect the similarity of any


(A) They are flipped horizontally
factor
(B) They are dilated by a scale
(C) They are translated down an
oton
mirrorimage O
(D) They are not the
Ans. Option (D) is correct. mirrorina
not the
Explanation: They are
Rotation or Turn another.
TRIANGLES
159

similar triangles have a scale factor of


two
Q2If which statement
regarding the two triangles
a:b,
i strue?
oftheir perimeters is 3a : b
(4) Theratio
altitudes have a ratio a:b
Their
(B) medians have a ratio :b
Their mne
(C)
bisectors have a ratio a. 2
Theirangle
correct.
(D)
Option(B)is Let ABC and POR be two similar
Erplanation:
Ans.
AD, PE are two altitudes:
trianglesand
AB BC AC
PQ QR PR
(corresponding sides) H
12 m
P
A E F

D M
N
K L 12 m

A 12 m B
E
R
B D
one of the
What is the length of EF, where EF is
. /B= ZQ andZADB= PEO(each 90) horizontal edges of the block ?
AD AB (B) 3 m
(A) 24 m
Now, PE PO (D) 10 m
(C) 6 m
(corresponding sides) Ans. Option (C) is correct.
shadow of a stick 5 mlong is 2 m. At the same Explanation: Length of the horizontal edge EF
.4 The
ime the shadow of a tree 12.5 m high is = half of the edge of pyramid
12
2
=6 m(as E is he mid-point of AT)
Tree II. Read the following text and answer any four
questions from Q1 to Q5.
Stick
Seema placed a light bulb at point O on the ceiling
and directly below it placed a table. Now, she put
a cardboard of shape ABCD between table and
Shadow
lighted bulb. Then a shadow of ABCD is cast on the
Shadow
table as A'BCD (see figure). Quadrilateral A'B'CD'
in an enlargemernt of ABCD with scale factor 1 : 2,
(A) 3 m (B) 3.5 m
Also, AB = 1.5 cm, BC = 25 cm, CD = 2.4 cm and
(C) 4.5 m (D) 5 m AD = 2.1 cm; ZA = 105°, ZB = 100°, C= 70
Ans. Option (D) is correct. and D= 85°, AOe+ A
Explanation: Let shadow of the tree be x.
By the property to similar triangles
we have 5 12.5
2

X =
(12.5 x 2) =5 m
5

o. Below you see a student's mathematical model of a


tarmhouse roof with measurements. The attic floor,
ABCD in the model, is a square. The beams that 105° B
-100°_70
Support the roof are the edge of a rectangular prism, 85°
EFGHKILMN. E is the middle of AT, F is the middle
0f BT, G is the middle ofCT.and Histhe
DT. A1 the edges of middle of
the pyramid in the model have
length of 12 m.
l60 Oswaal CBSE Question Bank Chapterwise &Topicwise, MATHEMATICS (STANDARD), Class-X
0. 1. What is the measurement of angle A' ? similar when one can become the other
photographafter
are.
(A) 105° (B) 100o size, flip, slide or turn. In the
(C) 70
Ans. Option (A)is correct.
(D) 80°
is 1:200
a
showing the side view of. train engine. Scale
bfealcttreor
Explanation: Quadrilatera! A'BCD' is similar to This means that a length of1 cm on the
ABCD.
LA' = ZA
above corresponds to a length of 200 cmphotogran,phof
oT 2
the actual engine. The scale can also be
LA' = 105° the ratio of two lengths. writ 20211
[CBSE QB en as
Q.2. What is the length of A'B' ? Q.1. If the length of the model is 11 cm, then
(A) 1.5 cm (B) 3 cm overall length of the engine in the find the
(C) 5 cm
Ans. Option (B) is correct.
(D) 2.5 cm

Explanation: Given scale factor is 1 : 2


above, including the couplings(mechanism we
to connect).
photograph 1
Sol. Length of model = 11 cm
A'B' = 2AB
.:. Total length of the engine = 2 x 11 = 22 m
A'B = 2 x 1.5 = 3 cm
Q.3. What is the sum of angles of quadrilateral A'B'CD'? 0.2. (i)) What will affect the similarity of any b
(A) 180° (B) 360° polygons?
(C) 270° (D) None of these (ii)Whatis the actual width of the door if the widthat
Ans.Option (B) is correct. the door in photograph is 0.35 cm?
Explanation: Sum of the angles of quadrilateral Sol. (i) They are not the mirror image of one another.
A'BCD' is 360°. (ii) Actual width of the door
Q.4. What is the ratio of sides A'B' and A'D'? Length of door in photograph
(A) 5:7 (B) 7:5 corresponding length in object
(C) 1:1 (D) 1:2
Ans. Option (A) is correct. 0.35
Explanation: 1/200
A'B' =3 cm = 0.35 x 200
and AD' = 2AD
70cm
=2x 2.1 = 4.2 cm = 0.7 m.
A'B' 3 30
0.3. Find the length of AB in the given figure.
A'D' 4.2 42 Also find the area of AADE. 2
5 or5:7 A

Q.5. What is the sum of angles Cand D'?


(A) 105° (B) 100 X Cm

(C) 155° (D) 140°


Ans.Option (C) is correct. B
Explanation: LC= 2C= 70° 3cm
and D'= D= 85° 4 cm
ZC'+ D'= 70° + 85° = 155°

Case based Subjective D 6 cm


Questions
I.Read the following text and answer following Sol. Since, AABC and AADE are similar, then their ratio
four questionsfrom Q1 to Q5. of corresponding sides are equal.
SCALE FACTOR AB AB+ BD
Ascale drawing of an object is the same shape at the BC DE
object but adifferent size. The scale of a drawing (*+4) cmn
is a comparison of the length used on a drawing
to the length it represents. The scale is written as 3cm 6 cm
a ratio. The ratio of two corresponding sides in 6x =3(x + 4)
similar figures is called the scale factor. 6x =3x +12
Scale factor = length in image / corresponding
length in object 6x -3x = 12
3x = 12
x=4
1
Hence, AB = 4 cm.
Here, AD = AB + BD
=4+ 4=8cm
and DE = 6 cm
If one shape can become another using revising,
then the shapes are similar. Hence, two shapes are
TRIANGLES 161

:ST || QR (Given)
Area 7xADx DE ps PT (By BPT)
PO PR
;x8x6 3 PT
8 28
= 24 cm',
Readthefollllowing text and answer the
ILquestions:
following PT =
1
The triangle proportionality theorem is a = 10.5 cm
geometric law stating that when you draw line Q. 2. Which property used in the given case ?
parallel to one side of a triangle, it will intersect Sol. BPT i.e., Basic Proportionality Theorem.
2
he other two sides of the triangles and divide Q.3. If QR = 32 cm, then find SI. Also find TR.
proportionally. n the given APQR, ST| |QR
themPS Sol. From the figure,
3 PS PT and ZP is common.
and and PR = 28cm. In APQR, PQ
SQ 5 PR
APST ~ APQR (By SAS)
PS ST
PQ QR
S
28
cm 3 ST
32
5
3
ST = x 32
8
R
= 12cmn.
TR = PR - PT
0.1. What is the length of PQand PT ? (28 - 10.3) cm
Sol. Length of PQ = PS + SQ [: PT = 10.5 cm
=3+5 proved above]
= 8 cm 1
= 17.5 cm.

Solutions for Practice Questions


Multiple Choice Questions Explanation: In ADEF and APOR, LD= 20 and
iAns. 2: Option (A) is correct. ZR= ZE. By AA similarity, we get ADEF ~ AQRP.
DE EF DF
Explanation: In ARQS and RPT Hence, .Option (B) is incorrect.
ZRÌS = ZRPT (Given) QR RP QP
ZR = ZR (common) Ans. 11: Option (B) is correct.
So, by using the theorem, Explanation: In AABC and ADEF, ZB = ZE and
are equal, then their ZF= 2C. By AAsimilarity, we get AABC ~ ADEF.
If the corresponding anglesproportional and the
corresponding sides are Thus,the triangles are similar but not congruent.
triangles are similar. Very Short Answer Type Questions
Ans.5: Option (C) is correct. Sol. 1:
Explanation: ST ||QR (Given) A
2 cm
PS ST 3 cm
(Corr. B.PT)
PQ QR
4 1.6
9 QR 4.5cm

9
QR = -4 x1.6 = 3.6 cm.

Using basic proportionality theorem


AD AE
4
BD CE [By using BPT]%
S
1.6 cm 3 2
5
R
4.5 CE
CE =3 cm
¡Ans. 10: Option (B) is correct. CBSE Marking Scheme, 2020-21)
162 Swaal CBSE Question Bank Chapterwise &Topicwise, MATHEMATICS (STANDARD), Class-X

REFLECTIONS

a) have you understood the criteria of similarly of triangles by AAA, SSS, AA, SAS Criterion
(b) Can you explain the basic
() Will you be proportionality theorem í
able to explain the basic proportionality theorem ?
h
Tangent to a circl~ at point of contact.
ndicular to t e
1. (Prove) The tangent at a11y point of a circle is perpe · le
radius through the point of contact. t to a czrc
an external poin
2. (Prove) The lengths of tangents draw11 from
I
are equal.
..
I
_____ .., __
------ -----~
- ---- - d he · 1 nd dif:r. r.."C'!'inrl,_,.., =7 . h · [ ----- - ·-
1.JJ erent terms assoCLated W1th t e arc e.
To understan t etrc e a
s of the important theorems related to the tangent
t

: Understand the concept of tangent to circle and proof


___ - - - -
1
of cirde.
.. -- - - • - • - - - - - - - - • - - • • - -•·•--•-•-•--
--•·••-••- •••••
•-••-- -
\I - "" ----- - -•----- •-•• ••

-
► Tangent:At a ng ent toact rde isa e a m !)

Tungentline
the circle at one point only. ~
is
► The common point of the circ~e and th e tangent
called the point of corit act ·
► Secant: Two common points (A
and B) between line PQ and
circle.
► Atangent to a circle is a special
case of the secant when the two
end points of the corresponding
Seca nt Jnd C.i.rcle.• A circle is a round shaped !•
chord are coincide. :
Tangent figure has no corners or edges.
► There is no tangent to ~ cir~Ie. _
Tangent: A line, curve or surface :
passing through a point Iymg ms1de the circle. touclling but not intersecting another. ,
► At any point on the circle there can be one
and only '
Cont act To touch or make touch :1
one tangent. physical. l
nd icular ► --- ------ ------- -- ---- --- ---·
► The tangent at any point of a circle is perpe -----
There are exactly two tangents to a circle throu gh a
to the radius through the point of cont a(t · point outside the circle.
\
~ '.~ .,.,./
:

In a fixed plane, 0
(II
The locus of a point
Only one common equidistant from a fixedr ,: :2 P
<
~
point (A) between ·. point is called 'circle'. G radius e.
()
circle and line PQ.
Here, PQ is tangent
at point A lying on
G r
1
J
The fixed point is
called the centre &
separation of
centr
I . The tangent at any
point of a circle is
------ . t:u
(/)
t%i
perpendicular to ·,.
the circle. ! points is the radius
of the circle.
the radius th rough
.L¼
(
Y · ' X
,0
C
(1)
the poi nt of contact. ' p ''
&_
LOPQ=90" 0
l--- - -- ~-- ~- ----=-.:.-"- -- :::
tD
Q §
2. The length of :;::.-
tangents drawn 0
from an external ::r
$l)
( point to a circle
are equal.
p~ ~
('J

Facts Tangent and g_


PQ=PR CD
tangent point Theorems "'~
-::

"O

I en·
s,
1. There is no tangent ~ a:
>
'
to a circle passing ~
through a point lying
inside the circle.
b ;!
{,q

Two common
points (A and B
~n
Q)
2. There is one and only
one tangent to a circle
0 p
~ between
line PQ and
-
passing through a point · Secant
circle. Q
lying on the circle. b
(ft ~
3. There are exactly two
_!
tangents to a circle through p n
a point lying outside ii,"
0,
the circle.
. 1, x"'
~ f~rtdl!Jnr
• Fl.est L<:~ L ..
':;] · v~J
• Thinl L.'1! 1.•.-J
---...... --...._
- ,,,gth of the segment of the tangent fr the
CIRCLES rm
fht 1,, point P and the point oftontact
~•n
11 with the
r cJL'ri.'. •ailed the length of the tangent
. •le,:- L' •

cif\- , ,gths of the tangents draw


n from
r11c kt . circle are equal. an external I
;
; (1) The word 'tangent' comes from the
~ ,jJll t()cl
r' 1, figure , Latin word 'tangere', which means ;
!fltlt: PB... - to touch and was introduced by '
PA= ·•--
-- -~..-- ...~·~ .._ .......
the Danish mathematician Thomas ;
Fincke in 1583.
i (2) The line perpendicular to the
tangent:

/( • 0
/ and passing throu gh the
contact, is known as the norm
poin
al
t of;

J (3) In two concentric


circles, the chord /
of the larger circle, which touches the ;

::c:ll:;a:~~cle, is bisected at the point:


~""-·•-~--~---·•-~~ · ~··--------~~·-- ~
--... .,/
\. _____ ___ ·-·-·••~·- ------ -- •- --~---- 1

fl~~~~.~ 13 cm from
Q. 3. From a point P which is at a distance of

II , LCOD is
the centre O of a circle of radius 5
tangents PQ and PR to the circle are
cm,

the area of the quadrilateral PQOR is:


the pair of
draw n. Then

Q.1. In the given figure, if LAOB = 125°, then


2
equal to: (A) 60 cm2 (B) 65 cm
2
(B) 45° (C) 30 cm2 (D) 32.5 cm
(A) 62.5°
(
(C) 35° (D) 55°
~- --- --- --- --- -·· ,: I

I: I p: ;, . ----if-"-1.J..Ul-t---f--➔
B/
ll
!
\,_ ___ -·····-• · •·· -- ------- -----
R
------ (I +DJ
Ans. Option (A) is correct.
s at
Explanation: PQ is tangent and QO is radiu
D contact point Q.
LPQO = 90°
circle and
Q. 2. ln the given figure, AB is a chord of the
B = 50°. If AT is :. By Pythagoras theorem,
AOC is its diameter, such that LAC
the tangent to the circle at the point A, then
LBAT PQ2 = OP2 - Orj
is equal to: = 13 -522

(A) 65° (B) 60° = 169- 25 = 144


(C) 50° (D) 40° ⇒ PQ = 12cm
WPQ~60PR
[SSS congruence]
:. Area of 60PQ = area of 60PR
B [Since, congruent figures are equal in areas]
Area of quadrilateral QORP = 2 area of fl.QPR
- 1:::;.__ _ _ _ _ _
_.,. 1
=2x base x height
_ _..,
A
....... ---~- . • . . _ . ,.. • .s ,_ ---·-------
T
- - - - - -- - "----
~m 2
,;, Th of tire clzapler
·-, itSl qriestirms are for practice and tlreir solutions are available at the eud
, yr

, , , . MATHEMATICS (STANDARD), Clafls-x


Oswaal CBSE Questio n IJa nl< Clin ptrrwi.:qc & 'J'opi cwrn r .

:.::: RJJ x Ol<

(
= 12 X 5 \
2
:::: 60 crn
Q. 4. At one end A of dianwter Al:J O ( a ctr· ck.of
,
radius 5
cm tangent XAY is drawn to t e c·ircle• Jhe •Jengt1t
t1
\
'
of the chord CD parallel to XY 31 td at ;i d1stance '
S cm from A is:
(A) 4 cm (B) 5 cm

(C) 6 cm (D) 8cm m+m Ans. Option (C) is correcl


I
Ans. Option (D) is correct.
Explanation: Join OA. OA is radius and AT is ta
Expla11atiott: XA y is tangent and AO is radius at at contact point A. n&r,
contact point A of circle.
LOAT= 90°,
B Given that, OT = 4 cm
/1-

9' ~~~D
I' , ,'
Now,
AT
4
base
= hypoten use == cos 30°
I ' r= 5cm
✓3
( o} ⇒ AT=4X - 2 = 2✓3,,,.,.. ,.
~

set s:cm
"'1V
\ r~
Q. 6. In the given figure, 'O' is the centre of circle, PQ
a chord and the tangent PR at P makes an angle of4
50° with PQ, then LPOQ is equal to:
X A y
(A) 100° (B) 80°
AO= 5 cm (C) 90° (D) 75°
LOAY = 90°
CD is another chord at distance (perpendicular) of 8 R
cm from A and CMD II XAY meets AB at M.

Join OD.
OD= 5cm
OM = 8 - 5 = 3 cm
L.OMD = LOAY = 90°
Now, in right angled dOMD m
Ans. Option (A) is correct.
MD2 = OD2 - MO2
Explanation: OP is radius and PR is tangent at P.
= 52_32
So, LOPR = 90°
= 25-9 ⇒ LOPQ + 50° = 90°
= 16 ⇒ LOPQ = 90° - 50°
⇒ MD =4cm ⇒ LOPQ = 40°
We know that, perpend iculars from centre O of In dOPQ, OP = OQ [Radii of same cirde]
circle bisect the chord. LQ = L.OPQ = 40°
CD =2MD [Angles opposite to equal sides are equJl]
=2X4 But, L POQ = 180° - LP - LQ
= 8 cm. = 180° -40° -40°
Hence, length of chord, CD = 8 cm. = 180° - 80° = 100°
⇒ LPOQ = 100°
Q. 5. In the given figure, AT is a tangent to the circle
Q. 7. In the given figure, if PA and PB are tangents to the
with centre 'O' such that OT = 4 cm and
circle with centre O such that Lt1PB == 50°, then
LOTA= 30°. Then AT is equal to: LOAB is equal to:
(A) 4 cm (B) 2 cm (A) 25° (B) 30°
(C) 2✓3 cm (D) 4✓3 cm (C) 40° (D) 50°
~ Drtail(•u <;nJutlrin:
OQ- 4 cm
OQ l flQ and Of< J KT
{·: R,adiu" J. t.at'ige n t 1

• correct,
(M isl AOAB, we have
;;of\
Of'., ·on: n
f'£,plfl"aft OA = OB [Radii of same circle]
LOAB = LOBA [Angles opposite to
equal sides are equal]
PA are radius and tangent respectively at LOQP = LORP = 90°
14and
MO ointA. and LQPR = 90" (given)
c0lltact p LOAP = 900_
Then LQOR = 90°
So, LOBP = 90°
,_;1arly, [By property of quadrilateral]
StJllV a.1..:1ateral PAOB,
iJ1 qu cuu
! t,1ow, +LA + LO + LB = 3600 So, PQOR is a square in shape.
i PO + 900 + LO + 900 = 3600 Hence, OQ = PQ = 4 cm.
th
~ SO LO= 360° -90° -90°-50° Q. 9. In the given figure, PQ is tangent to the circle wi
LO= 130° centre at 0, at the point B. If LAOB = 100°, then
~ LABP is equal to
. iJ1AOAB, (B) 400
(A) 50°
A~; + LOAB + LOBA = 180° (0) 80°
(C) flJ'
l30° +LOAB + LOAB = 180°
~ [·: LOBA = LOAR]
~ uoAB = 180° - 130° = 50°
LOAB= 25°
:::>
LOAB = 25°
Hence,
two
In the given figure, from an external pointf P, di
Q.S. . Ie o ra us
drawn to a cuc p
tangents PQ and PR are
4cm with centre O. If LQPR = 90°, then length of fi1 [CBSE Dclhi & OD, 2020}
PQis Ans. Option (A) is correct.
(A) 3cm (B) 4cm
(C) 2cm (D) 2✓2 cm

Detailed Solution:
OA =OB [Radius of circle1
LOAB = LORA
[Isosceles triangle propertyJ

Now is 60AB, LOAB = l&J - lOO = 40


2
Ans • DJ [CBSE Delhi & OD, 2020] !Sum angle property of .1.OABJ
·Option (B) is correct
Hence, LABP = LOBP-LABO
=90°-400
I
[·: LOBP :: ~, angles between
I
tangent and radius]
·· ... ___ ___ __ ______ _ ~· ··----------~ ·-- ------· = 500,
o "w~nJ C l3SE Qut" :.Uon Htt n k C h np terwi<.c &.
T · ·i ~e MATHEMATICS (STAND,h•-
opi cv. · ' · ~ ). CJ,
iJ ~ ... . :.:
J
~
Now, the perimeter of 6.PfM
~ ::: P[.
s
:::: (28 - Y) ., ~ l.;1,
. ~ { \ .•
;c:: 28 + 10 :i: •• ;, .
. ~ - · t/ ,.
Q. l. If PQ = 28 cm, then fi nd the perimeter of APLM. Q. 2. PQ 15 a tangent to a circle With - Cifi cr,1 ·,
If L\OPQ is an isosceles trian cent:, ~
p gle, t}i'1\ fi l!~
/\ \ rmJr.,

rcesc
1
~d <~
/ SQi'. •, '
\ Sol. '

T 0

./
In .10PQ,
mrcBSE SQP, 2020-21] LP+LQ+LO =180°
Sol.·: PQ = PT (LO = LQ isosceles .
PL + LQ = PM + MT 2LQ+LP = 1800 · tlit:i:, .,
PL + LN = PM + MN 2LQ+90° = 1800
(LQ = LN, MT = MN) 2LQ = 90"
(Tangents to a circle from a common point)
LQ = 45°
Perimeter (Ll.PLM) =PL + LM + PM ¼
[CBSE SQP Marking Scheme,2t'.-
= PL + LN +MN+ PM
Detai1ed Solution: '
= 2(PL + LN)
= 2(PL + LQ) LOPQ = 90° (Angle lt".', ~
=2X28=56cm ¼
{CBSE SQP MarJdng Scheme, 2020-21] tangent and n::
Dt>t:i ileci s,11ution:
Let LPQO be x then0
,

Given, PQ = 28 cm LQOP =x 0
[.10PQ is isosceles tric'".
PQ = PT
(Length of tangents from an
external point are equal)
i.e., PQ = PT = 28 cm
According to figure,
Let LQ = x, then
PL =(28 - x) cm I
and let MT = y, then I
PM =(28 - y) cm /
and LM = LN + NM ./
. . --= X .+ y - ·-· In L\OPQ,
LOPQ + LPQO .+ LQOP = 180° .
[Sum of angle; l,fatn,i:? ;
;
90° + x 0
+ x =180°
0
. :
0 9()" - l.):t
⇒ 2.t = 180 - :, - ·
0
}
900 5" f
⇒ ,,~·o = - 2 ;::;~ t
f
Hence, LOQP is 45°. to J,l .
• dra-WJ\ '
Q. 3. If two tangents inclined at 60 are , tall8'vl "·
of radius 3 cm, then find length of e~diQP. Zli;.►; "
g [CUSE t .J I

I1

~
CI RCLES .
Now, CR ~ AC- AR ,: 11 - 4 = 7 cm
Simil
. a rIY,CR and CQ are tangen ts.
.. CR = CQ = 7 cm
p
Now, BC =BO + CQ = 3 + 7 = 10 cm.
Hence, the length of BC is ·io cm.·

; i
t
\.. . . - ------ ----- ·--- -- __ __ _,,
I

111 1:,PAO, . ,• ► Some----students were not versed "


with the
tan 30 = -AO ~ .. • • properties of circle.
0
... .... _ ... __ .. ...
,
PA - rt

(Using trigonometry)½
1 3
:~_, ►___prop
I
✓3 PA It is necessary for the students to learn all
PA= 3✓3 cm. -~erties
•~•-•~ of circle .
--w------ ~~-- •~•• -----•~ •----
½ Q. 5. In the given figure, find the length of PB.
[CBSE SQP Marking Scheme, 2 •
020 211 r--· - - - - -
·1-d Solution:
petal C
PA= PB=?
Angle between tangents = 60° (Given)
:. Tangents are equally inclined to each other
.
LOPA = LOPB = 30°
and LOAP = 90°
[Angle between tangent and radius] ~ DJ (CBSE OD Set~!, 2020}
t' Sol. Since AB is a tangent at P and OP is radius.
1n6PAO,
LAPO = 90°,AO = 5 cm and OP= 3 cm
tan 300 = Perpendicular = OA In right angled 6OPA,
Base AP AP2 =Aa2- OF2
1 3
⇒ (By using Pythagoras theorem)
✓3 - AP
AP2 = (5)2 -(3) 2 = 25-9 = 16
(Using trigonometric Ratios) ⇒ AP= 4cm
⇒ AP= 3✓3 :. Perpendicular from centre to chord bisec
t the
chord
Hence, the length of each tangent is 3✓3 cm. AP= BP= 4cm .
Q. 4. In the adjoining figure, if MBC is circu Q. 6. If the radii of two concentric circles are
mscribing 4 cm and
5 cm, then find the length of each chord
a circle, then find the length of BC. of one
circle which is tangent to the other circle.
DJ [CBSE Delhi Set-I, 2020]
----- A I
\
lCBSE SQP, 2020}

Sol. Length of Tangent = 2 x ✓ 52 - 42


= 2 x 3cm = 6cm ½+ ½
[CBSE SQP Marking Scheme, 2020]
Detailed Solution;
I .§
il - ~ - -

In60BC,
Ca2+ BC2 = OW
. ------·--···•·-·•··-· Q _____ -·----- ---------
' Sot .. AP
· · . and AR are tangents to the circle from exter l 42 + BC2 = 52
point A. na
.16 + BC2 = 25

s· . AP= AR= 4cm BC2 = 25- 16


_unilarly , PB and BQ are tangents. BC2 = 9
\
BP:::; BQ = 3cm
I
I
BC= 3
. J,IATJIBMATJCS (STAlfDARD) , Class-x
Oswaal CBSE Question Bank Chaptcrwise & TopJcwise,
• 1 r
- =-
2 OP
OP = Zr

Q. B. nvo concenbic circles of radii a and b
given. find the length of the chord of(4 :i. hi
circle which touches the smaller circ] Ui~~1\,
ICBSE Delhi ; · . 9'
,
er)o11 J;_
,
ln AOAC,
OC2 + AC2 = OA 2
42 +AC2 = 5
2

AC2 = 9
AC =3
AB =AC+BC
=3+3
=6cm.
: ...l1'....~~=-,..- --
Q. 7. If the angle between two tangents drawn from an I ::1-_c.:..__.._.,__.
I
external point 'P' to a circle of radius 'r' and centre
0 is 60°, then find the length of OP. l I_ -~;;_"_~_f(), 7: ...?lt': -~---• -
7 ~- ~~-
;1_ • • ·-,
I
• --.
[CBSE SQP, 2020J
Q. 9. In the fig. there are two concentric circles ··:· ;
_!~fil {~ B~E Foreign Set-I, II, III, 2016J centre O. PRT and PQS are tangents to the .~
circle from a point P lying on the outer ;;: '
If PR = 5 cm, find the length of PS.

iC!Jr
SoL

I
1
OA j
In OBP,
. 'lno
= Slil.:,v =-1 ;
OP 2
I
OP =2r I i
T
fQJSE Marking Scheme, 2020]
Detailed Solution:
(2) 1'J [Delhi Comptt. Set-I, II, ill. .Wti (
Q. 10. PQ is a tangent drawn from an external poinm t
OA =r
a circle with centre O and QOR is the diameterol }
OP=?
the circle. If LPOR = 120", What is the measureri f·
Angle between tangents = 60° LOPQ? j
Tangents are equally inclined to each other r
R
⇒ LOPA = LOPB = 30°
l
In.10PA,
LPOA = 180° - 90° - 30° 1
= 600 f

OA ( Q p t
cos60° =- . . . . . -· ,. l
OP
0 I.IJ fCBS E Foreign Set-I, II, III, l Olt\) .

. ~ .
Q. 1. In the figure, quadrilateral ABCD is circUJllScrifOifr .;
~
ib~

a circle with centre O and AD.LAB. If radius · •


circle is 10 cm, then find the value of X, . ./ f \ i ·:
k -at-:the_ e_n_d o-if-th-ec-ha-pt-er~ -- - - - - - -- - - -
-------- f'f .
i'hes
;, ;-;T;:;:
~~ ue:st;:;
: e:-;q;: :;:a"-;:
; ions o,;p:t':<ad
efi;.:: and
;::::-:
: ice ei=--:
~ th:-=; ut~
, so:::;l= = ar;=e-a-:--va-i:la;-:b-;-
ions
1
CJRCLt?.f-:J -

(!JTI [D [CBSE Drlhi ';~1-L 2[)ZI)j


I [CBSE SQP, 2020·2Ij S [CBSE Delhi Sei-1, IL Ill, '(lft I
ol. Let L. OPQ b
e 9, then
LA = LOPA = LOSA :::: 900 ½
L TPQ 9Cr - 0
:i
i.SOP = 90° Since,
TP =TQ
AP =AS
LTQP = 90° - 9
/~1so1 osAP is a square. [opposite angles of equal sides]
.
He~ce, AP = AS == 10 cm p
'

½
CR= CQ = 27cm
BQ = BC-CQ I
·T
= 38-27 = 11 cm
½
BP= BQ = 11 cm
x = AB =AP+ BP Now, L TPQ + L TQP + L PTQ = 180° ½
= 10 + 11 = 21 cm ½
[Angle sum property of a triangle]
⇒ 90° - 0 + 90° - 0 + L PTQ = 180°
[CBSE Marking Scheme, 2020-21]
⇒ L PTQ = 180° - 180" + 20
. n figure, two tangents TP and TQ are ⇒ LPTQ =20
0
the give Hence, L PTQ = 2 LOPQ ½
jJI circle with centre O from an external
drawn to Hence Proved.
, T. Prove that LPTQ = 2LOPQ. ·[CBSE Marking Schemer 2020]
pa111t •
, 1 touches all the four sides of a quadrilateral ABCD. Prove that AB+ CD= BC+ DA.
~i.Aatc e 11 [CBSE OD Set•I 1
Ur III, 20161

7
C1t~n : d,-d() "tQ~t'hi'Y.\j_•~*~-~
0
b Aseo o ~ " , ~, R£ s.

: S-
!
:i,,
To f~~ -. AB HD e At>-\-BC'.
,.
'I
,J.

Iid ~- .
:I
ii
ii
;~
.,·
~ . , ,wlt1r 10 'l'nplawlo>o , IIIATll@ll'IJ\'1'108 (8TJINl)ARI)), Clt1 ,,. )( r
Iii , , . l 'I•~I( Q11r,I hl(ii ,,, II 1, l lI1111>.,,1' '>ll(1 1,c •
\''"'"'I I I'< >Q .., :-u,O" (OQn ' I (). I" '
. l 1, i , wltk , I.. '
(1. 4, In ilg. All( 1, • Ir ·" _Ill c •' ·I .. , , 1, 111.all'<'d 1t1 1h1•
!11,"1 '

,. Jhll" • ('10" " ~ii"


flll
, 'r, 1
·
~
t:
l-~ , nnd ,lll " J,I cnl, 1 I ~Y') , ,11' (
111 . 1,"o' 1":1 ,i rA1li11•.
of• It, drd•·· Su, ()1'£lU 1, 11 ; qunrr ,
. , who"' ctnl•~ · · .
1rl,111~k
11
· Thell or QH \ I IW . OQ ,. -, /'
,<'J 1 ',

It. Thu,, CQ ,, llC - (J/1 ~ 4R- ,


r
Uut· CQ r. rn I
1'1 h11gl'Hl 14 from ''X t
Cl~ .-J 118 - 1· .
" l'rr)
n11~lit11
I [

J\f> r-z /\B - /Jfl eii H- r '


nnd
Hut M' :., Al~
1'fllnf;l'nt11frotn (')(t·,~rn~1 .
,
/ / 11R ,.,:, 14 - I lln11111

rr
AC ::,: 50 cm [pr11v~.1
Now ~ ib .
AH + RC ::,: sn
, ⇒ 'J4 - r + 48 - r = 50
=> - 2,· ~ 50 - 62 ~ - 12
n 14 rm
fJ {CHSt o,mptl. Sd-1, II , Ill, 2lll8J ⇒ r = 6cm.
Q. s. In given figure, 0 is the centre of the ct' re1ea Q
AC = JABi + BC
2 is .n diameter. If PQ is a tangent to the drdnU
Sol.
and L KLN == 30°, find L'. PKL. e~i

j p;

P 14 cm A
1
/o
l!I [CHSE Comptt OD Set-I, II, 111,lfJ~
= J14 2 + 48 2 = J2500 = 50 cm ¼
Sol. Here, OK= OL [rn~
LOQB = 90° ⇒ OPBQ is a square LOKL = LOLK = 30°
BQ = r, QC = 48 - r = CR [Opposite angles of equal side, 1 1

Again, PB= r Since LOKP = 90° [Tangen:'. _

PA= 14-r ⇒ RA =48-r ½ LPKL = 90° - 30° = 60° I


[CBSE Marking Scheme,2~:'
AR + RC = AC ⇒ 14 - r + 48 - r = 50
r = 6cm ¼ 11
Q. 6. In fig., AP and BP are tangents to a circle . '

[CDSE Marking Scheme, 2018jv centre 0, such that AP = 5 cm and LAI'B=;


Det.1iled Solution: Find the length of chord AB.
I

In MBC, L B = 90° [Given] '


AC2 = AB2 + BC2 ,______!.. -- -__,...,.__
[By using pythagoras theorem]
= (14)2 + (48) 2 = 196 + 2304
= 2500
AC = J2500 = 50 cm
t
Here, LOQB = LOPB = 900 ..i(
(Radius is perpendicular to tangent) . . ·· ~-~· -. 111,i i ~
.-. In Quadrilateral OPBQ, m [CBSE Delhi Set-I, Jf, . !
r
~
)#:,;,,
~ ,
,
J

~~
~
,l
PA = PB
L PAB = L PBA = 60°
½ C1R C-LES tffl
. ½
\r ' ·' 0r, . n equilate raJ triangle .
p/l8Jsa ½
• ,')
. AB~ PA= 5cm . ½
,1e,,ce, .
l> ~C BSE ~ a>.1<i.ng Scheme, 2016]
. ure, AB 1s the diamet er of a circle 'th
·veil f ig w1
·
Q1,.
111 g1
tO ~•
~.,d AT is a tangen t. If LAOQ == 5 0 f' d
8 , In
Q\. . ~;~. ,I' • '!I ~.c -~ ~E ~~~~~!!:~et.I, II, III, 2015] Sol.
t'J {CHS E OD ';et-I, L , 1 . ' , WJ >I
A

, .:
\., ____ .... ... ··---•·"A ----..--.. ··- ______..
.. , Let AF= AE = x.
AB = 6 + x, AC = 9 + x and BC = 15
LAOQ = 58° (Given)
%
501, ar MBC = _! [15 + 6 + x + 9 + xJ.3 = 54
~\ .
LABQ = .!_ LAOQ 2
r 2
45 + 3x = 54 1
[Angle on the circumf erence or, x =3
AB= 9cm,AC = 12an
of the circle by the same arc) and BC= 15cm.
= .!_ X 58° [C BSE Mad<ln g Sd,emc , 201.51
2 Q. 10. In figure, 0 is the centre of a circle. PT and PQ a.re
= 29° tangent s to the circle from an externa l point P. If
1
L1PQ = 70°, find LTRQ.
LBAT = 90° [·: OA _l_AT]
LATQ = 90° - 29°
= 61° 1
[CBSE ·Markin g Scheme, 2015}
: ' Q. 8. In figure, PQ is a chord of a circle centre O and PT
is a tangent. If LQPT 60°, find LPRQ. = DI [Foreig n Set-I, U , HI, ::'. D15l
~) [CBSE, OD S e t-I, II, III, 2015] Sol. LTOQ = 180° - 70° = 1100 l
.,,,,..,,. ._ _ _ ......... ◄ . ... __. .. ~ & ..- - .. - ... , ...--,-..~~...,i.... . _,
[angle of supplem .entary1
i . - - ·-· ..,,

..,
0 .., .,.
( (
l

j
I
I
I
f

Q.9. In fi
....-- --==,.,.=---- .;~
- - . . . . - ---~ -.. . . ---, ....,.-· _,,,. -
! Then, LTRQ = ¼L TOQ
"-

. gure, a triangle ABC is drawn to circum scnbe (angle at the \.'ir1..,--:umfrrtm-."\"'
a Cttcle of radius 3 cm such that the segm.e nts BD ot th ~ circle by ~mt' a t'\:)
illld D '
It th C are respect ively of lengths 6 cm and 9 cm. = l X ll0"'=5 5" 1
of •e area of t\ABC is 54 cm2, then find the lengths 2
~
8
•des AB and AC.
"~tseq~esrtons are . --,
. - - ------ ---.. --
. ·- - -f O tfii.-cfitzi,ta
or practice andiliezr solut1011s are available al .tfie em/o[i
NDARD) , t J,1"! 11 Y.
k Chl\ptcrwlsc {\o\ Topl nv lsr , MAT l1EM ATI C8 (STA
_ _s_w_aal CBS E Que stion Don

. , -~----==------ -- -- - So,
Now
PT ;-;, QT
/'Q = PT + 7Q ,1
Q. l. Prove th a t ttte parallelogram circumscribing a
.
ctrd c is a rh. om b us. tJ fC USf ; Delh i Sci-IL W201

⇒ l
PQ "' /Yr + />T
!'Q = 2t-,-r
- r· I
⇒ 2
PQ = PT 1
Sol. Let ABCD be th~ 11!{1'1_ j
= BC ... (i) ½ to thl'. ,1
AB = CD and AD Hen ce, the com mon tang ent 7
and Q~ c!~ <"
DS + CQ bisects the com mon tang ents at p
AP + PB + DR + CR = AS + BQ +
1
of a bi f
or, AB + CD = AO + BC Q. 3. If a circle touches the side BC angJ, ~ j
and AC
at P and exte nde d side s AB
From (i), 2AB = 2AD or AB = AD . i th t at Q,~/
I,
I
respective y, pro ve atA Q = -2
(BC
+ C,1_ <, .,.,.

~ fJ fCBSEQ D S,l_, ,
Sol. BC + CA + AB
= (BP + PC) + (AR - CR) + (AQ - BQ)
= AQ + AR - BQ + BP + PC _ CR
A

1
or, ABCD is a rho mbu s.
[CBSE Mar king Sch eme, 2020]
02 .:-i'cd Soln tion :
Let ABC D be the parallelogram.
...(i)
.·.AB = CD and AD = BC
wn to a circle from
We kno w that the tang ents dra
t are equ al in leng th.
an exterior poin
Therefore, AP = AS, BP = BQ, CR = CQ and DR =
t the ton
DS. ·: From the sam e external poin , "' gea
e are equal.
Add ing the above equations. segm ents dra wn to a circl
AP + BP + CR + DR = AS + BQ
+ CQ + DS
From the poin t B, BQ = BP
⇒ (AP + BP) + (CR + DR)
= (AS + DS) = AR
From the poin t A, AQ
+ (BQ + CQ)
From the poin t C, CP = CR
⇒ AB + CD = AD + BC
:. Perimeter of MB C,
From eq. (i), + CR- CR;
2AB = 2AD AB + BC+ CA = 2AQ -BQ + BQ
=2A Q I

or, AB =A D !
Hence Proved. 1 !

Hen ce, ABCD is a rho mbu s. ⇒ AQ =- (BC + CA + AB) ;


2
h each other at the
Q. 2. In given fig., two circles touc Hen« proti'll, f
tangent to the
point C. Prove that the common
mon tang ent at P and Q.
circles at C, bisects the com th 8 cm of acin~~!
Q. 4. In figure AB is a chord of leng
e at AWi< · j
radius 5 cm. Tbe tangents to the circl
f
intersect at P. Find the length of AP.
i
f
i
i
I
I.
r
r
{

I
f!!ill.1
[': HSE Delhi Set-HJ, 20201
Sol Since, PT= TC
and QT = TC
poin t]
[tan gen ts of circle from exte rnal
~ •.

~ CIRCLES II
1
A ⇒ AM =4+x:=AN

',ol• p: Now AB = AN + NB = 10
⇒ X + 4 + X = 10
⇒ X =3
[Tangents from external point ,AJ

1
1
:. BL= 3 cm, c~_,~_5- ~ ~d AN =: 7 <:111·
AB= 8cm
AM =4cm.
OM= ✓oA 2 -AM 2 ; // \
AP== y crn, PM = x cm.
OM=
[By Pythagoras theorem)
✓52 -4 2 =3cm. !
l
\
r M
/ _..::: ,,ge= ---- C
/ B'--
L-et • . gl I

.. ~OAP is a nght ang1e trian e. ( -<- - Bern ~


' OF2 = OA2 +AP2
and
[By Pythagoras theorem] Q. 6. The radii of two concentric circles are 13 cm
8 cm. AB is a diameter of the bigge r circle and BO
(x + 3)2 = y2 + 25
is a tangent to the smaller circle touching it D at
x2 + 9 + 6x = y2 + 25 ... (i)
.
~
x2 + 42 = y
.;). ... (ii) and intersecting the larger circle at P on producing
Also,
x2 + 6x + 9 = x2 + 16 + 25 Find the length of AP. ~ DJ [CBSE SQP, 2018-19J
from
6x = 32 Q. 7. Prove that the lengths of two tangents drawn
32 16 an external point to a circle are equal.
x= - or - cm
6 3 FJ [CBSE OD Set-I, Il, III, 20181
g
Sol Given: AP and BP are tangents of circle havin
y2 = x2 + 16 = 2S6 + 16
9 centre 0.
400
9
20 2 I P.- -- --+ -- --+
y = - cm or 6- cm. I
3 3 I

in a
I
Q. 5. In the given figure a circle is inscribed i
MBC having sides BC =8 cm, AB = 10 cm and
AC = 12 cm. Find the length BL, CM and AN. To Prove: AP = BP

,- !:~\ --- Constructio n: Join OP, AO and BO


Proof: .l\OAP and .l\OBP
OA = OB [Radius of circle]
§ ~ OP =OP [Common side]
Mi
'!
,
~N

X \
d)~ \

B----.,.___;;:::,,...::::;_,,_- ...l C
41
:
L'.OAP = L'.OBP = 90°

WAP =.l\OBP
[Radius .l tangent)
[ByRHSJ
8-x . AP =BP [By cpctJ
Hence Proved.
lm)g [CBSE Delhi Set-II, 2019J Detailed Solution.:
[Delhi Set-I, II, III, 2-016}
- SoLLtt BL~; ~BN' '. ,. ,_.
{Tangents from exfemal point 51
CL =8-x = CM . ·
. [Tangents from external point C) -OH M•~ t~.~_.....,- _ __
~ ,. AC == 12 . - I- ~ ~ ~-. kl._.~ -~ --

-:--
:. ~ ~PTadia ::-aw
ons:--::art ilab
=:-;;- =d;-::of,;;tht~cha
;:-:-en
:-;:;lt:-::at;-;;tht = - --
pter
:;:::;- - --
z.sol
and thtir uli=
:::;:::
fer
ill Oswaa l CBSE: Qu estion Bank Ch npterwise & Topicw
ise , MATHEMATICS (STANDARD) , Class-x

'; ~ 0 f.._t , M~, pa..


⇒ AB2 == 32
t..,,ill(u,·o,,,, l,i,. on,oo ~ or. Hence, AB = ✓32 :: 4Ji err.
, f',-c.ct, !j,,,·{?)la,-,J . (•~ ~ (
0't_C
LOAP ~ Lc~f -:-qo• (~ 411"1Wl • [CBSE ~rkin,, S h
6 . C ~
Q . 9. ABC is a triangle. A circle touche s si'd
C: "~ ,. , ~ ~ , , es A•rr ~,i
produ ced and side BC at X, Y and z r~ ii "'t. .
~ t ~t ,oi"" o( ~ 1 Show that
1
Pe-r.1:,, ,
~t ~ Ot ( ~V~14 tt) ·
AX = !2 perimeter of MBC.
/ FJ {CBSETerl!i.11,.
I- Sol. Try yourself like Q. 3. SATQ-II.
I L.

\ -

lwlfili@~
Q.1. Prove that oppos ite sides of a quadti.i,,.
) O i.., ~OM ..ct -6,Qlt, _ circumscribing a circle subtend suppl- '<:i.
angles at the centre of the circle. •u•en~
)y i~ ~""" ~Y, ·
~ .:\o~P'~ AO~. f4 [CBSE Delhi Region,:-- :
by~ [CBSE Foreign Set-I, ll, IU.~:-'
=) Ai>-:: 81>- Sol. Given: A circle with centre O is inscribed ;.i, :
quadrilateral ABCD .
},u,t.a (11'o~ -
l
I In MEO and D.AFO,
~------ ----------- ------------•- •- -----------A OE = OF
Q. 8. In the given figure , PA and PB are tangen ts to a [radii ofciir.c
LOEA = LOFA = 90°
circle from an extern al point P such PA = 4 tha~
cm and LBAC = 135°. Find the length of chord [radius is 1r to tangen:i:
,..--- - - - -B
-
AB.
- 4 - - -

p A

I
\ F 1---.....e-'. .,..--, ,. I

The point of contact is perpendicular to the tanfw!:


OA = OA [conuooni:i \
I

DJ [CBSE OD Set I, II, III, 2017]


MEO = MFO [ByRf-; I
I

L7 = LS (By cp.:t) ...r · 1


Sol. PA =PB =4cm
Similarly, I
[Tangents from external point] ¼
Ll = L2 .I
LPAB = 180" - 135° = 45°
L 3 = L4 .- I
(Supplementary angles]
L S= L6
.I
LABP = LPAB = 45° L 1 + L 2 + L3 + L 4 + LS + L6 + L? +
,s- ;-_•·:
- . ·.
[Opposite angles of equal sides]¼ [angle around , f ·
LAPB = 180° - 45° -45° 2 Ll + 2 L S+ 2 L4 + 2 L5 == 36(l''
= 90° L l + L B+ L4 + L5 == lSO"
So, t:.ABP is an isosceles right angled triangle. (L l + LS) + (L4 + LS) == 1800
LAOB + L COD == 180° J'r(ll'~1·
AB2 =W 1 Hence
I
• I

I
I
• I
I
> L Ab~

-J> L f> b +-L


'I
I

Q, 2· a, b and c are the sides of a right triangle, where c Let circle touches CB at M, CA at N and AB a t P.
is the hypotenu se. A circle, of radius r, touches the Now OM ..L CB and ON ..L AC [radius ..L tangent}
OM == ON [radii]
. a+b-c
sides of the mangle. Prove that, r = CM = CN [1angents] 1
2 · :. OMCN is a square.
Let OM = r= CM== CN 1
E,i [CBSE Term-II, 2016] AN =AP, CN = CM an d BA1 = BP
Sot. [tangen ts fron.1 exkrnal point]
A AN= AP
⇒ AC - CN =AB - BP 1
/J - r = c-BAI
li - r =c- (11-- r)
b-r=c- a+r
2r =a+ li- c
!l+b - c .
,. = - - -. He1we Proved, 1
2
a
1 lCllSil Matking Schem e, 1-0161
ffflPI MATHEMATICS
~
I a. Toplr:wiNc (BTA.llfDAKD) (' )
f".l".-.,a o'"·'"I ens>: Qu<0llon Bnnk Ch,.ptcrw " • · ' - • •' " X
~
. - 71, ~ ll
1.. pol nl •u<h that OT J3 cm ,nd O I Int,,..c!B ~
Q. j , ht J1tg. 0 11'1 LIH' ('(!J\Uf' of a d rde of ra~i.1.18 5 Cff), r
ctn:I< ,i E. If /\B 1, , tangent to the circle al E,
StJl.
and
trr :,,g

TE ~ OT - OE., , .._
fi nd th,• lcrtgth of /\ D, where Tr and TQ ,re two = 8 cm • • l., - ;
Let PA :,::, AE a X. :~{
tartgcnts to the circle.
r Then, TA 2 .c: Tf!t + £Ai fr:z:t~ ·
/
A
or, (12 -xfl' = B2 + ?-
2Ax = 80
or, _ 3.3 cm. (Apfroiq
X ::: .

Thus AB - 2 x .r. == 2 :,: 3_3


\ = 6.6
,
cm. (Apn,, .
r·OX..j
[CBSE Markfrig s(; h~ c:,,
l!) !CBSE D<>lhi Set-I, TI, Ill, 2016]
Q. 4. Prove that tangent drawn at any point of a circle is perpendicular IO the radius through the point of ., •·
m [CBSEODc~
~1-11 ,

Sol. Jl~ - . R l ~ - ( ( O, q PJ...bM.d . f.tU1jtM:i -


--- - ---~·-- - __ . - --
---- .. -~ 1-,---
~ - =- _f)_f_ JP~.·-- -- - - - - - . \ - - + - ~
~ i ~ ly_fuu/. _a_;..cR.,__.t....,.JJl.J.S.J-j1~atL-·· ,.~--¼~-=:-~~~=-~::_~
1.Md(-7' Z<U MOJJI
ae ; o g ( Jv(MW.)4 )

. c) ~Of J. AB
A} ~ p J. p~
tiJM.&.bJ.NH<i
7
....... .. - ,... ..... - . .. . ,. ... . .- ............. _,_ ,... ____
Q. 5. In the given fi gure, o isthecentreoftheci
~ ~

- · - - - - - -I· - - • - - - - .• - - - - - - - - - - - - - - - - - - _- - - --- ------- --·-·· ··


re e. Oetermme L'.APC' if 0 A and DC are tangents and LADC : : ~•

A __...

:~ ·rlr& ➔;~,;t·;;;;~;for pnu:/i;-· ,L"".~·-


1111' , - ·- art
1t sol11tlo11s - '' e-Al the end
- 111N1il11L/ - of /lit-cha11ter
Ill
I • · ~u ~
.
CJP.CU ~S
_
.... .
... J 4 .. ' •. t_\)

I ·ffi(~~~--:
tfl: . ~~£-!"'~·1 . ·"·' ' ":'
~ -~~'::'l<l_~ ~ • 1

L l<OQ + 2Hf == 3€/f


assage below and answer the foll ,..:_ L.ROQ ::: 3fff - 210"
J~e P O"utg
iea"
I, uottS, LROQ = 1S<r
qoeS . wheel (or a big wheel in the U 't d Q. 2. Find LRQP.
ferf!S n1 e
A )
. dorn is an amusement .
ride consistin f
g o a (A) 75°
(B) €ff
!(illg upright wheel with multiple pas
iati11g senger- (C) 300 2
ro . components (commonly referred t (D) 90"
afl'Y111g . o as Ans. Option (A) is correct.
c er cars, cabins, tubs, capsules, gond
asseng o1as,
P ds) attached to the rim in such a way th t Explanation: In .1OQR
or po a as
eel turns, they are kept upright usuall b
thewh , y y LOQR = L.ORQ
gravilY· LROQ = 150°
After taking a ride in Ferris w~eel, Aarti came out and LROQ + LOQR + LORQ = 180"
fro!11 the crowd and _was observing her friends who
150° + 2 LORQ = 180"
were enjoying the nde. She was curious about the
different angles and measures that the wheel will 2LORQ = 30"
fortn• She forms the figure as given below. LORQ = 15°
- - LOQR = LORQ = 15"
Now LRQP = LOQP - LOQR
= 90° -15°
= 75°
Q. 3. Find LRSQ.
(A) 60° (B) 75°
(C) 100° (D) 30°
Ans. Option (B) is correct.

½
Explanation: LRSQ = LROQ =75°

Q. 4. Find LORP.
(A) 90° (B) 70°
(C) 100° (D) 60°
Ans. Option (A) is correct.
Explanation: L.ORP = 90°
m[CBSE QB, 2021] Because, radius of circle is perpendic;ular to tangent.
Q,l. In the given figure find L.ROQ. II. Read the following text and answer any four
(A) 60° (B) 100° questions from Q.1. to Q.5.
(C) 150° (D) 90° 1 There is a circular field of radius 5 m. Kanabh,
Ans. Option (C) is correct. Chikoo and Shubhi are playhlg with ball, in which
Kanabh and Chikoo are standing on tht' bow,dary
Explanation: LORP = 90° = L.OQP of the circle. The distance between Kanabh and
[·:radius of circle is perpendicular to tangent] Chikoo is 8 m. From Shubhi point S, two tangents
:. LROQ + LORP + LOQP + LQPR : : : 360° are drawn as shown in the figure.

LROQ + 90° + 90° + 30° = 360° [ijII liJ + 19


. . . .iaTl{EMATICS (STANDARD), Cla, 8 _.,,
. .,. Top1CWJ5C , ~ · "'-
~ . Bank Chaptcr.,nse °'
IL=£·: o~ial CBS£ Que oon5
· Ans· Option (D) is correct.
Explanation: llSKR and ..1RKO,
LRKO = LKSR
and LSRK = LOR.K
t:J<SR ~ tiOKR
SK RK
11,en KO RO

SK = i

S 3

[RO = 3 m, proved .
35K = 20 in fJ"

Q. 1. l-vh.at is the relation between the lengths of SK and 20



SK= -
SC? 3

I,. (A) SK # SC (B) SK== SC Hence, the distance benveen Kanabh and ShiJ.hl.,;
(C) SK> SC (DJ SK < SC 20 '•.\
Ans. Option (B) is correct. -m.
3
Explanation: We know that th e lengths of tangents
Q. s. What isthe mathematical concept related lo .
drawn tram an external point to a circle are equal.
So, SK and SC are tangents to a circle with centre 0. question? ~
1 (A) Constructions (B) Area
SK= SC
(C) Circle (D) None of these
Q. 2. The length (distance) of OR is:
Ans. Option (C) is correct.
(A) 3m (B) 4m
Explanation: The ~athematical concept (Cirde) ~
(C) 5 m (DJ 6 m
1 related to this question.
Ans. Option (A) is correct.
Explanation: In question 1, we have proved
SK =SC
Then ~KC is an isosceles triangle and SO is the
f~c. --
I. Read the following and answer any four quesf:ions
angle bisector of LKSC. So, OS 1- KC. from Q.1. to Q.5.
:. OS bisects KC, gives KR == RC == 4 cm.
Vanm has been selected by his School to design logo
= ✓oK 2 -
2
Now, OR KR for Sports Day T-shirts for students and staff. ThE
[By using Pythagoras theorem] logo is designed in different geometry and diliereni
colours according to the theme. In given figure, a
= ✓sz _42
circle with centre O is inscribed in a MOC, such
= ✓'25-16 that it touches the sides AB, BC and CA at points
D, E and F respectively. The lengths of sidesAB,BC
= ✓9 and CA are 12 cm, 8 cm and 10 cm respectively.
=3m. [CBSE QB, 20::;
Q. 3. The sum of angles SKR and OKR is: 1
(A) 45° (B) 30°
C
(G 90° (D) None of these
An.s. Option (C) is correct.
Explanatwn:
LSKR + LOKR = LOKS
= 90°
[Radius is 1.' to tangent]
1 A
Q. 4. The distance between Kanabh and Shubhi is:
10 13 Q. 1. Find the length of AD and BE.
(A) ~ m (B) m
3 Sol. Let AD be x can, then DB = (12 - x) cm
(C) 16 m 20 AD = AF, CF = CE, DB = BE
(D) -m
' 3 3 'ti
[Tangents to a circle from an external poli1
AF== x cm,
CJR.CJ.,ES Ill
CF == (10 - x) cm inversion centre as one of the limiting points. Two
concentric circles are of radJj 5 cm and 3 cm given in
BE == (12 - x) cm,
the following figures.
CE == 8 - (12 - x) === (x _ 4)cm -
CF== CE
10-x == x-4
: \
! 0
2x == 14 I

X == 7. A
AD== 7cm. 1
Q. 1. Find the length of the chord of larger circle which
BE == (12 - x) cm touches the smaller circle. Also find the area of
== (12-7) cm these two concentric circles.
Sol. Here AO = 5 cm and OC == 3 cm.
[ ·: x === 7 proved]
= 5cm 1 AC = l✓<A0)2 -(OC)2 I
the radius of the circle is 4 cm, find the area of
Q.2·:oAB, Also £ind the perimeter of MBC. [By using pythagoras theorem]
OD= 4cm
sot. RadiUS, = 1✓(5)2 -(3)21
AB= 12cm
and
= 1✓ 25 - 9 l=IJ16I
C

=4cm
Hence, the length of chord AB = 8 cm. 1

Area of concentric circles

A D B: = n(r/-r\)
~<------➔) :
12cm = n[(5)2 - (3)2]
Then, area of ~OAB
= 16nsq. cm. 1
Q. 2. If radius of circle is 7 cm, then find the area of
1
= -xODxAB circle. 1
2
Sol. Radius of the circle, r = 7 cm
1
= -x4x12 Area of the circle = nr2
2
22
= 24cm2• 1 = -x7x7
7
Perimeter of MBC = AB + BC + CA
= 154cm2•
= (12 + 8 + 10) cm
Q. 3. Find the area of ~OAC. Also find the perimeter of
= 30cm. 1
aOAB. 2
Q. 3· Find the length of CF. Sol. Given, OC=3crn
Sol. From question 1, and AC= 4cm
CF = (10-x cm) [Proved in question 1]
= (10-7) cm Area of aOAC = .! x oc x AC
= 3cm. 1 2
II. Read th
fr e following and answer any four questions 1
OQ\ Q.1. to Q.5.
= -x3x4
2
Concentn .
1'h c circles are circles with a common center. 6cm2•
===
e re!>io b etween two concentnc · or
· cles of 1
d.iff 0 • n
Perimeter of aOAB = OA + AB + BO
c erent r dii' ·
a 1s called an annulus. Any two ctr · cles
=5+8+5
an be made concentric by inversion by picking the
=== 18cm. 1
. )IA.TIIEMATICS (STANDARn), Class-}(

Im Oswaa l CBSE Quc~ tl<m Bn nk


C llll P( l'fWIS
' C &,
.,. picWISe,
0

.~·· -- . ,Sol~tion~Jor practice Questions ' •


-- ~ ' , ~ ,\ :;.·

; M.~ '2lw 2\lt. ttlo. m


: Ans.1: Option (0) is correct. Sol. 6:
circulJl sCribin g
I
, faJ •l1ma tio11: s·m<."l'. ,1u:1dri
-i '
later,l1 aneles at the
: 11 drde subt('nd!- ~upplementary 0

' ~"\>ntrc of the circle.


iAOf! + LCOD = 1800
12..~~ + i.COD = 180°
LCOD = 1800- 1250 = 550
\ Ans.2: Option (C) is correct. d LAPB = 90° [angle in serni.cir 1
Expla11ati.on: Since, the angle between
b dchord
d by an
the DB _ o[ di .
tangen t is equal to the angle su te~ e and LO - 90 ra us 1s Petpendi dej
same chord in alternate segment of mcle. tOCUlar
\ , MBP ~ llOBD tangent]
' ⇒ LBAT = 50°. AB AP
I
• = -
: Very Short Answer T~pe Que5ttons
I OB OD
I l.
: Sol. 9: PQ = PR = 5 cm
26
=-
AP
and PQ = QS 13 8
PS= 2PQ Hence, AP = 16 cm
I
= 2 x 5 = 10 cm. ¼ [CBSE Marking Scheme ,, ,. 1 .
I
[CllSE Marking Scheme, 2017] Long Answer Type Questions ' '"0.10-\91 '
'
I '
: Sol. 10: In t.OQP, LPOR = LOQP + LOPQ Sol. 5:
[Exterior angle]
LOPQ = LPOR - LOQP
= 120° - 90°
I = 300
: Short Answer Type Questions-I
I

: Sol. 8: Given, LQPT = 60°


I Given DA and DC are tangents from point Dtoa1 ·
circle with centre 0. :
LOPQ = LOQP = 90° - 60° = 30°
Ll = L2 = 90•
LPOQ = 180° - (30° + 30°)
[radios .l tangent] 1
= 180° - 60° = 120° L1 + L'.2 + L3 + L4 = 360°
or, 90° + 90° + 50° + L4 = 360° l ,
I
1
LPRQ = - Reflex LPOQ 1 or, L4 = 130°
2
Reflex L4 == 360°-130° = 230° 1 •
I·: Reflex LPOQ = 360° - 120° = 240°]
LAPC = .! reflex L'. 4
= !2 X 240°= 120° .2
[angle subtended at centre! ·
[CBSE Marking Scheme, 2015] 1 l
LAPC = -x230 ° = 115° I·
2

t
:I The use of a circle Storts &om the ti f l ~~--~~~~i~a ,.
i ~~~~ :)~~~~~~r~~: _o~~irde ?
. . ~, ..
p o t le pen to the shape of planets, Will you able to solve all comple
. . .. ...... ,._ - ..... ....... __ x questil-11);.. ·
- . . ...
-.. . . -
~ _, ,._ ......
- -. . ... .. ,. -
SELF ASSESSMENT) PAPER-4
CIRCLES 183

hour
Merimum Time: 1

the.correct alternative from the given options. MM: 30


Choose
givenfigure, AB is a chord of the circle and AOC is its
L
atInthe
circleatthe point A, then ZBAT is equal to: diameter, such that (1
ACB = 50. If AT is the 6 6
tothe tangent
55
B

(A) 65° (B) 60


(C) 50° (D) 40°
0.2.Thelengthofthe tangent drawn from a point 8 cm away from the centre of circle of radius 6 cm is:
(A) 2/7 cm (B) 7/2 cm
() 2V2cm (D) None of these
two radii of a circle is
If the angle between 130°, then the angle betweern the
their point of intersection is: tangents at the end points of radii at
(A) 70° (B) 60
() 50° (D) 40°
||
0.4.InAABC, DE BC, then the value of x is:
A

x+2

D
x+1
x+4

3
2 (B) 2

5
(C) 2 (D)
KP 5
Q.5.In the figure, PQis parallel to MN. If and KN =20 cm, then KQ is:
PM 12

M N
(A) 5.9 cm (B) 6.8 cm
(C) 5.7 cm (D) 6.7 cm
MATHEMATICs(STAMDARD
Topicwise,
184 Oswaal CBSE Question Barik Chapterwise & 0PR 46,them
centre O. I!
a cirde with
Q.6. In the givenfgure. PQand PR aretwo tangsnts to

(B) 134
(A) 67 (D)6*
(O 44
II. Case-Based Question questionsit Throwbal
given below
Q.7. Read the following text and. answertheOrganized afour davs Intechool
Throwtall team captain
The City Modern School in Delhi has this competition. The schoolpreparation onlv Piva,
The participants were very exited for
to pre-boardexam standing at the centeSwai d
due Act
her team members for practice match the plav ground. Priva waspositon. From the point thets
but
on
Anushree drew a ircle of radius 5cm fromthe centre Aas her own
and Anita as S and R ,t
away
Anushree markedia point N, 13andmNR and gave positions to Swati Priva throws if to respeTey.
Swat
drew two tangenial lines NSAnushree throws the ballto the Priva, Priva, Priva throwsitto
all started to plav throwball.
Anushree. Anushree again throws the ballto Anita, Anita
throwsit to
St

ofNSA?
(i) What is the measure (B) 45
(A) 30 (D) 90
() 60 Anushree is:
between Swati and
(iü) The distance (B) 14m
(A) 12 m (D) 18 m
(C) 15 m Anita?
to throw the ball to
(iii) How far will Anushree have (B) 14 m
(A) 12 m
(D) 18 m
() 15 m
following is correct ?
(iv) If LSNR = 8, then which of the (B) SAN = 90° -9
(A) ZANS = 90-e
(D) ZRAN = 0
(C) RAS = 180 - 0
(v) IfZSAR = 0, then NAS is:
(B) 90-0
(4) 90° 2
(C) 180 -20 (D) 90 +9
III Very Short Answer Type Questions
between the tang
Q.8. If achord AB subtends an angle of 60 at the centre of a circle, then find the angle
B.

A
tangentdrawn froM an external
nB oint to a
a i15 then f5nd the measure of ZOPO
AOR= oe
R
itticeitre ).Unt åarec

AD 3
DE|| BCand DB
=. IfAC = 5.6 cn, then find AE.

Questions-I [2 x3 = 6]
Answer Type
T Short concentric ircles are 4cm and 5
cm, then find the length of each chord of one circie whichis
the,radii oftwo circle.
Q11.If other
tangentto the Aat distance 6cm!from the centre of the circle is 5cm. Find the radius of the
length of atangent from a point
Q12The
circde.

6Cm

A
B 5 cm

that: AD = AB xAF.
and CD || EF, Prove
013.In the given figure, DE | |BC A

[3 x 2 = 6)

DE + FA.
CD +EF =BC +
V. Short Answer Type Questions-I
circumscribe a circle,then prove
that AB +
014. If ahexagon ABCDEF ZCABis isosceles.
BE ZCED, prove that
AD
and ZCDE =
Q15. In the given figure, if DC EC

B
A
186 Oswaal CBSE Question Bank Chapterwise & MATHEMATICcs (STANDARD), Class-X
Topicwise,
VIL Long Answer Type Questions
the
Q. 16. Ais a point at a distance 13 cm from AP andAQ are
the centre 'O' of a circde
Pand Q. If a tangent BC is drawn at point R yng on minor
of AABC.
of radius 5 cmn.
arc PO to intersect AP tangetehes peta
at BAQat C. Fina t

OSWAAL COGNITIVE
Finished Sotving the Paper ? LEARNING TOOLS
Time to evaluate yourself !

OR
SCAN THE CODE

For elaborated
Solutions
UNITV TRIGONOMETRY

CHAPTER

TRIGONOMETRY &

9 TRIGONOMETRIC
IDENTITIES
ratios of an acute angle of a
Iutroduction to Trigonometry: Thigonometric (well defined); motivate the
eristence
Syllabus right-angled triangle. Proof of their Values ofthe trigomometric ratios of
O'and90.
ratios whicheverare defined atbetween
Relationships the ratios.
30,45°and60. sin' A +cosA =1.. Onlysimple identities
of the identity,
Trigonometric.Identities: Proofandapplications
tobegiren.
List of Topics

In this chapteryOu will study ofa triangle


Topic-1: Trigonometric
between the sides & angles
Ratios and Its Values
Study ofrelationships
between 0° to 90o
Page No. 187
Trhigonometric ratios
ofaright angled triangle with their sides. Topic-2: Trigonometric
Trigonometricratios 6 trigonometric
ratios. ldentities
their
Trigonometric identities with Page No. 195

Values
Trigonometric Ratios andIts
Sx trlgonometrle
Concepts Coveredtrigonometrie ratios with their stdes of aright angled
101C ratios between 0 to 90
trlangle, Values of

ERevision Notes
angled at Bis given
In fig, aright triangle ABC right Here
and ZBAC= 0is an acute angle. side AB which
0s adjacent to ZA is base, side BC opposite to ZA is
O Key Words
perpendicular and the side AC is hypotenuse which
is opposite to the right angle B. Hypotenuse: The longest side of a
Perpendicular right angled triangle.
H y p o t e n u s e Base: Adjacent side of an angle is a
i base of a triangle.
Perpendicular: Opposite side of an
90° angle in the right triangle.
4.
B
Base
Toplewlne, MATHIEMATICH (TANDARD
Oawaal(CRNE QueutlonMant Chupterwine &

co= ()NoN
detined
AB AC C
ACBCAB
snt=
reiation
ACZA=* BC AB *through
proper
Brashing"tant )N
A
to A=
Site toc
opposite
Side A= ZA ofSecantA=
ZA
tiehare"Hair
ofSine ofCosineofTangeat
of of learn
Cotangent CosecantpeopieBrowa
to 45
How Curiy
<Some
Note: 30

(«)
Notdetined

cos
A tan
A Cot
A
A

ratios
between
trigonometric
triangle Values
of
between
Trigonometric
a
relationships
of
angles

&
of sides
Trigonometry Ratios

Trinometgor
<igonometr,
and
Study
the 1dentities

Relation
oftancos Trigonometric
Identities
&
sin
with Thid
Level
A
sin?
=J1- Map
A Second
Level
sin 1 Mind
A
of A cos
terms the
sin i.e. =1 seAA
1-sin2
A A
cosec? Traxe
in +A A sin²sin
A sin² "irst
Level
A cos tan?A=
+1=
cos =l- +
A
and Since, cos² A
A sin
A A +
A cos² cos
Solution: 1 cot
tan 0sAs90
0sA<0
0<As90
Express =
tan
A
TRIGONOMETRY AND) TRIQONOMETRIC IDENTTTIES 189

KeyFormulae
tronometrieratios of LAin right triangle ABC are defincd as Scan to know
The
more about
sine of ZA = sin =erpendicular or opposite side BC
this topic
Hypotenuse AC
cosine of ZA 080 Base or adjecent side AB
Hypotenuse AC

tangent of LA= tan 0 erpendicular or opposite side BC Trigonometric


ratios
Base adjacent side AB

cotangent of ZA = cot 0 Base or adjacent side AB


Perpendicular or oppsite side BC tan0 Scan to know
more about
secant of ZA sec 0 = Hypotenuse AC 1
Base or adjacent side AB COs0 23A0
this toplc

cosecant of ZA =cOsec0 == Hypotenuse AC 1

Perpendicular or opposite side BC sin 0

Perpendicular Trigonometric
ratlos with Angles
Hypotenuse

909

Base

Hiscdear from the above ratios that cosecant, secant and cotangent are the reciprocals of sine, cosine and tangent!
spectively.
sin0
Also, tan 0 =
cos0
cos
and cot 0
sin0
The trigonometric ratios of an acute angle in aright triangle express the relationship between the angle and
length of its sides.
The value of trigononmetric ratio of an angle does not depend on the size of the triangle but depends on the angle:
only.
30 45° 60° 90

1 1
Sin A 2
2

V3 1 1
COs A 1
2 2

1
0 1 J3 Not defined (0)
V3
1
Not defined (o) V3 1
V3
1 V2 2 Not defined (o)
2
Not defined (o) 2
MATHEMATICS
190 Oswaal CBSE Question Bank Chapterwise &Topicwise,
Bhole
(88TANDARD), Class-x
B
Fundamental cot 0 =
Prasad p Base
Facts
(1) The oncept of trigononmetry is sec =
Har
Badri
H
B
PerHypotpendienusecular ACO A

ompletely based on right angles. Har H


Base BA

PerHypotpendienusculear
(2) The three basic functions in cOsec =
Pandit P
trigonometry are sine, cosine and AC

tangent. 2. Trigonometric Ratios


Mnemonics: We learn these
(3) Trigonometry, as the name might ratios in
suggest, is allabout triangles.

Mnemonics
(i) "Some people have" sin = P

(ii)"Curly Brown Hair" cos 0 =


H

H
B
fol wingwAY
The relation of Trigonometric Ratios
Mnemonics: (ii) "Through proper Brushing" tan = P
B
In right angled AABC, we have BC P
(i) sin =
BC BA BC AC H
sin 9 = Cos = tan =
AC AC AB
Interpretation:
AC
cot =
AB
sec = COsec =
AC Some feople Have
BC BA BC

sin- cOS tan,


sin 0
Perpendicular Hypotenuse
Pandit Badri Prasad C
Har Har Bhole
Lcosec sec cot H P

90°
B

Hypoternus
B

AB B
(ii) cos =
AC H
B Base A Interpretation:
Brown
Interpretation: Çurty Hair

Here, Base Hypotenuse


sin =
Pandit P Perpendicular BC BC P
Har H Hypotenuse AC
(iii) tan 9=
AB

COs A
Badri Base BA Interpretation:
Har Through Brushing
Hypotenuse AC
J Proper
tan A Prasad P_Perpendicular BC tan 9 Base
Bhole B Base AB Perpendigalar

OBJECTIVE TYPE QUESTONS (1 markeach)

40
Multiple Choice Questions (A) 1 (B)
1 38
Q.1. If cot8= ,the value of sec0 + cosec0 is (C) 9 (D) 5;
OUCESE, Board erm-l,.
These questions are for practice and their solutions are available at
the end of the chapter
TRIGONOMETRY AND TRIGONOMETRIC IDENTITIES 191
7
rightangled at B, sin A = 25 then the Q.5. In the given figure AABC is an isosceles triangle,
right-angled at C. Therefore
cosCis: B
ralueof 24
(B)
25

24
(D) 7

RCBSE, Board Term-I, 2021]


Giventhat sec = 2, the value of 1+ tan9 is
C A
a3 sin 9
(A) AB = 2AC2 (B) BC = 2AB?
(A)2V2 (B) V2 (9) AC = 2AB? (D) AB² = 44C2
( )3/2 (D) 2 U[CBSE, Delhi & OD, 2020]
correct.
RICBSE, Board Term-I, 2021] Ans. Option (A) is correct.
Ans.Option (A) is
Eglanation: It is given that Topper Answer, 2020
sec = N2 ..)
Also, sec 45° = J2 ..ü)
From (i) and (i), we get
0 = 45° -AR°=Q4c
Put value of 0 in 1+ tan
sin
!
1+tan 1+ tan 450
sin 9 sin 45° Detailed Solution:
1+1 From the given figure,
AC = BC (given).i)
:.In right angled AACB,
AB = B+ AC2
= 2W2
(Using Pythagoras Theorem)
0.4.Inthe AABC show below, X
:Y= 1:2. = AC2 + AC
C [from eq. ()]
= 24C2
Q.6.If AABC is right angled at C, then the value of
cos (A+B) is:
(A) 0 (B) 1
1
(C) 2 (D)
2
1 1
0.7. Given that sin a = =and cos B= ,then the
What is tan x? 2 2
1
(B) 1 value of (a + B) is:
(A) 0° (B) 30°
V3 (D)V3 (C) 60° (D)90°
() 2 Ans.Option (D) is correct.
DOC[CBSE Q.B., 2021-22 Explanation: Given, sin g===sin 30°
Ans. Option (A) is correct.
Explanation: LX: LY =1:2 (given)
Let ZX = aand ZY = 2a, then in AABC
4 + 2a + ZB = 180° [Angle sum property of A]
[.: ZB =90°, given] a=30°
3a =90 And,cosß==cos60°
a =30°
Now tan X tan a B=60°
tan 30 =
1 yt B= 30°+ 60° =90°
These questions are for practice and their solutions are avallable at tthe end of thechapter
Oswaal CBSE Question Bank Chapterwise & Topicwise, MATHEMATICS
192 (STANDARD), Class-X
SUBJECTIVE TYPE QUESTIONS
0.6. If tan (3x + 30°) = 1, then find
Very Short Answer Type the value of z,
Questions
0.1.If sin A + cos B = 1, A = 30° and B is an acute
(1 mark each)
Q.7. What happens to value of cos uBoard
from 0° to 90° ? when
angle, then find the value of B.

ADRICBSESQP 2020-21]
Sol. cos e decreases from 1 to 0. A[Board Ter9min-c1re.xa
[CBSE Marking Scheme,
Sol. sin 30° + cos B = 1
1
2
+ cos B=1 Short Answer Type
Questions (2 marks each
cos B= 1-! 3 1
2 Q.1. If tan A = find the value of 1
4' sin A cOSA
ie., cos B = cOs 60° s60°=

tan A 3
ADRICBSE SQP 2020,
3k
Hence, LB =60. Sol. Given that,
[CBSE Marking Scheme, 2020-21] 4 4k
Sin A 3k 3
Q.2. If sin a =
2
and cos B=0, then find the value of
5k
B-a.
Q.3. Evaluate: sin² 60° + 2 tan 45° - cos30°
OU[CBSE SQP 2020 COs A =
4k
5k 5
ADACBSE OD Set-I, II, 2019]
1
Sol. sin60° + 2 tan 45- cos2 30°
sin A cosA 3 4
20+15
12
[For any two correct values] ½ 35
=2 (CBSE Marking Scheme, 2019] ½ 12
Q.4. If sin = cos 0, then find the value of 2 tan
+ cos² 0. [CBSE Marking Scheme, 2020-21)
U[CBSE SQP, 2018] Detailed Solution:
Sol. Given, sin 0 = cos 0 ’ =45° 3
5
We have, tan A =
4
2tarn + cos0 =2+
2
Perpendicular
[CBSE Marking Scheme, 2018] 1 Base
Detailed Solution: i.e.,perpendicular =3k and base = 4k. (ke
We have, sin 9 = COs 0 Let ABC be a right angled triangle, then BC =
We know that, cos = sin (90° -0) and AB = 4k
sin 9 = sin (90° -0)
0 = 90°-0
20 = 90°
0= 45°., 3k
Now, 2 tan 9 + cos²e= 2tan 45° + cos 45°
2
A
-2x1+( Now
4k

AC² = AB² + BC?


(By using Pythagoras theoren
= (4K) + (3k)
0.5.)If sec 0.sin = 0, then find the value'of 0. = 16k + 9k2
RBoard Term-1, 2016]
= 2532
available at the end of the chapter
These questions are for practice and their solutions are
TRIGONOMERY AN) TPIGOHOMETPIC IDEAKTITIES 193
AC 5k
BC 3k 1 1.732 0.577
sín A rs (i) tan 30 we
Now, AC 5k 5
AB 4k 4

and AC 5k (ii)sec 3r +cosec AÝ


2
+2
243+1)
1 1 1 55
Hence, sinA
4
COsA 3/5 4/5 3 4 21(3+3)
3
20+ 15 35
2.821 (Approx.h
12 12
Q.4. Evaluate:
3sin - COs
) = 0and 0° <0<90°, find the 3 tan 30 +tan' 6Y + cosec 30- tan 45
valueof0. ADUBoardSQP, 2020-21| cot 45
v3 sin 0 -cos ) = 0and 0°<0<90° DOUBoard Term-1, 2016]
Sol. Here =03A + B
1
J3sin 9 = cos 0 Q.5. If sin (A + B) =1and sin (A - B)
sin 9 1 1 and B.
<90 and A > B,then find AuBoard
Cos9 Term-1, 2016]
tan =
0.6, Find the value of:
sin 30°. cos 60 + cos 30. sin 60
cos 90° ?
Is it equal to sin 90° or AOUBoard
tan9=
sin 9 Term-1, 2016]
= tan 30°
cos
sin 60
1 Sol. sin 30° cos 60° + cos 30
9=30°, 1
antenna is fixed at right an
63.the rod AC of TVdisc
gles to wall AB and a rod CD is supporting the disc
AC=1.5m long and CD =3 1 3
as shown in figure. If .
+ cosec 4 4
m, find (i) tan and (ii) sec
1
4 1
4
90° as
A 1but not equal to cos
It is equal to sin 90 = Marking Scheme, 2016]
cos 90 = 0. [CBSE
60° geometrically.
. Findcosec 30° and cos U[Board Term-1, 2015]
D
B A

Wall Sol.
AOC+ RÊCBSE OD Set-II, 2020] 30
AC = 1.5 m
Sol. Given,
CD =3 m
and
1.5 cm BO60
AT D

each side equal to 2a.


3 cm Let a triangle ABCwith=LB = 2C =60
ZA
BC
Draw AD perpendicularto (by RHS) ½
ABDA ACDA
D
BD = CD
CPCT)
AC ZBAD = LCAD = 30° (by
sin 9 =
CD
Cosec 30 =
AB 2a =2
1.5 In ABDA, BD
3 BD
COs 60° =
and AB 2a 2

[CBSE Marking Scheae, 2015]


sin 0 =

030°
their solutions are available at the end of the chapter
fur practiæand
These questions are
MATHEMATIC8

194 Oswaal CBSE Question Bank Chapterwise &Topicwise, (8TANDARD), ClaS5-X


Short Answer Type Q.5.
(3markseach)
Questions-II
1
tan (A - B) = 0<A+
Q. 1. Iftan(A + B) = land
B.
find the values of Aand 2019]
B< 90°,A> B,then Delhi Region,
R(CBSE
TopperAnswer, 2019

In the given APQR,


'Sol.
and PR - PQ = 1 cm.
right-angled
sin R + cos R. DetTO e[Board
rmine tTerm-,
hee vne
P
Sol.

40 cm 41 cm

9 cm R

PO' + OR = PR?
b= 46-A (By Pythagoras tecr
IA33-5'A=2-5] PO +9 =PR?
3 PO + 81 = (PQ +1
(4sin - cos0+1
PO + 81 = PQ' +1+ 2PQ
PQ = 40
Q.2. If 4tan =3, evaluate 4sin0 +cos-1) or,
PR-PO =1
UCBSE Delhi/OD, 2018] PR = 1 + 40
Or,
PR = 41
0.3. If sin (A + 2B) = and cos(A + 4B) = 0, A > B, 40 9
2 sin R + cos R = 49
41 41
and A,B<90°, then find Aand B.
C+ UICBSE Comptt. Set-I, II, III, 2018] (CBSE MarkingSchem,

Sol. Given, sin (A + 2B) = 2 >A+ 2B=60 1 Long Answer Type


Questions (5 markseach
1
cos (A + 4B) =0,>A + 4B =90° 0.1. Evaluate:
Solving, we get A= 30 andB=15° tan" 30° sin 30° + cos 60 sin 90° tan
60-2i25
[CBSE Marking Scheme, 2018] RBoard Tera-1 2E
cos 0° sin 90°
Detailed Solution:
find the rea
Q.2. If J3 cot 0-4cot + 3 = 0, then
We have sin (A + 2B)= 2
of co 0 + tan² 0.
cot 0 = ,
sin (A + 2B) = sin 60° Sol. Let
A+ 2B = 60 then V3 co0-4 cot 0+ V3 = 0becomes
and cos (A + 4B) =0
cos (A+ 4B) = Cos 90° : cos 90 =0] h V3-4r+v3 =0
A+4B = 90 v3-3x-x+ 3 =0
Solving eq. (i) and (ii), we get
A = 30 and B = 15°. 1 (*- 3 V3x-1) =0 1
Q.4. If in a triangle ABC right angled at B, AB = 6units
and BC=8units, then find the value of sin A.cos C
+ cos A.sin C. UBoard Term-1, 2016)
These questions are foT practice and their solutions are aVailable at the end of the chapter
TRIGONOMETRY ANID TRIGONOMETRIC IDENTmie 195
cot = V3 or cot0 1 Sol. We have
1

0 =30° or = 60° sin (A +B-C)=sin 31


0 =30°,then Or, ..)1
A+ B-C= 30

o2 30° + tan 30° = (V3² + 1 and cos (B + C-A )= Cos 45°

Ot, B+C-A =45° ..Gi) 1


=34 10
3 3 1 Adding eqns. (i) and (ü), we get
2B =75
IA=60 then or
B= 37.5
Now subtracting egn. (ü) from eqn. (1),
cot60° + tan 600 2(A-C) = - 15°
OI, A-C=-7.5° ..(ii)
1 10 1
+3= 1
A+B+C= 180°
3 OI, A+C=142.5° ..iv)
angled triangle e ABC, if sin
acute, (A +B-C) Adding eqns. (ii) and (iv),
an
Qzn 2A = 135°
1
and cos (B +C-A ) = find ZA, ZB A=67.5°
V2 or
and C=75°
andLC.
Hence, ZA=675, ZB= 375 and C= 75° 1

Trigonometric Identities
Topic-2 Concepts Covered Three important identities are:
(0 sin 0+ cos 1, (ii) 1+tan 0 sec 0, (i) 1+ cof 0= cose 8,

RevisionNotes
An equation is
called an identity if it is true for all AB? BC2 Ac2 Scan to know

values of the variable(s) involved. AB? AB2 AB


more about
An equation involving trigonometric ratios of an this topic
angle is called a trigonometric identity if it is true for
allvalues of the angle. or
AC)'
AB
inA ABC, right-angled at B, By Pythagoras Theorem,
A
1+ tan A= sec A
Trigonometric
..ü) identities
Is this equation true for A = 0?
Yes, it is. What about A = 90? Well, tan A and sec A
are not defined for A=90°. So, eqn. (iii) is true for all
values of Asuch that 0 sA< 90.
Again dividing eqn. () by BC'.
AB² BC? AC?
BC² BC? BC2
AB + BC = AC ..)
Dividing each term of (i) by AC",
AB? BC? AC2
AC? AC cot A + 1 = cosec A ..iv)
Note that cosec A and cot Aare not defined for all
--
or (cos A +(sin A =
A=0.Thereforeeqn. (iv)istrueforallvalueofAsuchthat
0° <As90°.
1 Using these identities, we can express each
cos A+ sin A =1
This iss true for all valuessoof Asuch that 0° sAs90°,..ü)So, trigonometric ratio in terms of other trigonometric
ratios, i.e,, if any one of the ratios is known, we can
AK. trigonometricidentity. Now divide eqn.(i) by determine the values of other trigonometric ratios.
MATHEMATICS
(STANDARD), Class-X
Chaptewise &Topicwise,
I96 Oswaal CBSE Question Bank
OBJECTIVE TYPE QUESTIONS (1 mark etth
Q.3. Ifsec +tan =p, then tan is
Multiple Choice Questions (A) (B)
2p 2p
and tan 0 + cot = 2, then the
Q.Mfe is an acute angle is
value of sin' + cos0
1
(C)
p+1 (D) p+1
(B)
(A) 1
(D) 2
2
Ans. Option (B) is correct. R[CBSE, Board Term4:
2 UCBSE, Board Term-I, 2021] Explanation: sec 6+ tan =p
is the given equation.
Ans. Option (C) is correct.
Explanation: Since, 1+ tan'9 =sec'e
tan + cot =2
or sec = y1+tan²a
sin 8 COs Put this value in (i), we get
=2
Or sin 9
cos
1+tan 0 + tan =p
sin0+ cos0
2
cos® sin 9 J1+ tan 0 =p-tan
sin0 + cos0 = 2sin cos Squaring both sides, we get
1= 2sin cos 1+ tan0=p+
tane-2p tan
sin cos =
1
..) 1=-2p(tan 9)
2 1-p' =2ptan 9
(sin 0 + cos 0) = sin0 + cos0 + 2sincose tan = p-1
1 2p
=1+2 X
2 1
=1+1= 2 Q.4. The value of
tan0 + cot
Therefore, (A) cos sin 0 (B) sec 0 sin 9
sin + cos 0 = 2 ...ü)
(C) tan cot 9 (D) sec cosec 9
Now taking,
sin°0 + cos°0 = (sin + cos 0) AD R(CBSE, QB, 200:%
-3sin cos 0 (sin 0 + cos 0) Ans.Option (A) is correct.
1
Explanation:
=(V2)°-3xx2
tan 0 + cot0 sin Cos
3 Cos sin 9
=2/2-N2 = 2
1

0.2. If acot 9 + bcosec =p and b cot + a cosec = sin' 0+ cos²


q, then p²- = cos sin 0
(A) a-b (B) -a cos sin 9
() a+be (D) b-a sin'0+ cos0
B[CBSE, Board Term-I, 2021]
= COs sin 0.
Ans. Option (B) is correct.
[sin' 0 + cos'=l
Explanation: a cot 0 + bcosec 9 =p and bcot + a
cosec 8 = qare the given equations. 3 then cos =
Q.5. Given that cos 0- sin² 0=
Taking, p - 4
= (acot 0 + b cosec 0y- (b cot + a cosec 0)
= cot0 + b'cosec e + 2ab.cot 0.cosec 0 1
(A) (B)
-Bcot0-dcosec 0-2ab.cot 0.cosec 0 2

=a(cot0- cosec0) + (cosec0 - co0)


=a-1) + B) V7 (D)
(C)
2
=-a [using, cosec0 - cotte= 1) R|CBSE, Additional Question,.
TkIGONOMETRY ANID TRCONOMETRICIDENTITEs 197
(D); correct,
is
Otion 1
Eiylanation: Q.8. If sin A = then the value of cot Ais:
2
cos*0-sin0= 3
4 (given) (A) V3 (B)
QOs0-(l-cos²0)
co =
4
[sin? 0 = 1- cos') (C)
V3
2
(D) 1

2cos²0-1= 3 ORNCERT Exempl.]


4
Q.9. Given that sin 9 = then cos is equal to
3
2cos 0= 1+ 4 b
(A) (B)
cos² 0 = 7
(C) (D) -a
cOs 0 =
V7 RINCERT Exempl.]
Ans. Option (C) is correct.
4sin -cos0 Explanation:Given, sin = b
lan0= then is equal to:
3,
Q6If4 4sin + cos
sin'0 +cos'9=1 cose = 1-sin'
2 (B) 2
cos = b
3
1 (D)
(C) 3
4
Q. 10, If sin A + sin? A = 1, then the value of the
expression (cos A + cos*A) is:
RNCERT Exempl.]
is correct. (A) 1 (B)
Ans.Option(B) 2
Explanation: Given, 4 tan=3 (C) 2 (D) 3R |NCERT Exempl.]
3 ...)
’ tan - Ans. Option (A) is correct.
Explanation: Given, sin A+sin' A =1
4Sin0 ’ sin A=1-sinA= cos'A (: sin0+ cos0=1]
4sin -cos 0 Cos
4 sin 0 + cos 4
sin 0
-1 On squaring both sides, we get
COs sin' A= Cos A
1- cos A= cos* A
[Divided by cos in
both numerator and cos*A+cosA=1
denominator Q. 11. The value of 9 sec A-9 tan A is
(A) 1 (B) 9
4tan-1 sin0 (C) 8 (D) 0
4tan + 1
tan@ =cos ANCERT Exempl.]
0. 12. The value of (1 + tan + sec 0)
3 (1+ cot - cosec o)is
3-1_2_1 Put tan e =
3+1 4 2
4 (A) 0 (B) 1
|from equation ()| (C) 2 (D) -1
AOA|NCERT Exempl.]
Q.7.If cos A= 4 then the value of tan A is: Ans. Option (C) is correct.
5
Explanation:
3
(A) 3 (1+ tan 0+ sec®)(1 + cot-cOsece)
5 (B) 4 sin 1+
cose
4 Cos0 COs sin 9 sin
1
3 (D) 8 cos0+sin@+1)/ sint+cosf-1

These questions are efor practice and their :solutions RINCERT Exempl.] cos sine
are available at the end of the chapter
MATHEMATIC8
Chapterwise &
Topicwise, (STANDARD), Class-X
Question Bank correct.
198 Oswaal CBSE Ans.Option (D) is
Explanation:
(sin 8 +cose)' -()'
sin 9cos 8 (sec A+ tan A)(1- sin A)
sine+ os'e+ 2sincose-1
sin&ose sin A
cOs A
1+2sin&ose-1
COS A
sinose
1+ sin
2sin @cose
-2
cOs A (l- sin A)
sinose A
A) is
(sec A + tan A) (1 - sin cos' A
Q. 13. The value of (B) sin A COSA COS A
(A) sec A (D)cos A
(C) cosec A = COSA
N[NCERT Exempl.]
SUBJECTIVE TYPE QUESTIONS
/
=sedex cos e(:: 1-sin'9:
Very Short Answer Type
1
x cos8 [ sec 9
(1 mark each) cos 0
Questions
2 cos² 9 + 1, then find the
=1.
Q.1. If x= 2sin' 8 and y =DOCESE
value ofx + y.
SO, 2020-21] CBSE Marking Scheme x
x+y=2 sin 0 +2 cos0+1
½ 0.4. Write the simplest form of (1 - cos'A) (1
+
B[CBSE Delhi &OD.cotA,
Sol.
=2/sin0 + cos0) + 1
(As sinx + cos²x = 1)
=3. Topper Answer, 2020
[CBSE Marking Scheme, 2020-21]
Sol.
Detailed Solution:
We have
x= 2sin'e
and
y = 2cos0+
Then, x+y= 2 sin 0+ 2 cos² 0+ 1 ½
= /sin'0+ cos 0) +1 (-eesa) t eoa4 )
=2X1+1
I:: sin? + cos? 0 = 1]
=2+1= 3.
Ci-coaa)(sin'A+ essA)
1
sinA
Q.2. Find the value of sin'8+ 1+ tan'e, (I-c8sA)
SintA SifA
AOUCBSE Delhi Set-I, 2020]
1 1
Sol. sin0 + = sin'0 +
1+ tan'0 sece 0.5. If sin + cos = J2cos 0, (0 90) then the vabz
[::1+ tan0 = sec 9] ½
= sin0 + cos of tan is: OCBSE SQF
5
= COs ] 0.6. If tan a = 1ntind the value of seca.
seco
[CBSE Delhi Region, 2
=1. [:: sin' 0+ cos²0=1]%
Q.3. Find the value of (1 + tan)(1 -sin 0)(1 + sin 0).
TopperAnswer, 2019
AdUCBSE Delhi Set-I, 2020|
Sol. (1 + tan 0)X(1-sin 9)(1+ sín 0) Sol.
[: 1+ tan =sec 0] aa 5
sec 0(1-sin0)(1 + sin 0)
=sec 9(1 sin 0) keetantl

These questions are for practice and their solutions are available at the
end of the chapter
TRIGONOMETRY AND TRIGONOMETRIC IDENTI1ES 199
Sol.
LHS = (sin'+ cos o)
1-2sin' ocos 0
l25 (sin² e +(cost e
1-2sinecos²
(sin' 0+ cos² ey -2sin 9cos
1-2sin' 9cos
1-2sin cos² 0 =1= RHS 2
1-2sin 9cos² 0
Hence Proved
(CBSE Marking Scheme, 2015]

of cot0-
1 Commonly Made Error
Q8
Writethe value sin A Some students make mistakes to prove the sum
and become confused.
09.Ifkt1= sec 0(1
UCBSE SQP, 2018]
+ sin 9)(1 - sin 0), then find the
valueofk. C +uBoard
Term-1, 2015]
t+l=sec 0(1 + sin 0)(1-sin 0)
Answering Tip
Sol.
o,
k+1= sec 0(1-sin 0) Follow step by step simplification to avoid
t+1= sec 0.cos0 (: sin e+ cos? =1| errors.

k+1=sec²0 x 1
sec e
k+1=1 Short Answer Type
k=1-1 Questions-II (3 marks each)
k= 0. [CBSE Marting Scheme, 20151 Q. 1. sin + cos = 3,then prove that
tan + cot = 1
Short Answer Type ADACBSE SQP 2020]
ICBSE Delhi Set-I, 2020]
Questions-I (2marks each) Sol. sin 9+ cos = 3 (sin + cos 0) =3 1
cot a
0.1. Prove that: 1 + = C0Sec a.
’1+2 sin cos3 sin cos = 1
1+ coseca 1

sin 0 cose
AOACBSE OD Set-I, 2020] . tan 0t cot 0
Cos sin0
Sol. L.H.S =1+ cot
1+ cosec a [CBSE SQP Marking Scheme, 2020]
cOsec a -1 Detailed Solution:
=1+ Given
1+cosec a sin 9+ cos = J3

=1+ (1+cosec oa)(cosec a -1) 1


Squaring on both sides,
1+ cosec a (sin 9 + cos 0)² = (V3)
=1+ cosec a -1 sin 0+ cos + 2 sin cos =3
= COsec a = R.HS.
L.H.S = R.H.S. Hence Proved. 1 1+2 sin cos 0=3
2. Show that tan e + tan² = sect-sec0 2 sin cOs = 2

ADA[CBSE OD Set-1, 2020] sin cos =1 . () 1


. Express the trigonometric ratio of sec A and tan A
In terms of sin A. sin cose
AOT JBoard Term-1, 2015] tan + cot =
Q4Prove that: cos sin
(sin 9+ cos 0) sin*+ cos 0
=1 A [Board Term-1, 2015]
1-2sin0cos 0 cos@ sin e
These
questions are for practice and their solutions are available at the end of the chapter
MATHEMATICs
DO0Oswaal CSK QueStion lank Chaptenwise
A Topiewise, (STANDARD)
Sol. Given, 1 +t sin'0
, Clas8-X
sin cos 0
On dividing by sin'0 on tboth

sin0
sides,Wcote gt
+1=3
|nm vzation(0]1
1
Hence oved

Q.2ove that:
1 =0.
cOsec*0 +l=3cot 9
1+ cot+1=3ot o
2{sin° t cos')-3(sin'#+ os) +
ADACESE Delhi Setll, 2020) cot0-3 cot 0+2=0
ot0-2 cot - cot 0 +2 0
Sol. LHS
+l cot O(cot 0-2) - l(cot 0- 2) =0
-2 (sin°e + m'0)-3 (sin'e + us0)
}[
-2{(sin e)+(cos)}-3(sin'e+ uste) + 1 (cot 0-2)(cot 0- 1) =0
=2lsin~ + ussin'e - sin cos@+ os*] cot = l or 2
-3(sinte+ste) + 14 Then, tan ) = 1or
(r+*= (2 +Na-ab +b))
=2(sin'e - sin se + oste)
-3(sin e+ e)+ 1|*sin-0 +cos=1]1
2sine os +1 Q.5. Prove that: 2
cos'-cosA Hence prOVed
=-sin e - s sin-2sin 0 cot0,
=- (sin'e + cos + 2sin' oos) + 1 U[CBSE
=-(sin-6 +cos0)+1 Delhii &OD,
[ (a +)=a+t+2ab] 3% Topper Answer, 2020
=-1+1
=0= R.H.S. Hence Proved.
Sol.
cot6+ cosec8- 1
1+ cos 9 sinG- 2sine
Q3.Prove that: cot 8- cosec 8 +1 sin 9

ALABSE Delhi Set-lII, 2020] gese (2ceie -i)


cot8+ co sec -1 Kino (1- 2sinsy
Sol. LH.S, =
cot9-cOsec 8 +1
cEse 2(1-sine)-)
cos0 Sire (1-2 Sin'ey
Sin sin
Cose
+l
cesel2- 2sins -i]
sin sin Sins (1- 2sine
sin (cos - sin 0+1)
sin (cos8+ sin 8-1)
Suns
sin cos - sin 0+ sin
sin (cos0+ sin -1)

sin @cos + sin 8-(1-cos*0) 1 fesed,


sin (cos +sin &-1)
QA Prove that:
sin (cos +1)-[(1-cos0\(1 +cos®)) (sin cosec 0)* + (cos 0 + sec 0) = 7+tan'+
sin (cos0+ sin -1) cot aDACBSE Delhi Set-1, II, II, 201
(1+cos0)(sin8-1 +cos) Sol. LHS = sin'0 + cosec0 +2sin cosec 0 + os
sin (cos+sin 6-1) + sec² + 2 cos sec 1
1+cos0 = (sin*8 + cos0) + cosec²0 + sec0
2sin
sin 9
sin0
= R.H.S. Hence Proved. %
Q4. If 1+ sin? = 3 sin cos 0, prove that tan 8 = 1 =1+1+ cot0 +1+ tan'0 + 2+2
=7+ cot0 + tan²e= RHS Hence Proved N
or AOACSE OD Set-1, 20201 Scheme,2019
[CBSEMarking
provethat sec 0 + lan
"TRIGONOMETRYbeosAND TRIGONOMIUTRIC
0. 10. f 0
IDENTIES
M, then prove that coec ) + cot G
Z01

CBSE Delhi Setu,200191 9fCBSE Board Serm-i, 2013

4N Sol,

4x bk
1
2

lan²0 161:2 Given, (k+ 0)


1
tan*0 +
l62 AC2 AB -BC2
1 AC b-ak
tan 0 = -2:x
16x2 41
COSec ,Cot =

tan²0 = 1
b+a 3
COsec + cot =
both sides Vo?-a? Vb-a
square root on
laking
tan 0 =
[CBSEMarking Scheme, 20151
1 cos
1
Q. 11. Prove that: (cot 9 - cosec 9)= 1+ cos
tan = -
4x OUCBSE Board Term-1, 2015]
1
sec = X+
Given, 4x Long Answer Type
tan + sec = 2x Questions (5 marks each)
NOw,
1 sin A - cos A+1
tan = 1
1ßrove that
4x sin A + cos A -1 secA - tan A

sec 0 = X+
1 AUA[CBSE Delhi Set-I, 20191
Given, 4x
sin A - cOs A+1
1 1 1 Sol. LHS =
1 sin A+ cosA-1
sec + tan = +
Now, 4x 4x 2x
Dividing numerator & denominator by cos A,
Hence Proved.
tan A-1+ secÁ
2 cos -1 tan A+1-secA
0.8.Prove that: cot-tan =
sin cos
tan A-1+ secA
1
UCBSESQP 2018] (tan A - sec A)+ (sec A- tan A)
Q9Prove that: sin (1 + tan 0) + cos (1 + cot 0) = sec
+ coseco UCBSE SQP 2018] tan A-1+ secA
(tan A-sec A)(1-sec A- tan A)
M. LHS = sin 9(1 + tan 9) + cos (1 + cot )
sin COs -1 1 = RHS
= sin 1+ Cos0 con 1 sin
sin 0+ cos
tan A secA secA - tan A
Hence Proved
= sin 9| cos + sin
cos sin {CBSE Marking Scheme, 2019}
DetailedAnswer:
= (cos 0 + sin 0) sin9+ cos? 0 sin A- cosA +1
cossin 9 LHS
sin A + cos A-1
cos + sin 9 1
= COSec + sec 0 = RHS sin A cOs A+1 1+sin A
cossin 1
Hence Proved sin A+ cOs A-1 1+sin A
[C3SEMarking Schene, 2018}
ihe
questions are for practice and tleir solutions urc avuilable at the end of the
chapter
Oswaal CBSE Oucstion Bank Chapterwise &Topicwise, MATHEMATIC8 (8TANDARD), Class-X

202 sín 9(2cosec 9)


+ sinA)
1)(1
(sin A - cosA+ sin A
1
sinA+cosA sin A-
sin A+ cOsA -1+ =2
A) sin
(sinA- OsA+ 1)(1+sin
1
Hence =24
-1+ c0s A+(1- cos
A)+sin Acos A cosec + cot 9 Sin
A- cosA+1)(1 +sin A)
(sin
cos A
cOsA -cos A +sin A
Q.4. Prove that:
cot-CO%er
[CESE: Matking Sdets
1 1 1
(sin A- cos A+1)(1+ sin A) 1
1+ sin 1+ cose 1+ sec 1+coser
cos A(1-cos A +sin A)

1+ sin A
cosA
1
cOsA
+
sin A
cosA
A[CBS.Delhi, Regon
Topper Answer, 2016
1
= secA + tan A Sol.
(sec A + tan A)x(secA - tan A)
(secA- tan A)

sec' A- tan A 14tite


secA- tan A
1+eata (+Leerg
1 Hence Proved. 1
= RHS
secA-tan A

0.2. Prove that:


cosecA 1 l+ cort
tan A
tan" A-1 secA- cosecA 1-2cosA
AICBSE Delhi Set-II, 2019]

1
LHSE RH
sin A
Sol. LHS = cos"A -+ sin"A 1
sin A 1 1 sin A-2sin A
cos A sin A 0.5. Prove that: = tan A.,
cos A 2cos' A -cosA
sin A cos" A 1 UICBSE Delli/OD, 2018][Board Term4
sin A-cos A sinA-cosA sin A-2in A
1 Sol. LHS =
1 2 cos A-cosA
sinA-cos A
sin A(1-2 sin A)
1
Hence Proved 1% cos A(2cos* A-1)
1-2cosA
sin A(1-2(1- cos' A)
(CBSE Matking Scteme, 2019]
cos A(2cos A-1)
Q3. Prove that:
sin0
=2+
sin
= tanA
(2cos-A-1)
cot +cOsec o cot -cosec 0 (2cosA-1)
Hence Prorat
AUACBSE OD,Set-l, 2019] tanA RHS
Sol. [CBSE MarkingSchee,
sin sin0 Detailed Solution:
COSece+ cote cot-cosec 0
TopperAnswer,2018
sin sin6
cosec®+cot8 co8ec 9-cot.
Sol.
Topaova..kA-2sinA
sin G[cosec 0-cot0+ Coseo +cot 0)
1+1
cosec 6cot
LHS,
TRIGONOMETRY AND TRIOONOMETRIC IDENTITIES 203
zor'A-torA -2t 4
2(1+p) (1+)

Hence Proved 1

coÇA L2cas 4-(Ein'A4coa (CBSE Marking Schete, 20I8)


0.7, 1f scc 9 + tan 1
=p, show that sec 9 - tan

sinA <osA-sA7
Hence, find the values of cos 0 and sin 8.
co+Ã -sina AOABoard Term-1, 2015|
Sol. (seco-tan 9) 1
sec+ tan sec8-tan 9
fana: 1 sec-- tan
LHSRAE sec tan 1
sec* - tan'0
hece Ahoved.
5 Solving, sec 0 + tan = pand sec 0- tarn 9

Commonly Made Error We get sec = p+1 1


2p
Some Common errors obsserved were:
Working with both sides together. and tan 0
ö Skipping of necessary steps so as to get the 2p
answer.

oi) Some opened the LHS expression but failed p+1 andsin9
p-1 1
to simplify and come to the RHS. p+1
{CBSE Marking Scheme, 2015}
Q. 8. Prove that b'y?-ay= U², if :
Answering Tip (i) x=a sec 0, y = b tan , or
(ii) x= acosec 0, y =b cot .
Ensure that while proving identities students
proceed with either LHS or RHS but must not OUBoard Term-1, 2015|
work with both sides simultaneously. Q.9, If cosec - cot0 = V2 cot 0, then prove that
cosec 0 t cot 0 = V2 cosec 0.
06]fsec +tan =, then find the value of cosec 0.
U|Board Term-i, 2015]
UICBSE SQP-2018]
Sl sec O+ tan =p Sol. Given, cosec -cot V2 cot
1 sin Squaring both the sides,
cosec 0-+ cot0-2 cOsec cot 2 cot0
Cos0 COs0 or, cosec0- cot' 0=2 cosec cot 9 1%
=1+ sin pcOs
or, (cosec 0 + cot 0)\osec o - Cot 0) = 2 cosec & ot0
=p1-sin0 1
Given: (cosec @- ot = N2 cot 9)
(1+sin 9)² =p(1- sin²0)
1+ sin 9+ 2sin 0 = p 2 cosec 9 cot
cOsec 0+ cot
-p
(l+ p)sin' 0+2 sin 9+ sin" 0 V2 cot8
D= 4-4(1 + p)(1 (l-=0 cosec et cot V2 cOsec 134
Hence Proved.
Tuw 4-4(1-p)= 4p CISE Matking Schee, zO1S
dwestions are for practice and their solutions are availatble at the end of the chapter
Oswaal CBSE Question Bank Chapterwise &Topicwise, MATHEMATICS(STANDARD), Class-X

204
|COMPETENCYBASED QUESTIONS
Q.3. What should be the length of
sail in order to puli the
ship at he rope
O Case based MCQs at a vertical height of 200 m? the a
Attempt any 4 sub-parts from each question.
Each sub (A) 400 m (B) 300 n
ange
part carries 1 mark. (C) 100 m
answer any four questions (D)200 m
I. Read the following and Ans. Option (A) is correct.
from Q.1. to Q.5. Explanation: In AABC, we have
science that
Skysails' is that genre of engineering move energy to
9 =30°, AB = 200 m
uses extensive utilization of wind
sky sails technology
a vessel in the sea water. The
allows the towing kite to gain a height of
between 100 m to 300 m. The sailing
anything
kite is made
Then, sin 30° =
PerHypotpendienusculear
A
proper
in such a way that it can be raised to its AC
elevation and then brought back with the help of
raised
the kite to be 1 200
a telescopic mast that enables
2 AC
properly and effectively.
AC = 400 m.

Q.4. If cos A = then the value of 9cot


2 A-1i
(A) 1 (B) 3
Rope
(C) 2 (D) 4
Ans. Option (C) is correct.
A
B Explanation:
1
Given, cos A =

.RIn the given figure, iftan 0= cot (30° + 0), where 0 cos A COs 60°
Jand 30°+0are acute angles, then the value of Ois: A = 60°
(A) 45° (B) 30° Then, 9 cof A
-1 = 9(cot 60°-1
(C) 60° (D) None of these
Ans. Option (B) is correct.
Explanation:
Given, tan = cot(30° + )
= tan[90°- (30° + )J
= tan(90°-30°- 0)
=2
tan 9 = tan(60°- 0)
9= 60°-0 Q.5. In the given figure, the value of (sin C+«as A
20 =60 (A) 1 (B) 2
9 = 30°. (C) 3 (D) 4
0.2. The value of tan 30. cot 60 is: Ans. Option (A)is correct.
1 Explanation: We have,
(A) V3 (B) 4B
AB = 200m and AC=
[Provedin
1
(C) 1 (D) a AB + AB
Then, sin C + cos A =
Ans. Option (D) is correct. AC AC
Explanation:
1 1 AC
tan 30° X cot 60° =
200
2x =1
400
llowingtext and answer the following
TRICONOMETY AND TEICOIETPK iDNTYt 205
basis of the same:
tre CnstuCF a slide in a city park
KInts to Then 12 cotA-2-
slide was to be constructed for 1lt AI -2
The
beioy the age of 12 years. Authority
the sljde at a height of 4 m above
ren
tupof
inÈined at an angie_of 30 to the
ad

4-2 T2.
Q.4. 0n the given figure, the value of (sin C cos Aj is:
(A) (B) 2
1
1
() (D)
Ans. Option (B) is correct.
Explanation: Since, AC L BC,
fAB is:
istance of then 2C = 90
(B) 6m
sin Cx cos A = sin 90 X AC
(D) 10 m AB
coTect. 4
sCption(A)0s =1x 8
Erzaration: We have.
LB = 30 and
AC = 4 m -

AC
sín 30° =
AB Q.5. Inthe given figure, if AB + BC=25 cm and AC =
1 4 5 cm, then the value of BC is:
2 AB (A) 25 cm (B) 15 cm
AB = &m. () 10 cm (D) 12 cm
cos² 60° is: Ans. Option (D) is corect.
The value of sin30
1
1 Explanation: We have,
(B) 2 ZC = 90

3 AB + BC = 25 cm and AC=5 cm
(D) 2 Let BC be x cm, then AB= (25 -x) cm
iK Cptior (B) is correct. By using Pythagoras theorem,
Exglanation: AB² = BC² + AC2
2
(25-x) =+ (5)
625 - 50x + = + 25
1 50x = 600

4 4 600
12
2 50
2 Hence, BC = 12 cm

then the value of 12 cot² A -2 is: Case Based Subjectlve


(B) 4 Questions
Ss (D) 2 I. Readthe following text and answer the tollowing
(otirn (Di is
correct. questions on the basis of the sane:
ixplanation: In geometry, an equilateral triangle is atriangle in
which all three sides have the same length. ln the
2
familiar Euclidean geometry, an equilateral triangle
MATHEMATICS

206 Oswaal CBSE Question Bank


Chapterwise &
Topicwise,
(STANDARD), Class,
0.4. Find the length of AD
y
is, all three internal angles 3/3 cm
are alsoequiangular; thateach other and are each Sol.:: BD = and AB =
area als0 congruent
to 6V3 cn
an equilateral triangle which is AD
oU". Here we have U+ AE
.:. In AADB, sin 60
inscribed in a circle of radius 6 cm. AB
AD
2
643
2AD = 18
AD =9 cm.
0.5. Which mathematical
Sol. Trigonometric Ratios.
concept used
II. Read the following text and ansSwer
questions on the basis of the
Q1. What is the length of BD ? In plane Euclidean same: he it
Sol. ABC be an equilateral triangle. geometry,
a
.:.OA= OB = 0C = 6cm (given)
CO are bisectors
quadrilateral whose all
Opposite angles are
sides have the
equal. Opposite
thomog
Dis the mid-point of BCand BO& parallelto each other. s
of B and ZC. Diagonals
perpendicularly. Sum of any two bisect
.: cOsZOBD
BD
OB
180°.
Now, we have a rhombus of side
adiacent arg
BD two angles of 60 each.
cos 30° =
i.e., 6
D 20cm
BD = 6cos 30°

=6x -=3V3

Hence, the length of BD is 3/3 cm.


Ql2. Fnd the side of this equilateral triangle. 20cm

Sol. We have proved that Q. 1. What is ZAOB ?


BD = 3V3 cm
Sol. As we have given that the diagonals AC and Bh.
BC = 2 X BD arhombus are perpendicular bisector of each o
Then,
= 2x3V3 ZAOB = 90°.
Q.2. Find the length of OA.
= 6v3 cm.
Sol. Since, ZAOB = 90° and AB = 20cm .

Q.3.Findthe length of OD. . In AAOB,


Sol. Here, OB = 6cm and BD= 3W3 cm cos ZBA0 =
OA
AB
OD
In 4ODB, sin 30° =
BD OA
cos 30°=
20
1 OD
2 3/3 |Diagonals AC and BD are bisectors of ZBAD &L
20D = 33 V3 OA
2 20
OD 3/3
2 204 = 20/3

5.196 OA = 10V3 cm.


2
= 17.32 cm
= 2.598 cm
kewth
ot
fBO TRIGONONMETRY AND TRIGONOMETRIC IDENTITIES 207

|Proved above A0 - 10/3 cm]


OB 20y3 cm.
sin.N=

4.64 Cm
Q.5. Find the length of other diagonal BD.
Sol. Diagonal BD - 2xB0
OB = 10Cm. -2x 10 cm
AC. [: Proveabove OB 10cm
lengthof
diagonal
e '=2NAO - 20 cm.

Solutionsfor Practice
Questions (Topic-1)
ChoiceQuestions
corect
( )is
ton giventhet
atin:lItis 1
cot8 = = cot 600

e = 60°
B
ofA
value
itutingthe sec60° + cOsec 60° sum of three angles = 180°
Le., LA+ZB+LC- 180°
= (2)+ 2C=90° [Given]
ZA+ ZB+ 90°= 180°
1 A+B=90°
cos(A+ B) =cos90° =0
correct. Very Short Answer Type Questions
(A)is
s Option
Eranation: CBSE SORMarkineSheme2011
Sol. 2: 30°
C
Detailed Solution:

sin a =
2
25
7
sin a = sin 60° (: sin 60° =

B A a = 60
and cos ß=0
Perpendicular cos B = Cos 90° [: cos 90° = 0] 4
sin 0 =
B=90°
Hypotenuse B-a =90°-60°
Now,
Base =30°.
d
Hypotenuse Sol. 5: Givern, sec 8.sin9=0
BC 7 sin 6
Sin A =
AC 25 COS
BC tan 8 = 0= tan 0°
cOsC =
AC 25
Option (A) is correct. (CBSEMayking Schene, 2016)
Clanation: We know that, in AABC,
GE#aal CBSE Question Bark Chapterwise Topicwise, MATHEMATICS (STANDARD) Class-
208

Sol. 6: tan(3z +30) =1= tan 45* 4x 4


4sin &-cos8+1
3x +30= 45 or, I=5 1
4sin 8+ cose -1
5 5
C8SE Marking Scheme, 2015| 4x 3
5
Short Answer Type Questions-l 13
Sol.4: 3tan 30 +tan 60+cosec 30°-tan 45° 11
cot 45° (CBSE Marking
1 2
+(N3j +2-1
Detailed Solution:
We have 4tan =3
Scheme, A
1
(1 tan 9=

3x+3+2-1
1 We have, tan 9 Perpendicular
Base
=1+3+2-1=5 1 Let perpendicular = 3r and
[CBSE Marking Scheme, 2016]
Base = 4x
Also let ABC be a right angled A.
Commonly Made Error AC = vAB- - BC*
Sometimes students get confused with the (By using Pythagoras
values of trigonometric angles. They substitute theorem)
wrong values which leads to the wrong result.

Answering Tip
Memorize the values of trigonometric angles
properly and practice more such problems to
to get confused.

Sol. 5: Here, sin (A + B)= 1 = sin 90°


AC = vl6x* +9y?
A + B= 90 = v251* = 5:
...i)
sin (A B) = = sin 30° 1 Then sin =
BC 3x
2
A- B= 30° AB
...i) and 4r
Solving eq. (i) and (ii), COs =
AC
A= 60° and B = 30° 1
{CBSE Miarking Scheme, 2016] 4sin - cos + 1
Short Answer Type Questions-I| 4cos +cos -1 3
Sol. 2: Given, 4tan =3 4x l

3
tan = 13
4

3 16 11
sin =
5
and cos= 4 5
-1
5
13
11
TRIGONOMETRY AND TRIGONOMETRIC IDENTTIES 2091
Cormtnonly Made Error
and 6
sin C C0S C*
didates do not ind the values
cand 10
Mostlyand cosine. Some candidates do the :. sin A cos C +
cæs A sun C
Wrong
of
sinecalculation.
88
10 10 10 10
AnsweringTip 64 36
100 100
Candidatesshould find the value of sin and 100
=1. 1
theorem 100
nt by using yagoras
[CBSE Marking Scheme, 20i6}
Long Answer Type
sol.4:Here, AC= (8k)+ (6k) = 1003?
AC= 10k
Questions
Sol. 1: tan 30 sin 30° + cos 60° sin 90° tan 60
-2 tan 45° cos² o sin 90

8k

35 x1 x3-2x1x1 x 1.
A 6k B 1,3 1+9-12
6
6
sin A= COS A = 1
10 10
[CBSE Marking Scheme, 20151 5

Solutions for Practice Questions (Topic-2)


Multiple Choice Questions
Ans. 7: Option (B) is correct. cos A= :sin A+cos A=1
Explanation: ’ cosA=l-sinA
4
Given, cosA= V3
Cos A
sin A+cos A=1 Now,cot A= =2/B
. sin A=V1-cos² A sin A 1

. sin A= 1-cos? A 2
Ans. 11: Option (B) is correct.
sin A: Explanation:
9sec² A-9tan' A= 9(sec A- tan A)
=9(1) [: sec A- tan'A = 1]
Now, tan A = sin A =9
COSA Very Short Answer Type Questions
3 Sol. 5: J2-1 CBSE SOP Marking Scheme, 2020]
53 Detailed Solution:
4 4
5 sin + cos = 2 cos 0
Ans. 8: Option (A)is sin + cos
correct. or
Cos0
v2
1
Explanation: Given, sin A= Or
sin cose
cos Cos0
COS
A=i-sin' A=, or tan
tan
+l=
= 2-1.
2
10 0awaal ChsK Queation Bank Chapterwiae &Topiewiae, MATHEMATIOS

Sol. 1l:
(8TANDARD), Claan-X
LHS (cot0- cosec 0y
Sot. 8so cotuse0
sin cose 1
sin slrn0
1
cos-1
|CHST Marking Seheme, 20181 sin
Short Answer Type Questions-l
Sol. 2: L.HS, = tan' 0 + tan0 (-cos0)²
tan 0(1 + tan 0) sin 6 (: sin
tan 0X sec 1
(1- cos0)
9+ cos9
=sec9- sec 9 R.H.S. (1- cos* 0)
L.H.S = R.H.S. Hence Proved. 1

Sol. 3 1 1
(1-cos0)(1-cos0)
(1-cos0)(1+ cos¬)
sec A 1
cosA N1- sin A 1-cos0
sin A sin A 1+ cos
and tan A= 1
cosA =RHS
1-sin A
[CBSE Marking Scheme, 2015] Hence proved.
[CRSE Marking,
Short Answer Type Questions-ll Long Answer Type Questions Scheme, 25
Sol.8: LHS = cot - tan 9 1 Sol. 8: (1) =sec'6, = tan0
cos sin
sin cos0
or I = sec0 tan'9 =1.
cos 0-sin²e
1
sin cos
cose-1+cos e Hence Proved..
sin cos (ii)
a
= cOsece, y
2cos -1 RHS Hence Proved 2
sin cos or, =cOsec 9- co9 =1
[CBSE Marking Schenme, 20181
Hence Proved.?

REFLECTIONS

The trigonometric ratios of an acute angle in a light triangle express the relationship between the angle and
the length of its slides.
Willyou be able to define the trigonometric ratios for angle Cin the hypotenuse right triangle?
C
side
adjacent
C
angle
to

B
A side opposite
to angle C
CHAPTER

Syllabus
10 HEIGHTS AND
DISTANCES
(Angle of elevation, Angle of Depression)
Stmple problems on heights and distances, Problems should not involve
more than two right triangles. Angles of elevation / depression shoula be
only 30, 45°, 60.

In this chapter youwill study


To understand the concept of angles of elevation and depression and able tosolve the questions based
on it.
To solve the problems based on trigonometry (Heights and Distance) of day-to-day life.

E Revision Notes
The line of sight is the line drawn from the eye of an (i) By height AB, means object is at point B from the
observer to the point on the object viewed by the point A located at the ground.
observer. Scan to know
(iü) AOisthe distance of the observer from the point
º The angle of elevation of a point more about A.
on the object being viewed is the this topic The angle of depression of a point on the object
angle formed by the line of sight being viewed is the angle formed by the line of sight
with the horizontal when it is with the horizontal when it is below the horizontal
above the horizontal level, ie., level, i.e., the case when we lower our head to look
the case when we raise our head Heights and at a point on the object.
Distances
to look at a point on the object. (Observer) Horizontal level
A
Line of sight, angles and altitude
(height).
LAninegle of osifghtdepres ion
B(Object)
B
(Object)
Line of sight
The height of object above the water surface is equal
to the depth of its image below the water surface.
Angle of elevation A The values of the trigonometric ratios of an angle do
o(Observer) Horizontal level
not vary with the length of the sides of the triangle, if
elevation. the angles remain the same.
(1) ZAOB is the angle of
o
MATHEMATICS (STANDARD)
Bank Chapterrisc & Topicwisc.
Question
R1 Oswaal CBSK
60
AB =
equatioas
ZC m
tan
=
30 . ( ü ) object V3
Distance
etwega B=90°,
BC =15
C 60
tan
) AB
m of = AB
200 both 60
k height tan L.e.,
and
-x-’B sofve AABC,
30 x Here,
Pind and
determine
In -Third
Level

S
mB

Level
To
60
Second
C

Level

First
Examples
Height

equal
Depressiun
Hortzontal
levd

is
Elevation
of
Objet

Angle
dev tiosof JAnge
LAne -
Measuring
Angles
neig1staces
ht
ang

of
Angle
to

ofExamples
Distance

...)..(i1) m
object 90 =343B=450,
length =4+DC
ßtan D=90° m
AB
DB D= i.e.,
BD=3 1)
width +
an flag solve
and
it +
AD30°,
AD m=3(N3
of (i1) Findtan
ox B
ie.,
Height/Length determine =AB =
A ZA PDAD PD BD3)
figure,
AAPD, +
tan ight
30o
= In
ABPD,=
45o
tan (3/3
trec broken
part
=
DC
AC From
a CD if
is =
is To right AB
BD
In .:.
()
HEIGHTS AND DISTANCES 213
OBJECTIVE TYPE QUESTIONS
(imark each)

Multiple Choice Questions


tan 9 = V3x
shadow
6m high casts a 2N3 mlong on the
tan = J3
aApole
thenthe
Sun's elevation is:
ground (B) 45° tan = tan 6â
(4)
600
(D) 90° 9= 60P
IC 30°
angleof
depression of a car parked on the
150 m high tower is 30°. The
he tromthe top of
az
road of the car from the tower (in mnetres) is:
distance f V3x
(4)50 3
(B) 150 V3
(C)150
V5 (D) 75 A

length of a string
between a kite and a point
o3The groundis 85 m. If the string makes an angle 2. 5. The angle of depression of a car, standing on the
onthe 15 ground, from the top of a 75 m high tower, is 30
withthe
groundlevel such that tan = then The distance of the car from the base of the tower
8
8 (in m.) is:
:height from the ground ?
at what
the kite is (A) 25 V3 (B) 50 3
(B) 79.41 m
(A) 75mn (C) 75 V3 C+ A
(D) 72.5 m A (D) 150
(C) 80m
correct. Q.6. In the figure below, what is the length of AB?
(A)is
Ans.Option tan =
15 D
Explanation: 8

15
sin = ..)
17
45 m
sin 0 = ...(ü)
Now, 85
30° 45°
(ii),
From, equation (i) and A B
15
17 85 45
x =75 m (A) 45V3 m (B) J3 m

() 45(V3-1) m (D) 45(V3 +1) m


85
m AOACBSE AdditionalQuestion, 2021-22]
Ans. Option (C) is correct.
Explanation: According to figure,
DC
In ADBC, tan 45° =
B
BC
45
Q4.If the height of a vertical pole is V3 times the 1=
BC
length of its shadow on the ground, then the angle
of elevation of the Sun at that time is: BC= 45 m.
(A) 30 (B) 60° DC
In ADAC, tan 30° =
(C) 45° (D) 75° +Ä AB+ BC
Ans. Option (B) is 1
correct. 45
Explanation: Let the length of shadow is x, AB + 45
Then height of pole = V3 x AB + 45 - 453
AB = 45(V3 - 1)m
Now, tan =
CB
AB
These equestions are for practice and their solutions are available at the end of the chapter
214 Öswaal CBSE Question Bank Chapterwise &Topicwise, MATHEMATICB(STANDARD) Clasa-X

SUBJECTIVETYPE QUESTIONS
A

Very Short Answer Type


Questions (1 mark each)
Q.1. The ratio of the length of a vertical rod and the
length of its shadow is 1: /3. ind the angle of
30
elevation of the sun at that moment ? B 30 m
ADACBSE Delhi Set-I, 2020] C
Sol. Let AB be a vertical rod and BC be its shadow.
From the figure, ZACB = 0.
ADCBSE Delhi &OD
In AABC, Topper Answer, 2020
A
Sol.
hat hte{ tousen-.ABehm
-Inxinht AABC,
tan 30 i
B 30

30rm
tan =
AB
BC

tan 0
1
e10234m
tan = tan 30°
1
0=30° 0.4. The ratio of the height of a tower and the
Hence, the angle of elevation of the sun is 30°.
Q2 In the given figure, find the angles of depress
its shadow on the ground is
angle of elevation of the Sun ?
J3:1 Whatleisnghthe
ions from the observing positions O, and O, UICBSE Delhi Set-I, 11, 2017|[CBSE Term-lI, 2019
respectively of the object A. 0.5. If a tower 30 mhigh,
casts ashadow
on the ground, then what is the angle 1043
AOA[CBSE OD Set-l, 2020] mlong
of elevation
of the Sun (UCBSE OD && Compt. 0D
Set-I, II, II 2017| [CBSE Foreign Set-, I, I, 20151
60°

TopperAnswer, 2017
(A45
Sol.

Sol.
60 Touei A8 is 29m.4nd _shodow BUSJDAm

lonAA
4445
B tane

Draw O,X || AC 1
ZAO,X = 90° -60° = 30 s0, anqle of eleyaHon of sun i3 60
and LAO,X = LBAO, = 45° 26. In the given figure, AB is a 6 m high pole and
Q.3. In figure, the angle of elevation of the top of a DCis a ladder inclined at an angle of 60' o u
tower from apoint C on the ground, which is 30 horizontal and reaches up to point D of po
laddes
m away from the foot of the tower, is 30. Find the If AD = find the length of the
2.54 m,
height of the tower. (use V3 = 1.73)
These questions are for practice and their solutions are available at the end of the chpter
HEIGHTS AND DISTANCES 215
allows the towing kite to gaina height of anything
Detween 100 metres -300 metres. The sailing kite
2.54 mn
1S made in such a wav that it can be raised to its
proper elevation and then brought back with the
D help of telescopic mast that enables the kite to be
raised properly and effectively.
61 Based on the following figure related to sky

Lader sailing, answer the questions:


In the given figure, if sin Ç = cos (30 - 30°, where 6
and 38-30 are acute angles, then find the value ofb.
B

+A[CBSE Delhi Set I, II, III, 2016] Rope


figure, a tower
AB is 20 m high and BC, its
shadow on the ground, is 20 J3
a7in mlong. Find the
Sun'saltitude.
A

(9,What should be the length of the rope of the


kite sail in order to pull the ship at the angle
20 m (calculated above) and be at a vertical height of
200 m ? C[CBSE SQP 2020-21]

B Sol. (i) cos (90°-0) = cos (30-30)


20 V3 90-030-30
0=30 1
AOC+ACBSE OD Set-I, II, III, 2015] AB = sin 30°
(ii)
0.8.If thelength of the ladder placed against a wall is AC
wice the istance between the foot of the ladder 200 1
and the wall. Find the angle made by the ladder AC 2
Length of rope =AC = 400 m 1
with the horizontal, ACBSE SA-I, 2015] CBSE SQP Marking Scheme, 2020
Sol. Let the distance between the foot of the ladder and y, standing
the wall, ABbe x. Q.2. The tops of two towers of height xand 30° and 60°
on the ground, subtend the angles of
Then AC, the length of the ladder = 2x respectively at the centre of the line joining their
feet, then find x:y.
AICBSE Delhi Set-i, II, III, 2015]

Sol.
Wall A

30 60
D
In AABC, LB=90°

cos A = Let M be the centre of the line joining theu feet.


2x
Let BM = MD = 2
1
Cos A = = cos 60°>ZA = 60° tan = Perpenicular
2
base
ShortAnswerpe In AABM = kan 30°
Questions-I ch)
Q.1.'SUseskyvesselextensive
sails' is that genre of engineering science that
in the seautilization of wind energy to move
water. The 'Sky sails' technology
1
)
These equestions are for 1Practice and their solutions are avatlable at the end of the chapter
Chapterwise &
Topicwise,
MATHEMATICS
(STANDARD), Class.X
16 Oswaal CBSE Ouestion Bank tan 30 = AB
In 4DB,
BD)
tan 60
In ACDM 1

X h=
From (i) and (ü),
V3
Multiplying (i) and (i), we get
3
1
I : = :3
2015]
[CBSE Marking Scheme,
water,
light house, 40 m above the
Q. 3. From the top of h= Vabcm
of a small boat is 60.
the angle of depressionfrom the base of the light
Find how far the boat is UCBSETerm-II, 2015] Hence, the height ofthe candle is
house. Q. 2. Fromthe top of a 120 mhigh tower, a man
ShortAnswer ype two cars on the opposite sides of the
(6marks each) straight line with the base of tower
Questions-II
of depression as 60° and 45°. Find the
between two cars. disteg
A
{CBSE Compt. Delhi/OD Set-I,
B
Sol. X-- 6045°

à = 120m
Tower

- iCm
A

top of the candle In AABD, ZADB = DBY= 45 (alternahe :nl


If the angles of elevation of the b' cm (a > b)
from two coins distant a' cm and and in AABC, ZBCA = ZXBC = 60
straight line from it
from its base and in the same AB = tan 45
the candle.
are 30° and 60°,then find the height of In AABD,
AD
C+AICBSE SQP 2020-21)
120
=1
AD

Sol. AD = 120 m
Candle AB = tan 60
Now, In AABC,
hm CA

120

60° CA
30
C-bB CA = 120 -403 m
CD = AD +CA
Let AB = candle
CandD are two coins. = 12) + 405
AB
In A4CB, tan 60°
BC 189,231
120 + 69.28 =
Hene the distance between two
[CBSE Mfarkiug Schene
h b3 ..0)
HEIGHTSAND DISTANCES
217
shadoyof a tower at a time is
three times as
Teasitsshadow when the angle of elevation of
Sun is6 Find the angle of elevation of the
thetim of the longer shadow.
Sanat h-50 =
ACBSE Foreign Set-1, 1i, II, 2017) Hence
straight line passing through the foot of a
twopoints Cand Dare at distances of 4 m
d16m.from the foot respectively. If the angles of
V3h-50/3 =h
evationtrom Caand Dof the top of the V3h -h = 50V3
qAnplementary,,then find the height of tower are
the tower.
A|CBSE OD Set-1, IL, II, 2017] i(V3-1) =s048
h=
5043
Topper Answer, 2017 3-1
h=
soV3(V3 +1)
tenprmntory 3-1

h=
s0(3+ W)
2
ione
- h=75+25 V3 =75 +43.25
= 118.25 m 1
[CBSE Marking Scheme, 2016]
6JA man standing on the deck of a ship, which
is 10 m above water level, observes the angle of
(oi elevation of the top of a hill as 60° and the angle
of depression of the base of hill as 30. Find the
distance of the hill from the ship and the height of
the hill. A[CBSE OD Set-II, 2016)
em ond
Topper Answer, 2018
Sol.

ns iqnenng-e value) wtuu Bl 10md EL:30


heghi of towerm
0.5, The angles of depression of the top and bottom of
a50 mhigh building from the top of atower ar 45°
and 60° respectively. Find the height of the tower
and the horizontal distance between the tower and
the building. (Use y3 = 1.73)
A CBSE DelhiSet I, I, III, 2016] 17-32 Ym
Sol.
J456

10J3

Tuaat afhil:h10m
50 n
40m
3
OA7 m long flagstaff is fixed on the top of a
standing on the horizontal plane. From point tower
tan 45°=
h-50
the ground, the angles on
of elevation of the top and
bottom of the flagstaff are 60° and 45°
x =h-50 Find the height of the tower respectively.
correct upto one plaçe
of decimal. (Use V3 =
tan 60°.= 1.73)
the chapter
ACBSE Foreign Set li, 2016}
O These questions are for practice and their solutions are uvailable at the end of
218 Oswaal CBSE Question Bank Chapterwise &Topicwise, MATHEMATICS (STANDARD), Clans.x
Q. 2. The angles of depression of the
50 neters
of a building
Long Answer Type(5 marks each) as high
Questions
top of a tower are 30° and 60°

oberved t
1. The two palm trees are of equal heights and a
standing opposite to each other on either side
the height of the
distance between
tower, and
the also Tespehcetivetrwe.
building and the
of the iver, which 80 m wide. From a point 0
between them on the river the angles of elevation
of the top of the trees are 60° and 30, respectively.
Find the height of the trees and the distances of the
Sol.
X
30
R

a1CBSESO
(h-50) m
point Ofrom the trees. (use N3 = 1.73) 30
A
ADAICBSE SQP 2020-21
Sol. Building
50 n

60°
Tower
60 30
B 80m -X m

Let BD = width of river = 80 m Let height of building, AB = 50 m


AB = CD and height of tower, RT =hm
= height of bothpalm trees =h BT = AS = xm
BO = Xm AB = ST = 50 m
OD =80-x m RS = TR-TS =
In AABO,
(h-50) m
tan 30°
RS
In AARS,
AS
tan 60° =
1 h-50

.)
RT
In ARBT tan 60° =
h= V3x BT
In ACDO,

tan 30 =
(80-x) Solving () and (i), we get
h=75
1
..i) 75
(80- x) From (i), X=
V3
Solving (i) and (i), we get
= 25/3
X= 20 m
h= V3x Hence, height of the tower=75 m
[From eqn. (0)]
Distance between the building and the tower
= 1.73 X 20
= 25V3
=34.6 mn
= 25 × 1.732
The height of the trees = h=34.6 m
= 43.30 m
BO =X =20 m
DO =80-x Q. 3,A vertical tower stands on horizontal plane si
= 80-20
surmounted by a vertical flag-staff of height
The angles at a point on the bottom and
= 60 m
the flag-staff with the ground are 30 anto
: The distances of the point O from the trees are respectively. Find the height of the towe
20 m and 60 m respectively.
(Take y3 = 1.73) ALuCHSE Delhi Set.!.
HEIGHTS AND DISTANCES 19
question.
Acordngto
al. flagstaff.and BD is atower.
ADisa

6m
60 20 m
D
45
A
P

\20 In APAD,
B AD 20 + BD
tan 60° =
AP 20
h AABC, 20+ BD
tan 45°= AB 20
BC
20 + BD = 20/3
h+6
1=
BC
BD = 20/3 -20

BC = h + 6 .)
- 20(V3-1)
DB = 20(1.732-1)
InADBC, tan 30°= = 20 x 0.732
BC
= 14.64 m.
1
[from (/)) Hence,the height of the tower is 14.64 m.
h+6
Topper Answer, 2020
hN3 =h+6
A
hv3 -h= 6
h(v3-1) =6
6 V3+1
h=
J3-1 3+1
h= 6V3 +1) ut 'AB’_ratsmussion touse
2
Be Buitdum -2om.
h=3(/3 +1) R- fsint on tfia ground
=3(1.73 + 1)
h=3X2.73
Io A PBC.C=90
h=8.19 m tam4s'.RO20
. The height of the tower is 8.19 m
Q4.From apoint on the ground, the angles of elevation
of the bottom and the top of a transmission tower
are 45° and 60° respectively above of 20 m high
tam6o'-ite0
building, Find the height of the tower.
AICBSE Delhi/OD Set-i, 2020}
Sel Let the height of the tower be BD
In APAB
AB
h20s.-2o.
tan 45° =
AP

20
AP

AP = 20m
220 Oswaal CBSE QuCS AB 3000 m
the top ofa building from
Q.5. The angle ofelevation of elevation DC
tower is 30° and the angle of In ADAC,
AC tan 30°
the foot of a buildingis
top ofa tower from the foot of the height
of the
tower is 50 m high, then find the 2020] AC = 9000 m
60 IF the Set-I,
CBSE OD BC =,- AC- AB
of the building.
Sol. Accornding to question, 6000
. Speed of aeroplane = 30 m/sec
= 200 m/sec
Building = 720 kmh
[CBSE SQP
Detailed Solution:
Marking, Scheme,
60R
201
D /AED = 60° and ZBEC = 30
X

AD= BC= 3000 3 m


In AABD,
AB Let the speed of the aeroplane =I m/s
tan 60°=
BD
50
J3 =
BD
50
|3000/3 m
60
BD =
V3 30
A B
Now in ABDC,
CD
tan 30° BC = DE = 30 X x
BD Then,
= 30x m

50 In AABE, ZD= 90°


50
BE
tan 60° =
AB
3h = 50 De
5
h=
50
= 16.67 V3 = 3000
3 DE

Hence, the height of the building is 16.67 m. AB = 3000 m

he angle of elevation of an aeroplane from In ADAC, ZC= 90°


30
a point on the ground is 60°. After a flight of DC
becomes 30. If tan 30° =
seconds the angle of elevation AC
the air plane is flying at a constant height of
1 300043
3000 V3 m, find the speed of the aeroplane.
DE+ CD
AO CBSE SQ 2020]
DE + CD = 3000 ×3
Sol.
3000 + 30x = 9000 [from eqs. () and
30x = 6000
3000/3 n x= 200 m/s
3000/3 m Hence, the speed of plane = 200 /s.
18 =720 kh
A 200 x
5
30x m standingopus

Correct figure. 1
gawo poles of equal heights
to each other on either
are
side of the
road
on
theat
BEE between
In dABE, tan 6Y 80 m wide. From a point in
the top
AB
road, the angles of elevation of
eland
30° respectively. Findthe
ang the distances of the point height of
Pfrom the HEIGHTS AND DISTANCES21
poles

A0A(CBSE1 Delhi, the


fsimilarto Q.No. 1LA.IO. Set-f, 20191 sin 45 PR
a
standing on
horizontal
indlyingata distance of 200 mfrom himand
Ant,
at ana plane, 1

P)
of 30°. Deepak
devation
standing on the roof of a
highbuilding, andthe PD 042 n
0m angle of
elevation
of
samebird to be 45°, Amit and
the
Deepak are on 5) 1A14 m
sides of the bird.
gposite Findthe
g distance of the 70.7 m
from
hid
Deepak. BAICBSE OD Set-1, 2019) 70.7Thus,m. the distance of the bird from Deepak is
(.9. Fromapoint Pon the
201 0
of the top of a ground, the
tower is 30 and angle of elevation
45
DDeepak) flag-staff fixed on the top of thethattower
of the top of the
length of the flag-staff is 5m, ind is 45°. If the
30 |50 m
tower. (Use 3 =1.732)
the height of the
AAmit

InAAPQ, PQ = sin 30= 1 Sol. Let AB be atower JCBSE OD Set-II, 2019


AP 2 1 and BC be aflagstaff.
PQ =(20)) = 100 m 1
5m
PR = 100-50 =50 m
1 B
PR
In APRD, = sin 45° = 450
PD
30
P A
PD = (PR)(W2)= 50N2 m 1 1

[CBSE Marking Scheme, 2019] In APAC, according to


tziled Solution:
question,
AC
Let Pbe the position of the bird, Abe the =tan 45° =1 1
position AP
of Amit, Dbe the position of
Deepak and FD be x+5 =y
the building at which Deepak is
50 m. standing at height
In APAB, = tan 30°=
Given, AP = 200 m and FD = 50 m 1

In APQA, ZQ=90° 1
(irom eç. (i))
sin 30° = PQ
PA
5

PQ
2 200

200
PQ = = 100m
2
13.66
PR = PQ-RQ 2

= PQ- FD 6.83
= (100-50) m Height of tower = 6.83 n1
n APRD, 50 m
[CBSE Marking Schene, 2019}
ZR= 90°
Oswaal CBSE Question Bank Chapterwise &Topicwise, MATHEMATICS (STANDARD), Class-X
0. 10. The shadow of atower standing on alevelgroundis foundto be 40 mlonger when the Sun's altitude
/3 =1.732)
when it was60. Find the
height of the tower.
(Given
|CBSE Delhi
Topper Answer, 2019

Sol.

tona at aun's albilusle


at 6o

a 30

BC

tan 30 A& AB
40tbC

AAX 2 40 2

AL 20x/·73 2ny 3464m

1
Q. 11 The angle of elevation of the top of a hill from the(0. 12./Two points A and B are on the same side ot
tower and in the same straight line with its bas
foot of a tower is 60 and the angle of depression
from the top of the tower of the foot of the hill is The angle of depression of these points from t
respeetively. If the
30°. If tower is 50 meter high, find the height of the top of the tower are 60 and 45°
height of the tower is 15 n, then find the distani
hill. ACESEComptt. Set-I, II, TII, 2018j
[CBSE Delhi Set-1, I, III, 2015] between these points.
C +[CBSE
Deli Se-l 208

These questions are for practice and their solutions are available at the end of the chapter
60° the same line.HEIGHTS AND DISTANCES 223
he angles of From the pointA of an
ZBCA =45
depresslon
Z0AD = 30 i.e., BDA
of two ships Daeroplane,
=30 and 20ACand Care
Let distarnce = 45° i.e.,
15m
between two ships
Tower and
DCXm
BC
60
A5
30
A(Aeroplane)
DC
InADCA, = tan 60
CA
15 7500 m
= J3
15 30
D 45°
B
+y
x= 5W3
1
DC -= tan45°= 15 In AABC, AB
= tan 45°
n ADCB, =1
CB BC

x+y= 15 7500
=1
5V3+y =15
y=7500
y= 15-5/3 AB
In AABD, = tan 30°
s(3-J)m BD
7500 1
Hence, the distance between the points
X+y
=3(3-43) m x+y= 7500 /3
[CBSE Marking Scheme, 2017|1 x+ 7500 = 7500V3
D13. The angle of depression of two ships from an
aeroplane flying at the height of 7500 mare 30° and x= 7500/3-7500
45°, If both the shipsare in the same line that one
ship is exactly behind the other, find the distance - 7500(V3 -1)
between the ships. = 7500(1732-1)
C+ A[CBSE Foreign Set-II 2017| =7500 X0.732
Sol. Let AB be the height of the aeroplane, then AB = =5490 m
7500 m. Hence, the distance between two ships
Also let D and C be the positions of two ships on =5490 m
V4 As observed from the top of a light house,100 m high above sea level, the angles of depression of a ship,
sailing directly towards it, changes from 30° to 60°. Find the distance travelled by the ship during the period of
observation. (Use 3 =1.73) C[CBSE OD Set-II, 2016]

Topper Answer, 2016

100m

136
CyI
MATHEMATICS (STANDARD),
Question Bank
Chaptenwise &
Topicwise,
Class-X
Oswaal CESE

ZADB 30
BD

100 N3-160m

[3-1] 10o% 2
3
106 2xJ3 2O3
3

200Y173m
CD:
3
3.44m
115.33 m.

VCOMPETENCY BASEDQUESTIONS (4 marks egch)


Explanation: From figure,,the
electrician
to reach at the point Bon the pole AD.
is
O Case based MCOS So, BD = AD - AB
sub = (5-1.3) m = 3.7m
Attempt any 4 sub-parts from each question. Each
Q. 2. What shouldbethe length of
part carries 1 mark.
LRead the following and answer any four questions
ladder,when inclined
at an angle of 60° to the horizontal ?
from Q.1. to Q.5. 3.7
(A) 4.28 m (B) m
An electrician has to repair an electric fault on the
pole of height 5 m. He needs to reach a point 1.3 m
(C) 3.7 m (D) 7.4 m
below the top of the pole to undertake the repair
work (see figure). AD[CBSE QE, 2021] Ans. Option (A) is correct.
Explanation: In ABDC,
A sin 60° =
BD
BC
3.7
2 BC
B 3.7x2 3.7x2x3
BC=
V3 3

BC = 4.28 m (approx.)
Q.3. How far from the foot of the pole should he pac
the foot of the ladder ?
(A) 3.7 m (B) 2.14 m
602 (C) (D) None of these

Q.1. What is the length of BD ? Ans. Option (B) is correct.


(A) 1.3 m (B) 5 m Explanation: In ABDC,
(C) 3.7 m (D) None of these
cot 60° = DC
Ans.Option (C) is correct. BD
HEIGHTS AND DISTANCES 225
DC
3.7

DC =
3.7 3.7/3
3 42 m
DC= 2.14 m (approx.)
horizontalangleofisthe
changedto 30°, then what
ladder ? R
IHthe length 42 m
should
24 bethe (B) 3.7 m
A)74
n AB
(D) 5 m Now, tan =
C 1.3nn BC
( A )is correct.
Option,
Erlanation:In ABDC. 42
tan =
ins 42
BD
sin 30° tan 9=1
BC
tan = tan 45°
3.7 =45°
2 BC Hence, angle of elevation = 45°
BC= 3.7 × 2=74 m 2.2. They want to see the tower at an angle of 60°. So,
should
value ofZB ? they want to know the distance where they
i sthe
What stand and hence findthe distance.
a5(4)60° (B) 90° (A) 24.24 m (B) 20.12 m
(D) 180 (C) 42 m (D) 24.64 m
( )30°
correct.
(C)is Ans. Option (A) is correct.
ins.Option D is 90e
Ernlanation: In ABDC, angle Explanation: Height of IndiaGate = 42 m
angle sum property.
Angle of elevation = 60°
So, byB+D + 2C= 180 Let the distance between students and India gate =
ZB= 180° (90° + 60°) X m.

= 30° A

IL. Readthe:
following and answer any four questions
Q.5.
from Q.1. to 42 m
students of class X visited India Gate on
Agroup of students had
an education trip. The teacher and
interest in history as well. The teacher narrated that
B
607
India Gate, official name Delhi Memorial, originally
called AI-India War Memorial, monumental AB
sandstone arch in New Delhi, dedicated to the Now, tan t =
BC
troops of British India who died in wars fought
42
between 1914 and 1919. The teacher also said that tan 60° =
India Gate, which is located at the eastern end of
the Rajpath (formerly called the Kings way), is 42
/3 =
about 138 feet (42 metres) in height.
42
[CBSE QB, 2021]
V3
42x V3
V3x3
4243
3
.What
at a
is the angle of elevation if they ate standing = 143 m = 24.24 m
distance
(A) 30°
of 42 m away from the monument ? 0.3. If the altitude of the Sun is at 60°, then the height
(B) 45° of the vertical tower that will cast a shadow of
(C) 60° length 20 m is:
Ans. Option (B) is (D) 0°
correct. (A) 20V3 m
20

Expl
Distanceanatebetween
ion: students and India Gate
HHeight of India gate = 42 m
15
(B)

= 42 m (C) (D) 15/3 m


MATHEMATIC8 (8TANDARD),
&Topicwise,
OswaalCBSE
Question Rank Chapterwise
0.1. The distance
of the Clas -Xthe
satelite from
226 Devi is:
Ans. Option (A) is
corect. top
Explanation:
A 60
(A) 1118.36 km
(C) 1937km
(B) 577.52 km
(D) 1025.36 km
ofNang
Ans.Option (A) is correct,
Explanation:

60 Satallite
F
20m

h
A 30T600
Let, the height of tower = H
AB
tan 8 =
Now, BC
B D
tan 60° =
20
Q S R
Nandadevi

h= 20/3
20
Now, AG = 1937 km
2
Mul anyangiri
shadow is
Q.4. The ratio of the length ofa rod and its
1:1. The angle of elevation of the Sun is: COs
AG
(A) 30 (B) 45° AF
(C) 60 (D) 90°
Ans. Option (B) is corect. 1937
Q.5 The angle formed by the line of sight with the cos 30° 2
horizontal when the object viewed is below the AF
horizontal level is:
(A) corresponding angle 1937
(B) angle of elevation 2 2AF
(9 angle of depression
(D) complete angle
Ans. Option (C) is correct. AF = 1937 1937V3
IL Bead the following and answer any four questions 3
from 0.1. to 0.5.
Asatellite flying at height h is AF = 1118.36 km
of the two tallest mountains in watching the top Q. 2. Thedistance of the
Uttarakhand and satellite from the
Karnataka, them being Nanda Devi (height 7,816 Mullayanagiri is: top df
m) and Mullayanagiri (height 1,930 m).
of depression from the satellite, to the topThe angles
of Nanda
(A) 1139.4 km
(B) 577.52 km
Devi and Mullayanagiri are 30 and 60° (C) 1937 km
If the distance between the peaks of tworespectively.
mountains
(D) 1025.36 km
is 1937 km, and the satellite is Ans. Option (C) is correct.
the midpoint of the distance verticallytheabove
mountains. between two Explanation: In AFPH,
UCBSE QB, 2021]
cOs 0 PH
FP

COs 60°= 1937


2FP
1937
2 2FP
FP = 1937 km
Q.3. The distance of the
satellite from the ground ls:
(A) 1139,4 km (B) 567 km
Nanda Devi (C) 1937 km
Mulayanagiri (D) 1025.36 km
Ans. Option (B) is
correct.
HEIGHTSAND DISTANCES227
Eylanation:
satellite from the ground= Fi
ofs
stane = FG + GI Mullanyangiri
|FG= AGtan30°1937 I930 m

1937
-+7.816
-|243 B
45
Mile stone
= 566.99 =567 km
theangle
Whatis
of elevation if a man is standing Case based Subjective
distance of 7816 m from Nanda Devi ?
a
sta
(B) 45°
Questions
(4)30 IRead the folowing text and answer ny four
(D) 0° qáestions from Q1 to Q5.
:Option(B) is
coTect. A fast jet-powered miitary aircraft designed for
attacking other aircraft. Domination of the airspace
Eyplanation: Height of Nanda Devi Mountain above a battle fled permits bombers and attack
= 7816 m
alrraft to engage in tactical and strategic bombing
mountain
ctance between man and of enemy targets. The united States Air Force is
= 7816 m. composed of 5217 active airraft making it the
A
largest, the most technologically advanced, and the
most powerful air leet in the worid. The angie of
elevation of ajet ighter fom point A on ground
7816 m is 60. After flying 10 seconds, the angle changes to
30. If the jet is flying at a speed of 648 kh.
B 7816 n

AB
tan =
BC

7816
tan 1816
Q.1. Find the distance in meter travelled by Jet in 10
tan =1 seconds.

tan = tan 45° Sol. Lhour =60 x 60 =3600 seconds.


0= 45
ánd 1km =1000m
·. Distance traveled by a jet in 3600 sec = 6S000 m
5.If a mile stone very far away from, makes 45° to
the top of Mullanyangiri mountain. So, find the Distance travelled by a jet in 10 sec
64800
istance of this mile stone from the mountain. 3600
-x10
(A) 1118.327 m (B) 566.976 m
= 18O m
(C) 1930 m (D) 1025.36 m
Ans, Option (C) is .2. According to case, the figure is:
correct.
Explanation:
AB
tan 45 =

1930 3
1=

X= 1930 m
Then findthe value ot a.
Topicwise,
MATHEMATICS
(STANDARD),
223 Oswaal cBSE
Question Bank Chapterwise & Class-X
Sol. In AABC,
tan60°= 5
45R60
1

h=3 ...)
1
|300 m
tan 30°:
In AAED, r+1800
I+ 1800
B 60°
...i)
V3 0.1. Find the value of y.
Sol. In APBM,
From eq. (i) and (ii), we get
I+1800 tan 45° = PM
BM
V3
300
3.1 =x t 1800 1
2x= 1800
X= 900 m. y= 300 meters.
0.2. Find the value of x.
find the constant
Q.3. Find the figure of question 2, Sol. In AAMP
height at which the Jet is flying.
tan 60° = PM
Sol. Since, x = 900 m (Prove above) AM
.. Substitute the value of x in eq. (i),
V3 = 300
h= xv3
= 900 V3 meter. 300
=100V3
Q.4. How many aircrafts composed by USAF?
= 100 × 1.732
Sol. USAF is composed of 5217 active aircrafts.
= 173.2 meters.
Q.5. Which mathematical concept used in this case?
0.3. Find the width AB of the rive.
Sol. Applications of Trigonometry or Height and
Distance. Sol. We have proved that
II. Read the following text and answer any four x= 173.2 mand
AB = x +y
y=300 m
questions from Q1 to Q5.
An aeroplane is a vehicle with wings and one or = (173.2 + 300) m
= 473.2 m
more engines that enable it to fly through the air.
Q. 4. Find the distance of PB.
Most people think about inventors of aeroplane
Sol. In APMB,
was Wrights brothers, we have discussed about an
PM = 300 m,
aeroplane as following:
and ZPMB = 90.
BM=y=300 n
An aeroplane at an altitude of 300 meters observes
the angles of depression of opposite points on the
(PB) = (PM) + (BM²
two banks of a river to be 45 and 60°. [Using Pythogoras theorem
= (300) +(300)
[Use, v3 = 1.732] = 90000 + 90000
= 180000
PB = V18x10000
= 100x342
= 300/2 m.
Q.5. Which mathematical concept used in this case!
Sol. Application of trigonometry.
Solutiornsfor HEIGHTSAND DISTANCES 229
Practice
Mlticle
e Questions
Choice
(4). correct.
:Option is
QuestiVeryonsShort
Etylanation:In AABC, ZB = 90° Answer Type Questions
6
tanB=. N3= tan 60° ’ 0=60°
2/3
A
Sol. 4:

lower
6m

B
2/3 m B Shadow
Option(B) is correct.
Ans2: Let the height of tower be AB and its
Elanation: In AABC, ZB=9no BC. shadow be
30yC AE
= tan 9
BC
150 m

30° 1
= tan 60
Hence, the angle of elevation of Sun = 60°.
tan = CB
AB [CBSE Marking Scheme, 2017|
Sol. 6: Given, AD = 2.54 m
tan 30° = 150
DB = 6-2.54 = 3.46 m
In ABCD, ZB = 90°
150 BD
sin 60 =
V3 DC
X= 150N3 m 3.46
ns. 5: Option (C) is correct. 2 DC
Erplanation: In AABC, LB =90° DC =
3.46 x 2

30
V3
3.46 x 2
1.73
75 m
= 4m
. Length of ladder = 4 m.
)30°
Sol. 7: Let the ZACB be , ZB= 90°
CB A
tan = tan =
AB BC
20 1
75 tan = =tan 3° 1
tan 30° =
20/3
1 75 0=30
V3 Thus, the Sun's altitude is 30,
[CBSE Marking Scheme, 2015]
x= 75/3 m
Topicwise, MATHEMATICS
230 Oswaal CBSE Ouestion Bank Chapterwise & (STANDARD), Clas -X
Short Answer Type Questions-l
Sol. 3:Let AB be the light house and Cbe the position of In AABD, AB
the boat. = tan 0
AD 1
Since, PAC =60° ZACB = 60°
Let BC be x m.
AB = tan 60° = tan 0
In AABC, 3r
BC
40 x/3 1
= /3 3 Atan 30
40 =30
[CBSE Marking Scheme,2017
P A
Sol. 7:

7m
40 m

Light house B

60 Tower
B

40
x=
45°
60

40V3 m

4043 BC
Hence, the boatis maway from the foot of In ABCTD, = tan 45°
3 CD
the light house[CBSE Marking Scheme, 2015] 1
Short Answer Type Questions-Il i.e., = tan 45° = 1

B
*=y
Sol. 3: x+7
In AACD, > tan 60° =3
y3 =x+7
Putting y=x, then
Tower x3 =x+7
60°
7 =(V3-1)x 144
D
3 75+1) 2
7(2.73)
AB 2
In AABC, = tan 60°
AC 19.21
=9.6)
2
h
=3 =9.6 m
[CBSE Marking Scheme, 2016|
HEIGHTSAND DISTANCES 231
rType Questions
Or,
BC = 50/3 m
Again in ABCD, ZBCD = 90
DC
tan 60=
BC

30° hill DC = BC tarn 6O


=
50/3 x V3 m
Tower DC = 150 m
60° 30° .:. The height of hill is 150 m.
B

Commonly Made Error


50 m the
be height of the tower and CD
LetAB = ofhill,
height The concept of angle of depression is not clear
hethe to many students. That's why they are not
NOw,inAABC.
ABC =90 able to draw the diagram correctly.
AB
tan 30° =
BC Answering Tip
50 50xV3
BC = tan 30 1
m The concept of angie of depression and angle
of elevation must be ciear to the students.

REFLECTIONS
. ul wou be able to solve the questions based on the concept of angles of elevation and depression?
How can you apply the concept of heights and distances in solving your day-to-day problems? lIlustrate with
eamples.
SELFASSESSMENTPAPER-5

Maxinum Time: I hour

given options.
I.Choose the correct altemative from the =
45°, then x
Q. 1. If tan 45° - os30= I sin 45° os
(B)
(A) 2

3 (D)
(C)

Q.2Iftan then cos e- sin' e is equal to:


4
7
(4) I (B) 25
7
(C) (D) 25

Q.3.lf cos = , then 2sec² 8+ 2 tan² -7 is equal to:


(A) 3 (B) 4
(C 0 (D) 1
Q.4.lf x= a sec Oand y= band y = btan ., then br -ay =
(A) at (B) ab
(C) - (D) a²+
Q.5. If cos A+ cos² A= 1, then sin' A+ sin Ais:
(A) 0 (B) -1
(C) 1 (D) None of these
Q.6. lf sin 9 - cos = 0, then the value of sin+ cos 0is:
1
(A) (B)
4 2
3
4 (D) 1
II.Case-Based Question
0.7. Read the following text and answer the given below [1 x4=4
questions it:
Neelesh is having a garden near Bangalore. In the garden, there
One day due to heavy rain and storm one of the are different types of trees and flower plank
trees got broken as shown in the figure. The height of he
unbroken part is 15 m and the broken part of the tree has fallen at 20 m
away from the base of the tree.

15 m

20 m
HEIOHTrs AND DISTANCES 233
paragraph and
the above
carefully
length the broken part?
of answer any four questions:
Resd i sthe
1What
A)
15m (B) 20 m
C)
25mn
of the full tree?
(D) 30 m
the height
was
What (B) 50 m
A)
40m
()
35mn
right-angle triangle (D) 30m
formed what is the length of the
A4)
i)
In
15m
thet
(B) 20 m hypotenuse?
(D) 30 m
( )25
mn
oftho formed right angle triangle?
the area
isnn?
What
(A)
100 (B) 200 m²
()
60m2 (D) 150 m²
perimeter of theformed triangle?
isthe
What
(B) 50 m
(A)60m
( )45
m (D) 100 m
ShortAnswer Type Questions [1 ×3 = 31
a
Very 12
then find tan B- sin² B.
8hcotB 5

0) cOsec 0 = 1.
that: (1 - cos
18
Prove sin2 = 1, prove that: cos 9 + cos 0 = 1.
+t
sin Answer Type Questions-I [2 × 3 = 6]
KShort
tan + cot 0 - + 2 = sec 0cosec² 0.
that:
Prore
V2-1, show that sin Acos A =
A= 4
itan
ladder15
metres long just reaches the top of a vertical wall. If the ladder makes an angle of 60° with the wall,
A of the wall.
indtheheight
60°
15 m

A
[3 × 2 = 6]
Questions-II
VShort Answer Type of elevation of the top of tower is found to be equal to
the angle of
o1 From the top of a cliff 20 mn high, the angle height of the tower.
depression of the foot of the tower. Find the making an
so that the top of the tree touches the ground ground is 8
BAtree breaks due to storm and the broken part bends the tree to the point where the top touches the
angle of 30 with it. The distance between the foot of
m. Find the height of the tree. [1 x 5 = 5]
Answer Type Questions a car at an angle
1L. Long
straight highway leads to the foot of a tower. A man standing at the top of the tower observes
speed. Six seconds later,
lh A towards the foot of the tower with a uniform
of depression of 30°, which is approaching be 60. Find the time taken by the car to reach the foot of the tower
the angle of depression of the car is found to
from this point.
OSWAAL COGNITIVE
Finished Solving the Paper ?
Time to evaluate yourself ! LEARNING TOOLS

OR
SCAN THE CODE

For elaborated
Solutions
MENSURATION
UNIT VI

CHAPTER
AREAS

11 RELATEDTO
CIRCLES
Area of sectors andsegments of a circle. Problems based on areasS and
FAEI Syllabus circumference of the abovesaidiplane figures. (In
perimeter/
area of segment of a circle, problems should be restricted to centra
only)
calculating
angle of6îand90°

Inthischapter you will study


About circle with its parts. circle.
minor and major sectors of the
To understand the area of the and major segments of the circle.
To understand the area of minor
To solve the shaded problems in acircle.

Revision Notes
A
circde is a collection of all points in a plane which
are at a constant distance from a fixed point in the
same plane.
A Radius
A

Scan to know
s
segment joining any two points of acirde
more about Aline
this topic
Aline segment joining the called a chord. Achord passing through the ent
centre of the circle to a point of circle is called its diameter. Adiameter is
on the circumference of the largest chord of the circle. Here AB is a diame
ircle is called its radius. which is a longest chord.
Area related to Adiameter of a circle divides a circle into two
circles arcs, each known as a semi-circle.
AREAS RELATED TO CIRCLES
235

Segment
Minor triangle
SegmentB sector
Major
4OAB
corresponding
corresponding
A
Meaning
arc
corresponding
thebetween
enclosed
region
circular
the
Formula of
area
9360sin
ofPortion and Tr-
thethe
chord of of 360X
ircumference
X Area
Area P

a
=
arc
of
Length Area

360° 1=60°
Segmernt

Circles
Formula Relate
to

of circle
a
Sector

Circle

Level
Third
Sector
Minor
ofArea circle
Sector Major
Sector
B

2ar
of
Map
area Meaningenclosed
region
circular =
diameter
the arc
corresponding MindLevel
=oArea
X
360°
ofPortionand
the
Formulae Second
radius X the
Area :Area 1
Tace
two Circumference
=

by Level
+e
= First
Area

" "
Topt
Chapterwiseö
OuestionRank
OswaaiCBSE
236
A

R
Semi-irce B

(or arcs) are said to


Two circles be
on placing one over
the other cong
completely.
The distance around the circle or the
circleis calledits circumference or
The mid-pointt ofthe hypotenuse of a perimeter.
from the vertices of(the triangge.
Dhamete
is equidistant at the.e
Chord Anglesubtended
is always a right
angle. circumferenceby diame
a

called an arc.
circumferenceofcircleis than thatofa
Apartof a Jength is less
arcofa circle whose minor arc.
An the same circle is calleda
semi-circle of that of
whose length is greater than
Ån arc of a circle same circde is called a major arc. A
the
a semi-circde of two
bounded by an arc of a circle and
The region
is called a Sector,
radiiat its end points

Major Sector Angle described by minute hand in 60 minutesS


360°.
Angle described by hour hand in 12 hours is 3k
O- Key Formulae
A 1. Circumference (perimeter) of a circde = z
Minor Sector
2r, where d is diameter and r is the radius of th
circle.
2. Area of a circle = r.
1
O- Key Words 3. Area of asemi-circle = -2
Circumference: carrying around means the
perimeter of circle. 4. Perimeter of a semi-circle = r+ 2r = (r +2r
= z(R + r\R
5. Area of a ring or an annulus and
Arc: Any smooth curve joining two points. where R is the outer radius r is the ine
Sector: A part of a circle made of the arc of the radius.
circle along with its tworadi. 2rr , where 8is te
6. Length of arc, I= 360° 180°
Achord divides the interior of acircle into two
parts, each called a segment. angle subtended at centre by the arc.
or area of sectet
7. Area of a sector =
360°

x), where lis the length of arc.


Major Segment
8, Area of minor segment = 360° 2sina
circle-Arä
Minor 9. Area of major segment = Area of the
Segment of minor segment segtt
= y2 Area of minor cent
atthe
Circdes having the same centre but different radii 10, If a chord subtends a right angle
then area of the corresponding
arecailed concentric circles.
AREAS RELATED TOCIRCLES237
subiends an angle of 60 af e
todarea of the corresponding segment 13. Number of reuolubons in one minute
then Distance moved in1minute
Circumference
14. Perimeter of a sector = -+2r.
moved
by a wheel in 1 revolution =i 180
Ce
irumterence the wheel.
of

Fundamenta Facts
mathematican
Srinivas Ramanujan worked out the identity using the value of n correct to millíon
ndian
decimal:
asof
mathematician Aryabhatta gave the value of n as 62832
.ndian 20000
(madeagreater discovery" this mnemonic help us in getting the value of r =3.14159
,eitundersseparate reading with explanation how to use
CAN I HAVE ASMALL |CONTAINER OF COFFEE
Noof
1 4 1 5 9 2 6
Letters
Arhimedesscalculatedthe area of acircle by approximatingit to asquare.
sreaofsectorrofacircle ddepends on two parameters-raius and central angle.

(1 mark each)
OBJECTTVE TYPE QUESTIONS
Ans. Option (C) is correct.
Multiple Choice Questions Explanation: Length of minute hand = Radius of
where the the quadrant/sector soformed= 84 cm.
i1 The area of a quadrant of a iscircle In 1 minute,minute hand makes an angle of 6°.
dirqumference of circle is 176 m, Therefore, in 15 minutes it makes an angle of
(A) 2464m? (B) 1232 m?
(D) 308 m? 15 x 6°= 90°
C 616 m Distance covered by the tip of the minute hand
s Option (C) is correct. = Length of arc
Explanation: It is given that circumference of
the circle is 176 cm
3600
2tr = 176

2X Xr =176 900 22
7 x 2 x 7 x 84
360°
176x7 = 132 cm.
=28 cm
2×22 The diameter of a car wheel is 42 cm. The number

Also, in a quadrant = 90° of complete revolutions it will make in moving 132


km is
Area of quadrant = -Xy2 (A) 10+ (B) 10
3600
(C) 10 (D) 10
900 22 Ans. Option (B) is correct.
x 28 x 28
36007 Explanation: Diameter of wheel = 42 cm
= 616 cm? Radius of the wheel = = 21 cm
ie minute hand of a clock is 84 cm long. The 2
distance covered by the tip of minute hand from Distance in 1revolution = Circumference of the
10:10 am to 10:25 am is
wheel
(A) 44 cm
(B) 88 cm = 2r
() 132 cm (D) 176 cm 22
=2X x3
A|CBSE, Board Term-, 2021] 7
MATHEMATICA
Chapterwisc &Toplcwise,
238 Oswaal CBSE Question Bank
Explanatlon:: (AccorBTANDRO)
Crcumference of a dcircleingto , Clua,y
ralu<syuest on,
s 132 cm
132 kn 132
lotal distanco coveredby the wheel Let ' and ' be
ihe Pe
X 100000 cm 132(0XX000 Cm square. of
Number of revolutions 2r 4u cdrco
Total distance covered by wheel 22.
Distance covered in 1 revolution
-r 2u

13200000
100000 10 |r7a
132

Q.4. If the perimeter of a circle is half to that of a square, Area of circle,


area of
then the ratioof the area of the circle to the From eqtuation (), we have
the square is
(A) 22:7 (B) 11:7
(C) 7:11 (D) 7:22
Ans. Option (D) is correct. 22 49a
Explanation: Let radius of the circle be r cm and side 1\ 121
of the square is a Cm. 14a'
According to the question,perimeter of the circle is
half of perimeter of the square.
Area of square 4, =
2rur =(4a)
2 From equation (i) and (ii), we have
20 4,4
2r
1 A, >A,
or Area of circle is greater than
the
Q. 8. Area of the largest triangle that area of square.
asemi-circleof radius' units is:can be
Areaof the circle
Area of the square
insaribe
(A) sq. units (B) 1 Sq. units
11 7
=T X 0r 22 (C) 2* sq. units (D) V2 sq, units
Q.5. If the sum of the areas of two circles with radii R,
Ans. Option (A) is correct.
and R, is equal to the area of a circle of radius R, Explanation: Take a point C on circumiete
of the semi-circle and join it by the
then: the end poit :
diameter AB.
(A) R, + R, = R (B) RË +R; - R?
(C) R+ R, <R (D) R-R;= R?
Q.6. If the sum of the circumferences of two circles
with radii R,and R, is equal to the circumference
of a circe of radius R, then:
(A) R + R, = R
(B) R+ R>R
(C) R, + R, <R
(D) Nothing definite can be said about the relation
among Ry R, and R ZC=90° |Angle in a semi-circle is right a
Q.7.If the circumference of a circle and the perimeter So AABC =xABxCD
of a square are equal, then: 2
(A) Area of the circle = Area of the square
(B) Area of the cirde > Area of the square =x2rxr =tsq, units
(C) Area of the circle < Area of the square thatr
Q. 9. If the perimeter of a circle is equal to
(D) Nothing definite can be said about the relation Square,then the ratio of their areas is:
between the areas of the circle and square, R (A) 22:7 (B) 14:11
Ans. Option (B) is correct. (C) 7:22 (D) 11:14
These questions are for practice and their solutions are available at the end of thechapter
AREAS RELATED To CIRCLES 239
B)
iscorrect. whose
Letthe
radius of circle be ' and side Explanation:
diameter is 16 m,
Area of first circular park
ratio:
are
b ea question,
given
auare
circe =
gt o
Perimeter of square
of
2TT= 4a
epineter
=(8)
..) = 64n m?
Area of second circular park whose diarmeter is 12 m,
Area
ofcircle [From equation (i)] 2

Area
ofsquare

14
result as
equation(i), we get =36 n²
aispropOsed
saking
1to build a single circular park equal According to question,
sum of
areas of two circular parks of Area of single circular park
the
and 12 min alocality. The radius = Area of first circular park
to
anes
16 m
would be: + Area of second circular park
idameters
n
newpark
the
(B) 15 m TL= 64 + 36
A 10m (D) 24 m U ²=100
G 20
m correct r= 10 m
is
0otion(A)
SUBJECTIVE TYPE QUESTIONS
Type Then, the length of arc APB
Very Short Answer x 2r
Questions (1 mark each) 360°
diameter 42 cm, if an arc subtends
circle of
na 22 360
60
X2xx10.5
the centre, where n = , then
n angle of 60° at
= 11cm
of arc.
whatwill be the length Now, the perimeter of the sector OAPBO
AO U[CBSE SQr 2020-21] =OA + length of an arc APB + BO
tinthe given
figure, find the perimeter of thesector = (10.5 + 11 + 10.5) cm
60°
ta circle with radius 10.5 cm and of angle =32, cm.
Q.3. Find the area of the sector of a circle of radius
|Take = 6 cm whose central angle is 30°. (Take n = 3.14)
Sol.
AOU [CBSE OD Set-III, 2020]

B
A

60°
o30°
A 22 cmB
OUCBSEOD Set-I, 2020]
LWe have, radius (r) =10.5 cm Given, radius of a circle,
and OA = OB =6 cm
angle (0) = 60°
(Assuming in figure) ½
P and central angle = LAOB = 30°
By using formula,
10.5
cm area of the sector of a circle

60° 360°
30°
x3.14 x6×6
360
9.42 cm²
questions are for ppractice and
their are available at the end of the chapter
solutions
MATHEMATICS

Bank
Chapterwise &
Topicwise,
(STANDARD), Clas-X
2x 5.2x
240 Oswaal CBSE QUestion
3609
Short Answer Type
(2markseach)
Questions-I 6x360°
into the form of 2Tx 5.2
piece of wire22 cm long is bentangle of 60 at its
Q. 1.A subtending an Now, area of sector =
an arc of a circle
22 X
360°
thecircle. Use n =
centre. Findthe radius of 2020] 6x360
A{CBSEDelhi Set-I,
Sol. AB is an arc of a circle. hand
= 15.6 sq. 2x5.2x360
units.
0.3. The minute of a
Let radius of acircle be r cm. clock is 12 c
the area of the face of the
clock
minute hand in 35 minutes,

aAICBSE Delhi Set-u


Short Answer Type
600 Questions-II
(3 NarkS ene
A B 0.1. The area of a
Find the cost
circular
of fencing playground
this is
22 cm ground
at the
of acircular play ground = cSet1,
ABSE OD
50 per metre. rate
Sol. Area
i.e.,
AB = 22 cm
i.e., 22176
=22176 cm
and [where rr is the radius of a
Length of arc = 360°
2,r0 1
2= 22176 x 7 play grounA
2x22xrx600
22. =7056
r = 84 cm =
22 = 0.84 m
7×360° Cost of fencing this ground =50 x 2r
22xr =50 x 2x
22, = 7
21
22 × r = 22 x 21
=7264.
1
Q. 2. Sides of aright triangular field aree25 m, 24
r= 21
1 7m. At the three corners of the field. mand
Hence, The radius of the circle (r) is 21 cm. buffalo and a horse are tied separately with mn
radius 5.2 cm of 3.5 m each to graze in the field. Find the atea
Q.2. The perimeter of a sector of a circle of the field that cannot be grazed by these animals
is 16.4 cm. Find the area of the sector.
ACBSE Delhi Set-II, 2020] A (CBSE SQP 20
Sol. Given, radius of circle (r) = 5.2 cm Sol. Required Area = Area of triangle
-Area of 3sectos
i.e., OA = 0B = r=5.2 cm
1
and the perimeter of a sector = 16.4 cm Area of Triangle = x 24 × 7 = 84 m 1
As we krnow that perimeter of the sector
2Ttr0 Area of three sectors =
= 2r + 360°
360°
2x5.2x x (Sum of three angles of triangk
16.4 = 2x 5.2 + 22 x7x7x 180°
360°
7x2x2x 360°
77 or 19.25 n
4
Required Area = (84- 19.25) m
/5
cm2 = 64.75 m
[CBSE SOP Marking Scheme, 202
Detaiied Solution:
Given that, a triangular field with the three corn
a horse are tid
A B of the field a cow, a buffalo and
separately with ropes.
These questions are for practice and their solutions are available at the end of the chapter
APEAS feLATED TO CIpCLESN
ania) grazed the field in each corner of
scctorial form. So, arca of thefield whiciu cannot he gyazeo
as a cach
r field
C

radius of sector by three anirmals Area of AI!


that Area f three seciors
,rn C(Cow)
Ne (BA 1925) rn
f4.75 r
3 in Figure, a nquare OABC is inscribed in a quadrant
OPBQ. If OA 15 cm, findthe area of the shaded
region. (Use n 3.14)
B

-18 m-5m 1 5
c m

ACRSE OD Set-J, 2019}


-3.5 m
3.5 m
Sol. Since OABC is a
square.

(Buffalo)B 7m
r
3.5 m>-3.5 m’H (Horse) 15 cm
(r) =3.5 m
1 5
c m

ZC =
Area of sector 360°

3600n(3.5)²
7 m²
Then, ZOAB = 90
ZB .. In AOAB,
ZB=
Area of sector 360° Radius of quadrant = OB = V15+ 152
ZB
360 (3.5) m? (By Pythagoras theoremn)
OB = 15/2 cm
ZH Now, area of quadrant OQBP
Area of sector ZH = 360°-T2
ZH 360
n(3.5) m2
360° 90°
x 3.14 x (OB
Sum of the areas of three sectors 360
ZB 1
T(3.5) + 360° n(3.5)° x 3.14 x (15/2
360° 4
ZH =353.25 cm?
360°
n(3.5)2 Area of square = (0A) = (15
= 225 cm?
2C+ B+ ZHY
-( 360°
Hence,
area of shaded region = area of quadrant
22
XX3.5 ×3.5 - area of square
7 =(353.25-225) cm²
= 128.25 cm
f: ZO+B +ZH= 180°]
180°
x 38.5
360° Commonly MadeError
= 19.25 m?
Since ACBH is a right angled Many candidates use incorrect formula for
triangle, finding the area of shaded region.
then area of ACBH = 2CBx 1 BH
AnsweringTip
x 24 x 7
2 Remember all the formulae related to square
=84 m2 and quadrant of the circle.
MATHKMATICA
Chaptewine Topewinc, (STANDARD), Clana X
Queatíon tank
ae 4cnm atd
long hants of aclock
Q4 1he ahort atd Nnd the sun of
distances
6m log vespectvcly
travelled by their ts in 48 hours.
20151
|FovignSetLL, L
cicle of radius 28 em and Jem
Q5ARIN isa qtadrant of a BC as diamcter.
drawn with
a senntine BEC is
n
Nind the area of theshaded reglon. Use A|CBSE OD Set
(I\, 20971
Topper Answer, 2017
shtdd. regton Aea of miccle
4
ArtaoT michhele unln d s,

A|CBSESQP 2017]
will be
Sol. As ABC is a quadrant of the drecde, BAC
measu ,
In AABC, B= AC+ AB²
= (28)+ (28)
=2(28)2
BC = 38V2 m 2.2.5

V
Radius of semi-cicle drawn on BC = 2

=142 cm 2415

Area of semi-circle= x (14 V2}² 12.38 Cm.


anta of shaded teqion is \2.315 (m
= 616 cm²
3
Area of AABC = x 28 x 28
Q.7. In the given figure, two concentric circles with
=392 cm? have radii 21 cn and 42 cm. If ZAOB = 60,
centre O
22
Area of of quadrant =
1
xx x 28 x 28 find the area of the shaded region. Use =

= 616 cm
Area of the shaded region
=Area ofsemi-circie +Areaof&
-Area of quadrant
616 392-616
=392 cm?
60°
[CBSE Marking Scheme, 2017| D

0.6. Three senicircles each of iameter are 3 Cm, a


circle of diameter 4.5 cm and a semicircle of radius B
4.5 cm are drawn in the givern figure. Find thearea
of the shaded region.
These questions are forpractie and thetr solutions are gtilable at the end of the chapter
TOPper Answer, a017 AREAS RELATED TO CIRCLES
Q.9.Iancircle
the given 243
OB and lgure, AOB a sector of angle 60 of
with centre andis radius
AP =15 cm, find the 17 cm. If AP
region. atea of the shaded

R: 42cn,t1 21¢m

(s6p-co)
nodedvyion

Sol. As OA = 17 AJCBSE S.A.2, 2016]


cm, AP = 15 cm and
triangle.
. Using
AOPA is right
( u2-21)(42 42)
Pythagoras theorem in AOPA,
OP = OA- Ap?
= V(17 -(15)2
Area of the shadedOP =8 cm 1

t shaded gion is 346m? region


= Area of the sector AOBA
showstwo arcs PAQ and - Area of AOPA 1
Fgure PBQ.
1partofcircle with centre O and radiusArcOP whileis
PAQ 600
3608X -xbxh
; semi-circle drawn on PQ as
PBQissa 2
= PQ=10 cm, show diameter
rc
Nithcentre .If OP
M. 60°
that area 22

ofshaded region is
360: x17x17-x8x15
= 151.38-60
-91.38 cm? 1
P Q.10. In fig., APB and AOP are semi-circles, and AO = 0OB.
If the perimeter of the figure is
47 cm, find the area
10cm

5 cm of the shaded region. Use =


M B ACBSE Delhi Set-1, II, III, 2015]
5 cm

ACBSE Delhi Set I, Ii, III, 2016]


LGiven,
OP = 0Q = PQ = 10 cm
and ZPOQ = 60
Area of segment P
PAQMP
100 100v3) Sol. Let' be the radius of given circle.
6 4
cm. 1
Perimeter of given figure 47 cm
Area of Perimeter of fullcircle -perineter of
semicircle = 252 Cm2 circle
= 47-2r
Area of shaded
region = 2572 50
3
Or,
2sur-(2r) =47-2.
3r+2r = 47 cm
1
2
Hence Proved
CBSE Marking Scheme, 2016]
244Osaal (RSE Question Barnk Chapterwise &Topicwise, MATHEMATICS (STANDARD), Clasg-X

7
Radius -3.5 m
47 Cn
The width of path = 0.7 m
Cm .Radius of park with path
47
= 3.5 +
1 0.7 =
Area of the path =
(4.2) - 42 m
Now, area of shaded rgion (3.5

B7-x5.643-912 5)
1
-area of circle
A= area of circde 7

22
area of circie =22 x
= 16.94 m²
0.77
4 Cost of the cementing the path
= 16.94X 110
3,2x7x7 m
4 7
=1863.40
ICBSE
Q. 2. The given fig. shows a Marking
sector OAP of aA 20)Scheme,
= 115.5 cm 1 centre 0, containing 20. AB is ircle with
(CBSE Marking Scheme, 2015)
to the radius OA and meets OP
Prove that the perimeter of
tan 9 + sec 0+
perpendiculat
shadedproduced
at B.
region is
Long Answer Type 180°
Questions (5 marks each)
Q. 1.A park is of the shape of a circle of diameter 7 m. B
It is surrounded by a path of width of 0.7 m. Find
the expenditure of cementing the path. If its cost is
110 per sq. m. CA Foreign Set-I, II, III, 2017]
Sol.

A
t-7M

Given, the diameter of park =7m rCAICBSE OD Set-I,II, III, 2015, 16]

Topper Answer, 2016


Sol.

Puimsa fuhachd qin


AkLAH LAE) TO
stan9_ CRCLEA245

BP

AP 360

5
beads o Gferent sizes are Joined
such that the distance between their Or,
130r n(r+r
14 cmn. The sum of their areas is r+ r
together
130 ...(i) 1
is
entres Findthe radius each bead. (r t r) +r +2ra
130n
cm2
CAICBSE Term-2, 2015| Or,
(14) 130 + 2r,
Or,
cm, 2rr, 196-- 130
the circles are r cm and r,
theradi of
let
66
(ri-r=r+r-2r
130 -66
Oeicm T Cm
64
Or, 1 - , =8 .(iii) 1
From (i) and (iii), 2r, = 22
Or, r 11 cm
I;tn = 14 (Given) ..) and r; 14- 11
and sum of their areas = tri+ nr
130r = n(r+ r,)
=3cmn. 1
(Given) 1 [CBSE Marking Scheme, 2015}

cOMPETENCY BASEDQUESTIONs 4marks each)


Q.1. What is the area of the grass field?
O Case based MCOs (A) 225 m? (B) 225 m
LRead the following and answer any four questions (C) 255 m² (D) 15 m
from Q.1. to Q.5. Ans. Option (A) is correct.
Ahorse is tied to a peg at one Corner of a square
staped grass field of sides 15 m by means of a 5 m Explanation: Area of square = (sidey?
= 15 X 15
Iing rope (see the given figure). AOC+AE
= 225 m²

Q. 2. The area of that part of the field in which the horse


can graze.
(A). 19.625 m? (B) 19.265 m²
(C) 19 m? (D)78.5 n?
Ans. Option (A)is correct.
Explanation: From the figure, it can be observe that
the horse can graze a sector of 90° in a circle of 5 m
radius.
Area that can be grazed by horse
= Area of sector
246 Owaat CHSE Queation Bank Chapterwise &Topicwine, MATHEMATIC8 (8TANDARD), Clasn-X

9
-X

aX3.14 x 5 x 5

19.625m?
were 10 m long instead
Q.3. The grazing area if the rope
of 5 m.
(A) 7.85 m² (B) 785 m?
(C) 225 n² (D) 78.5 nm? A

Ans. Option (D) iscorrect.


Explanation: Area that can be grazed by the horse
when length of rope is 10 mlong
90° X 2
3600

x3.14 x 10x 10
4

= 78.5 mn Design I: This design is made


withaa
Q.4. The increase in the grazing area if the rope were 32 cm leaving equilateralt
triangle circle radius
ABC intheof
as shown in the given figure.
10 m long instead of 5 m.
(A) 58.758 m² (B) 58.875 m² Design II: This Pookalam is
middle
made with 99
(C) 58 m? (D) 78.5 m? design each of radius 7 cm. drqular
Refer Design I:
Ans. Option (B) is correct.
0.1. The side of equilateral triangle is
Explanation: Increase in grazing area
(A) 12/3 cm (B) 32/3 cm
= (78.519.625) m
= 58.875 m² (C) 48 cm (D) 64 cnm
Ans. Option (B) is correct.
Q.5. The given problem is based on which concept?
0. 2. The altitude of the equilateral triangle is
(A) Coordinate geometry
(A) 8cm (B) 12cm
(B) Area related to circles
(C) 48 cm (D) 52 cm
(C) Circe
Ans. Option (C) is correct.
(D) None of these
Refer Design I:
Ans. Option (B) is correct. 0.3. The area of square is
II. Read the following and answer any four questions (A) 1264 cm² (B) 1764 cm?
from Q.1. to Q.5. (C) 1830 cm2 (D) 1944 cm?
AREAS RELATED TO CIRCLES Ans. Option (B) is correct.
Pookalam is the flower bed or flower
patterm Explanation: radius =7 cm
designed during Onam in Kerala. It is similar as
diameter = 2 X 7 cm
Rangoli in North India and Kolam in Tamil Nadu.
= 14 cm
During the festival of Onam, your school is
side of square = 14 cm + 14 cm+ 14 cm
planning toconduct a Pookalam
friend who is a partner in competition. Your = 42 cm.
two designs given below. competition, suggests
(CBSE QB 2021] Area of square = side
Observe these carefully. = (42 cm)
= 1764 cm²
each circular
design is AREAS RELATED TO CIRCLES 247

A)
124
cm2
of (B) 132 cm²

O 144
cm?
(D)154 Cm²
correct,
Option(D)is radius =7 cm
Eplanation:
Aas
each circular design
Areaof

22
*7x7 Design A: BroochA is made with silver wire in the
form of a circle with diameter 28 mm. The wire used
= 154 cm2
for making4 diameters which divide the circle into
theremaining portion ofthe square ABCD 8 equal parts.
05.Areaof
Design B: Brooch b is made two colours i.e. Gold
is
(A)378cm2 (B) 260 cm? and silver. Outer part is made with Gold. The
(C) 340cm2 (D) 278 cm? circumference of silver part is 44 mm and the gold
part is 3 mm wide everywhere.
correct.
Option(A) is Refer to Design A
Ans. Erplanation: Area of 9 circular design Q.1. Find the total length of silver wire.
=9x 2 Ans. Diameter = 28 mm
22 radius = 14 mm
9xx7x7 Total length of wire = length of 4 diameter
+ circumference of circle.
= 1386 cm?
= 4 x 28 + 2r
m2
Area of square = 1764
ABCD = 112+2xx14
Area of remaining portion of square
= Area of square - Area of 9 = 112 + 88

circular design = 200 mm


= 1764 cm²-1386 cm? Q.2. Find the area of each sector of the brooch.
=378 cm2 Ans. Area of each sector of Brooch
1
-x Area of Brooch
Case based Subjective
Ouestions
8
LRead the following and answer any four questions
from Q.1. to Q.5.
A Brooch
xx14x14
8
=77 mm?
A brooch is a small piece of jewellery which has
a pin at the back so it can be fastened on a dress, Refer to Design B
blouse or coat. 0.3. Find the circumference of outer part (golden).
Designs of some brooch are shown below. Observe Ans. : circumference of silver part = 44 mm
them carefully. [CBSE QB 2021] i.e., 2r = 44 mm

2x Xr = 44
7

. Radius of golden part (R) = (7+ 3) = 10 mm


Now, circumference of golden part = 2rR
22
=2X

= 62.86 mm.
MATHEMATICS
248 Oswaal CBSE: Questíon Bank Chapterwisc & Topicwisc,
r7mm.
(STANDARD), Class-X
areas of golden and silver
Q.4. Find the ditference of radius of whole Brooch
parts. = 7mmn +3 mn
Ans. Area of golden part = nR² = 10 nm.
= n(10)
100x mm? Circumference of outer edge
= 2rr
Area of silver part = n
n()= 49r mm²

.:. Difference of areas of golden and silver parts


=

2xx10
= 100n -49r 440
mm
= 51 n mm². 7
brooch B. He makes let the number of
Q.5. A boy is playing with revolutions =n
many
revolution with it along its edge. How Now, According to question,
take to cover 80
complete revolutions must it n.2r = 80n
mm? 440
1. 80
Ans. Circumference of silver part of Brooch 7
= 44 Cm 440
n. 80x 2
2r = 44 mm 7 7
2x = 44 n= 4.

Solutions for Practice Questions

Multiple Choice Questions


Ans.5. Option (B) iscorrect.
Explanation: According to the given condition,
Area of circle =Area of first circle
+ Area of second circle
60°
nR2= nR,2 +R,?
R²= R?+R,? B

Ans.6. Option (A) is correct.


Explanation: According to question, .. The length of arc, l = -x2Tr
Circumference of circle 360°
= Circumference of first circle 60° 22
+ Circumference of second circle
360X2xX21
7
2rR = 2R, + 2rR, = 22 cm
R=R, + R, Commonly Made Error
Very Short Answer Type Question
Some students used incorrect formula for
Ans.l. Length of arc = 360(2r) length of arc and some made mistakes in
calculation.
60°
Answering Tip
= 22 cm Remember the formula of length of arc and
[CBSE Marking Scheme, 2020-21] the concept of angle subtended at the centre.
iDetailed Solvtion:
Given, diarmeter of the circle = 42 cm Short Answer Type Question-l 360°
42 = 6° ½!
Then, radius of the circle = Cm = 21 cm. Ans.3, Angle subtended in 1 minute = 60°
2
.. Angle subtended in 35 minutes=6° x 35
and angle subtended at the centre, = 210°
=60
AREAS RELATED T0 CIRCLES 249

Area of the face of the dock described by the Total distance traveled =(32a + 576z) m
rünutehar:d in 35 minutes = 608, Cm

= Area of the sector (210) [CBSE Marking Scheme, 2018]


Detailed Solution:
36 Short harnd makes 4 rounds in 48 hours
Long hand makes 4S rounds in 48 hours
=
22 12x12x 210 Radius of the circde formed by short hand =4m
360
and radius ofthe irde formed by long hand=6m
663280 Distance travelled by short hand in one round
=ircumference of the crde
2520
=2%4 x=8% m
= 264 cm rounds
Distance travelled by short hand in 4
=2x 4 X 4z =32: cm 4
iShort Answer Type Question-l! round
hours Distance travelled by long hand in one
Ans4 Distance travelled by short hand in 48 =2X * X6= 12z cm
1 rounds
=4x 2 X4 Cm=32z m. Distance travelled by iong hand in 48
Distance travelled by long hand in 48 hours = 48 x 12 = 376z m 4
608: amk
Sum of the distances = 32 + 576x=
1
= 4S x 2r X 6m =576z Cm

REFLECTIONS

cirde?
Are you able to calculate the area of a segment of a
L arc?
formulae related to length of an
2 Did you remember the major and minor segments of the
cirde? Give examples.
area of
3. How will you alculate tBhe
CHAPTER
SURFACE
,AREAS

12
Sylabus
and
cubes,cuboids,
Surface
rolumes
AND VOLUMES

of
combinations of any two0 of the
areas spheres, hemispheresandright circularrcylinderslcones.
fol owing
chapteryouwillstudy volume and surface area of solid figures.
this concepts relatedtothe
alimortant
AZfalae and soid fgures.
problems reiated to the life.
sahe the complexproblems in day to day
sirple and
Apoyforruas ir

Revision Notes
A Cone is a three dimensional geometric shan
spherr is a perfecty round geometrical object in tapers smoothly from a flat
A Scan to know
three dimensionalspace. base to a point called the apex more sbo
or vertex. this togc

Scan to know
ore about
h Combining and Car
this topic version ofSoliós

A hemisphere is half of a
sphere.
Surfece areas and
object
Acylinder is a solid or a hollowcircular
volumes

that has a circular base and a


top of the same size.
SURFACE AREAS AND VOLUMES 251

cm hemisphere
glass. 25
the capacity
cm cylindrical a
of cm =;,ifr
32.71
=
cm
5
diamneter of 98,125-32.71
= Height
Exanple 5cm rh 5
capacity Apparent
glass X Volume Cm
65.42
-’ of
4~5
cm 2.5
ylindrical
Inner = hemisphere
capacity
glaSs
X
Apparent cm
2.5 a98.125
cm -
theX
(2.5) Actual
capacity
Given:
Actual of 3.14 x
:
Solution of 3.14
Volume
Latetal
nurtace
ren
Find: x
b)
+ CSA2rh
= 2r(R+r)h
CSA
=rl CSA
=
4
2h(l CSA
=
2
LSA
=
4d
=
LSA
Combination
R-) Solids
of

hl) 4+
otaI
suface
Ara r)h
+ r) ) jolume,
and
Surfa
bh + + + Areas
2rh
2r(R
Tl Formalas
+
2(lb TSA
=
4 3TSA
=
TSA
=
6a = =
= = TSA TSA
TSATSA

V=
n(R-h
V=lx
bx
h
Volume
V=-h
V=
uh
3V=
3 4
V=
V=3
Surface
Area
Hollow
cylinder
(R>)

Nate
of
solici

Hemisphere
Cylinder Sphere Level
Third
Cuboid Valume
Cube Cone

Sum
of
surface
faces
solid
of Level
areas
of
the by
hollow/closed
a
enclosed Second
Quantity
of
solid
space
Level
3-D
Fist
MATHEMATICS
Question Rank Chapterwisc
& Topicwise, (STANDARD), Class-X
Oswnai CBSE Right Circular Hollow Cylinder:
Total surface area =(External
O Key Words surface area) + (Area of brinm)surface area +
Surface area: The anount
ing the outside ot a
three
of space cover
dimensional shape.
occupied by a
=(2rRh +
=(2rd:(R+ ) 2ri) + \nterTa
:Volurne: The amount ofspace +
three dimensionalobject.
of sub
=|2r(Rt
Curved surface area =(2rRl 4
h+ R-nI 2n(R-
Materials: A substance or mixture
stancs that constitutes an

Key Formulae
object.
2rrh)=2rhR
Cuboid: walls
Lateralsurface area or area of four
2(l + Wh R
Total surface area = 2(lb + bh + hl)
Volume= | xþ x
Diagonal= '+b +h Volume of the material used
= (External volume) - (Internal volume)
= IR1 - rh=Th(R?
= tRh-Trh= TI(R-)
height of the
Here, I is length, b is breadth and h is Here, Rand r arethe external and internal
cuboid.
Cube:
Lateral surface area or area of four walls
his the height of the right circular
Right Circular Cone: hollow cylradüinder.and
=4 X a
Total surface area = 6X a Slant height,
Volume = Area of curved surface = rl
DiagonaB of a cube = /3 xa

Here, a is edge of cube.


Right Circular Cylinder:
Area of base or top face = Ir²
Areaof curved srlace or curved surface area
= perimeter of the base x height Total surface area = Area of curved surtae + Area o
= 2rh
base
Total Surface area (incduding both ends)
= 2rrh + 2rr² = 2rtr(h + r)
Volume = (Area of the base x heiglht)= rh
1
Volume
3
vertical height ar
Here, r, h and Iare the radius,
circular cune
slant height respectively of the right
Sphere:
Surface area = 4nr

Here, r is the radius of base and is the height of the Volume


3
right circular cylinder
SURFACE AREAS AND VOLUMES 253
Here, K andF äre he external ana intersal tadi öf.
the spherical shel.
Hemisphere:
Area of curved surface = 2rr2
Total surface area= Area of curved surface + Area:
of base

the radius of the sphere.


Here,r is =3rr2
P SphericalShell:
(outer) = 4nR2 Volume = n
Surface area
4 TRI_
material =
Volumeof 3 3

n(R°-)
3

R
Here, r is the radius of the hemisphere.

F Fundamental Facts
regular polyhedra. Five such solids are : dodecahedron,
called the regular solids or
The platonic solids also tetrahedron. earth, the icosahedron
icosahedron, octahedron
and
'element' fire, the cube with
equated tetrahedron with the the constellations and
heavens
Greek mathematician Plato dodecahedron with the stuff of which
with air and
with water, the octahedron
were mnade. Oxford.
platonic solids are kept in Ashmolean Museumn in cylinder circumscribing it.
Thestone of a sculpture consisting of a sphere and
Archimedes carried
The tomb of

(1mark each)
oBJECTIVE TYPE OUESTIONS
= 10m

Multiple Choice Questions Area of circle =


=1 (10) = 100 rm?
it by the lamp
Q.1. Savita has a lamp placed at the center of her square Hence, the area of the yard is not
yard, each side measuring 20 m. The light of
lamp = Area of square yard cincle
covers a circle of radius 10 m on yard. What area of - Area of

the yard is NOT lit by the lamp? (400 -100x) n


(A) 400t Sq. m (B) 100Tt sq. m water is emptied through
Q2A vessel having 30 m² ofand
(C) (40- 10r) sq. m (D) (400- 1007) sq. m the other large. Water
twoopenings, one small opening at the rake
AO CBSE, Q.B., 2021-2022} flows out through the smaller one at the rate V
Ans. Option (D) is correct. of U mh and through the larger vessel
Explanation: m°h. Given that 3U 2V = 70 and that the
what is V?
Each side of Savita's square yard = 20 m gets fully emptied in 1hour,
(B) 20 m'h
Area of square yard = (side)2 (A) 10mh
= (20) = 400 m? (C) 30 mh (D) 50 nmh
Radius of circle, where light of lamp covers aO BCESE, Q.B, A2-282
Ipiwe, MATHEMATICA
waal SS uext Rank hatewise &
QAAPlumbline (Sahul) in
t he
Ans Oion (8) is ont. (A) a0e and a evlser
(1) a hemishee and a cone
()
and
combnatlon
a
o
Nate ofsnalkr wing Unt' h
(D) Nyhere and cyltnder cylxder
Sniar in 1hour, water disland by larger
.i)
Ihe n .
.)
Nultilying by in v, (0 and sutbtracting with q
().

3U +V 70
()

Hence V'is 20 m'h. Ans. Option (B) is correet.


Q.3, The radius ofa sphere (in c) whose volume is 12n Eylanation: lumbline is an
cm, is instrument
check the verticality of an object. It is
a

gilli-danda cgame
ombination
(B) 3 V3 ot ahemisphere and a cone,
(A) 3
Q.7. The shape of agilli, inthe
() 323 (D) 3 figure) is a combination of: (see in
ACBSE Delti & 0D, 2021)
Ans.Option (C) iscorrect.
Topper Answer, 2020

(A) two cvlinders


(B) a cone and a cylinder
(C) two conesand a cylinder
(D) two cylinders and a cone
033A hollow cube of internal edge 22 cm is Alled
with spherical narbles of diameter 0.5 cm and
1
3 cnm it is assumed that space of the cube remains v
rAdi
unfilled. Then the number of marbles that the volu
cube can accommodate is: ol. Let
(A) 142296 (B) 142396 cOne

(C) 142496 (D) 142596


Q.4. Acylindrical pencil sharpened at one edgeis the Ans.Option (A) is correct.
combination of: Explanation: Let the spherical nnarble has radius r.
(A) acone and a cylinder Diameter of the nmarble = 0.5 cm
(B) frustum of a cone and a cylinder 0.5 Cm= 0.25 cm
Give
(C) a hemisphere anda cylinder
(D) two ylinders
Q. 5.Asurahi is the combination of: Length of side of cube l 22 cm
(A) a sphere and a cylinder Let n marbles can fill the cube.
(B) a hemisphere and a cylinder . Volume of nmarbles =(1-)partof
(C) two hemispheres
(D) acylinder and a cone volume of cube
Ans. Option (A) is correct.
4
Explanation: A
surahi is the combination of a sphere
and a cylinder.
O These (uestions are for practice and their solutions aTe atailable at the end of the chupter
Henc
SURFACE AREAS AND VOLUMES255
7 3
7x3x22x22x22x7
22
8x4x22X0.25x0.25x0.25
x2x7
n=7X3X22 x 22
X7
=42x 484
n=142296
accommodate142296 marbles. 22 25 25 11 121
ubecan -=03601
meedicine-capsuleis in the shape of acylinder
S0,Ct 7 100 100 6 336
cm with two hemispheres stuck :. Volume of capsule = 0.3601 cm = 0.36 cm'.
44
diameter 0.5 lengthofentire capsuleis
to,itsends. The is: Q. 10. If tWo solid hemispheres of same base radius
ofthecapsule
each
to
Thecapacity
2ct1.0.36cm3
4)
(B) 0.35 cm are joined together along their bases, then curved
(D)0.33cm² A surface area of this new solid is :
()0.34cm? (4) 47
correct. (B) 6r
Option(A)is of2henmispheresand () 3rn (D) 8 A
Elanation:Capsuleconsists Ans. Option (A) is correct.
acylinder. -2cm4
0.5 cm = 0.25 cm +05m Explanation: When two hemispheres of equal radi
2
are joined base to base,new solid becomes sphere
and curved surface area of sphere is 4rr.
Q. 11. Aight circular cylinder of radius r cm and height
r= 0.25cm
capsule =r+h+r
Totallength of 2= 2r + h hcm (where h > 2) just encloses of sphere of
2=2x0.25 + h diameter :
h=2-0.5 = 1.5cm (A) r cm (B) 2r cm
hemispheres (C hcm (D) 2h cm
Volumeoftwo
Volume ofcapsule = + Volume of cylinder Ans. Option (B) is correct.
Explanation: As the cylinder just endloses theare
sphere so the cylinder and diameter of sphere
equal, ie., 2r and height h = 2r.

SUBJECTIVE TYPE QUESTIONS


volumes in the ratio 1: 27.
Q:2. wo cubes have their
Find the ratio of their surface areas.
Very Short Answer Type R[CBSE OD Compt. Set-l, Il, IIL,
2013]
(1 mark each)
Questions and A, then
in the ratio 1:3 and Sol.Let the sides of twocubes be a
01)Two cones have their heights 1
: 1.What is the ratio of their
radii in the ratio 3ADACBSE
volumes ?
Delhi Set-I, 2020]
and y t, be radii of two
Sol. Let h, and h, be height volumes
Cones, then ratio of their
A 3
area of acube is 6(side)
3 We know that the surface
1

h 1 3
:Raio of surface areas = 6A? -6
{CBSE Marking Schemte, 201SI
and
Given, 1 what is the
Q.3. If the volume of a cube is 8 cm, then
ADO((O.EB)
-Ale) surface area of a cube.

Short Answer ype


-6e) Questions-I
(8 marks each)
cylinder with its
Q1. Thevolume of a dght circular 1
25 - cm. ind the
Hence, ratio of their volumes is 3:1. height equal to the radius is
Chapterwise & Topicwise, MATHEMATICS (STANDARD), Class-X
256 OswaalCRSE Question Bank
Hence, the ratio of the
22 radius and the
height of the cylinder Usen=
ADNCBSE OD Set-1, 2020|
0.3. From asolid right circular height
and base radius 6cm, a
rightcylinder of
Sol. Given, height and same base
of the remaining solid.
circular
removed. heig
cone ht A
Find the sasne.
oi
Volume of a right circular cylinder = 25- cm
ADACBSE OD vo24lume
Height (h) =14 cm
176
7
Sol. Given,
and Base radius (r) =6 cm
Volume of the remaining solid
Set-1l ,
wher h is height and r is radius then = Volume
22 x xh= 176 cylinder -of a right
7
circular Vol
coneume ofciracular
176 =8= 23 =uth-uhh- 1
22 3

Hence, height of the cylinder = 2 cm 2

Q.2. Asolid is in the shape of a cone mounted on a


hemisphere of same base radius. If the curved ( 2 22
surface areas of the hemispherical part and the 3 7 x6X6 x14
conical part are equal, then find the ratio of the =1056 cm3
radius and the height of the conical part.
ADAICBSE O Set-II, 2020] Q3sha is 10 years old girl. On the; result
Sol. Let ABCbe a cone, which is mounted on a hemi hor father Suresh were very happy as day, she Isha
got fand
sphere. position in the class. While coming back
Given: OC= OD =rcm home, Isha asked for a treat from her to their
Curved surface area of the hemispherical part reward for her success. They went father as a
1 and asked for two glasses of juice. to ajuice shop
(4nr) Aisha, a juice seller, was serving juice to hes
customers in two type of glasses. Both the glace
had inner radius 3 cm. The height of both the
glasses was 10 cm.
First type: A Glass with hemispherical raised
bottomn.

r Cm

T Cm

D
Second type: A glass with conical raised bottom of
Slant height of a cone,
height 1.5 cm.
Isha insisted to have the juice in first type of glass
So, curved surface area of cone and her father decided to have the juice in second
type of glass. Out of the two, Isha or her father
Suresh, who got more quantity of juice to drink
ADyCBSE SQP 2024]
and by how much?
[Given
2r = h?+r2 Sol. Capacity of first glass = H
Squaring both the sides, we get =1 X 9(10-2) 1
4 = + =721 Cm³
42-=
32 = Capacity of second glass= rH- ;
IX3x 3(10 - 0.5)
3 85.5x cm
more quantity of juict.
1
" Suresh got 42.39 cm³ MarkiugSchemne, 2.!:4
{CBSE SOP
SURFACE AREAS AND VOLUMES

olution
type glass,
we have = 4.5n m3
first Now, capacity of Juice in second glass
»of( ) 3 Cm
radius 10cCm
= in
Heght
(H)
cylindricalpart 3
= NPH n(3) × 10 - 4.57
10
=1X (3) x
= 85.5 x 7
= 268.71 cm³ (Approx.)
3 Cm Since, Suresh used second glass for drinking juice,
cm
10 so he got more quantity of juice.
1.e., Suresh got 268.47 - 226.08 = 42.39 cm more
juice than Isha.
Q. 5. A right circular cylinder and a cone have equal
bases and equal heights. If their curved surface
areas are in the ratio 8:5. show that the ratio
between radius of their bases to their height is 3 : 4.
hemispherical part U{CBSEComptt. OD Set-l, II, III, 2018j
Capacityof
=
2, Sol. Let r be the radii of bases of cylinder and cone and
h be the height
Slant height of cone =
3
nx(3) 2rh 8
5

4
5

16
2+h? 25
3 Cm 25h = 16² + 16H
92 = 16

: Capacity of Juice in first


glass
16
= IX(3)x10. 3
4
-nX(3°[10 -x3] ¿CBSE Niarking Shene, C18]
=9r X 8 Detailed Solution:

3cm

= 72r cm
= 72% 22 = 1584 Let r be the radii of bases of cylinder and cone and h
7 7 be the height, then Curved surface area of cylinder
-
In case 226.28
of i
cm (Approx.) and c°rved surface area of a cone
second
tadius of base (r) =3glass, we have
am
teight of bottom part (h) = 15cm
:Capacity of conical part where,
Given,
2arh
1
7XRX(3) ×1.5
258 Ostan! CBSE Qrestion Bank Chapterwise &Topiwise, MATHEMATIC8 (8TANDARD), Clnsa-X

Short AnswerType
VA.2 Questions-II
0. 1. A solid is in the (3 maks eaeh)
hemispherical end. form
The of a
25
is 20 cm and the diameter totalof heightcyliofnder w\th
-16 16r tind the total volume of the the cylindertheis 1sol d
solid. (|Use x 2cm.
Sol. Height of cylinder=
ACBSE ODOSet-, 2019
3
20-7s13cm.
.. The ratio eween radius of their bases to their
heights is 3:4. Hence Proved. 3.5

Commonly Made Errors 20 cm 13 cm

In such type of problems, mostly students write


incorret Yormulae of surface area of cylinder i3.5
and cone and also they do errors in calculation.
Students write the formula of cylinder in place
of cune and viceversa.

Answering Tip
Total volume
-x13+ 1

22 49
Adequate practice and remembering of
formulae is necessary. 77× 53
6
Q.6. A cylindrical glass tube with radius 10 cm has
water upto a height of 9 cm. A metal cube of 8cm 680.17 cm (Approx.)
edge is immersed completel. By how much the
water level will rise in the glass tube ?
[CBSE MfarkingSciheme, 2019)
Detailed Solution:
OACBSE Term-li, 2015]
Q.7. Asphere of diameter 6 cm is dropped in a right Height of the cylinder (h) = (20-7) cm = 13 cm
circular cylindrical vessel partly filled with water. 7
The diameter of the cylindrical vessel is 12 cm. Radius of circular part (r) = Cm
if the sphere is completely submerged in water,
by how much will the level of water rise in the Volume of solid= Volume of
ylindical vessel ? AICBSE SOP 2016} cylinder
+ 2 X Volume of
Sol Diameter of sphere = 6 cm
hemisphere
Diatneter of cylindrical vessel = 12 cm
Volume of sphere V=h + 2x
3
4
xrX3x3 x3
=36t cm
Volume of sphere = Increase in volunme of
cylinder
=

. Leve! of water ise in vessel 1cm = 680.17 cm (Approx.)


/C85E MMarkiny 5cihene, 2016]
GThse qustions Sre for practice and their solutions are available at the end of the chapter
ylindrical
tank of radius 40 cm is filled upto SUREACE AREAS AND VOLUMES 59
other
cylindrical pipe with the 2.A heap of rice is in the form of a cone of base
byan diameter 24 m and heicht 3.5 m. Findthe volue
m 1
Jeht3.15
km/hi n hour. Calculate the diameter of the rice. How much canvas cloth is required to
2
? just cover the heap ? ACBSE Deihi/OD 0181
pipe
tindrical Sol. Radius of conical heap 12
sa G+A[CBSE Delhi Region, 2019) m
[CBSE LDelhi Set-1, II, III, 2015] Volunne of rice x 12 x 12 x 3.5m

Topper Answer, 2019 = 528 m

Area of canvas cloth required = rl


!= J122 +(3.5)² = 12.5 m
saliaylinoluital
22
i.Area of canvas required *x 12 x12.5

= 471,42 m²

[CBSE Marking Scheme, 2018]1


Detailed Solution:

|yeal Topper Answer, 2018

Corical heap of nei

152 4

Tae voke of he iie hog i

03A wooden article was made by scooping out a


hemisphere from each end of a solid cylinder, as
Shown in fig. If the height of the cylinder is 10 cm
and its base is of radius 3.5 cm. Find the total sur
face area of the article.

ThE +CBSEDelhi/OD 2018]


Qstions are for practice and their solutions are available at the end of the chapter
Oswaal CHSE Question Bank Chapterwise %Topicwine, MATHEMATIC#(8TANDARD), Cla*,-%

CommonyMade Error
Someimes the students find TS.A. of tUhe canvas
1*45x3/2A
=20r crn.
Internal radius of cylindrical
in place of CSA. vesAelbeh 19
Let the heizht of emptied water
. Volune of water in cylinder
3
Answering Tip
They should have clear jdea about C.S.A. and
7xYolume of cone
3
h
TSA.and volume,
.5)A toy is in the form of acone of radius 3.5 cm
mounted on a hemisphere of same radius on its X10 × 10 =Volume ofcong
h= 150
drcular face. The total height of the toy is 15.5 cm.
Find the total surface area of the toy. Hence the height ofh=15 cn
water = 1.5 cm
ACBSE Delhi/OD 2017)
Q.7. Rampal decided to
ICHSE Matking Scheme, 201
Opper Answer, 2017 conical in shape with
donate canvas for
height 24 mto a centre for
base 10
diameter 14 mtants
welfare. If the cost of 2 m
metre, find the amount by wide handicapped
canvas
which
2r,
is ?person's
15.5Cm the centre.
Rampal helpet
So.
C+ACBSEComptt. OD Set-i, i1, I T
ICISL OD Set-I, I, III. 2
eighi of hemisphe
Sol. Diameter of tent = 1# mand height
radius of tent = 7 m ,2015,215
=24 m
heiql 6f: cont 15 Sm- Sa Slant height = 2-
12cm h
=576+ 49 -25 m
Slan! height of one th Surface area of the tent = r!
22
1s6.2S X7x25
= 550 m
cSA o (onei CSA herni-phci . Surface area of 10 tents = 550 x 10
= 5500m
Total cost = 5500x

=110000
Hence, the amount by which Rampal helped the
:12 ( 6.25 3s) centre =110000
[CBSE Marking Schene, 2011
Q. 8. A well of diameter 4 m dug 21 m deep. The earth
Total suface arca of toy fs 214-Stm? taken out of it has been spread evenly all around it
3
in the shape of a circular ring of width 3mBo form
Q. 6. The th part of a conical vessel of internal an embank1nent. Find the height of the embank
radius ment. CBSE Delhi Set-l, li, Ui, 291o. 2015)
5 cm and height 24A cm is full of
water. The water Soí. Dianeter of earth dug out = 4m
emptied into a cylindrical vessel with internal
radius 10 cm. Find the height of water in cylindrical Radius of earth dug out, r = 2 m
vessel. Depth of the earth, h=21 m,
ACBSE elhi Setl 2017| Volume of earth dug out = 1uh
¡65F OD bei-1, 2016]
Sol. Radius of conical vessel 5 cm
7
and its height = 24 cm
Voiume of this vessel = w'h Width of embankment 3nt
Outer radius of ring =2+3= 5m
heightofembankment be #,
ofenbanknent Volume of
Q.11. In fig., SURYACE ARE.AS AND
he
ihume
n(R-)h = 264 earth dug out
, from a cubuidal solhdVOAIMES 26)
22 x (25 --4) xh = 264
7 1
diateamensiof thedionsaneter7 cm
hole of 15 cn x 10 cm x metallic block of
is
5 cm, a
cyindical
drilled out. Find theesurface
264 x7
Heightofembankment = 4 m
h=
22 x 21 =4 remaining Hlock.Ue
7cm
rCBSE Marking Scheme, 20151. 1

the radius of base


right
sumof and heightsurface
Thecicularcylinder is 37 cm. 1E the total
of asolid
area
1
is
otihesolid cylinder 1628 sq. cm, find the volume of
5 Cm
hecylinder.Usen=7
mCBSE Delhi Set-I, 2016]
Herer
+h 37 and 2tr(r + h)= 1628
2r X 37 = 1628 10 cm
1628 15 cm
37
O,
ACBSE Delhi Set-i, I1, IIL, 20731
r=7 cm Sol. Total surface area= 2416 +
and
h =30 cm bh+ hl) + 2rrh -2tr
22 Here,l = 15 cm, b= 10 cm, h =5 7
Hence, volume of cyinder= ²h x7x7x 30 cm, r= CIm
2
TSA of cuboidal block = 2(15 x
= 4620 cm3 10 + 10 x 5
[CBSE Marking Scherne, 20161 =550 cm²
+5 x 15)
1
UA metallic cylinder has radius 3 cm and CSA of cyinder = 2rrh
height
sem. To reduce its weights, a conical hole is drilled
in the cylinder. The conical hole has a radius of
=110 cm?
-2xxx5 2

-3 cm and its depth cm. Calculate the ratio


2 Area of two circular bases = 77
of the volume of metal left in the cylinder to the
volume of metal taken out in conical shape. -77 cm²
Required area 550 +110-77 = 583 cm²
ACBSE Foreign Set-I, II, III, 2015] [CBSE arkingScheme, 2IIS)
Volume of cylinder = rh = n(3) x 5 Q. 12. From a solid cylinder whose
height is 8 cm and
= 45n cm3 radius 6 cm, a conical cavity of same height and
same base radius is hollowed out. Find the total
Volume of conical hole = surface area of the remaining solid. (Take x = 3.14)
UCBSE Comptt. ODSet-l, 31. 11T, 2017
Cm3 Sol. Height of cylinder = height of cone = 8 m
radius of cylinder = radius of cone = 6 ct
. Slant height of cone= v8 +6²
Metal left in cylinder = 45
3
133T
3
m
1 = 10cm
Again, the required ratio Total surface area of emaining solid
= Cuved surtace area of cyERdes
Volume of metal left
-+ Surface ara ce
Volume of metal taken out + Area tup cver
133
133:2
= Kh+t)
2

Hence, Volume of metal left : Volume of metal


taken HenceIoial surtac area of naining s:N
out =
133:2.CBSE Marking Scherte, 2015}
Ceaal CRSE Qrestion ank Chaptewise &Topicwise, MATHEMATICS (STANDARD), Class-X
262

Long Answer Type Length of water moved in 1minute 1


Questions (S marks each)
Length of water moved in 30 minutes
Q. I.Asoid is in the share ofa hemisphere surmounted 30
by a cone. If the radius of hemisphere and base x 10
60 =5 krn
radius of cune is 7 cm and height of cone is 3.5 cm,
22 Now, volume of water in canal 50OO ro
find the volme of the sotid. Use-!
= Length x
5000 × 6 × Breadth
SoB. Here.
ADICBSE OD Set-III, 2020]
radius (r) = 7m
Now, volume of water in canal
1.5 m3
Depth
volume of
and height of cone (h) = 35 cm irrigated water in
5000 x 6 × 1.5 m= Area arez
s000x 6 × 1.5 m²=
irrigated x8
cm
Area irrigated x
3.5 cm 100
:. Area irrigated = 5000x 6x1.5 x100
n
= 5.625 x 10 m?
p. Water is flowing at the rate of 15 km/h
cylindrical pipe of diameter 14 cm
pond which is 50 mlong and 44 minto a a through
ime the level of water in pond rise bywide. cuboida!
In what
:. Volume of the solid 21 cm ?
= Volume of hemisphere ÄCBSE SQP 2020, 2018]
SoL. Quantity of water flowing through pipe in 1
+ volume of a cone
7 hoy
=1X × 15000 m³
100 100

2 22 122 Required time


*7x ()+x x(7 x3.5
(2156 + 539) x15(00
= 2 hours
x 2695 (CBSE Marking Scheme, 2120)24
3 Detailed Solution:
= 898.33 cm Speed of water flowing through the pipe
9. 2 Water in a canal, 6 m wide and 1.5 m deep, is = 15 km/hr = 15000m/hr
flowing with a speed of 10 km/hour. How much Volume of water flowing in lhr = R²H
arez will it irrigate in 30 minutes; if 8 cm standing
water is needed ? 7, 7 x 15000
7 100 100
ADAICBSE Delhi Set-I, 11, III, 2019, 2018]
CBSE OD Set-III, 2020, 2017} = 231 m3
Volume of water in the tank when the depth is 21 cam
SoL. Lergth of canal c¡vered in 30 min = 5000 m 1/%
=lbh
:Volume of water fiown in 30 min
21
=6X 15x 5000 m² 2 = 50 × 44 x m²
100
If8 Cn standirg water is rneeded
= 462 m?
then area irigzted = 611.5x5000 = 5625(00 n 12
0.08 462
Time taken to fill 462 m = 2hrs.
231
[CBSE Marking Schene, 2019)
Detailed Solution: Q. 4. A solid iron pole consists of a cylinder o
Canal is of the shape of cubojd, where height 220 cm and base diameter 24A Cm, which
15 surmounted by another cylinder of height
Breadth 6m 60 cm and radius 8 cm. Find the mass of tt
Depth 1.5 m pole, given that 1 cm of iron has approximately
and Speed of water = 10 khr 8gmmass. (Use t =3.14)
Length of water moved in 60minutes = 10 km {CBSE OD Set-I, 2019)[CBSE Term-li, 2012)
SURFACE AREAS AND VOLUMES263

8 Cm Commonly Made Error


60cmn
Many candidates use formula for finding
volume of cylinder, 2nh instead of rh.
km
220cm/
Answering Tip
12 cm Remember the correct formula for finding
volume of cylinder.
(12)2(220)+ 3.14(8)(60) cm 1 25. Water is flowing at the rate of 5 km/hour through
volume = 3.14 12057 6
99475.2 + apipe of diameter 14cm into a rectangular tank of
ea al 111532.8 cm3 1 dimensions 50 m x 44 m. Find the time in whiclh
111532.8x8 the level of water in the tank will rise by 7 cm.
Mass
kg 1 A|CBSE Comptt. Delhi Set-I, II, III, 2017]
1000
= 892.262 kg 1 Sol. Try yourself similar to Q. 3. L.A.T.Q.
TCBSE Marking Scheme, 2019] ICBSE Marking Scheme, 20171
2.6. A vessel full of water is in the form of an inverted
/Solution: cone of height 8 cm and the radius of its top,
massofpole,
we needto find volume of pole.
small cylinder which isopen, is 5 cm. 100 spherical lead ballsare
pole = Volume of
Bind
dropped into vessel. One-fourth of the water flows
ilume of + Volumne of large cylinder out of the vessel. Find the radius of a spherical
cylinder. ball. [Foreign Set-I, II, III, 2015]
aN,forsmall
Radius, r = 8cm
Sol. Volume of water in cone
Height, h = 60cm
cylinder
iuneofsmall 3
= th
= 3.14 >x (8) x 60 1
=3.14 × 64 X 60
x(5)'x8
= 12057.6 cm3 200
Hlume of large cylinder, 3
Haight =h= 220 cm Volume of water flown out
Radius = R=
diameter 1 200

24 14
= 12 Cm Let the radius of one spherical ball be r cm
2
4 50
3*
tume of large cylinder = R²h, 3
=3.14 x(12) x 220 50 1
=3.14 x 144 X220 4x100
= 99475.2 cm³
1
Volume of pole = Volume of small cylinder 0r,
r= =0.5cm
+ Volume of large cylinder
=12057.6 + 99475.2 CBSE Marking Sckeine, 2015)
= 111532.8 cm 0.7. Asolid toy is in the form of a hemisphere
Snce,1 cm of iiron has 8 gm mass
surmounted by a right circular cone of same
, iven
radius. The height of the cone is 10 cm and the
yolume of iron has mnass
= 111532,8 x 8 gm radius of the base is 7 cm. Determine the volume
= 892262.4 gm of the toy. Also find the area of the coloured sheet
required to cover the toy.
892262.4 22
1000
kg (Use t = and v149 12,2)
892.2624 kg
= 892.262 kg
ICBSE Deihi O.0, 2020}
Question Bank Chapterwise & Topicwise, MATHEMATIC8
(8TANDARD), Class-X
Topper Answer 2020

Fon Hemisphass
Fox cAMe
Sol,

5 1232m3

COMPETENCY BASED QUESTIONS (4 marks each)

It was originallycommissioned by the emperor


Casebased MCOs Ashoka in the 3d century BCE. Its nucleus was a
I. Read the following text and answer any four simple hemispherical brick structure built over
the relics of the Buddha. It is a perfect example of
questions from Q1 to Q5.
combination of solid figures. Abig hemispherical
The Great Stupa at Sanchi is one of the oldest stone dome with a cuboidal structure mounted on it.
structures in India, and an important monument of
Indian Architecture.
Take - -) [CBSE QB, 2021)
A--Chattra

Donne
-Harmika
Balustrade Q.4.The totalSUREACE, AREAS AND
surtace area the VOLUMES 265
hemispherical domedimensions
(A)shaped top with
1200
(C) 1392. 5q.sq,mm
of
with radiuscombined figure ie.
14 mand cuboidal
8 m 6 m %4 n
is:
Staiss (B) 1232 n4. TIN
Ans. Option (C) is (D) 1932 sq rm
correctsurface
Erplanation: Total . Area of
-2 Conbined figure
2(b + bh + hË - lb
714x14 +2(8 x6+6x4+4x8)
0 10m) -8x6] m²
= |1232 + 208 -
lo 30)
- 1392 m? 48] m
Q.5.The volume of the cuboidal shaped top is
dimensions mentioned in question 4is:
(A) 182,45 m
with
(C) 292 m (B) 282,45 m
Ans. Option (D) is (D) 192 m
correct. of the
Explanation: Volume
=lxb x h
cuboidal shaped top
=8m X 6 m X 4 m
= 192 m
I. Read the
leulatethevolume
vo of the ions fromfollowing text and answer any
01 to Q5. four ques
the dome is 21 m:
hemispherical dome if On a Sunday, your Parents
heheightoff could see lot of toys took you to a fair. You
A)19404cu. m (B) 2000 cu. m displayed,
them to buy a RUBIK's cube andand you wanted
015000 Cu. m (D)19000 cu. m cream for you. strawberry
AD CBSE
ice
QB, 20211
sOpion (A) is correct.
Enlanation: Height of hemispherical dome
ytius of hemispherical dome = 21 m.
2
Volume of dome =3 r
2 22
3xX21x21x21
7
= 19,404 m' The length of the diagonal if each edge
Ihe formula to find the volume of 6 cm is: measures
sphere is:
2 4 (A) 3/3 cm (B) 3V6 Cm
(B) () V12 cm (D) 6N3 cm
(D)2u2 Ans. Option (D) is cortect.
xOpbon (B) is correct. Explanation: We know that,
AThe doth require to cover the hemispherical dome Length of the diagonal = 3xside
he radius of its base is . Length of the diagonal of cube with edge 6 cm
IA 1222. sqm 14 m is:
(B) 1232 sq.n1
= 3x6
q 1200 sq,mn 63 cm
(D) 1400sq.m
ption (B) is
planation: correct
The . cloth required to cover the
Q.2. Volunme of the solid figure if the length of the edge
is 7 cm is:

Memispherical dome it the radius of its base is 14 cu (A) 256 cm?


(C) 343 cm
(B) 196 cn'
(D) 434 cm
Ans, Option (C) is correct.
22 Explaation:
7 Volume of cube (side
(7343 cnt'
1232 sq. ni
Oswaal CBSE Question Bank Chapterwise &Topicwise, MATHEMATICs (STANDARD), Class-X
266
0.1. What is the volumne of
Q.3. What is the curved surface area of hemisphere (ice
cream) if the base radius is 7 cm ? depression? cuboid and
(A) 309 cm?
() s03 m?
(B) 308 cm?
(D) 903 cm?
Sol. For cubojd
(I) l= 15 cm, b = 10 cm and
Conical
Ans. Option (B) is correct. Volume of the cuboid 1h=3.5
=
cm
lxbxh
Eplanation: We know that, = 15 X
10 x 3.5
CSA(curved surface area) of hemisphere = 525 cm
For conical depresion:
Given, r=7cm
r= 0.5 cm,
CSA = 2x
So,
7*7 h=14 cm
(I) Volume of conical depression
-308 cm?
1 22 1
Q.4. Slant height of a cone if the radius is 7cmand the x 0.5 x0.5x14
height is 24 cm
(A) 26 cm (B) 25 cm 11
(C) 52 cm 30
cm
(D) 62 cm
Ans. Option (B) is correct. 0. 2. What is the total volume of conicalI
Explanation: Slant height of cone, Also find the volume of wood in
the
Sol. (I) Volume of four conical depressions entire
depressiostns?and?
Hence, r =7 cm and h = 24 cm 11
= 4x = 147 cm
30
l =J +(24)
(1) Volume of the woodiin the entire stand =
J625 = 25 cm
Q.5. The total surface area of cone with hemispherical
of cuboid- Volume of 4
conical depressionsVolume1
= 525- 1.47
ice cream is:
(A) 858 cm? (B) 885 cm? = 523.53 cm3
(C) 588 cm? (D) 855 cm² 0.3. The given problem is based on which mathematiesl
Ans. Option (A) is correct. concept?
1
Sol. Surface Areas and Volumes.
Explanation: Total surface area of the cone with
hemispherical ice-cream = curved surface area of II, Read the following text and answer the
questions
the cone t curved surface area of the hemisphere that follows, on the basis of the same:
= 550 + 308 Abird-bath can be a garden ornament, small reflect
= 858 cm2 ing pool, outdoor sculpture and also can be a part
of creating avital wildlife garden. Mahesh made a
Case based Subjective bird bath for his garden in the shape of a cylinder
Questions with a hemispherical depression at one end. The
I. Read the following text and answer any four ques height of cylinder is 1.45 mn and its radius is 30 cm.
tions from 01 to 05. [Use n = 3.14]

A carpenter made a wooden pen stand. It is in


the shape of cuboid with four conical depressions
to hold pens. The dimensions of the cubod are
15 cm by 10 cm by 3.5 cm. The radius of each of the
depressions is 0.5 cm and the depth is 1.4 cm. (See
Figure).

Q.1. Find the curved surface area of cylinder hemt


1
sphere.
Sol. Given, radius (r) = 30 cnm
and height (h) - 145 m=145 cm
. Curved surface area of cylinder = 2zrh
=2X3.14 x 30 × 145
$URPACE AVLAS ANt VAIJMLS
27318 cm2.
radius () 30 cm
of henisphere 2n2 Total surface are of red stsrface srea ct cyr
surface area
Given,
curvod 2X3.14 × (30) der + curved surface arez f hernásphere
5652 cm2 1

cylinder and the total (Prored fros à1


the
volume of the 2
Find the bird bath.
oft 145 cm o.3. Which mathematical concept used is this case?
surfacearea h
radius (r) = 30 cm, Sol.Surface areas and Volumes.
o.
Flere, cylinder = h
Volume of
Questions
Solutions for Practice
Short Answer Type Questions-11
Multiple Choice Questions Ans.3.Total surface area of artice CSA of cyarder
correct. + CSA of two hemispheres
Option (A)is part of the pencil is
Ans.4. Explanation: The sharpened cylinder. CSA of cylinder = Zrrh
un-sharpened part is =2x 2
35 x19
cone and correct. gilli 7
Ans.7. Option
(C) is
The shape of gilli, in theand a =23 can
Explanation: cones
combination of two Surface area of two henispherDcal scoops
danda game is a =4xx35 x 35
cylinder. 7
Short Answer Questions cm3 = 154 cm?
Very ofa cube =8
volume articie=220 + 154
Ans.3. Given, cm, then :. Total surface areaof =374cn?
Let the side of a cube be a
a =8
a = 8
JCBSE Marking Schene

/2x2x2 Comnonly Made Error


= 2cm
acube unable to ind the
Then, surface area of = 6a Mostly students are
=6 X 2 × 2 hemisphere also they use the vaiue
radius of
= 24 Cn2 and they do errors
of,3.14 in place of 7
Questions-I
Short Answer Type raised measured be h in caBculation, they
subtract the area of
Ans.6. Let the height of
water
hemisphere from TS.A. of cylinder in place
Cm. of adding these.
.:. Volume of water displaced in cylinder
= I(10)h
Volume of cube =8x8x 8 cm³ Answering Tip
n(10)h =8x 8 x 8
8x8x8x7
h=
They should read the
question clearly and
22 ×10×10 correct caBculation for
use right formula and
= 1.629 cm
ICBSE Marking Scheme, 2015) which good practice isis necessary.

REFLECTIONS
basic soids naeBy.
Will you be able to determine the surface area of an
cCube, cuboid, sphere and hemisphere? object formed by combining any two of the
ESELFABSSSMENT PAPER-6

Maximum Tëme: I hour

Lhooe the comet altemative from the given optoS. 924 m'. Then Hh
L1.f the cuvd surface area of a cylinder in 264 m² and its volume is ratio of its
dhaneter is:
{A) 3:2
(B) 7:3 height to ite
(D) 7:1
{O1:3 end, then the surface
area of resulting cuboid is
Q.2 Two qubes each of volume &cm' are joined end to
(B) 40cm?
(A) 20 Cm (D) 80m²
(C) 10m
Q.3. The total surface area of the given solid figure is.

(A) rl + 2urh + r? (B) rl-2rh + Irl


(D) rl + 2rrrh
Q.4. A cylinder, acone and a hemisphere have same (B) base and same height. The ratio of their volumes is
3:3:1
(A) 1:3:3
(D) 2:1:3
( 3:1:3
Q.5. The circumference of acircle of radius 4.2 cm is:
(B) 24.6 cm
(A) 61.6 cm
(D) 28.4 cm
(C) 26.4 cm
angle 90° is:
Q.6. The area ofa sector fo a circle of radius 1.4 cm and central
(A) 1.54 cm? (B) 154 cm²
(C) 3.08 cm? (D) 308 cm?
IL Case-Based Question
carries 1 mark.
Attempt any four sub-parts from each question. Each question
1 x4=4)
Read the following text and answer the given below questions it: they decided to go to
City Modern School was organized one day educational trip for their students, in which narrated the facts of
Delhi. School took 10th standard students to show the Red Fort. Their Mathematics teacher
Red Fort to students.
2 pillars which are
Then the teacher said in this monument one can find combination of solid figures. There are
smaller domes at the centre. Flag
cylindrical in shape. Also 2 domes at the corners which are hemispherical. 7
hosting cerernony on Independence Day takes place near these domes.
clothmaterial will be required to cover 2 big dormes eachSURFACE
Howmuch AREAS
of radius 2.5 AND? VOLUMES69
metres

(B) 78.57 m²
(D) 25.8 m²
formula
to find the volume of a cylindrical pillar:
fhe
(B) rl
(D) 2r
area of two pilars if height of
surface
laterals the pillar is 7 mand radius of the
(B) 123.2 m²
base is 1.4 mis:
AThe
I123 cm2
:
(D) 345.2 cm²
volume ofa hemisphere if the
What I the
w ibe radius of the base is 3.5 m?
A)
859m3
(B) 80 m3
(D) 89.83 m
ratioofsum of volumes oftwo hemispheres of radius 1 cm each to the volume of a sphere of radius
the
Whatis
'cm?
(B) 1:8
4)
1:1
)8:1
(D) 1:16
Question [1 X3 3)
Short Answer Type
Very volume and the surface area of a solid hemisphere are numerically equal, then find the diameter of
fthe
hemisphere.
swingsthrough an angle of 30° and describes an arc 6.6cm is length. Find thelength ofthe pendulun.
9Apendulumperimeter of a sector of a circle whose central angle is 90° and radius is 7cm?
isthe
AWhat Questions-J [2 x 3 = 6j
Type
ShortAnswer
and area of a circle are numerically equal then find the radius of the circle.
LIthe perimeter
of a protractor is 72 cm, calculate its area.
LItheperimeter minute hand of a clock is 14 cm. Find the areaswept by the minute hand from 9 a.m to 9.25 a.Im.
1
The length ofthe {3 x 2 = 6}
Questions-II
rShort Answer Type other. Find the total surface
5h direular discs, each of radius 7 cm and thickness 0.5 cm are placed one above the
side 25 cm if the solid formed is
anea of the solid so formed. Find how much space will be left in a cubical box of
placed inside it.
of the spherical part is
BÁ Spherical glass vessel has a cylindrical neck 8cm long, 2 cm in diameter; the diameter
cm². Check whether she is
volume to be 345
85 cm. By measuring the amount of water it holds, a child finds its
Correct, taking the above as the inside measurements and = 3.14.

l Long Answer Type Questions


and 50 m broad. What is the rise of water
BU persons are taking a dip into a cuboidal pond which is 80 m long
Erel in the pond, if the average displacement of the water by a person is 0.04 m'?

Finished Solving the Paper? OSWAAL COGNIIVE


Time to evaluate yourself ! EARNING TOOS

OR
R9 detailed sotutions
Can hoOR Code
STATISTICS AND PROBABILITy
UNIT VII

CHAPTER

STATISTICS
13
Sylabus Mean, median and mode ofgrouped data(bimodal: situation to be
avoided).
In this chapter you will study
To find the arithmetic mean bå direct method, assumed mean method and step
grouped data. deviation method of
To understand how to fnd mean of grouped data.
Calculate the median and mode of grouped data.
Students will be able to analyse the mean values.

Revision Notes
Statistics deals with the collection, presentation and Mode: Mode is the
analysis of numerical data. observation which occurred
maximum times. In ungrouped
Three measures of central tendency are: data, mode is the
(i) Mean, (iü) Median and (iüi) Mode
observation having maximum frequency. In a
grouped frequency
determine the modedistribution,
Mean: In statistics mean stands for the
it is not possible to
arithmetic by looking at the frequencies.
mean of the given items. To find the mode of
grouped
with the maximum frequency.data, locate the clas
iie., Mean = Sum of given items This class is known
No. of items as the modal class. The mode of the
data is a value
Median: It is defined as the inside the modal class.
Scan to know
middle most or the central more about
value of the variable in a set this topic - Key Formulae
of observations, when the Mean:
observations are arranged
either in (a) For Raw Data:
ascending or
descending order of their If n observations x, Xy ., ,are
given,
magnitudes. Statistics then their arithmetic mean is given by :
It dívides the arranged
in two equal parts i.e., series
50%
of the observations lie
below the
remaining are above the median. median and the
ciasses
adding
class
class
the
preceding
bythe limit
Lower
all
obtained Level
hiN
of
frequencies class 2
Frequency limit
Upper
interpretation
analysis,
data
quantitative
of Level
Second
the
collection,
A

Level
ºFirst

Class
MMark
Frequency
Cumulative

term
frequencies Definition

of
Sum Statistic
n=
even,
Median
Where
is Data)
Ungrouped
(For
n
If

Median

marks.
frequencies
f,=
term

class
n+1) of
Sum
2
frequencies =
x,
odd,= = Grouped
Data
Median
n Mean Where
If
isn Where

succeeding
preceding
class
class class
interval class
of class
frequency
modal
modal median
interval
median d,
XËaftequencies
Here,
class class frequencies
of
n=Sum median clasg
marks
X,

of of of class
modal
to class
ciass
of bdal of
limit
frequencyfrequency
frequency Limitclass to
of Cumulative
of frequency
preceding
lower towidth
mod
of ,=
lower
with
I== = f.= k=
f. l= h== c.f
=
Where,
MATHCMATICN (ETANDAKD), Clnhn X
Topiowae,
Chaptruwine h
OwelCRSE Ourtion nk Man o the Oiped data
caleulated usng the lormula:
(b) orUngmuped Data:
listict obsevations
II here a
vanabwithfquenes Mvlian (M,) I+
lyt, t arithowtic
, spwtivch. then the
mean is given b lower imit of
whe
frequency of melan class, n nmedlan clas, fc
observatons, cf. (umulativg nunber
class-inte{rrvclealasn,p,uey
of the cass preceding the median
class-size or width of the h
Mode of Grouped Dala:
(c) For Grouped Data:
of groupd dataj Mode of the grouped data can be
(i) To find the mean
it is dssunned that
of ech
the frequency
centred
class-interval is
by using the formula: calculated
arvund its mid-point.
Mode(M) -1+
(ii) Diret Method:
Mean (* )= where l= lower limit of the
size of the modal class, H
the mid
where the x, (class nnark) is and f
interval
width or
frequency of the modal class-interval, f,
class, fo =frequenc)
of the class preceding the modal
point of the ;h class class,
frequency of the class succeeding the r f
is the corresponding frequency.Short class. modal
Method or
(iii) Assumed Mean Empirical relation between mean, mediar
cut Method: and mode:
Mean (a)= a+ () Mode = 3 median - 2 mean
1 mode + mean
(ii) Median =
and
where a is the assumed mean
3
of x
d = , - a are the deviations (ii) Mean =
3 median - 1 mode
from a for each i. 2
(iv) Step-Deviation Method:

Mean(x)=a+h Fundamental
Facts
where a is the assumed mean, h is
In statistics, that single value is called
the class size and u; = the central tendency and mean,
median and mode are all ways to
Median of Grouped Data: describe it.
find The mean is the average of a data set.
Let n =f +f +fs +. + ,.First of all The mode is the most common
and then the class in which lies. This
2 number in adata set.
class is known as the median class. Median The median is the middle of the set
of the given distribution lies in this class. of numbers.

OBJECTIVETYPE QUESTIONS (1 mark each)

(A) lower limits of the classes


Multiple Choice Ouestions (B) upper limits of the classes
() mid-points of the classes
Q. 1. In the formula x=a+ for finding the mean (D) frequencies of the class marks
data,
Q. 2. While computing mean of grouped
of grouped data d{s are the deviations from a of: assume that the frequencies are:
These questions are forpractice and their solutions are available at the end of the chapler
STATISTICS 273
(A)evenlydistributed over all the classes The modal class is the class having the maximum
8)centredattheclass marks of the clagses
Irequency.
IC)
centredat the upper limits of the classes The maximum frequency 20 belongs to class (15-20).
(D) centredatthe lower limits of the classes Here,
correct. 66
Option (B)is So, 33
Eyplanation:In grouping the data from 2

data, all (he observaticons between lower ungrouped


limitsof classmarks.sare taken in one groupthen mid- and upper 33 lies in the class 10-15.
Therefore, 10-15 is the median class.
valueor class mark is taken for further
calculation. So, sum of lower limits of (15-20) and (10-15) is
Thereíore, frequencies or observations must be (15 + 10) 25
centredattthe class marks offtthe classes.
xI xj6are the mid-points of the class intervals of Q.5. Consider the following frequency distribution:
erouped data, f(s are the corresponding frequencies Class 0-56-11 12-17 18-23 24-29
and+ is the mean, then Sfx,-) is equal to: Frequency 13 10 15 8 11

(A) 0 (B) -1 the upper limit of the median class is:


(C) 1 (D) 2 (A) 7 (B) 17.5
Ans. Option (A) is correct. (C) 18 (D) 18.5 A

Explanation: :x ) Ans. Option (B) is correct.


Explanation:
..) Cumalatiye
Class Frequency frequency
0.5 -5.5 13 13
24 X+*+x+..n times
5.5 - 11.5 10 23

nx .(i) 11,5 -- 17.5 15 38

17.5 23.5 46
From equations (i) and (i), we have
23.5 29.5 11 57

(57 +1)
The nedian of 57 (odd) observations 2

58
29th term
0.4. For the following distribution:
2

0-5 5- 10 10 15 15 - 20 20 - 25 29th tern lies in class 11.5 - 17.5.


Class
10 15 12 20 So, upper limit is 17.5.
Frequency Q. 6. For the following distribution :
the sum of lower limits of median
class and modal
class is: Marks Number of students
(A) 15 (B) 25
(D) 35 A Below 10
(C) 30
Ans, Option (B) is correct. Below 20 12
Explanation:
Cumulative Below 30 27
Class Freqúency frequency Below 40 57
10
0- 5 Below 50 75
15 25
5- 10 Below 60
12 37
10 - 15
the modal class is:
20 57
15 - 20 (A) 10-20
66 (B) 20-30
20 - 25 (C) 30- 40 (D) 50 - 60
thelr solutitas avalable atthe end of
for practtee and the chapter
Ihese questions are
274 Oswaal CBSE Question Bank Chapterwise &Topicwisc, MATHEMATIC8(8TANDARD), Clasg-X

SUBJECTIVE TYPE QUESTIONS


95
Very Short Answer Type = 475
Questions (VSAQ) (1mark each) Q.4. Consider the following frequency
the heights of 60students of a clase
Q.1. Find the class-marks of the classes 1025 and 35
AOACBSE OD Set-l, 2020) Heights (in cm)
55.
10 + 25 150-155
No. of sludents
Sol. Class mark of 10 - 25 = 15
2 155-160
13
35 160-165
=17.5 10
2 165-170
35 + 55 170-175
and Class mark of 35 - 55 =
2
175-180
90
= 45 Find the upper limit of the
2 given data. median class in the
Q.2. Find the class marks of the classes 15 - 35 and
45 -60. A [CBSE OD Set-II, 2020)
Sol. ACBSE SQP 20203h
15+35 Heights No, of
Sol. Class mark of 15 -35 = 2
(in cm)
150-155
students
15
Cumul
frequencyaive
50
= 25
15
2 155-160 13 15 + 13 = 28
45 + 60 160-165 10 28 + 10 =38
and Class mark of 45-60 =
165-170 8
38 + 8 = 46
105 170-175
= 52.5. 46 +9 =55
175-180 55 +5 =60
Q.3. Find the class marks of the classes 20 - 50 and
ACBSEOD Set-III, 2020| Since total frequency is 60.
35-60.
20+50 = 30
Sol. Class mark of 20 - 50 = 2
2

70 And cumulative frequency greater than or equal


= 35
30 lies in class 160-165.
So, median class is 160-165.
35 + 60
and Class mark of 35 -60 = .:. Upper limit of median class is 165.
2 1

Q.5. Following distribution gives cumulative frequencies of'more than type':


Marks obtained More than or More than or More than or More than or
equal to 5 equal to 10 equal to 15 equal to 20
Numberof students 30 23 2

(cumulative frequency)
Change the above data to a continuous grouped frequency distribution. UCBSE Term- 1, 2013!
Sol.
5-10 10-15 15-20 More than 20
15 2

[CBSE Marking Scheie, 20151


Q. 6. In the folowing frequency distribution, find the median class.
eght ncm 140 - 145 145-150 150 -155 155 -160 160-165 165-170

equenc 15 25 30 15
Meighn Isequeneg
14)-145
145-19) 15
1)-15
J55-16) 75
1)- 165

165-170 10

Nf 10

100
2

The cumulative frequerncy just greater than 5/) is 75 and the aresprding das is 155-15).
Hence, snedían class is 155-160.
that Mode =124and Mezn = I05.
OFind1the median of the data, using an empiical relation when it is given
1 2
Sol.
Median = ; Mode + Mean
1 2
=;(124) +(10.5)
3
124 21
3 3
124+21 33.4
3 3
1
33.4
3
=11.13 (Approx.)
(2 marks each)
Questions-I
Short Answer Type
following distribution:
0.1. Find the mean of the 7-9 9-11 11-13
3-5 5-7
Class 10
10
5 ADA:CBSE Delhi Set-L 2020
Frequency
of the following data: 120-140
Q 2 Findthe mode 80-100 100- 120
20-40 40- 60 60- 80
0-20 6 5 3
Class 10 12

Frequency
6 QDACBSE Delhi Set-I. 020)
frequenc.
modal class having the maximum
class is theModal
Sol. Since the class = 60- 80
f xh
Mode =I+ 2f-f-2
10,f =6and h= 20
Hence,I = 60,f = 12,fo = 12-10
Mode 60 + 2x12-10-6 X 20

2x20
= 60 +
24 -16
and their solutions are available at the end of the chapter
arefor practice
These questions
276 Owanl CRSE Questin Bank Chaptenwine &Toplcwise, MATIEMATICS
(6TANDARD), Clas, X
65.
Q.3.Compute the mode for the following frequency distribution :
12-16 16-- 20 20 - 24
Size ot itens 4-8 8- 12
(in cm)
17 12 10
Frequency 6

Sol. Her Modal class = 12- 16 ACBSE


I=12f 17,f = 9, (, = 12 andh=4
Mode = +
(2f-fo-h}
= 12 + X4

8x4
= 12 +
13
=12 + 2.46 = 14.46. (Approx.)
Q.4. Find the mode of the foliowing frequency distribution :
20- 25 25-30 30-35 35 -40
CBSE, O.D. Set-tIl, 2021
40 -45
1

Cass 15 - 20
8 10 3 2
Frequency 3

Q.5. Find the mode of the following distribution : U CBSE, Delhi &
.30 - 40
0.D, 20N
10 -20 20-30 40-50
Marks: 0-10 50-60
4 6 7 12 5 6
Numberof Students:

Topper Angwer, 2020

Mankso. s4 Shudtnts)
0-I0. 4
6

ta-40 Rneltalass
40-50.:
6

ModeLt!of..

Mode. 30.t. 12-7


24 s
30+

Hed alnaks 34olmaxks. (arrox)


equestions are for praçtice and their solutions are availabie at the end of the chapter
the mode of the following distribution: STNTISTICS 277
Class 25-30 30-35 35 -40 40 -45
45-50
Frequency 25 34 50 42 38
50-55
14
UCBSE Outside DelhiSet-1, 219)
Maximum frequency 50,class (modal) = 35 -40.
Sol.

Mode = l+ xh
2f,-6-h)
5034
=35 + x5
100 -34- 42

=35 +
16
x5= 38.33 (Approx.) [CBSE Marking Scheme, 2019] %
24
7Findthe unknown values in the following table :
Class Interval Frequency Cumulative Frequency
0-10 5 5

10-20 7
20- 30 18
30 40

40 - 50 30
UBoard Term-1, 2016

Sol. X =5+7=12
X= 18-X = 1812 = 6
X3 = 18 + 5= 23
x4= 30- X =30- 23 = 7 [CBSE Marking Scheme, 2016] h x4=2
and
respectively. The value of the largest observation is
0.8. The mean and median of 100 observations are 50 and 52 mean and median. ACBSE Term- 1, 2016]
110 not 100. Find the true
100. It was later found that it is

Mean =
Sol.

50
100
Zfx = 5000
Correct, Ef =5000- 100 + 110 = 5010
Correct Mean = 5010
100
= 50.1
median= 52. {CBSE Marking Scheme, 2016]
Median will remain same i.e.
regarding marks obtained by 48 students of a class in a class test is given below:
Q.9. The data
Marks obtained 0-5 5-10 10 -15 15 20 20 -25 25-30 30- 35 35-40 40-45 45-50
Numberofstudents 1 2 10 25 7

students. ACBSE Term- 1, 2015)


Calculate the modal marks of
class is 30 -35,l=330,f =25,f =10,f, =7and h = 5
Sol. Modal
25-10
Mode 1+ 2,--h > Mode =30 + 50-10-7 x5

=30 + 2.27 or 32.27 approx. .CHSE Marking Scheme, 2015| 2


(STANDARD). C
Chapterwise & Topicwise, MATHEMATICS
278 Oswaal CBSE Question Bank by students of a class
Q. 10, Given below is the distribution of weekly pocket money received Calculate the
by mostof the students.
money that is received
20 - 40 40- 60 60 - 80 80 - 100 100 - 120 120-140 pocket
Pocket nonev (in ) 0- 20 18 2
3 12
Numberof students 2 2
AOACBSE
Term- 1, 2015)
Sol.

Cass Enterval Frequency


2
0- 20
2
20 - 40
3
40- 60
12
60 80
80- 100
18
5
100- 120
2
120 -140
44
Total
Modal Class = 80- 100
Here, and h= 20
I= 80, f = 18,f, = 5, fo = 12
Mode =1+ x h
2f-o-f)
18-12 X 20
= 80 +
36-12-5

o6
80 + x 20
19
= 80 + 6.31
= 86.31 (approx.)
2
Hence, mode = 86.31.
(3 marks each)
Questions-II
Short Answer Type
frequencies a andb, if the total of the frequencies is
data is 16. Find the missing
Q.1. The median of the following
70. 25-30 30 - 35 35 -40
15-20 20 - 25
10- 15
Class 0-5 5-10 6 4
6
12 15
Frequency 12 A[CBSE SQP, 2020-21]

Sol. Cumulative frequency (cf)


Class Frequency ()
12
0-5 12
12 + a
5- 10
24 + a
10-15 12
39 + a
15 - 20 15
39 + a + b
20-25
45 + a+b
25-30 6
51 + a +b
30-35
55 + a +b 1
35 - 40 4

70
Total
STATISTICs 279
Agordingtoquestion. 55 +a + b=70
a+b= 15
N
Median = |+2 -xh
f

16 = 15+ 35-24- a.
-x5
15

1= 11-a
3
a =8
Substituting the value of ain equation (), we get
8+b=15
b= 15-8
b=7.
Q2Themode offthe following data is 67. Find the missing frequency x.

Class 40 - 50 5060 60- 70 70-80 80 -90

Frequency 5 15 12 7

[CBSE, Delhi Set-III 2020]


o3. Themean of the following distribution is 53. Find the missing frequency k?
80- 100
Class 0-20 20-40 40 -60 60 -80
k 13
12 15 32
Frequency
UCBSE Delhi Set-il, 2019)

Sol.

Class Frequency Class Marks


Interval ()
12 10 120
0-20
15 30 450
20 - 40
32 50 1600
40-60
70 70k
60 80
13 90 1170
80 -100
Ef = 72 +k Ef;= 3340 + 70k 1%

Mean =53
Given,
Mean =

3340 +70k
53 =
72+k

53(72 + k) = 3340 + 70k


3816 + 53k = 3340 + 70k
70k-53k =3816-3340
17k = 476
k =28
is 28.
Hence,value ofk {CBSE Marking Scheme, 2019}
rhractice and their solutions are available at the end of the chapter
280 Oswaal CBSE Question Bank Chapterwise & Topicwise, MATHEMATIC8 (8TANDARD), Class-X

given below:
Q.4. The marks obtained by 100 students in an examination are
45-50 50-55
Marks 30-35 35 40 40 - 45 55-60
Number of Students 14 16 28 23 18 60--65
3
Find the mean marks of the students.
a class for the wholo
CBSE. OD, Set-l.
Q.5. Aclass teacher has the following absentee record of 40 students of term. Find the
number of days a student was absent.
Number of days 0-6 6-12 12-18 18 24 24-30 AJCBSEI
30-36
mezn
Number of students 10 11 7 4 4 3
36-42,
TopperAnswer, 2019

methost.
uaing stapdosiakiou
(aloutakig meam

21 3 oX-3 =-30,

q-2] 2 11X-2 a -22


b-12 |
12-I8 15
Lar 24
24-3 O 2-21 z
35-1 a 2
D0-36 33
36-42 39 Deta

Total: |Zfiu;-46

A+ Zfiui x h

0.6. The table below show the salaries of 280 persons :


Salary 5-10 10 - 15 15 -20| 20-25 25-30 45 -50
30-35 35-40 4045
(in thousand )
No, of Persons 49 133 63 15 6 4
Calculate the median salary of the data. U[CBSE Delhi/0D2uS!
SIATISTrIC$281
Salary
(inthousand)
No,of Jesons
5-10 4
10-15
133 =f 49
15-20 63 182
20-25 15 245
291
25-3 6 260
266
35-40 273
40-45 277
45-50 1 279
N
280
280
2 140.
1
Median class = 10- 15.
Median =-f2
-f
5
= 10
133 (140-49) 1

=10 + 5x91
133
= 13.42
13.42
Hene,
median salary is thousand or 13420 (approx.) [CBSE arking Scheme, 2018 1
Solution:

Topper Answer, 2018


Salay iHheusgmt Rs lo-is

io-1s.
- mectian cass -ts.
2=o, he5, f (33, HQ, ct 4q.
We knot, medign

342.
Ihe. baedian salany is 3.42|4eyd 7pees..
fhe
mean of the
following data is 14.7, find the values of pand g.
Class 0-6
Erequency 10
6-12 12-- 18
4
18- 24 24 30 30-36 36-42 Total 3
40
ICBSE TRrm-i, 2ae/
MATHEMATICS (8TANDARD), Clas-X
Chapterwise b Toplcwise,
282 Oswaal CBSE Qurstion Bank
Sol

30
)-6 9p

12- 18 147
21
7
18 - 24 27
A-30 27
132
30-36 33
39
1
36--42 Exf, = 408 + 9p + 27q
Total f; =26 +p+q= 40
Given,
Lf, = 40
26 + p + q= 40 or, pt q=14

Mean, x =

408 +9p + 27q


14.7 =
40

p+ 3q =20 1
Subtractirng eq. (i) from eq. (i), 9=3
24 =6 or,

Putting the value of qin eq.(),


p=14-q= 14-3= 11
p=11,q =3
Q.8. Hence,
The mean of the following distribution is 48 and sum of all the frequencies is 50. Find the missing gfrequercies

x and y.
40 - 50 50-60 60 -70
Cass 20-30 30- 40
6 11
Frequency
UCBSE Term- 1, 2016, 2015)
Sol.

-2 - 16
25
20-30
35 -1 -6
30-- 40 6

40- 50 45 = a
55 1 11
50-60 17

60-70 65 2 2y
Ef = 25 +x+y Lfu, = 2y -11
Total

Mean =a+

48 = 45 + 2y -11 × 10
50
1
48-45 = 2y-11
5
3 5 =2y -11
15 = 2y - 11
y= 13
Also E{, = 25 + x +y= 50
X+y =25
x = 25- 13 = 12
Scheme 2016 !
..x12 and y 13. {CBSE iatking
STATISTICS 283
Long.Answer Type Questions
D
(5 marks each)
t Themedian ofthe following data is 525. Find the values of x andy, if total frequency is 100.
Class
0-100
Frequency
2
100 - 200 5
200- 300
300400 12
400 - 500 17

500 - 600 20

600-700
700-800 9

800 - 900
4
900 --1000
A(CBSE Delhi Set-, 2020)
Sol. Cumulative frequency
Class Interval Frequency
2 2
0-100
5 7
100-200
7+x
200-300
12 19 +x
300 - 400
17 36 + x
400- 500
56+ x
500-600 20
56 +*+y
600 -700
9 65 + x +y
700800
7 72 + x+y
800-900
4 76 + x+y 1
900 -1000
N= 100
Total
76 +x+y = 100 ..)1
Also, x+y=100-76 = 24
500 -600.
Median = 525, which lies between class
Given, Median class = 500600

-cf. 1
Median =|+2 xh
Now,

100
2
-(36 + x)|
525 = 500 + X 100
20

25 = (5036-x) 5
25
14-x = =5
5
1
x=14 5=9
value of xin eq. (), we get
Putting the 1
y =24-9= 15
y=15.
Hence, =9and
(STANDARDI
Chapterwise & Topicwise, MATHEMATICS Class-X
Oswaal CBSE Questio Rank
234 frequency fof the class 19-21.
distribution is 18. Find the
Ihe mean of the followins 15 - 17 17- 19 19-21 21-23
Clas 11 -13 13- 15 23
f 13
Frequency
AOOUCRSE OD Set-i 220 1CBSE

obtained in a suryes, are tabulated belaw :


Dbi0.D
Q.3. Dily wages of 110 workers.
Daily Nages fin) 100-12O
L20-140 140- 160

20
160- 18O 180 -20
18
2N0-2
12
22-as
15
Number of Norkers 10

Compute the mean daily wages and


modal daily wages of these
workers.
ACBSE SQP
Sol.
Numberof
DailyWages (in ) Workers ()
100 - 120 10 110 -30
15 130
10-140
150 -1
140 -160 29
I70= A
160- 18O
190 1
180 - 20 18

12 210
200-220
13 30
220-240
Total 110

1 x 20 = 170.19 (appron)
Mean daily wages = 170 +
110
22-20 x 20 =* 166.67 (appro) (CBSt Marking Scheme .
Mode = 160 -
4-0-IS

Detailed Solution:
1. Calculation of mean:
Class mark No. of Norkers
Daily Wages
(Cass interval)
1100
100 - 120 110
15 1950
120 - 140 130
150 20 3000
140-160
3740
160- 180 170
190 3420
180 - 200
210 12 2520
200-220
230 13 990
220-240

Total f, = 110 Sx= 18720


18720
Mean, X= = 170.182
110

Hence, mean daily: wages are? 170152 (Approx.)


2 Calculation of node

Daily Wages tin) 100 - 120 120- 140 140- 160 l60-180 180- 20

Numbe of Worker 10 15 20’f 22’ 18 ’f:


Here, maximum frequency,f =2
STATISTICS 285
ponding.class 160- 180 is modal class,
loundaryof modal class 160
= 22.
nNimum trquency
fnqueneyolpre-nnodal class =20.
firquency of pasttmodal elass 18,
modal class =180 160 20)
widthof

Mode =I 1

22 - 20
x 20
2(22) -20 - 18
2
160 + x 20

160 +6.67
Mode l66.67
Therefore, modal wages of workers is 166.67. (Approx) 1

the median of the following frequency distribution is 32.5. Find the values of fj and f:
Class 0-10 10-- 20 20- 30 30- 40 40- 50 50-60 60-70 Total

5 12 3 2 40
Frequency
ACBSE Delhi Set-l, 2019]

Class Frequency Cumulative frequency


0-10
10-20 5 f+5
20-30 9 f+ 14

30-40 12 f+ 26
+ + 26
40- 50
3 f+6t 29
50-60
60-70
2 i+ht31
1
40
Median 32.5 ’ median class is 30 - 40.
10 1
Now
32.5 = 30+(20-14-h)
12
1
f 3
f+ +31 =40
Also h=6 |CBSE Marking Scheme, 201911
Detailed Solution:
Cumulative
Frequency () Frequency (of)
Gass.
0- 10
f
10-20
5 f+5
ft14
20-30
30-40 12 +26
40-50 htfst26
50-60 3 ftht29
60-70 2 htf+31
N=f= 40
286 Oawaal CBSE Question Bank Chapterwise &Topicwise, MATHEMATIC8 (8TANDARD), Cln94-X

S++31 = 40
Now,
hth =9
fh=9-f
Given that median is 32.5. which lies in 30- 40
Hence, median cdass = 30 - 40.
40
Here; l 30, = 20,f= 12 and cf = 14 +f
2 2
Now, median 32.5
N
xh = 32.5
I+

30 + x 10 = 32.5

(6-A
12
X10 = 2.5

60-10j1
12
=2.5
6010f = 30
10f, = 30
f=3
1
From eq (),we getf=9-3= 6
Hence, f, =3 and f,=6

Commonly Made Error


formulae has to be an.
Some candidates use incorrect formula for median. Some get confused that which
to find median.

Answering Tip
known and unknown data.
Students should read the question carefully and keep in mind the
obtained by the students in an examination :
0.5. Find the mode of the following distribution of marks
0-20 20- 40 40 -60 60-80 80-100
Marks oblained
15 18 21 29 17
Number of students
relationship estimate the value of its meäan.
Given the mean of the above distribution is 53, using empirical
UCBSE SQP20148
Modal class =60- 80
Sol.
Mode = I+. -xh
2f-f
Here,I = 60,f = 29, fo = 21,f, = 17 and h = 20 1
29- 21
Mode = 60+ 2x 29-21-17X20
8
= 60+ x 20
58-38
=60 +8 = 68
STATISTICs 287
Frpinialrelationship, Mode =3
median-2 mean
Mode = 68 and mean = 53
3 median = mode + 2 (given)
mean
3 median = 68+ 2 x 53
Median = 174 = 58

Median = 58.
annualdayofaschool..A00 studernts partiipated in the function. Frequency distribuion showing their azes
Hence,

inthe
following table :
show
as
is 05-07 07-09 09-11 11-13 13-15
Age(inyears) 15-17 17-19
70 120 32 100 45
Numberofstudents 28 5
data.
median of the above ACBSE Term- L, 20131
Find
meanand

CI.

70 70 -210
05-07
120 190 -2 -240
07-09 10
32 222 -1 -32
09-11 12 = a
100 322
11-13 14 45
45 367
13-15 16 2 56
395
15- 17 18 3 15
400
5
17-19
Ef= 400 Ef4=-366
mean = 12
Let a = Assumed

Mean, X =Q+

- 366 x 2 = 12 183 = 12-1.83 = 10.17 1


Mean = 12 +
400 100

N
Median class

400-- 200 = 09- 11


2
(N
Median =1+

200 -190 Uy2 =9+ 0.625 =9.625 2


Median =9+
32 x2=9+
[CBSE Marking Schene, 2013|

COMPETENCY BASED OUESTIONS 4 arks each)

acute respiratory syndrome corona virus 2 (SARS


<0) Case based MCOs CoV-2) among humans.
I. Read the following text and answer any four ques
tions from Q1 to Q5.
COVID-19 Pandemic
The COVID-19 pandemic, also known as corona
COVID-19
virus pandemic, is an ongoing pandemic of corona
disease caused by the transmission of severe
288 Oswaal CBSE Ouestion Bank Chapterwise &Topicwise, MATHEMATIC8 (STANDARD), Class-X

The following tables shows the age distribution of case admitted during a day in two
different hospitals
Age (n yeats) 5-15 15 25
Table 1
25-35 35 45 45-55
ADICBSE QB, 2021
No. of cases 6 11 21 23 14 55-65
5
Table 2
Age (in years) 5-15 15 25 25-35 35-45 45 -55 55 -65
No. of cases 16 10 42 24 12
Refer to table 1:
Q. 1. The average age for which maximum cases occurred is:
(A) 32.24 (B) 34.36 (C) 36.82 (D) 42.24
Ans.Option (C) is correct.
Explanation: Since, highest frequency is 23.
So, modal class is 35 - 45.
Now,
Mode = 1+10xh
2f-fo
Here, I= 35, h= 10,f = 23,fo = 21,f, = 14,
23-21 x10
Mode = 35 +
46-21-14
2
= 35+x10
11

11
=35 +1.81
=36.818 ~ 36.82
Q. 2. The upper limit of modal class is:
(A) 15 (B) 25 (C) 35 (D) 45
Ans. Option (D) is correct.
Q.3. The mean of the given data is:
(B) 32.4 (C) 33.5 (D) 35.4
(A) 26.2
Ans. Option (D) is correct.
Explanation:
Class marks Frequency Deviation
Age (in years} ()
10 6 -20 -120
5-15
11 -10 -110
15-25 20

30 = a 21 0
25-35
40 23 10 230
35 45
14 20 280
4555 50
30 150
55-65 60
Eff=n= 80 Lfd, = 430

Now, Mean () = 4+

430
= 30+
80
=30 + 5.375
= 35.375
=35.4
STATISTICS 289
table2:
Referto
modeof the given data is:
04The
(4)41.4 (B) 48.2 (C) 55.3 (D) 64.6
Ans.ption(A)is5correct.
Erplanation:
Here, ModalI class is 35-45..(As highest frequency is 42)
Now, Mode = I + o -xh
2{-fo-fh
Here, I=30, f = 42,fo = 10,f, = 24, h= 10
42-10
Mode 35 + -x10
2x42-10- 24
32
= 35 + x10
50

=35 + 6.4
= 414
0.5.The median of the given data is:
(A) 32.7 (B) 40.2 (C) 42.3 (D) 48.6
Ans. Option (B)is correct.
Explanation:
Frequency ð)
Age (in years) (No. of cases) Cumulative frequency (ef)
5-15 8 8

15-25 16 24

25-35 10 34

35 - 45 42 (frequency) 76 (Nearest to )
45-55 24 100

55 - 65 12 112

Ef,= n= 112
112
Now, = 56.
2 2

l=35 (lower limit of mediarn class)


c.f, = 34 (Preceding to median class)

Median =
Here,

35+
56-34 10
42

22
= 35 + x10
42

35 +

110
354+
21
= 40.24 s 40.2 (Approx.)
MATHEMATIC8 (8TANDARD), Class-X
290 Oswaal CBSE Question Bank Chapterwise &Topicwise,
from Q1 to 05.
II. Read the following data and answertany four questions
100 m RACE
students to run 100 m.
groupo of
Astopwatch was used to find the time that it took a

60-80 80-100
Tine (in sec) 0-20 20- 40 40-60
6 3
No. ofstudents 10 13

|CBSE SQP, 2020-211


Q. 1. Estimate the mean time taken by a student to finish the race.
(D) 50
(A) 54 (B) 63 (C) 43
Ans. Option (C) is correct.
Explanation:
Tìme (in sec)
10 8 80
0-20
10 300
20-40 30
50 13 650
40-60
70 6 420
60-80
90 3 270
80-100
40 1720
Total
1720
Mean = = 43
40

Q.2. What will be the upper limit of the modal class ?


(A) 20 (B) 40 (C) 60 (D) 80
Ans. Option (C) is correct.
Explanation: Modal class = 40-60
Upper limit = 60
Q.3. The construction of cumulative frequency table is useful in determining the:
(A) Mean (B) Median (C) Mode (D) All of the above
Ans. Option (B) is correct.
Explanation: The construction of c.f. table is useful in determining the median.
Q.4. The sum of lower limits of median class and modal class is:
(A) 60 (B) 100 (C) 80 (D) 140
Ans. Option (C) is correct.
Explanation: Median class = 40-60
Modal class = 40- 60
Therefore, the sum of the lower limits of median and modal class = 40 + 40= 80
0.5. How many students finished the race within 1 minute?
(A) 18 (B) 37 (C) 31 (D) 8
Ans. Option (C) is correct.
Explanation: Number of students who finished the race within 1minute = 8+ 10+ 13 =31.
TATISTICS 291

baed Sublective Ouestions


Case
following etiand answer the
the.
questions that follow, onthe hasis of the sane,
Lhesd as frlkrw%.
maximumbowling speds, In kn per hour of 33 players at acrieket coaching centre are yfven
The

H5100 1(0- 115 115- 13) 1)- 145

11
Nonberoplayers

class and medlani clans of the glven data?


Whatisthe modal frequencyle,85- 100
01.
clans with highest
l.Modal dass is theobservations33 115
H-Number of observation, which liesinclass 100- 2
observations 16.5
Median o 33 npeed?
medlan of bowling
0.2.What isthe
Median m |4
Sol. 100 - 85 15
Iw 100,f 9, c.f, 11, h

Modian14

(33
7-11)
1004 x15

10 b.5x15

82.5
J004
9
100 +9.166
109,17 kn/h (Approx.)

Hence, the median bowling speed is 109,17 kn/h(Approx.) of medlan class7


0.3. What is the sum of lower limit of modal class and upper limit
Lower limitof modal class » 85
Sol,
and upper limnit of medlan clasy l 15 200.
Bum 85 4 115
follown, onthe basis of the same.
II. Readhe following text and answer the questions that
Theweights (in kg) of 50 wrestlers are recorded in the following table:
130.-140) 140- 15O
Weight (n k¡) 100-110 110-120 120 130)

No,of Wrestless 14
292 Oswaal CBSE Question Bank Chapterwise &Topicwise, MATHEMATIC8
(STANDARD), Class-X

Q.1. What is the upper limit of modalcdass? Also find the mode class frequency of the given data?
Sol. Modal Class = 120-130 1%
Upper limit = 130.
Mode class frequency of the given data is 21.
Q.2. How many wrestlers weights have more than 120 kg weight? Also find the class mark for class 130 -
Sol. No. of wrestlers with more than 120 kg weight = 21 + 8+3=32. 140,
For class mark of 130- 140
130 + 140
2
270
= 135.
2
Q.3. Which method is more suitable to find the mean of the above data?
Sol. For finding mean, Step-Deviation method is more suitable.

Solutions for Practice Questions (Topic-1)


Multiple Choice Questions Short Answer Type Questions-l
Ans.1. Option (C) is correct. Ans.1.

Explanation: In the given formula, a is assumed Class Frequency () Mid-Value ()


mean from class marks (x) and d,= x;-a
3-5 5 4
Therefore, d, is the deviation of class mark (mid 20
value) from the assumed mean 'a'. 5-7 10 6 60
Ans.6. Option (C) is correct. 7-9 10 8 80
Explanation:
9-11 7 10 70
Marks Number of students 11-13 8 12 96
0- 10 3-0=3 3
Sf= 40 Sfr = 326
10 - 20 12-3=9
mean =
20 30 27- 12=15 15

30 40 57 - 27=30 30 326
=8.15. 1
40
40 - 50 75 - 57=18 18
Ans.4. Here, modal class
50 60 80 - 75=5 5 =30-35

Modal class has maximum frequency (30) in :l=30,fo =9,f, = 10,f, = 3andh=5
class 30 -40.
(2f-fo-h |x
1
Mode =l+
STATISTICS 293

10-9
)4
a5 Mode =1+h-h
210-9-3) 2f,-h-6
5 15-x
4
A) 3 0 4 0,625 -110
67 =
30-12-x
30).625, 1 15-1
7= -x10
short AnswerTypeQuestions-|| 18-%
Sol.2.Frorn the table of gjven question, the modal class
is the clas having the maximurn frequency, 7 (18-x) = 10(15- )
rmodal class 60-70 126-71 = 130-10x
ie.,
3x = 150- 126
Then, l 6),f, 15,f, 12 and h=10 3x = 24
I =8.

SolA.
32.5 37.5 42.5 47.5 52.5 575 62.5
14 16 28 23 18 3 2= 110
-2 1 2 3
SM,42 -32 -28 18 16 9, Efu, = -59 1

Mean 47.5 59 x5
475-2.68 = 44.82
110
Note: If Nis taken as 100, Ans. 44.55 {CBSE Marking Scheme, 2019)2
Detailed Solution:

Number of students Mid Vafues


Marks

30-35 14 32.5 -3 42

35-40 16 37.5 -2 -32

40 ~45 28 42.5 -1 -28

45 -- 50 23 47.5 0

50-55 18 52.5 1 18

55-60 57.5 2 16

60-65 3 62.5 3 9

N= Ef =110 Lfu, = -59

Here, Assumed mean, A = 47.5


Mean = A
+ yh
N
47.5 + (-59) x5
110
47.5- 2.682
= 44.818
294 Oswaal CBSE Question Bank Chapterwise &Topicwise, MATHEMATIC8 (sTANDARD), Class-X

Long Answer Type Questions


Soi2
Class Class mark (x) Frequency ()
36
I1-13 12
6 84
13- 15 14
15-17 16 9 144
13 234
17-19 18

19-21 20 20f
21-23 22 5 110

23-25 24 4 96
f=40 +f Ef =704+ 20f
f= 40 +f
Efr = 704 + 20f
704 +20f
Mean = 18 =
40 +f 1
720 + 18f= 704 + 20f
fe8 (CBSE Marking Scheme, 20181
OR

Topper Answer, 2018

Clar

I-I5

21-13
2A
404

720-704 2Of-i84

REFLECTIONS
1. Will you be able to represent a cumulative frequency distribution graphically as a cumulative frequency curtei
or an ogive of the less than type and of the more than type?
PROBABILITY
-~-•~ -~ ---- H~- -- -----~--~ -------- ~-- -- ~-- ~---•·--- -1 I
I

Classical defittiHon of probability. Simple problems 011 findittg tlte i


probability of an event. :
I

------------------------------
I

~~ ----- ------- ----~ -- ---- ----~ --~------~ ---- ---------~----·----- '


-~ui ¼IM .
., . -- ~~ ~
J, ■-■ - ~ ~ ~

rneoretical probability and experimental probability.


Problems depend on the definitions of sure/ certain event and complementary event.
[}.jJerent types of problems based on dice, coin and cards .
.. ~.~-- -- --- ~ - -- -- ----------- ------- ---------- ----- --~-- -------- -----~--- --- ---- --- -- -

► Probability is a branch of mathematics that deals ► Probability can never be negative and more than
v.;th calculating the likelihood of a given event's one.
occurrence. ► A pack of playing cards consists of 52 cards which
I>- A random experiment is an experiment or a are divided into 4 suits of 13 cards each. Each suit
process for which the outcome cannot be predicted consists of an ace, one king, one queen, one jack and
with certainty. e.g., 9 other cards numbered from 2 to 10. Four suits are
spades, hearts, diamonds and dubs.
(i) tossing a coin, (ii) throwing
a dice, (iii) selecting a card ► King, queen and jack are face cards.
and (iv) selecting an object ► The sum of the probabilities of all elementary events
etc. of an experiment is 1. ·
t> Outcome associated wi th ► Two events A and Bare said to be complementary to
an experiment is called an
~f each other if the sum of their probabilities is 1.
event. e.g., (i) Ge~g a h~ad
on tossing a coin, (n) gettin~
Proba bility
► Probability ~f an event E, denoted as P(E), is given
a card ts by:
a face card when
drawn from a pack of 52 cards. = Number of outcomes favourable to E
P(E)
> Theevents whose probability is one are called sure/ Total possible number of outcomes
certain even ts-
► The eve~ts whose probability is zero are called ► For an event E, P( E) =1- P(E), where the event E
unpassible events- representing 'not E' is the complement of the event E.
An event with only one possible outcome is called ► For Aand B two possible outcomes of an event,
► an elementary event. (i) H P(A) > P(B), then event A is more likely to
► In a given experiment, if two or more events are occur than event B.
equally likely to occur or have equal probabilities,
then they are called equally likely events. (~) li P(A) = P(B), then events A and B are equally
likely to occur.
r,,- Probability of an event always lies between oand 1.
faL t gw--- \, 1 ,, -
Diec·
Two dia- are rolled, what ii probability S un: or C t>rta lrt ri ve n I Ct,mpl, m,ot,.r1 J. ~•r.t
I ?
of gettl.11g 12 Ma sum? When oent ha~ln,g Pr,r CTt'11t 1::, ;-•1 .....,,,.,,..,~•4rr J. ,,,..t,t 1:;,
prnbabillty to occur as I £,.--i,,.,,y;,,, ~
r:.omplrmenf nt11t, P(ft ) x I - l'(E) ,,._
SolarJon : Number of pos•ible ,,n/y ,,n,e "'7W.- ,,f n-,~
~~ ?.
outcomes c 6 2 =36 :;r,:

Numberoff11TOW2ble outcomes= 1 i.e. (6, 6)


5-
P(E) =2._ 1
36 J
?
~
r,
Oef"initlon~
~
,-------__ WJ=at~·
~

~
n-~... ,_ ......_;,,"'.,,.._
Coin ,..., - 7 7 Jr• - e:,r~
r'o
When a com is toased, what ~
would be the probabt"lity of
appearing head 7 !-1-.~ r , l ~ ~
,......,
,~~..;:,,,,: tr; £.
Pf.E) "% ~ ! , . a - at .-11 ~
S'
Sohm.on : Total onfcome,, = 2 i.e. (H, T) ~ uf ~ ~
Fnom:able outcomes = I ~
~ :>-
~ufred Prob. P(E) = 2I 1:.

Vc:lue
;;;·
r:
>

Saciof~~
o,f~,e'~
~
~
~i1 L ~
C a:rd FcreT==ts.A, 3 , C; PV-J-:f'(:B}+?(C) = l -!
What is the probability of getting
~'(obaou;ti! ,"\
• >
., . • z:
au ace from a pad: of S2 cards? 0
\ . ~ 0.s:P(E).s:J >
- :i~_ ,r:
',
Solution : Number of fnomable E
OU1CO?l3<!$ =4 /A W!1:z:t,-.~u., 1,.,
JI' )" ~ t?l'- ~..=c:tt
,· v.:::,- ~
Number of posru,le outcomes = 52
\, "';':,.;( "
Y-
P(E) ~ = _:_
=52 ~~ !'t'rz:r::ca a,£ ai,aa; a
13 r,-,,. . . ";,od ~......
;~-r·,
• Fi,,,,t LL9d • S«ood L 6'Yd • n,;,,.4 ,......d.
~ t b c ~ ls.¥; (
P(E) - T- ~«a"-m
~ -- . -- ·-----
.JS/I ~
~ -►- . -- -isi:=- -- r --- --
l'l< OD/\ BJLJTY
f ..JJ,:
'i .
• Tn 1ne · c.' •nt "w i 'i
11 .th~ exp~ri,ni.
assum e th, :
expe nrnents have ,
equQJ/y likclY OU. lco
es from an

'
rn,•s
. Tht' scl of outcom .._ .. · '
-~known as e ve nt
s. "" A deck. of playing ca rd s co.n.s.,615
c11tS· t 1 .
· 1en ·t· Something tlrnt .rn n be
iv,,,,.,,, of 4 suits· .· 8 paue
.,
s (<t) , h earts (• ),
,. ,/ ·menat •has i1 set of possible rc- . ( )
di am on d s • and clu bs ( . Oubt,
<fo)
£JPc~~ th an d sp ad es are of black
.,,,.,rll ,
- .. ' . d' co lou r,
- - ... - ... ... ... .. while he ar ts an d iarnond s are of

.
'r r ... ... ...... - - - ... - I
' 11/t..;
. .. ---
,_ , I re d co /o ur.
., __.,. ____ ___
------

~ m&:J:nimr.&:m,~,\--Q:m?,ibi$;;.-)~,.:-,:►-. -_; ·1:


\
.. I
,...,,• ' _ _ . ,rim enta l or empm d
ca
! . . . on
·he expt of t 1s ba se
, , 1 il bT tY an even ,d wIu·1,e ,
, , .
rob . . actu ally ha pp en c
,' P a t 1ia~ ·1· f th ,
' 1 t
vh J· eoretical probab1 ity o h et 'ra nd om toss'
• i
1he tt attempts to pre c w
1 cfj t
ce
a
rta in
'
:
,
;I
► By th e ph rase
m ea n th at th e
coin 1.s aUow , we
ed
th e ba sis of b'
c~enh pe n on (aJl freely w ith ou t an y 1as or
,v,ll ap . . .to e.
' as~ umptionu ns. :I m terfe re nc
m be r of tri,al s m • an he
' mcr.easrng, ,, on probability 'T
I . . As
1
tJC
• ·ient go on ► Th e fir st bo ok an ce ' wa s
' " ex penn nm en ta l , of Ch
1
y expect the ex.pe . . s to be ' Book on G am es m at ia'
; , n mat he an
' wei ma th eoretical probab
1hhe , w rit te n by Italia
,, a.n c . ) ]. Ca rd an .
nearl y the sa me. k of a com, w e on of
:
' u1J 1en we sp ea , •t • I
; f ►- The elassi ~ ! definiti re
I is
I
ir r.e.,_ give n by Pi er
I ► r,
to be 'fa pr ob ab ili ty w as
i ass ume it ;
that th ere 1s no Si m on Laplace.
: sym metrica l 50
------- -~-----·
;
me do w n more
: reaso n for it to co an the o_ther.
often 0 11 one sid e
th
:
:
~-- --- ---- --- ---- ----
ty of th as e com
We call this pr op er
being 'unbiased'
.
,
... .... ------
I
- -- -- ~
- -~ -- -
-- - -- - -

l
is correct.
Ans. Option (A) total sp ad e
o11: To tal ac e ca rd s = 4 an d
Expla11ati on g aces is also
a
12 (O ne ca rd am
ca rd s = 13 - 1 =
sp ad e) ad e
v~IJ e of ne ith er ac e or sp
P(E) + P( E) =x, then the Ca rds w hi ch ar e
Q, 1. For an event E,
= 52 -1 6 = 36
' r3 -3 is
(B) 2 36
(A) -2
(D) -1 Re qu ire d pr ob ab
ili ty = 52 = J_
13
(C) 1 rm -I, 202(]
[g [C BS E Boa rd Te in g cannot be the pr
obability
th e fo Jl ow
I
correct. Q. 3. Which of
Ans. Option (A) is
planation: Gi ve n of an event?
Ex ... (i) (B) 3%
P(E) + P(E) = X , (A) 0.01
ob ability
to th e law of pr (D) 17
Also, according ...(ii) (C) 16
P( E) + P(E) = 1
16
17 rm-r, .20 7 : 1
From (i) an d (ii),
w e get Ci) CJ [C E Board 1ethat 5 will
BS
\
X =1 obabiJity
d twice. The pr
- we ge t Q. 4. A dice is rolle
Put value of x in x3
3, er tim e is
3 = -2 no t come up eith
x3- 3 = (1)3 -3 = 1- 1
card fro m a pack of
ty that the drawn (A) 11 (B) 3
Q. 2. The probabili ad e is 36
52 cards is neith
er an ace nor a sp
(B) 35 13 (D) 25
(A ) 9 52 (C) ~
13 % 1-I1 ~! Vi ]
10 ~ ~ BJ [C BS E Board T~w
(C) 13 26 .
O .
---- -
mICllS E Board 'fl'rm-J 2fJ2J J 1
A
ns. ption (D) IS
correct •
- - --- -·-
- - 411d 1J,; ; ,; /- -- -- ll --- - - -- -
ns (lrt/orpract/c, sou t ons,1r,: 1va,•1", e ,11 lhc c11J of fhtd1ap1,,; -
~.: -.,:;;-,u qur.,tlo
\ti ,,
,

--
1 ~
----·- -~---,.-- ___ _
\1 J
__ _
.. ,,,.., __ ,
- - ~~·
I
:a o,;._'lUl C'BSE Q\i,esb.,,~ &nk C'htF-'ff"~'K ti T c ~~ . MATH.DlAT1C8 (STAKDARI>), Cl.aai. ;,c

~ Ali f ' ~ e\'\c"!\~ ~ ".mm t:,d(.m-: £.~'lft: ~ ·:_'_~know that the~:..,,,


rother i.~ negativl' tlJ ..... nor b(, &Teater »-.. · ~ •
II
0 1) \l : ) (l 3) l l -1) tl 5) (1 6) s...,. - 0.5 and J.lXU)l can not l'l'pt~~-r v ~ ~- ~
G l) (: 2) (1 3) (2 ~) (2 5) (: 6) events- .. -~ -
(:3 l ) (3 .1\ t] ) n. .n ) H~only (n).[111},(iv) and ('<i) ~ - _ :-l
1 3 , ... ~ I (3 S) {3 6 C\t e'o--ellls. · r•,~t ~
(4 l ) (.t 2) (4 3) ('4 4) (! S} (~ 6) Q. i. Two identic.a.l fair di-ce h.ave nulll~
(3 l ) fS 2) (5 3) {:- ~) _ (5 5)
_ __ t5 E,) on their hces. Both are tossed 5~ I to 6 'ftrl....
f. )
( 1 \6 ~) (6 3) (6 -1) (6 5) (6 6)
\\'hat is the probability that the 1.DlllJ~~
num~ thn turn up is U 7 Pmd!J.ct Of i
T«.a! e"\° 'a\~ = :x, (A) 1 {B) ~
Out of ~ c'\-en~ in which 5 will nc>t rome ur 36 9
&.he. ti:c:R a."'l' (l, 1) (1. 2l (1. 3) (1. 'l) (1, 6) (2. 1) 1
C:... l \ G., 3) (2. <Q (2.. 6)(3. 1l (3, 2) (3. 3) (3, 4) (3, 6)l-L (Q - {D) ~
1) (-l. 2) ( t 3} (4, ~) (-t ~ (6.. 1) (6, .Z) (6. 3) (6, 4) (6, 6). 6 3
~ o.. oi ~ e---mts in = 23 Bl lCBSE Q.B.. 1. :1.-.
Ans. Option (B) is correct.
. ~ 25
R ~ p~t,;..lfu.·
.
= -36 Erplmurtion: When dice is rolled ht.ice • .__ ·
e'\cTT!s in which the product of the mun~.. ~ ~
Q. 5.. f-omy is Nind_ :o!<led and ~W to pick OTii' ball
tram. ,c',ach of ~ jars.. up 12. · that~

-- -----~
.
= {(2. 6), (6, 2), (3, 4), (-!. 3)}

Jar l
----------
~ ,....----......
..__...,..
JarJ'Z- Jar~
!\o. of events= 4
Total no. of events = 36
Probability (product of U) = 2- = ~
. ••••
•••• •o•
••• ••••
••
36 9
Q. 8. If an eYent that cannot occur, then its 1

4 red hills 3 red. balls 4 red balls is: pro~

..______..,
.; green balls.
~
3 green balls
,._______,,
2 green bails
(A) 1 (B) i
4
1
Th-e ch..ance of Romy picking a red ball is same in (Q 2 (D) 0
(A) jars 2 and 3 (B) jars 1 and 3
Ans.. Option (D) is correct.
fii
(q jars 1 and 2 (D) all the three jars
Erpla~ An event that cannot OCCur • • ·
~ [!l + fil [CBSE Q."B., 20~1-22] probability, such an event is called im ~ ,0
event flOS>ib,e
Ans. Option {Q is ro:rrect. ~

Explllnlltion: Q@ An event is very unlikely to happen. Its probabiJi.tv !


is closest to: • ·
. 1, ......-,h~J-.hf..T
For Jar . ·n,. red ba.Il - 4 - 1
....,...,;
y,...........,.,...._. Y~'b a - 8- 2 (A) 0.CXXJ1 (B) 0.(X)l
(Q 0.01 (D) 0.1
Ans. Option (A) is correct. [I
For jar 2, probability prling a red ball = ¾= ½ 1
~la.nation_: The probability of the event, 1,-ruch
15 very unlikely to happen, will be ve.rv c!OSe lo ~-
f-or jar 3, probability picking a red ba.Il = ¼= ¾ zero. So it's probability is o.cxxn which is .minimum
amongst the given values.
:. Probability picking a red ball is same in jar 1 and 2.
Q. 6. Look at the numbe:r!i shown below. Q. 10. If the probability of an event is p, then the ~
probability of its complementary event ~ill be:
(i) - 05 .(ii) 0.CXXXl1 (ill) _! (A) p-1 (B) p
2
1
(iv) 1 (v) llXXXll (vi) 99% (Q 1-p (D) 1--
p
Which of the above nwnbers represent
prob-abilities of events? Q. 11. The probability expressed as a percenbgt oi 1 ·
(A) only (i) and [llll particular occurrence can never be:
(B) only (i), (il"), (ml and (iv) (A) less than 1()()
(Q only (n), (iii), frv) and (v) (B) less than 0
(D) only (ii), (llll, (iv) and (vi) (C) greater than 1

llJ {CBS E Q .B., 2021-22] (D) anything but a whole number


Ans. Option {B) is correct.
Am. Option (D) is c.orrect.
PROBABILITY •
, probability lies between 0 and 1 and
~ e probability that a non-leap year selected at
/iJ!l''rlClrl• verted into percentage it will be
. pf . is con random will contain 53 Sundays is: .
¾
-
...
,hen ,t
r' (loan
i,t'
111•ee
deno e
d 100. So, cannot be negative.
ill f
t s the probab ty o an event A, then:
(A) i (B) :

~
. l:lf ~,;f(A) 0 (B) P(A) > 1 5
r r(Al <
i~ . 1~1 .c. P(A) ~ 1
(C) (D) -
7
m
Clo ,~
(D) -1 :5 P(A) :5 1 m Ans. Option (A) is correct.
, I · (C) is correct
'l.\ ·011 Explanation: Number of days in non-leap year
Y. f apb tion: As the probability of an event lies = 365
le £,p/a11a .
o and 1. , 365 1
t<l\1•een Number of weeks = - = 52- = 52 weeks
is selected from a deck of 52 cards then 7 7
,acard • • • '
,tU bability of its bemg a red face card 1s •
• •~fO • Number of days left = 1
For example, it may be any of 7 days which from
3 B) ~ Sunday, Monday, Tuesday, Wednesday, Thursday,
•) ' -26
·, (,~ ( 13
:) Friday and Saturday; so, T{E) =7
2 1 F(E) =1 (Sunday)
(D) -
(C} 13 2 P(F) = F(E) = !

. . . . . . . .~~..-~~==
~
T(E) 7

J
-~
.'P.!Wl~!P.~
l ~!:i.- ~ - - - -
Rilfllf;I
li,llllliliiliillllllliiiiillilM•
· ·is the probability of losing it 7
Sol. P(winning the game) = 0.07
. , what

l ,•

p. find the probability of getting a doublet in a throw PQosing the game) = 1- 0.07
of a pair of dice. e) ~ DJ {CBSE SQP, 2020-21] = 0.93 1 !'
{CBSE Delhi Set-II, 2020] Q. 6. A die thrown once. What is the probabilHy of
.,
1,2. Find the probability of getting a black queen when getting an even prime number?
a card is drawn at random from a well-shuffled m
[CBSE Delhi Set-II, 2020]
· pack of 52 cards. ~ DJ (CBSE SQP, 20Z0-21l Sol. Total possible outcome n(S) =6
2 1
. Sol Probability of getting a black queen = = 1 Favourable outcome = {2}
52 26
i.e., n(E) =1
!ktailed Solution:
. . n(E) _ 1
Total no. of cards = 52 = 6·
P(getting an even pnme number) n(S) - 1
No. of black queens = 2
2 1 <Arf'a number x is chosen at random from the
So, Probability of black queen = = 1
52 26 ~ numbers - 3, - 2, - 1. 0, 1, 2, 3, then find the
Q. 3_A letter of english alphabet is chosen at rand~m. probability of x2 < 4.
What is the probability that the chosen letter 1s a ~ m[CBSE OD Set-I, 2020]
consonant.
l'ifi1
~
UJ [CBSE Delhi Set-I, 2020) Sol.
Sol. In English language, there are 26 alphabets.
Consonants are 21.
So total no. of outcomes,
' n(S) = 26 Total possible outcomes= n(S) =7
and favourable outcomes,
n(E) = 21 Favourable outcomes = :t2 < 4 i.e., x = - 1, 0, 1
. The robability of choosing a consonant n(E) =3
.. p n(E) 21
½ P(x2 < 4) = n(E) = ~ . 1
= n(S) = 26 n(S) 7

4 A die is thrown once. What


is the probability of is the probability that a randomly taken leap
Q. · getting a number less than 3 7 year has not 53 Sundays 7
$ ~ m[CBSE Delhi Set-I, 2020] 0 ~ !i1 [CBSE OD Set-I, 2020]
~ . ; ,,011, arejar practice and th,lr solutions are available at the end of the chapter
~, . lcwise, MATIJEMATI CS fSTANDARD), Class-X
~~~, O~waRl CBS!!: Qu<"st ion Urrnk Chuptcrwrnc &. Top
-
., ( Q 13 Out of 200 bulbs in a box, 12 b-u.lbs
Q. 9, A die i~ thrown once. Whal is the probnln_hty, ~ · · One bulb is taken out at random te defe(ij.
gdti ng a prim"' number !CIJSE OD Sel-l, Z0.2 ] ?m What is the probability that the dra:orn the ~
Sol. Total J'X l~~ibk outcomes :=, 11(S) :::-, 6 defective 7 (Cas~ ~~lb ls m
F,n-our,ihk out-comes = {2, 3, 5} Sol. Total no. of cases = 200 • OP,2);11
1.t:., n(E) = 3 ½. Favourable cases = 200 - 12 == 188 ·l
11(£) 188
..
Probability == - : . Required probability = -
- n(S) 200
3 1
47
=-=-
6 2 so
1
Q. 10. A number is chosen at random from tlte numbers ~What is the probability that a non-leap Yea
- 5, - 4, - 3, - 2, - 1, 0, 1, 2, 3, 4, 5. Then find the <\../Mondays? DJ fCBSETerrn.~ h~ss3
~,2o·t1
prob,1bility that square of this number is less than Sol. There are 365 days in a non-leap year. ,,
or equal to 1. ~ DJ [CBSE SQP, 2020-21] •: 365 days = 52 weeks + 1 day
[Delhi Set-I, II, III, 2017] :. One day can be M, T, W, Th, F, S, S -::::7
½
:. P(53 Mondays in non-leap year) == I
3 [CBSE SQP Marking Scheme, 2020] 1 7 ½
Sol. -
11 [CBSE Marki.no- Sc...\e.,., ,, /i
i, u,e, -'l/l3j .
Dt.?t.nled Solution: Q. 15. Two different dice are tossed together. F' !
Given numbers, - 5, - 4, - 3, - 2, -1, 0, 1, 2, 3, 4, 5 probability that the product of the number on~ the
Total outcomes = 11 ½ of the dice is 6. ,~lop
f.i.1 [CBSE OD Set-I II III .u]15J I
I /

Squaring all the numbers we get


Sol. Product of 6 are (1, 6); (2, 3); (6, l); (3, 2)
25, 16, 9, 4, 1, o, 1, 4, 9, 16, 25 No. of possible outcomes = 4
Probabili P(E) = Favourable outcomes Total number of chances = 6 x 6 == 36
ty, Total number of outcomes
3
P(Product = 6) = ..! = I
36 9
P(E) = TT J.
[CBSE Marking Scheme
. 7,1, -
, ~i.111 1 1\

Hence, Probability= ~. ½ Q. 16. A game of chance consists of spinning an arrow


. h . .
11 wh 1c comes to rest pornting at one of the nlllllbm ,
~ The probability of selecting a rotten apple 1, 2, 3, 4, 5, 6, 7, 8 and these are equally likely .
randomly from a heap of 900 apples is 0.18. What outcomes. Find the probability that the arrowwfil
point at any factor of 8 ? . '
is the number of rotten apples in the heap ? 51
ID [CBSE OD 2017] DJ [Foreign Set I, H1 III, 2015!
Sol Total number of points = 8
Since, the factors of 8 are, 1, 2, 4 and 8.
= (1 X 8), (2 X 4), (8 X 1), (4 X 2) 1

'
A, ~ iH IT"><j . o.
nC5) =- qoo
__ !'{AJ ": _Q• l ll
'ro l\Pn

O\>ple.
l o la l oppll."S . PN(oF. otf favofu8r)a~e ~:c~;:v:u!a ble outcom6 "
.. ac ors o -
Total no. of possible outcomes
r.
I
,l
J
r, (s) ~ qoo
1"HPn Ofple.
- lola1 o.pple& i
4 1
= -=-
8 2
~~ !1"·

' ___ ?._(, .) < q •....'._'il


: ~
..1 ! .\ f'C~L k,~ _ f)vlnP~l 01 ro:llt..n o.F ple.1 . : [CBSE Mat·king Scheme, 2;1rJ .)

.-,-iiilil~ I\.•,
.• 9 U\) '· __ 'I"\( 12.

:j
To Ul , , 'l'ICAJ
nc 1)
. .'100~

. -0:1~ ./'.,~OA~ : I:=?7 / :~'-. : l®I . '. ff • -


1

..fo ·- lt!.~ npJ\~$ i~ ..-lhe .he.o.p.. 1 :


o w. 1~ 2- :niH~n ' ; • -- " • ' • - ~ •

a numberx is chosen atrandom ~~ the n~ 47 •


.. __ ____ _~ __ . ___~ _______ __ . ___,___.. __ ~ .. _______~

'f
Q
Q. 12. A die is thrown once. Find the probability of ✓ -3, -l_-1, 0, !, 2, 3. What is probability ~atxz{i,,,, "f ..
getting'' at most 2''. DJ fCBSE S.A.-2 2016] ~ ®' m[CBSE Di'llti set-!,
'•'~
Sol. S = {1, 2, 3, 4, 51 6} Q. 2. 'A child has a die whose six faces show the Jette.D ·j :,
J
n(S) = 6
0 0 @[QJ w• ·GJ
as shown below :
A = {1, 2}
baoililYI)! :
·,; ~
n(A) 2 • • .. •, ..
· = The die 1s thrown once. What 1s the pto
P(A) = n(A) = ~ _! l getting (i) A, (ii) D 7 . •J)'i 1
D '- ! J ;i - i '
6 - ::.
-;;;-;;;= ~-= ...__-:---.----:-:-:----n~ - ~
(S)
3_ __ _ _ _ _ __ ~~ J ~
· -.,!JJ
.J@Jl~ _9_.:.~ / J~?h,
J
f i 'lhcse queitlons arefor pr.iclice 11,uf their solutions arc aw1f/11/Jle III the
e,id of tire chapter

1
1 1
= =

ii.

JJ
1
i:
D ~ r? d .5o!.:±a: .=..-...: 5'::,-~ ;:- ~~ :~
L..-=t::: 2n d _z:re ffi
e' TI -:: :T X '- -c'3::= 2
2.?•game consists of tosstimine, if getting the same re-
g~ com! times and not- r--;e; = 1:
.\"o. of greruffii:n- - 1:
e each 'J JE !~ = .r - _;,
-~ fu e ou:fcom d me prooobil- Jo tl ln um h: :r of
es is a 511Cces-s,, fin
ci t in ail the toss ro th e prt"t-!e:m,
me ,-krnroing
ify of losing th e ga 1
Di:1\ i 5~ -I, .:;;J.9) Pt'black IIB m t~ )
=-
Ia] IJl ~G SE 4
¼
omes = 8
S:1 To.z1 nUL..-r:ilier of oll!tr I) -- "1~½ 4
~ IT .r - ;_r - H
.,1-.le mu:nber oi outcomes \" ........ .,
::..=,,-,.,.,..
v,<a.> .
•-•
OL
X = _ : ~:- ;:
2 J 1
su cr es s) = ;:- or J p (h-tue 111.lro.~)
=
Prooacilitr (c-etti.rig - '° 0, - ~

l 3
9" the gam e) = 1- -4 =--t;
A!,:i
n_;u
. rn • sin o
..-bilit,r (1o
..
Sc hem e, 10I9J
·cBS E :~fa1kin,g t =
or; 4

(i ) .m<J {i; ). , ,~
~..:w~'
tio n
N ow, from eq ua

writing ' 3y - 11

I ► Most of tth
co rre
ou tco
ct
m es
_
ar
-tu de nt s co mSo
e !> outcome_.
O ta1
e
"
m it er ro rs in
m
also in co rre ctl
etimes, favorable
y written by

· · · ·· - - - · · · · · ·

). 3 '- 5 - I1
stu de n ts.. From £"<-7u,1tio n ti l ·•
J]
j
w r t"t n1..1rh ks in th.~j.i ,
Hence, to ~al nu m
mes of <:: x -,- •• ~. i t
fav(1rable ou ko
-; ; to lis t t-h e 4 + 5 i" 1!
times .
► • · · · •-
-=,
Re m em . · 1 . . - · - · · ·- · ·· ·
,f
-
tossing a cou: : .
.. ,A - • .. - - - ~ ....
m Oswnal CBSE Question B1mk ChRpterwise & Toplcwlse, MATHEMATICS IBTANDARD), Class-X

Q. 7- A die is thrown once. Find the probability of getting Sol. Probability of either a red card or a q
(i) a composite number, (ii) a prime number. _ 26+2 28ueen
DJ !CBSE Delhi Region, 2019] - 52 : : 52
l

[ •, =~~,J
r '
I
P(neither red card nor a queen) ::: 1-~

= 52-28 = 24 =~
52

52 52 13
l
[CDSE l\1arking s~i
'1eo1e ,,.
Q. 11. Two dice are thrown at the same ti ' "lilSj
product of numbers appearing on th:e .and ~
Find the probability that the product"-' IS not~.'
a "·
number. m [CRSES Pnine
. . Qr,
IS

2o1~
Sol. Total number of outcomes = 36
&o4,M~ • '":,:e~ .,_,, Favourable .outcomes are (l, 2), (2, 1), (l, ) 1·
°)

3 '(3, 1),
(1, 5), (5, 1) ,.e.,6

,1§?? c. ..:1, v
zc: . 2,
I
Required probability = i._ or !
36
[CBSE Matkin!'
6
~ .-1. _
1
" " '-'u:ute 4l
'~cf s:Cards numbered 7 to 40 were put in a box. Poon~ Q. 12. Two different dice are thrown togeth ' 18l
selects a card at random. What is the probability the probability that the product of th er. Find
appeared is less than 18. e nllniber
that Poonam selects a card which is a multiple of 7?
[CBSE Delhi, 2019] DJ [Foreign Set-I II III I ; , 20)~
Sol. No. of all possible outcomes = 62 ::: 36
No. of favourable outcomes = 26
½
(4, 2)(4, 3)(4, 4)(5, 1)(5, 2)(5, 3)(6, 1)(6, 2){l I
: Sol. (1, 2)(1, 3)(1, 4)(1, 5)(1, 6)(2, 1)(2, 2)(2, 3)(2, 4)(2 )
(2, 6)(3, 1)(3, 2)(3, 3)(3, 4)(3, 5)(4, 1) '~
I•
'I
:. P(Product appears is less than 18) = 26 ::: ~
36 18 1½
[CBSE Marking Scheme, 20lij
L..k~~H~~~=l.t.l!M~~r~~~~· Q. 13. A letter of English alphabet is chosen at rand
find the probability that the letter so choseni/111,
I
(i) a vowel,
2: (ii) a consonant.
'~-- -~--- --~-~- -~ --~- --- -------
---~- ---- ------'
I
a [CBSE Delhi Term-2, Set-I, II, Ill, 2013]
Q. 9. An integer is chosen at random between 1 and 100.
Sol. Since, total number in english alphabet is 26. ~
Find the probability that it is: which 5 vowels and 21 consonants.
(i) divisible by 8.
(ii) not divisible by 8. t . ~, (i) P(a vowel) =~
! 26
DJ fCBSE Delhi/OD, 2018]
Sol To,tal number of outcomes = 98
.. . ,:,- ii '·~ -
~.. ii.
~ l ... (ii) P(a consonant) = qf)
(i) Favourable outcomes are 8, 16, 24, ...., 96, i.e., lf ½.
. ~ \i'
.. [CBSE Marki1tg Scheme, 2ill,J 1
12
Probability (integer is divisible by 8) =
98
or '-
49
i.
~
Q. 14. Ab~g contains cards with numbers written on it
fro~ 1- 80. A card is pulled out at random. Find the
t '·.probability that the card shows a perfect square.
(ii) Probability (integer is not divisible by 8) = 1-# J_ m
[CBSE S,A.-2, 2010)
[CBSE Delhi. Set-I, ll, lll, :Oti]
43
Sol. S = {1, 2 ............... SOl •!
49 n(S) = 80 ,:
[C,BSE Mru1<lng S<:}tcmc, Z0lB] :½.- A = {1, 4, 9, 16, 25, 36, 49, 64)
))
n(A) = 8
Q. 10. A card is drawn at random from a well shuffled
n(A) 8
deck of 52 cards. Find the probability of getting P(A) = n(S) = 80
neither a red card nor a queen.
1
m[CBSE SQP, 2018] =-
10
[CBSE O.D., Set-I, II, Ill, 2016]
PROBABILITY . .
(ii) Suppose she is allowed to pi k lliiiiil
the bag, what is the probabilityc fa tn:1tble ROD\
.. . o getting a p .
when it 1s given that the bag conta1ns · nu,
1. t , out of which 6 are black?
20
marble

~
is chosen between 70 and 100. Find the
[I + fl [CBSE Delhi & OD, 2.020]
. ,i-:;~~f~ thbaterit is.
r '(Ile null\
·11pfl r.r.l m [CBSE SQI~2020-21]
.11 • • i!,le by 7. i4!.i i.:a
jl dil'ls T~tal n~ber of~teg~; ~ 29. · ·- · ......,,;
: Sol.
iL • (p · · · i.../ ). -'6-::- · ' :I (b1aiol.
- \ • '
Ol'f'bcttl DD "ft'Vllh • I,

' , probability . rune numut:r = 1 ,naa,t,,': .,,


-,,,.01,,n,' .:. ~
p) · -29 _· < '. , '.;J : · . " 1.!err

e
J1f'J'O
bability (number divisible:b¥7)= .i
· . •
. _' "f",'\.•··'f1;
. 29 ·-·c., : \ ~-
.
' t-·'
J ~
I

l)i ' .

; rc~~E s9~ ~ar~~t~·~¢:n}~J9l9Jij


I 'Jedsolution:
pl Total number of outcomes = n(S) = 29
(i) Total prime numbers between 70 and 100
= 71, 73, 79, 83, 89, 97
i.e., favourable outcomes = n(E) = 6
P(prime number) = n(E) I
I

n(S)
2:
'~- ----- -- --- ------- -- -- ------------ -~ --- -----·
6
1 Q. 3. From all the two digit numbers a number is chosen
29 at random. Find the probability that the chosen
number is a multiple of 7.
(ii)Numbers, which are divisible by 7
ID [CBSE OD Comptt. Set-Ill, 2-0171
/ = 77,84,91,98 f'l"--~· "", -~· ..... , ....... . -1,·•.-•,4·--~ -;:~-f }Cl"'t"¥~~-~ -." ( ~ ~ , ~ ...,.

i.e., favourable outcomes = n(E1) =4 ¼ Sol. All_pqssible ou~mes:-:¥e-,~0;'~1, .J2 '.'t••· 98,.99.t - -. ,
No. of all posstble .9uh;orrt~s.= 90;' ~\ .~ . . 1
:.P(number divisible by 7) = n(fi) All favourable outcomes are
14, 21;2$::'.: 9S · -,-_, '
n(S) No. offavoural,le outcomes = 13 ' '.'. . , \J '•' ,,.,'1• .•
t• -:f."' • , • . ,,-..· •~
l

4 •'- P(getting a ~umber multiple o£7f~ -~ \~ ''.. /


29
1

, . . ..;,;_ ,_,_ ~
-
' . .. •, ·\>~:.
CBSE
~,~•. L. ¥~ktll g Scl.i.e.IRej 201.1] .
• : <gqi:rC{f -i
•, Q, 2. Read the following passage and answer the Q. 4. A box contains c,;ds; ~~ ~
n~i,";: thl 90. A
questions given at the end: drawn at random from the box. Find the probab ility
DiwaliFair that the selected card bears a •
Fair involves using a (i) Two digit number ·
A game in a booth at a Diwali
spinner first Then, if the spinner stops on an even (ii) Perfect square number
number, the player is allowed to pick a marble
_ . _ . ·."' P- , ..,..., ., ... ,,'!J [~elhi Comptt. Set-I, 20171
from a bag. The spinner and the marbles in the bag :"' . .·,4"!-'<f'-'-"
.Sol.No; o(a.ll:P9&sible:oii.icomei - i, :(,' ' ·•~:"'·'"';. "li
tt··-::.-~! ,
·,- ,':i -
1 1
are represented in Figure. (i)No.of~:.'....i · '-;.. "' •,·2 ,fi,.;t ··. ·: , t-
, ~US •\QV10g"' ~ numL.... :,= 90:- •:~~-,'JJ.¼~
IY.:,;::~ <,
Prizes are given, when a black marble is picked. . N ff utU"" '
. ' o. o avourable outcomes"" 81_ , :· ._...,,~,- , . '':
Shweta plays the game one. ?(selected card ~ -two digj.t.n\ im~); · ~- , ,
.,,,----- ) = No. of favourable outcomes .., Si , 9
r----- ----
I
No· ofall PoSS1'b·'!le outcomes 90 -10 ,. \ '"I_. =-- · ,·
(ii) Perfect square number s be . . .. . . _ . · ,. : •.. :
l; 4, 9, 16, 25, 36I '?, ' .
40 64 ·s ·
1 twe~ .1.
to 90 ~ - >:.·•,>
,.,,. '" l
.-. N0 · of favourabl·e ou-;.;.. ' · .· '. .;>. :
P(S I . ...0 mes == 9 . ., . . .
. eectedcar:dbears i • . . . . '. ,.,
. peri;ec t~are ntm\~i,:s)< ,.'.
_ 9 . ·1 · ;:
@)What is the probability that she will be allowed to _-:-:_ ,~l::; 10 ';J:.i
pick a marble from the bag?
~o;-~\i
.:!S~~-\M~.~ ~$che~t:.~.,~~,,;~=1:2
\.- ,,.~ ........,,.,1, .(.', :. : ."i- ...

~ese questions are for practice and their solutiolll are available al the end of the chapter

.
M ._ .
. . . MATHEMATICS (STANDARD), Clas s X
Oswaal CBSE Qurstion Bank Chnpterwise & lopicwtse , --~ ~~~~- -

Q. S. Two different dice are thrown together. Find th e


probability that the number obtained,
(i) have a sum less than 7.
(ii) have a product less than 16.
(iii) is a doublet of odd numbers.
m
[C BS E D elh i Set-I, II, III, ZOl 7]
2
Sol. Total number of all possible outcomes = 6 = 36
(i) The sum less than 7 = (1, 1), (1, 2), (1, 3), (1, 4), (1, 5),
~4~~~~~~~~~~~~~~~~~~
). n,e,... bl• t l:'. b o..UJ In ~ t bo_
No. of favourable outcomes = 15 Q>f '1 .1
I ~ I

P(have sum less than 7) = -15 = -5


36 12
1 --@ ~-; ;1;~1:-~~~ ~i ~·;;~·~-/duie~:~; ~1----.:j
f is the probability of getting (i) a Prime n ce, Wh~t
(ii) Product less than 16 = (1, 1), (1, 2), (1, 3), (1, 4), (1, 5), each dice 7 (ii) a total of 9 or 11 ? lltriber Ott (il
(1, 6), (2, 1), (2, 2), (2, 3), (2, 4), (2, 5), (2, 6), (3, 1), (3, 2),
~~~~~~~~~~~~~~~~~~ m[CBSE Delhi Set 1
(6, 1), (6, 2) , 20151
Sol. (i) Favourable outcomes are _(2, 2) (2, 3) (2, 5) (j ' (U)
No. of favourable out outcomes = 25
(3, 3) (3, 5) (5, 2) (5, 3) (5, 5) 1.e., 9 outcomes. ' 2)
:. P(have a product less than 16)
25 P(a prime number on each die) = -2_ 0 !
1
36 36 r 4 ½ (Ji)
(ii) Favourable outcomes are (3, 6) (4, 5) (5, )
4
(iii) Doublet of odd numbers = (1, 1), (3, 3), (5, 5) (5, 6) (6, 5) i.e., 6 outcomes (6, 3)
No. of favourable outcomes = 3 6 1 1
P(a total of 9 or 11) = or ½ (iV)
:. P(a doublet of odd number) 36 6
3 1 [CBSE Marking Scheme, 2016] !,
1 1
36 12 Q. 9. Two di~~rent dice are thrown together. Find th .
fCBSE Marking Scheme, 2017] probability of : e Q. 2
(i) getting a number greater than 3 on each dice. 1
016:lreter throws two different dice together and finds
~ e product of the two numbers obtained. Rina (ii) getting a total of 6 or 7 of the numbers on two di
ce.
throws a die and squares the number obtained. DJ + m[CBSE Delhi Set II, 2016] (i
Who has the better chance to get the number 25. (ii
[COSE Delhi Set-I, ll, III, 2017] Sol. (i) Favourable outcomes are (4, 4) (4, 5) (4, 6) (5, 4) (iii
~~~~~~~~~~ 1
Sol. Peter throws two dice together :. No. of favourable outcomes = 9
:. Total number of possible outcomes = 62 = 36 ½
He get 25 only when he gets (5, 5) P(a number > 3 on each dice) = _.2_ or ! ½
Q.
36 4
:. No. of favourable outcomes = 1 ½
= __!_
(ii) Favourable outcomes are (1, 5) (2, 4) (3, 3) (4,2) (5,J) !
P(getting the numbers of product 25) 1 (1, 6) (2, 5) (3, 4) (4, 3) (5, 2) (6, 1) l ,
36
Rina throws one dice
:. No. of favourable outcomes = 11 1
:. Total number of aUpossible outcomes =6 11 S
P(a total of 6 or 7) =- . ½
The number where square is 25 is 5 36
:. No. of favourable outcomes = 1 ½
[CBSE Marking Sd1eme, 2016\
1
?(getting a number whose squar~ is 25) =6
1 1
6> 36
Q.1. The King, Queen and Jack of clubs are removed
Hence, Rina has better chances to get the number from a pack of 52 cards and then the remaining
25. [CBSE Ma rking Schem e, 2017] cards are well shuffled. A card is selected from the
<t)A bag contains 15 white and some black balls. If remaining cards. Find the probability of getting a
the probability of drawing a black ball from the card.
bag is thrice that of drawing a white ball, find the (i) of spade (ii) of black king
number of black balls in the bag. (iii) of club (iv) of jacks ·
DI [CBSE OD Set-HI, 2017] lJJ [CBSE Complt. Set 1, II~
- - - - - -- - - - - - - - - - - - - - - - - - - -- -- - -- - -- -
(~ These que5tions are for practice and their solutions are available at the e11d of the chapter
~
•I fot~l 11
umber of cards = 52- 3 = 49 1
PROBABILITY •

I
1 Sol. No. of all possible outcomes= 125 ½
~ol• P(spade) = _]_ 1 (i) Ramla'l will buy a good shirt
• (1'I 49 :. No. of favourable outcomes = 110 ½
P(black king) =~ 1 :. P(Ramlal will buy a shirt)
(ii) 49 No. of favourable outcomes
=
P(club) = 10 1
No. of all possible outcomes
fiill 49
:. P(Ramlal will buy a shirt)
PQack) = 2_ No. of favourable outcomes
1 =
11vl 49 No. of all possible outcomes
I
I
I [CBSE Marking Scheme, 2018] 110 22
=-=- 1½
') l,' ued Soluti' on : 125 25
oeta f
Total number o cards = 52 - 3 == 49 1 (ii) Naveen will reject the shirt which have major
Number of spade cards = 13 defects.
Ii)
:. No. of favourable outcomes = 125 - 3 = 122
P(getting a spade) = 13 1 :. P(Naveen will buy the shirt)
i] 49 No. of favourable outcomes
(ii) Number of black king = 2 _ 1 = 1 - No. of all possible outcomes
P(getting a black king) = _!__ 1 122
49 1½
125
(iii) Number of club card = 13 - 3 = 10
[CBSE Marking Scheme, 2017]
P(getting a club card) = 10 1 Q. 5. The king, queen and jack of clubs are removed
49
from a deck of 52 cards. The remaining cards are
(iv) Number of jacks = 4 - 1 = 3 mixed together and then a card is drawn at random
P(getting a jack) = 2- 1 from it. Find the probability of getting (i) a face
49 card, (ii) a card of heart, (iii) a card of clubs (iv) a
Q. 2. A box contains cards numbered from 1 to 20. A queen of diamond.
card is drawn at random from the box. 0 f.i1 [CBSE Delhi Comptt Set-II, 2017}
Find the probability that number on the drawn Q. 6. A box contains 90 discs which are numbered 1 to
card is r 90. If one disc is drawn at random from the box,
(i) a prime number find the probability that it bears (i) a two digit
(ii) a composite number number, (ii) number divisible by 5.
(iii) a number divisible by 3 f.i1 [Foreign Set-I, II, Ill, 2017]
0 Ei.1 [CBSE Comptt I, II, IIl,2018] Sol. Total number of discs in the box = 90
[CBSE Term-2, Set I, II, 2015]
:. No. of all possible outcomes = 90 ¼
Q. 3. Two different dice are rolled tog~ther once. Find
the probability of numbers com.mg on the tops (i) Discs with two digit number are 10, 11, ....... 90
whose product is a perfect square.
No. of discs with two digits numbers = 90 - 9 = 81
Ei.1 [CBSE OD Comptl Set-I, 2017]
¼
Sol. No. of all possible outcomes = 62 = 36 1
:. No. of favourable outcomes = 81
All favourable outcomes are P(a disc with two digit number)
(2, 2), (3, 3), (4, 4) (5, 5), (6, 6), (1, 1), (4, 1), (1, 4) No. of fav,ourable outcomes
¼
. No. of favourable outcomes = 8 1¼ No. of all possible outcomes
·· . the numbers, whose product is a
:. P(getting 81 9
8 _ 3, 1¼ = - =-
perfect square) = 36 - 9 · 90 10

[CBSE Marking Scheme, 2017] (ii) The numbers divisible by 5 from 1 to 90 are
5, 10, 15 .... 85, 90
tains 125 shirts of which 110 are good
Q. 4. A box co;inor defects and 3 have major defects :. No. of favourable outcomes = 18
12 ha~~ will buy only those shirts which are good :. P(a disc with a number divisible by 5)
R~ Naveen will reject only those which have
wh~: defects. A shirt is taken out at random from
maJOb Find the probability that :
18
= - =-
90
1
5 .
1¼ i
i
the ox,
Lal will buy it. [CUSE Marking Scheme, 2lll 71 i
I
(1') Ratil
n will buy it. ffl [CBSE OD Set-III, 2017]
(,i)l'1•.-v/../._-:.

lflr4
.. &TJCS (STJll'IUlln.<,,, '- ' " " •' " I\

)I,\TJIJ:iun
. "'· Toriicwlse,

'
-1 111 prerw1s e ,. (I) An cve11 11un1bcr
1'-11!11 c
&liil Oswaal CBS E Qur stion n
1111k

Jack s and kings A nibcr which is a multlplc of 13.


18' ( ck colour (fl) nu re num ber
Q. 7. From a deck of 52 p!llying
can squ a
~ 0 J,1a d4 (Iii) A perfect
and Que en and Ace .
of red colo ur
s arc ,nix ed an al (Iv) A prime number less t~an 20. IC BSr.· SQJ> 2n
arc removed. lltc remaining card ,J{CJJS I: Delhi Com ptl. Sct -J, 201 7/
"t'nd the prob.ibility tlt • · ' .,,16
ca rd is d raw11 at random. r es ,,, 100 I
Sol. Total number of possible outcom
the drawn card Is: I lo 100 ,,, 50
(i) Number of even numbers from
(i) A black Queen. .. P(card taken out· has an even num
ber)
(ii) A card of red colour. ' 50 1
. rl == 100 =2
(iii) A Jack ofblac.k colour. Sd -1,
20 I
OD Com ptt.
~ A face car,:I, r:l [C'USE
'Cl (ii) Multiple of 13 from 1 to 100 are
"'1
2 2
Sol. No. oi an possible outcomes ==
52- (2 + + + ~) 65, 78 and 91.
= 44 13, 26, 39, 52,
No. of favourable outcomes = 7
(i) No. of black Queens in the dec
k == 0
0
::::
. P(card taken out has multiple of 13) 10()
J....
Que en) == =0 .'
k

l
P(ge tting a blac 44
(iil) Perfect square numbers from
1 to 100 are
1
Hence it is an impossible event 1, 4, 91 16, .ZS, 36, 49, 64, 81 and 100.
No. of red card s = 26 - 4 == 22 No. of all favourable outcomes =
10
(ii)
22 1
P(getting a red card) == -44 = -2
1
:. P(card taken out has a per 10
fect square numb
== - = -
I er) I
100 10
(iii) No. of Jacks (black) = 2
2, 3, 5, 7, 11, 13 17.
:. P(ge tting a black coloured Jack) (iv) Prime numbers less than 20 are
2 1 '
= -44 == -22 and 19.
es = 8
No. of all favourable outcom
ber less than 201
(iv) No. of face cards in the deck = 12 - 6 == 6 :.P(card taken out has a prime num
2 I
8
6 3 1 == - = -
/i P(getting a face card) = 44 = 22 100 25 1

[CBSE Marking Scheme, 2017} [CBSE Mark; ng Scheme, 20l7J

together. Find the Q. ~ n fig. a disc on wh:ch a player


spins an arrow I
Q. 8. Two different dice are thrown ined have where 'a' is the
probability that the num bers obta · twice. The fraction b is formed,
(i) even sum, and stops on the first
rJ [CBSE OD Set-III, 2017] number of sector on which arrow
(ii) even product sector in which
spin and 'b' is the number of the
On each spin, each
the arrow stops on second spin,
ction by the arrow. Q
sector has equal chance of sele

: t}J I, • h• Tfl o~ ehq i h ic 1'"9"1"'1 .


.
Find the probability that the fraction
> 1. i
'~ - ,folc,J fD'Uil,"' 01.1lu ,me.
: ;_ "-C.Sl •. r ~, . .
3 ,5),
C~-•()... C.1~0 , CJ,;), _(_
.. :
[C 1,,)J (1.5') , ( 1, (),l t ,'21,{' 1, u 1...!
:- ~ 1 •-
,_C5 cl),..C.".;_l:2., (J,.~l ,!•,~
'lk,j 1
. (~.•1 , C<,O) , C•. • J, Cir.1J

.a,~--;;g.- ~:~
_
1 _: _.
~ - - - · - ·- - - · -1
1--:: . 1, · / ·

r A
1 I o, 0 - 5" •· l
i
I - 7 - . •
I _ . _ _ _ _ _ __ _
-~ :
on t ,.,;·,..r' tum i,.L _ ~t g_ ~
,
: __ _...::-~ ~l:li ll.~ _,f ~,.,.+;"'1 l
I

I • :

p,n,dur'- of diqll " b i> \rl"r"I ~-


: i_Q'; A 1:
__ _ --··- __ ~ ___ __ _ __ I

I
.
-_ _
I _ . ,~ ~-~ - - ___ :
:~
_ ___
~ ._(J,_4'),_(.l_,I,,, ( ?, (),_ -•~'),
_ _ '-2_.~ t._c_~1«.'lr J J ,f ). Ctt, ,
ci.,,1 ._<tt,\) , .
{tlJ) ,..C,&, JJ, ,(), .(!Jj_, _
c~~,
c~ r,, cc.,1it)., _ 1

f,.tt, ,_t.,rJ,(, ,o]. ,
1
__{fi.'11.1.~!•'-'1, (s ,r:), ,,. c,,'1, ,c.,1),
;
; -.- ---·L..2]._ _-· Ei,1 [Foreign Set I, II, III, 2016)
: .!'.<~J...:-....~.t~l - ~ ..ll ~
l

1
~
. - - --
3

-H 7 -
,-.(.),

,. ~-:.->:
~--3 4 5 . · ·.
S
1 'Sol. For t > J , :when a = 1, b can not take any
value:
C
r -
p,,lxl biH~ .of ,-,H" 'I ,...,n P""J"'t !4 ~ _,,., o-1r. 1
take 2 valu~,
; •

~----- -~-ch----
---ds---on---whi ·---1,---2,---3 -~-
---bers -----··~;
...... 100 are
a =; Z,·bdri take 1 valu e, a = 3, b can
. 2¾
,· b ca'\,n
Q. 9. Car num a = 4; take 3 values
and no number ,_
written (one number on one card
and are mixed :when a;,,·5, b can tak,e4 values,
is repeated) and put in a bag
I
\
\
thoroughly. A card is drawn at ran
dom from the ._:_:,, a_::;::,61 b can.,take 5. values.
taken out has.

\
bag. Find the probability that card
,. nuntber :r is selected at r-,u'.\dt\01 fn\n\ th"
~ u.tnbel'S 1, 2,. 3 md -b. Another number y is ~1~,w
1crandom from
th~n ~ 1.{. 9 ind lf\. find th<'
r rl:\l'l-~u. that product of x md -~ is ~ ~n 1.:.,,
-1.,i..:JitY

m{CBSf 0D S..:i -ll. : ,,\(";

Ans. Option (B) is com>.\'!.


Q. -l. Find the probabilit y of getting a red focc rnni.
3 I
(A) 26 (H)
1:'I

I
(C) !
!12
(IJ)
4

Ans. Option (A) is correct.


E.\1,1miatio11: No. of Red faCt' card "" 6
- ...._..~ ... -- •·.
Toti1l no of cards = 52
Q.1. Find th~-~robability of getting a king of red colour.
. 1 1 Probability of getting a foa.: cnrd
(A) 26 (B) 13 No. of fuce cards
1 Total no. of c,1rds
1
(C) 52 (D) -
4 6 3
_ = 52 .:: 26
, Ans. Option (A) is correct. .
Explanation: No. of cards of a king of red colour - 2 Q. 5. Find Ute probability of gcttiug a spade.
Total no. of cards = 52
.!_ (B) ~~
13
Probability of getting a king of red colour (A) 26

=
No. of king of red colour
Total number of cards (C) ;2 (O) t
2 1 Ans. Option (D) is correct.
52 26 Explm111tio11: No. of face card "' 13
Total no of cards "" 52
Q. 2. Find the probability of getting a face card.
Probability of getting a face card
! (A) ..!__ (B) .!_ No. of face cards
26 13 Total no. of cards

lt (C) 2
13
(D) 2_
13
. ,MATJJJtMATICS (STANDARD), Class-X
Oswaal CtlSE Question Bonk Chnrt rrwisr. & Topk:WJSC, . . -

r ·p/allflho11: No. ofoutcomes when the
II Rea d the following text .ind nnswer ,rny i:;l surr, ·
· 'fotal outcomes = 36 IS 13 "' IJ
Rahul and Ravi pkmncc.1 tn rl,1y 13u~incs~ (boa rd o
g;ime) in which tht·y were surr<'sl'd lo use two Probability = - :::: O
dice. [CBSE Qll 202 l l 35
Q 3 Now it was Ravi's turn. He rolled the <f
. ·. .. . ·~iit·t~~ ..,. ~~" ' .

., ·:,~~jj·.. :r
.·. yr,' .·. ;' .,.
... /;._..~·t;
· · is the probabilitr that he got the surn o?· What
~ - ·.,·-~~;::.t·;:l"~~~U ~r~i. .. .-~
numbers appearing on the top face of th the tiv
- ~~ less than or equal to 127
5
e dice~

,,. f (A) 1 (B) 36


-
\
~)
. .
'·.
' .
.· .
~ 1· • •
(C) 18 (D) 0
· ~
-·~··=
. _ "£111/ . ··~
~= -~~
Ans. Option (A) is correct
c--
Expla11atio11: No. of outcomes when the SUJn ,
.?~·\:-_"~-:- . _.7~\i~--:~I''. ~~~ than or equal to 12 = 36 1s less

Total outcomes = 36
probability =
36
=1
..
36
Q. 4. Rahul got next chance. What is the probability
he got the sum of the two numbers appe . that
the top face of the dice is equal to 7 ? ilring on
5 5
(A) 9 (B) 36

1
(C) 6 (D) 0

Ans. Option (C) is correct.


Q. s. Now it was Ravi's turn. He rolled the dice Wh
is the probability that he got the sum of twat the
Q. 1. Ravi got first chance to roll the dice. What is the numbers appearing on the top face of the dice ;
probability that he got the sum of the two numbers greater than 8 ?
appearing on the top face of the dice is 8 ? 5
1 5 (A) 1 (B) -
36
(A) 2h fB) 36
1 5
1 (C) 18 (D) 18
(C) 18 (D) 0
Ans. Option (D) is correct.
Ans. Option (B) is correct.
Explanation: The outcomes when two dice are
thrown together are:
= (1, 1), (1, 2), (1, 3), (1, 4), (1, 5), (1, 6)
(2, 1), (2, 2), (2, 3), (2, 4), (2, 5), (2, 6) I. Read the following text and answer any four
(3, 1), (3, 2), (3, 3), (3, 4), (3, 5), (3, 6) questions from Ql to QS.
(4, 1), (4, 2), (4, 3), (4, 4), (4, 5), (4, 6) Two customers Shyarn and Ekta are visiting a
(5, 1), (5, 2), (5, 3), (5, 4), (5, 5), (5, 6) particular shop in the same week (Tuesday to
(6, 1), (6, 2), (6, 3), (6, 4), (6, 5), (6, 6) Saturday). Each is equally likely to visit the shop on
Total outcomes = 36 any day as on another day.
No. of outcomes when the sum is 8 = 5
Probability =~
36
Q. 2. Rahul got next chance. What is the probability that
he got the sum of the two numbers appearing on
the top face of the dice is 13 ?
5
(A) 1 (8) -
36
1
(C) 18 (D) 0
Ans. Option (D) is correct.
.....
PROBABILITY -
I ,\l'C
the total numbers of favourable
JI· ,1'J1ao111eS 7 Q. 3. What is the probability that spinning arrow will
J 1
01 1' Total outcomes = 5 x 5 = 25. point at a number greater than 27
1 Sol. No. greater than 2 = 3, 4, 5, 6, 7, 8 = 6
5° · ;s the probability that both will visit the
, ,~t,at .. same day?
,v -hOP 0 " P(getting no. greater than 2) = ~ = ~-
~ ' No. of possible outcomes that both will visit 8 4
1
5° · 1op on the same day
the s l - 5
= (T, T), (Yv, W) , (Th , Th) , Q. 4. What is the probability that spinning arrow will
(EF), (S, S) - point at an odd number?
th will visit the shop on same day) =~= ! Sol. Total odd numbers = 1, 3, 5, 7 =4
p(b
O
25 5 4 1
P(getting odd number) = =2'
\f11al are the total numbers of favourable outcomes 8
Q-
3
:f
· both will visit the shop on consecutive day? Q. 5. What is the probability that spinning arrow will
Sol. No. of possib~e outcomes that both will visit the point at a number less than 9?
shop on consecutive days = (T, W), (Yv, Th), (Th, F), 8
(F, S) or 0N, T), (Th, W), (F, Th), (S, F) = 8. Sol. P(getting no. less than 9) =
8 = l.
4_What is the probability that both will visit the III. Read the following text and answer any four
Q. shop on consecutive day?
questions from Ql to QS.
Sol. P(both will visit the shop on consecutive day)
In a club men are playing the ca rd game. A man
8 named Ramesh draw a card from a well-shuffled
=-
25 deck of cards.
Q. s.What is the probability that both will visit the
shop on different day?
Sol. P(both will visit the shop on different day)
= 1- P(sarne day)
= 1- -5
25

= 1- -1
5

4
5
Q. 1. Find the probability of getting a king of red colour.
II. Read the following text and answer any four Sol. No. of Red king = 2.
questions from Ql to QS. 2 1
P(getting a red king) = 52
=
26
·
e of chance consists of spinning an arrow
A ~am es of rest pointing at one of the numbers Q. 2. Find the probability of getting a face card.
which co~ 6 7 8 (see figure) and these are equally Sol. No. of face card = 12.
1, 2, 3, 4, , , ,
likely outcomes. 12 6 3
P(getting a face card) = = =
52 26 13
·

Q. 3. Find the probability of getting a jack of hearts.


1
Sol. P(getting a jack of hearts) =
52
Q. 4. Find the probability of getting a spade card.
Sol. P(getting a spade card) = 13
52
total possible outcomes? 1
1 Whatare the
1 Total poss1'ble outcomes = 8.
Q. · 4
So · bability that spinning arrow will Q. 5. Find the probability of getting a queen of
Q. 2. What is the pro
diamonds.
oint atS?
p 1 1
=8 Sol. P(getting a queen of diamond) = •
Sol. P(getting 8) 52

1

-
~- - -
• & T0 picw1se , MA
Oswaal CBSE Questi on Bank Chaptc rwtse

Multiple Choice Questions


. THEMATICS (STANDARD), Class-X

Ans. 3: Option (D) is correct.


= 3 1
Explanation: Probability of an event is always a Ans. S: Numb er of days in a leap year = 366
proper fraction. 366
7 = 52.28
Numb er of weeks =
Also, 0 :s P(E) :s 1
17
But >1 So, there will be 52 weeks and 2 days
16
So, every leap year has 52 Sunda ys
17 Now, the probab ility depen ds on remain ing
Theref ore, can never be probability of any
16 2
event. days
Ans. 10: Option (C) is correc t The possible pairing of days are
Explanation: Probability of an event + Probability Sunda y and Monda y, Monda y and Tuesday,
of its compl ementa ry event = 1 Tuesday and Wedne sday, Wedne sday and
:. p + Probability of compl ement = 1 Thursd ay, Thursd ay and Friday, Friday and
Saturday, Saturd ay and Sunda y. ½
Probability of compl ement = 1 - p
Ans. 13: Option (A) is correc t
There are total 7 pairs and out of 7 pairs, only 2
pairs have Sunday . The remain ing 5 pairs does
Explanation: In a deck of 52 cards, there are 26 red
cards. not include Sunday .
Numbe rofred faceca rds = 3ofhea rts + 3ofdiar nonds Therefore, the probab ility of has not 53 Sundays
=6 . 5
7•
in a Leap year 1s ½
So, probability of having a red face card
6 3 Short Answer Type Questions-I
=-=-
52 26 Ans.1 : Try yourself, similar to Q . 7, of very short
Very Short Answer Type Quest io ns answer type questio ns.
Ans.5: E1 {(1, 5),(2, 5), (3, 5), (4, 5), (5,
Ans. 1: Probability of getting a double t =1 1
6 5), (6, 5), (5, 1), (5, 2), (5, 3), (5,
Double t means same numbe r on both dice. 4), (5, 6)}
[CBSE Markin g Schem e, 2020]
:. P(5 will come at least once) =P(E ) = 11
Detaile d Solutio n: 1 1
36
The outcom es when two dice are thrown togethe r
are: P(5 will not come either time)= 1- 11 = 25 1
(1, 1), (1, 2), (1, 3), (1, 4), (1, 5), (1, 6) 36 36
(2, 1), (2, 2), (2, 3), (2, 4), (2, 5), (2, 6) [CBSE .Marking Schem e, 2019J ,
(3, 1), (3, 2), (3, 3), (3, 4), (3, 5), (3, 6) I

tA:~•l · '
I
(4, 1), (4, 2), (4, 3), (4, 4), (4, 5), (4, 6)
(5, 1), (5, 2), (5, 3), (5, 4), (5, 5), (5, 6)
(6, 1), (6, 2), (6, 3), (6, 4), (6, 5), (6, 6) i
Total numbe r of outcom es = n(S) = 36
Favourable outcom es are: (1, 1), (2, 2), (3, 3),
:
I
► In die based proble ms, studen ts often get J :I I

• confus ed in favorable outcom es. : :


(4, 4), (5, 5) and (6, 6) i.e., n(E) = 6 , - - - - - - - - - - - - - - - - - - - ~-~ - ~ ~
-~-------- -- - --~- - - ~ I
I
n(E) 6
Required Probability = - (S) = - = -1
I

n 36 6
Ans. 4: Total possible outcom es = n(S) = 6
Favourable outcomes = 1, 2 ► List all the events of throwi ng a die and not I
; :
I

i.e., n(E) = 2 ,,.,.,.., ,_ _ .., assume in mind.


__ __ _ .,. _ ___ __ _,, ,.._,.. _ _
_ _ _ __ _ _ _ _ _ ._.,,.-...., ___ ,. ,. .\ !
; I
P(num ber less than 3) = n(E) I

n(S)
PROBABILITY I---l--l~ 1
I --- --
- -- -- -- -- -- --
---- ----- ----- -------- ---------- --- ----- -- ------ ---
-~ n sw er Type Q~;;il~-~- --
., I

I I
I'
sol, 2: (i) Pri me nu mb ers fro
2, 3, 5, 7, 11, 13, 17, 19
m1
i.e. 8_

P(p rim e nu mb er) ==


to 20

!
are

or ~ 1½
► All necessary outcomes mu
st be listed be fore
sw ers mu st be in
I
I 5 finding probability an d all an
.
I
I (ii) Compos1te nu mb ers fro m to
1 20
20 _are ~ ----. ~: ~~r~e!! ~~~: ----. ----
--,. ---. . ------!
4, 6, 8, 9, 10, 12, 14, 15, 16, 18I / l,e, , 11
I
I
I

' P(C om po sit e nu mb er) == !!_ mes = 52 - 3 = 49


Sol. 5: No. of all possible outco
I
I 1½
20
No. of face cards = 12 - 3 =
I
I 9 1
I
(iii) Nu mb ers div isi ble by 3 fro m 1 to 20 (i)
I
18 i.e., 6 are 9
I
I 3, 6, 9, 12, 15,
Required probability = 49
~ ~
I

'I
I
P(n um be r divisible by 3) == 20 0 r 10
2
I
(ii) No. of car d of he art in
the de ck = 13
8}
[CBSE Marking Scheme, 201
I
I
I 13 1
I
:. Re qu ire d probabill ty =
I
j
49
I
I
I
(iii) No. of cards of clubs
= 13 - 3 = 10

'~~~~........
~l:...~Lt~W
-..ii...:...;.~~~.;,it.l~the 1
I
co mm following :. Required pro ba bil ity = :~
1
,
I I
I I err ors .
I t are incorrect. of dia mo nd
(i) Total ou tco me s of ev en (iv) There is on ly on e qu een
I
I I
I I
incorrect.
I
I (ii) Fa vo ura ble ou tco me s are plest : ~ 1
I
I (iii) Th e res ult s are no t
giv en in the sim
, I :. Required probability = 4
I
I f orm. e.g. 2017]
I
8 2 [CBSE Ma rki ng Sc he me,
20
= 5
-- --- - - -- - --- --- ~- - A
~----~ -- -- -- -- - -- -- -• - -- -
I

ility?
een exp eri me nta l pro bab ility an d the ore tic al probab ,
fer ent iat e be tw
: (1) Will yo u be ab le to dif eth er it is cor rec t or inc orr ect ?
for the be low sta tem en t wh on e of eac h. :
me s-t wo he ad s, tw o tails or
I
: (2) Ca n yo u giv e rea son are thr ee po ssi ble ou tco
ult ane ou sly the re :
"If tw o coi ns are tos sed sim
eac h of the se ou
.!. ".
tcomes, the probability is 3
:
e, for
---------- --~--- --- -~-- ----l
Th ere for
----- ------- --- -- -- -------"------
~-- ---- ---~------- -- ---- -- -------
I

I □□□
I
J
SELFASSESSMENT PAPER

Maximum Time:1hour
MM: 30
given options. onthe two
I. Choose the comet alternative from the probability that the sum of numbers appearing dice is 5;,
Q.1. Two ditferent dice are toSsed together. The
is:
1
(4)
1 (B) 9
4

()
1 (D)
cards, the probability of getting a face card is:
Q.2. When a card is drawn from a deck of 52 3
(A) (B) 13
26

3 4
(C) (D) 13
26
numbers 3, 4,5, ...,25 is prime;is:
Q.3. The probability that a number selected at random from
5 7
(A) (B) 23
23
8 9
(C) (D) 23
23

Q.4. Median class of the following data is:


Size of the agricultural holdings (in hec) Number of Families (f)
0-5 10

5-10 15

10 - 15 30

15-20 80

20- 25 40
25-30 20

30- 35 5

(A) 0-5 (B) 5-10


(C) 15- 20 (D) 20- 25
Q.5. The weights of tea in 70 packets are shown in the following table:
Weight (in gram) Number of packets
200 -- 201 13
201 - 202 27

202-203 18
203- 204 10
204- 205
205-206
The modal class is
(A) 200-- 201 (B) 201 - 202
(C) 205- 206 (D) None of these
tamilies is tabulated below: PROUADILITY13
lowerlnit of the median class?
WCkly income (in)
(0- 1000
Number of families
50
10X)-2oO
10
X*)-3O000
100
30- 400
-40
4000-5000
50000 - (o00
Total
600
(A) 000
(B) 1500)
(C) 000
iL. Case-Based Question (D) 4000
[1 x 4 4)
Read the following text and answer the given below
questions it:
Mr. Sharnna is a Maths teacher who is Working in City Modern Schoo1, Bangalore. In his class X,
ne there. He decided to teach them as per their capabilities So. he total 80 students
conducted one revision test on the bsis o
class I\ result. Themaximum marks were 50. Thee Wee 12 students vho scorcd less than 10 marks. Sneha who
gut 3 mark was handed over a red card as an intination to work hard for one month and show improventent, as
she scored the least in the class. Anirudra was presented a bado of hoour tor scoring the highest in the cass.
He
for seonedt #s marks. Best performer badge given to Anirudra. ir. Sharma prepared frequency distribution table
the data of the marks obtained by the students in the revision test as tollows:
Marks Number of students
0- 10
10-20
20-30 21
30 40 13
40--50

questions:
Read carefully the above paragraph and answer any four
'N Tho lower limit of modalclass of trequency distributionobtained by Mr. Sharma is:
(B) 20
(A)10
(D) 40
(C) 30
frequency distribution is:
(ii) The median class of
(A) 10-20 (B) 20-30
(D) 40 - 50
(C) 30 - 40
marks obtained by the studentsiis:
(iii) The mean
(A)23.25 (B) 24.25
(C) 26.125 (D) 31.375
MATHEMATICs (STANDARD), ClasA-X
&Topicwise,
a14 Oswaal CBSE Question Bank Chapterwise median class is:
class andlower limit of
(vi) Average of upper limit of modal
(B) 25
(A)20 (D) 45 who
(C) 40; Scction Bfor thOse O scored between
30 and
40; 35
Section Cforthose whoscored between
those who 20 and
scored above
30 and Section D
for those wh0sCOred below 20. Nunber
(v) Mr. Sharma formed Section. Afor
of students in Section D
were: (B) 20
(A) 15 (D) 28
(C) 25 ([1 x3=3]
letter js
III. Very Short Answer Type Questions
random. Determine the probabilitythat the consOnant.
Q.9. Abag contains
Q.8. Aletter of 9
englishblack and
alphabet 12
is white
chosenballs.
at One ball is drawn at random. What is the probability that thet ball drawn

is black.
weights (in kg) of 40 persons.
Q. 10. The following is the distribution of
Number of person
IWeight (in kg)
4
40 - 45
4
45 - 50
50 - 55 13

55 - 60 5
6
60 - 65
65 -70 5

70 - 75 2
1
75 -80
distribution (of the less than type) table for the above data.
Construct a cumulative frequency
IV Short Answer Type Questions-I [2 x3=6]
Q. 11.Consider the following data:
Class Frequency
4
65-85
85-105 5

105 125 13

125- 145 20
145 -165 14

165-185 7

185 - 205 4

limit of the modal class,


Find the difference of the upper limit of the median class and the lower
Q. 12. Consider the data:
4 -7 8-11 12- 15 16 -19
Class
5 4 10
Frequency
Find the mean of the above data.
0. 13. Abag contains7 red, 5 white and 3black balls. Aball is drawn at random from the bag. Find the probability that
the drawn ball is neither white nor black.
[3 x 2 =6]
V.Short Answer Type Questions-lI
Q. 14. The following table shows the ages of the patients admitted in ahospital during a year:
5-15 15-25 25-35 35-45 45-55 55-65
Age (in years)
6 11 21 23 14 5
Number of patients
Find themode of the data given above.
Q. 15..Cards marked with numbers 3, 4, 5, ...50 are placed in abag and mixed thoroughly. One card drawn at random
fron the bag. Find the probability that the number on the card drawn is:
() divisible by 5, (i) a perfect cube, (iü) a multiple of 4.
VI. Long Answer Type Questions PROBABILITY 315
0. 16. Alife insurance agent found the [1 x5=5]
following data for distribution of ages of 100 policy holders. Calculate the
age, if polhies are given only to persons having age 18 years median
onwards but less than 60 years.
Ages (in years) Number of policy holders
Below 20 2
Below 25 6
Below 30 24
Below 35 45
Below 40 78
Below 45 89
Below 50 92
Below 55 98
Below 60 100

Finished Solving the Paper ? OSWAAL COGNITIVE


Time to evaluate yourself ! LEARNING TOOLS

OR
SCAN THE CODE

For elaborated
Solutions

You might also like